You are on page 1of 223

Глигор Тренчевски

Костадин Тренчевски

MATEMATIKA
za osmo oddelenie
PREDGOVOR

Nastavnata materija po matematika za VIII oddelenie, vo u~ebnikot e izlo`ena vo


~etiri temi, a sekoja tema e razdelena na soodveten broj - pottemi, od koi, sekoja
pretstavuva edna zaokru`ena celina - nastavna edinka.
Vo prvata tema - Sli~nost na triagolnici, }e se zapoznaete so u{te eden vid na
preslikuvawe vo ramnina. Napomnuvame, dobroto sovladuvawe na ovaa tema }e vi ovoz-
mo`i lesno snao|awe vo re{avaweto na niza problemi od prakti~niot `ivot i uspe{no
sledewe na nastavata po fizika i drugite predmeti. Vo ovaa tema }e se zapoznaete i so
edna od najprimenuvanite teoremi - Pitagorovata teorema. Vo vtorata tema - Linearna
ravenka, linearna neravenka i linearna funkcija }e gi pro{irime i zaokru`ime znaewata
za tie osnovni i najva`ni poimi vo celata algebra. Vo narednata, treta tema }e se zapoznaete
i }e re{avate Sistemi linearni ravenki. Vo ~etvrtata tema - Geometriski tela, }e se
zapoznaete so zaemnite odnosi na to~kite, pravite i ramninite vo prostorot. ]e
presmetuvate plo{tini i volumeni na geometriskite tela: prizma, piramida, cilindar,
konus i topka. Ovaa tema e od posebno zna~ewe za prostornata pretstava na svetot {to
n opkru`uva.
Sekoja nastavna edinka vo u~ebnikot e pridru`ena so nekolku re{eni primeri i kon
nea se dadeni dovolen broj zada~i za ve`bi.
Zada~ite za ve`bi se podeleni vo tri kategorii:

a) Zada~i so sino kruk~e . Toa se zada~i so koi se vr{i uve`buvawe na soodvetni-


te poimi i barawa vo nastavnata edinka.

b) Zada~i so violetovo triagolni~e . Toa se poslo`eni i pote{ki zada~i, ~ie


re{avawe e prosledeno so soodvetno upatstvo.
v) Zada~i zatvoreni so crveno kvadrat~e . Toa se zada~i, {to mo`at da se koristat
za podgotovka na narednata lekcija.
Na krajot od sekoja tema dadeni se zada~i za povtoruvawe i utvrduvawe, kako i zada~i
za samokontrola. Sekoja od zada~ite za samokontrola nosi po 6 bodovi, koi mo`at da
bidat podeleni na ednakvi delovi, ako zada~ata e podelena na podzada~i (pod a, b, v,...), a
skalata za samoocenuvawe e slednata: 0-22 (nedovolen), 23-35 ocenka 2, 36-50 ocenka 3, 51-
62 ocenka 4 i 63-72 ocenka 5.
Na krajot na u~ebnikot dadeni se odgovorite na site zada~i, upatstva za re{avawe,
kako i celosni re{enija i dokazi na poslo`enite zada~i. Vi prepora~uvame da gi
razgledate i re{enijata na pote{kite zada~i. Toa }e ve zainteresira i kaj vas da se
pojavi pogolem i s pogolem interes kon matematikata.
Avtorite
4
A PROPORCIONALNI OTSE^KI

I. 1. RAZMER
Na geografskite karti sekoga{ e napi{ano vo koj razmer e taa karta.
[to prakti~no zna~i toa? Ako razmerot e na primer 1 : 25 00 000, toa zna~i deka ako
rastojanieto me|u dve to~ki na kartata e 1 cm, vo prirodata toa rastojanie e
2 500 000 cm = 25 km, a ako na kartata rastojanieto e 2 cm, vo prirodata toa rastojanie e
2 . 25 km = 50 km. Zna~i razmerot na kartata poka`uva kolku pati rastojanijata na kartata
se namaleni vo odnos na vistinskite rastojanija vo prirodata. Od druga strana, pak, vo
u~ebnicite po biologija mo`e da vidite sliki na razli~ni kletki vo zgolemena forma
koi bez mikoroskop se nevidlivi. Ako slikata e zgolemena na primer 20 000 pati, mo`eme
da ka`eme deka crte`ot e pretstaven vo razmer 20.000 : 1.
Vsu{nost, i vo dvata prethodni primera se raboti za odnos na dva broja. Za dadeni
D
dva broja a i b, koli~nikot a : b, odnosno se narekuva razmer (ili odnos) na brojot a
E
sprema b. Brojot a se vika prv ~len, a brojot b vtor ~len. Samiot koli~nik N D  E kako
realen broj se vika vrednost na razmerot. Na primer, vrednosta na razmerot 7 : 4 e 1,75.
Neka e daden razmerot a : b. Toga{ za razmerot b : a velime deka e obraten razmer na
razmerot a : b. Na primer, razmerot 5 : 3 e obraten razmer na razmerot 3 : 5.
Za dva razmera a : b i c : d velime deka se ednakvi, ako nivnite vrednosti se ednakvi,
D F
t.e. ako , odnosno ad = bc. Taka, na primer, razmerite 6 : 9 i 14 : 21 se ednakvi
E G
bidej}i 6 . 21 = 9 . 14 (=126). Zabele`uvame deka ako prviot i vtoriot ~len na razmerot
se pomno`at ili podelat so ist broj, vrednosta na razmerot ostanuva nepromeneta. Taka,
 ˜   ˜   
i , pa ottuka .
 ˜   ˜    
^estopati dva ili pove}e razmera gi zapi{uvame zaedno. Na primer, razmerite a : b
i b : c mo`eme da gi zapi{eme kako a : b : c. Taka, na primer, ako a : b : c = 3 : 4 : 5, toa
ozna~uva deka ako a = 3x za nekoe x, toga{ b = 4x a c = 5x. Navistina, a : b = (3x) : (4x)=3: 4 i
b : c = (4x) : (5x)=4: 5.
5
Razmerite od oblik a : b : c ili poop{to a1 : a2 : ... :an se narekuvaat prodol`eni
razmeri. Dodeka na obi~nite razmeri, na primer a : b, nie mu pridru`uvame realen broj
D
, kaj prodol`enite razmeri toa ne e taka. Na prodol`eniot razmer a : b : c mu
E D E
pridru`uvame dva realni broja i no ne eden realen broj.
E F
Na primer nema smisla ako zapi{eme a : b : c =5.

Zada~a. Razmerite a) a : b = 3 : 5 i b : c = 5 : 6, b) a : b = 3 : 5 i b : c = 15 : 17, da gi zapi{eme


kako prodol`en razmer.
Re{enie. a) Prodol`eniot razmer o~igledno }e bide a : b : c = 3 : 5 : 6.
b) Zabele`uvame deka vtoriot ~len (5) na prviot razmer e razli~en od prviot ~len
(15) na vtoriot razmer. Od a : b = 3 : 5 dobivame deka ako a = 3x toga{ b = 5x. Brojot x go
opredeluvame taka {to 5x = 15, od kade x = 3. Zatoa prviot razmer go zapi{uvame kako
a : b = 9 : 15, a za prodol`eniot razmer dobivame a : b : c = 9 : 15 : 17.
Prodol`eniot razmer ne se menuva ako broevite vo razmerot gi pomno`ime ili
podelime so ist broj. Na primer, a : b : c = 5 : 7 : 10 e isto {to i a : b : c = 10 : 14 : 20 ili, pak,
a : b : c = 15 : 21 : 30.
Ravenstvoto me|u dva razmera a : b i c : d se narekuva proporcija. Na primer proporcija
e ravenstvoto 5:7=15:21.
Sekoja proporcija e sostavena od ~etiri ~lena. Ako tri ~lena ni se poznati, ~etvrtiot
go opredeluvame ednozna~no.
DG
Imeno, ako a : b=x:d, toga{, E[ DG , pa [ .
E
Ovde se pretpostavuva deka ~lenovite na razmerot se razli~ni od nula. Neka e dadena
proporcijata a:b=c:d. So pomo{ na broevite a, b, c i d mo`e da se sostavat u{te 7 proporcii.
Na primer, od proporcijata 2:3=6:9 mo`at da se formiraat i slednite proporcii: 3:2=9:6,
6:9=2:3, 9:6=3:2, 2:6=3:9, 6:2=9:3, 3:9=2:6 i 9:3=6:2.
Vo narednata lekcija a i lekciite {to sledat }e razgleduvame razmeri za koi dvata
~lena se dol`ini na nekoi otse~ki.

1. Napi{i dva razmera so razli~ni vrednosti.


 
2. Koja e vrednosta na x, ako vrednosta na razmerot 2:x e: a) 1, b) 2, v) , g) ?
  
3. Daden e razmer ~ija vrednost e . Koja e vrednosta na obratniot na negoviot obraten

razmer? [to zabele`uva{?
     
4. Uprosti gi slednite razmeri: a)  , b)  , v)  .
     
5. Najdi gi x i y taka {to da va`i a)      [  \ , b)    [    \ .
6. Najdi gi broevite x , y i z taka {to [  \  ]    i [  \  ]  .
7. Najdi go x od proporcijata a) 7 : x = 49 : 35, b) 5 : 2 = x : 3.
8. Najdi go x od proporcijata   [ [ 
9. Izberi dva ednakvi razmera a : b i c : d, a potoa formiraj gi
site proporcii so pomo{ na broevite a, b, c i d.

6
I. 2. PROPORCIONALNI OTSE^KI

1. Na crte`ot 1 dadeni se dve otse~ki $% i &' , ~ii dol`ini se $% FP i


&' FP . Da go odredime koli~nikot od dol`inite na tie otse~ki pri merna edinica:
a) 1 cm, b) 1 mm, v) 1 m.
$% FP  
Baraniot koli~nik e soodvetno : a) ,
&' FP  

$% PP   $% P  
b) , v) .
&' PP   &' P   Crte` 1

Zabele`uvame deka koli~nikot od dol`inite na dve otse~ki ne zavisi od izborot na


mernata edinica, so koja se mereni.
$%
Definicija 1. Koli~nikot (ili $%  &' ) od dol`inite na otse~kite
&'
$% i &' , pri ista merna edinica, se vika razmer na tie otse~ki.

$%  FP  
Zna~i, razmerot na dadenite otse~ki $% i &' (crt. 1) e ednakov na .
&'  FP  
1. Dadeni se otse~kite: $%  FP i &'  FP . Odredi go nivniot razmer. Dali }e se
promeni razmerot, ako nivnite dol`ini bidat izrazeni vo decimetri (ili vo
milimetri)?
$%
Spored toa, za koi bilo dve otse~ki $% i &' , va`i N kade {to k e nekoj
&'
pozitiven realen broj.
$%
Namesto N ~esto pi{uvame $% N ˜ &' i velime otse~kata $% (odnosno
&'
dol`inata $% ) e proporcionalna na otse~kata &' (odnosno na dol`inata &' ) so
koeficient na proporcionalnost N .
$% &'  $% &'
Ako N , toga{ . Za razmerite i velime deka se recipro~ni ili
&' $% N &' $%
obratni eden na drug.
2. Neka se dadeni sega dva para otse~ki $% &' i ()  *+
(crt. 2), takvi {to, razmerot na prvite dve da e ednakov so
$% 
razmerot na drugite dve otse~ki, na primer: i
&' 
()  Crte` 2
.
*+ 
$% () $% ()
Bidej}i razmerite i se ednakvi, zatoa }e va`i proporcijata .
&' *+ &' *+

Definicija 2. Ako otse~kite $% &' () i *+ se takvi {to da va`i proporcijata


$% ()
, toga{ velime deka otse~kite $% i () se soodvetno proporcionalni so
&' *+
otse~kite &' i *+ .

7
$% ()
Ako va`i proporcijata (odnosno $%  &' ()  *+ ), toga{ za sekoja od
&' *+
~etirite otse~ki velime deka e ~etvrta geometriska proporcionala na ostanatite tri
otse~ki.
2. Dadeni se tri otse~ki: $%  FP &'  FP i ()  FP . Odredi ja dol`inata
na otse~kata *+ , taka {to da va`i proporcijata $%  &' ()  *+ .
Definicija 3. Ako otse~kite $% 01 i &' se takvi {to da va`i proporcijata
$% 01
, toga{ za otse~kata 01 velime deka e sredna geometriska proporcionala na
01 &'
otse~kite $% i &' , ili geometriska sredina na otse~kite $% i &' .
Na primer, geometriska sredina na otse~kite so dol`ina 2 FP i 18 FP e otse~ka
~ija dol`ina e 6 FP , bidej}i ˜  
Definicijata za proporcionalni otse~ki mo`e da se obop{ti i za proizvolen broj
na otse~ki. Imeno, ako otse~kite $ %  & '  0  1 i $% &' 01 se takvi {to, da va`i
$ % & ' 0  1
proporcijata    N , toga{ velime deka otse~kite $ %  & '  0  1
$% &' 01
se soodvetno proporcionalni na otse~kite $% &' 01 , a brojot N se vika koeficient
na proporcionalnost.

3. Poznato e deka so zapisot $%  FP e zadadena dol`inata (merata) na otse~kata $% .


Tuka brojot  se vika meren broj, a kratenkata FP - merna edinica ili edinica
dol`ina. Ako mernite broevi na dol`inite na dve otse~ki se  i  , dali mo`e da se
tvrdi koja od niv ima pogolema dol`ina?
4. Dadeni se otse~kite: a) $%  FP &'  GP , b) $%  FP &'  GP ,
v) $%  GP &'  FP . Sporedi gi nivnite dol`ini.
5. Dali }e se promeni razmerot na dve otse~ki, ako nivnite dol`ini gi: a) zgolemi{ 3
pati, b) namali{ 2 pati?
6. Od to~kata $ na pravata S naneseni se ednopodrugo pet ednakvi otse~ki:
$% %&  &' '( ()  Odredi gi razmerite:
$%  &(  $&  $)  &)  $' %(  &)  $)  %( .
7. Na crt. 3 zadadeni se otse~kite $% &' ()  *+ i
./ . Vrz osnova na crte`ot odredi gi razmerite:
a) $%  &' , b) $%  () , v) ()  *+ ,
g) ./  *+ , d) *+  () . Crte` 3

8. Vo eden pravoagolen triagolnik eden od aglite iznesuva 30o. Kolkav e razmerot od


pomalata kateta i hipotenuzata na toj triagolnik?
9. Zapi{i deka otse~kite $% i 56 se soodvetno proporcionalni na otse~kite 01 i
34 . Odredi ja dol`inata na otse~kata 56 , ako: $%  FP 01  FP 34  FP .
10.Ispitaj dali se proporcionalni otse~kite {to se dolgi:

8
a)  GP FP FP i  FP , b)  FP FP FP i  FP , v)  FP FP FP i  FP .
11. Otse~kite $% i 01 se soodvetno proporcionalni na otse~kite &'  FP i

34  FP so koeficient na proporcionalnost N . Odredi gi dol`inite na

otse~kite $% i 01 .
12. Nacrtaj otse~ka $% i podeli ja na 4 ednakvi dela.

I. 3. DELEWE NA OTSE^KA NA
EDNAKVI DELOVI

Na crt. 4a daden e triagol-


nik $%& , kaj koj to~kite 0 i
1 ja razdeluvaat stranata $&
na tri ednakvi delovi (otse~ki).
Da si postavime zada~a i
stranata $% da ja razdelime na
tri ednakvi delovi.
Nasetuvame deka, ako niz
Crte` 4
to~kite 0 i 1 povle~eme
pravi S i T - paralelni so stranata %& (crt. 4b), toga{ tie }e ja razdelat i stranata
$% na tri ednakvi delovi.
Ovaa postapka za podelba na dadena otse~ka na ednakvi delovi se zasnovuva na slednata:
Teorema 1. Ako nekolku razli~ni paralelni pravi gi se~at kracite na daden agol i
od edniot negov krak otsekuvaat ednakvi otse~ki, toga{ tie i od drugiot krak na
toj agol otsekuvaat ednakvi otse~ki.
Dokaz. ]e razgledame slu~aj so tri paralelni pravi. Neka paralelnite pravi S T i
U gi se~at kracite na agolot $2% soodvetno vo to~kite $  $  $ i %  %  % i neka
2$ $ $ $ $ (crt. 5). Treba da doka`eme deka 2% % % % % .
Za da go doka`eme toa, niz to~kite $ i $ povlekuvame dve polupravi $0 i
$ 1 - paralelni so krakot 2% . Taka gi
dobivame triagolnicite 2$ % , $ $ 0 i $ $ 1
i paralelogramite $0% % i $ 1% % . Od
vtoriot priznak za skladnost na triagolnici
dobivame deka D 2$ % # D $ $ 0 # D $ $ 1 .
Ottuka sleduva deka 2% $0 $ 1 . No bidej}i
$ 0 % % i $ 1 % % , pa zatoa dobivame deka
2% % % % % .

Zada~a 1. Dadena otse~ka $% da se razdeli


na pet ednakvi delovi.
Re{enie: 1° Od to~kata $ povlekuvame
proizvolna poluprava $0 , {to ne ja sodr`i Crte` 5
otse~kata $% .
9
2°. Na polupravata $0 izbirame proizvolna
to~ka 0  i otse~kata $0  so {estar ja nanesuvame
u{te ~etiri pati ednopodrugo na polupravata $0
(crt. 6). Taka na polupravata $0 dobivame pet
to~ki 0   0   0   0  i 0  takvi {to:
$0  0 0  0  0  0  0  0  0  .
3°. To~kata 0  ja soedinuvame so krajnata
to~ka % na otse~kata $% i niz to~kite
0   0   0  i 0  povlekuvame paralelni pravi so
%0  . Tie pravi }e ja razdelat dadenata otse~-
ka $% na pet ednakvi delovi (crt. 6).

Crte` 6

Zada~a 2. Dadena e otse~ka $% . Da se odredi


to~ka & , koja ja razdeluva otse~kata $% vo
razmer    .
Re{enie: Na otse~kata $% treba da se odredi
to~ka & , takva {to $&  &%    . Za taa cel,
prvo, dadenata otse~ka }e ja razdelime
na     ednakvi delovi (kako vo zada~a 1).
Potoa na otse~kata $% }e ja izdvoime otse~kata
$& , koja sodr`i 4 takvi delovi (crt. 7).
Navistina imame: $&  &%    . Crte` 7

1. Razdeli ja otse~kata $%  FP na tri ednakvi delovi.


2. Dadenata otse~ka $% razdeli ja na: a) 7, b) 11 ednakvi delovi.
3. Dadenata otse~ka $% razdeli ja na dva dela vo odnos   .
4. Dadena e otse~ka so dol`ina 01 D . Konstruiraj gi otse~kite $% i &' , ~ii dol`ini
 
se $% D i &' D.
 
5. Dadena e otse~ka $% so dol`ina S . Konstruiraj ramnostran triagolnik, ~ij peri-
metar e ednakov na S .
6. Dadena e otse~ka 34 so dol`ina S . Konstruiraj triagolnik, ~ij perimetar e edna-
kov na S , a stranite da mu se odnesuvaat kako broevite      .
7. Dadena e otse~ka $% FP . Na nejzinoto prodol`enie (otkaj to~kata % ) odredi to~ka
0 , takva {to $0  %0  .

10
8. Na otse~kata $% odredi to~ka & , takva {to da va`i:
a) $%  %&    , b) $&  $%    .
9. Konstruiraj pravoagolnik, ~ij perimetar e /  , a stranite da mu se odnesuvaat
kako broevite    .
10.Konstruiraj dve kru`nici so centri vo to~kite A i B, takvi {to, tie da se dopiraat
odnadvor, a radiusite da im se odnesuvaat kako 3 : 2.

I. 4. TALESOVA TEOREMA

Nacrtajte proizvolen konveksen agol 021


i ozna~ete so A O i B O po edna to~ka soodvetno
na kracite 20 i 21 na toj agol. Na krakot 20
konstruirajte to~ka $ , taka {to 2$  ˜ 2$ .
Potoa niz to~kata $ povle~ete prava para-
lelna so pravata $% i nejziniot presek so 21
ozna~ite go so % (crt. 8).
Od konstrukcijata na to~kata $ imame
deka 2$  2$   . Ottuka, i od $ % & $% , a vrz
osnova na teoremata vo I.3 zaklu~uvame deka Crte` 8
va`i i 2%  2%  . Spored toa, va`i propor-
cijata 2$  2$ 2%  2% ,t.e. otse~kite 2$ 2$  2% i 2% , {to paralelnite pravi $%
i $ % gi otsekuvaat od kracite na agolot 021 se proporcionalni. Toa ne naveduva
na mislata deka }e va`i i slednoto poop{to tvrdewe:

Teorema 1. Ako kracite na eden agol se prese~at so dve razli~ni paralelni pravi,
toga{ otse~kite {to gi otsekuvaat ovie pravi od kracite na agolot,
se proporcionalni me|u sebe, t.e. ako $ % & $% , toga{ 2$  2$ 2%  2% (crt. 9).

Ovaa teorema e poznata kako Talesova teorema*.


P
Ako koli~nikot 2$  2$ e racionalen broj , kade P i
Q
Q se prirodni broevi, toga{ postoi otse~ka koja P pati se
sodr`i vo otse~kata 2$ , i Q pati se sodr`i vo otse~kata
2$ . Vo toj slu~aj, dokazot na Talesovata teorema sleduva
od teorema 1 (I. 3.). Ako pak koli~nikot 2$  2$ ne e racio-
nalen broj, toga{ dokazot na Talesovata teorema e Crte` 9
pokompliciran.

Posledica 1. Ako $ % & $% , toga{ 2$  $$ 2%  %% (crt. 9).

*Tales od gradot Milet e starogr~ki matemati~ar koj `iveel VII - VI vek pr.n.e. Spored
predanijata, Tales se smeta za tatko na gr~kata matematika. Toj bil trgovec, koj vo VI vek pr.n.e.
patuval vo Vavilon i Egipet.

11
2$ 2%
Dokaz. Neka $ % & $% (crt. 9). Toga{ va`i proporcijata , odnosno
2$ 2%
2$ 2% 2$  2$ 2%  2% $$ %%
ravenstvoto    . Ottuka , ili , odnosno
2$ 2% 2$ 2% 2$ 2%
2$ 2%
$$ %%
, {to treba{e da se doka`e.

Posledica 2. Ako edna prava e paralelna so edna strana na daden triagolnik


i gi se~e drugite dve negovi strani, toga{ taa otsekuva od nego triagolnik,
~ii strani se proporcionalni so stranite na dadeniot triagolnik.

Dokaz. Daden e D $%& i 01 & %& (crt. 10). Treba da doka`eme deka
$0 $1 01
.
$% $& %&
Gledame, agolot %$& e prese~en so paralelni pravi 01 i
%& , pa vo soglasnost so Talesovata teorema imame:
$0 $1
. (1)
$% $&

Crte` 10 Niz to~kata M da povle~eme prava 03 paralelena so


stranata $& na D $%& , koja }e ja prese~e stranata %& vo
to~kata 3 (crt. 10).
Vo toj slu~aj za agolot $%& i 03 & $& , vo soglasnost so Talesovata teorema va`i:
$0 3&
.
$% %&
$0 01
No bidej}i 3& 01 , zatoa . (2)
$% %&
$0 $1 01
Od proporciite (1) i (2) dobivame .
$% $& %&
Va`i i obratnata teorema na Talesovata teorema:

Teorema 2. Ako dve razli~ni pravi, od kracite na eden agol, otsekuvaat


proporcionalni otse~ki, toga{ tie pravi se paralelni.

Dokaz*. Neka pravite $% i &' gi se~at kracite na agolot 021 i, pritoa


2$  2& 2%  2' (crt. 11). Treba da doka`eme
deka $% & &' . Niz to~kata & da povle~eme prava {to
e paralelna so prvata $% i neka taa go se~e drugiot
krak vo to~kata ' , t.e. neka &' & $% . Toga{ spored
teoremata 1, imame: 2$  2& 2%  2' .
Ottuka, a vo soglasnost so pretpostavkata:
( 2$  2& 2%  2' ), dobivame 2%  2' 2%  2' .
Spored toa, 2' 2' , t.e. ' { ' . Zna~i: $% & &' .
Crte` 11
*Za onie koi sakaat da gi pro{irat svoite znaewa.

12
Ovaa teorema ni dava u{te eden na~in za utvrduvawe na paralelnosta na dve pravi.
Posledica 3. Ako edna prava presekuva dve strani na triagolnikot i gi
razdeluva niv na proporcionalni otse~ki, toga{ taa prava e paralelna so
tretata strana na triagolnikot.

1. Kracite na agolot 021 se prese~eni so dve paralelni pravi $% i &' , pri {to
to~kite $ i & se od edniot krak, a to~kite % i ' od drugiot krak. Odredi ja dol`i-
nata na otse~kata 2% , ako:
a) 2$  FP 2&  FP 2'  FP , b) 2$  FP $&  FP %'  FP .
2. Kracite na agolot 021 se prese~eni so dve pravi S i T , pri {to na krakot 20 se
dobieni otse~kite 2$ i $% a na krakot 21 - otse~kite 2& i &' . Ispitaj dali
pravite S i T se paralelni ili ne, ako: a) 2$  FP $%  FP 2&  FP i &'  FP ,

Crte` 12 Crte` 13

b) 2$  $%    2&   FP i &'  FP .


3. Na crt. 12 pravite a,b,c i d gi se~at kracite na agolot $ . a) Koi od niv se paralelni i
zo{to?, b) Koi od niv ne se paralelni i zo{to?
4. Na crt. 13, kracite na agolot 2 se prese~eni so ~etiri pravi. Doka`i deka
a) ~etiriagolnikot $%&' e trapez, b) ~etiriagolnikot 3456 ne e trapez.
5. Nacrtaj agol 021 i dve paralelni pravi S i T koi gi se~at kracite. Neka S gi se~e
kracite 20 i 21 vo to~ki % i ' soodvetno. Najdi gi site odnosi me|u otse~kite
2$ 2% 2&  2' $% &' $& i %' koi se ednakvi na a) 2$  2% , b) 2$  $% , v) 2$  2& .

I. 5. PRIMENA NA TALESOVATA TEOREMA

Talesovata teorema nao|a {iroka primena vo geometrijata. Eve nekolku


primeri za toa:
Zada~a 1. Dadeni se tri otse~ki so dol`ini D E i F . Da se konstruira nivnata
~etvrta geometriska proporcionala so dol`ina [ , taka {to D  E F  [ , odnosno
EF
[ .
D

13
Re{enie. Crtame eden proizvolen agol 021 i
od temeto 2 po edniot negov krak gi prenesuvame
otse~kite 2$ D i $% E , a po drugiot krak ja
prenesuvame otse~kata 2& F (crt. 14 a ). Potoa gi
svrzuvame to~kite $ i & , a niz to~kata %
povlekuvame prava paralelna so $& , koja }e go
prese~e krakot $1 vo nekoja to~ka ' . Otse~kata
&' [ e baranata ~etvrta proporcionala na dadenite
otse~ki so dol`ini D E F . Navistina, taa ja
zadovoluva proporcijata D  E F  [ .
Zada~ata mo`eme da ja re{ime i na drug na~in,
kako {to e poka`ano na crte` 14 b). Od temeto 2 na
edniot krak gi prenesuvame otse~kite 2$ D i Crte` 14

2% E , a na drugiot krak ja prenesuvame otse~kata 2& F . To~kite $ i & gi povrzuvame,


a niz % povlekuvame prava paralelna so $& . Ako ' e prese~nata to~ka na taa prava so
krakot 21 , toga{ 2' [ e baranata ~etvrta proporcionala. Navistina, spored
Talesovata teorema va`i proporcijata D  E F  [ .
Zada~a 2. Dadena otse~ka $% da se razdeli na delovi, {to se proporcionalni na
otse~kite P Q i S .
Re{enie. Od to~kata $ povlekuvame poluprava $0 , koja
so otse~kata $% zafa}a nekoj agol. Potoa, na polupravata
$0 gi prenesuvame ednopodrugo dadenite otse~ki P Q i S
(crt. 15). Pritoa na polupravata $0 gi dobivame to~kite
$  $ i $ . To~kata $ ja soedinuvame so % , a niz to~kite $
i $ povlekuvame pravi paralelni so $ % . Tie }e ja prese~at
otse~kata $% vo to~kite & i ' . Vo soglasnost soTalesovata
teorema va`i
Crte` 15
$& &' '%
$&  &'  '% P  Q  S , odnosno .
P Q S
Zada~a 3. Vo trapezot $%&' kracite $' i %& se
prodol`eni do presekot vo to~kata 0 (crt. 16). Poznati
se dol`inite na dvete osnovi: $% FP '& FP i krakot
$' FP . Da se odredi dol`inata na prodol`enieto '0 .

Re{enie. Niz temeto ' povle~i prava '1 - paralelna so krakot


%& , koja }e ja prese~e dolnata osnova $% vo to~kata N. Vo
paralelogramot 1%&' }e bide 1% '& FP , pa zatoa:
Crte` 16
$1 $%  1%     FP 

14
Vrz osnova na Talesovata teorema (kracite na agolot $ se prese~eni so dve paralelni
pravi '1 i %0 ), imame $1  1% $'  '0 , odnosno      '0 , a ottuka, dobivame
deka '0  FP .
Zada~a 4*. Da se doka`e deka: Bisektrisata na sekoj vnatre{en agol na
triagolnikot ja razdeluva sprotivnata strana na delovi (otse~ki), {to se
proporcionalni na stranite koi go obrazuvaat toj agol.
Dokaz. Neka &' e bisektrisa na agolot & na triagolnikot $%& (crt. 17). Treba
da doka`eme deka otse~kite $' i '% se proporcionalni na stranite $& i &% , odnosno
$' '% $' $&
deka va`i proporcijata , odnosno .
$& &% '% &%
Niz temeto % da povle~eme prava - paralelna so
bisektrisata &' , koja }e ja prese~e pravata $& vo to~kata
( (crt. 17). Pritoa se dobiva triagolnik %&( , za koj lesno
se poka`uva deka e ramnokrak (Zo{to?). Zna~i, &( &% .
Vrz osnova na Talesovata teorema (kracite na agolot $
se prese~eni so dve paralelni pravi &' i %( ),
Crte` 17
imame: $'  '% $&  &( , no, bidej}i &( &% , zatoa }e va`i
i $'  '% $&  &% .
*Za onie koi sakaat da gi pro{irat svoite znaewa.

1. Na edniot krak na agolot 021 od temeto 2 naneseni se ednopodrugo otse~kite


2$ $% i %& , koi se odnesuvaat kako broevite     . Na drugiot krak e nanesena
otse~kata 2$ FP a niz to~kite % i & se povle~eni pravi %% i && paralelni
so $$ . Odredi gi dol`inite na otse~kite $ % i %& .
2. Daden e triagolnik $%& . To~kata 0 ja razdeluva stranata $& na dve otse~ki
$0  FP i 0&  FP . Odredi go odnosot na rastojanijata na to~kite 0 i & od
stranata $% na '$%& .
3. Vo trapezot $%&' kracite $' i %&
se prodol`eni do presekot vo to~kata 0 .
Odredi go prodol`enieto '0 , ako:
$' GP %& GP &0  GP .
4. Na crt. 18, na kracite na agolot 021 ozna~eni
se to~ki $ % & i ' . Na krakot 21 odredi gi:
a)to~ka 5 , taka {to 2&  2$ 2'  25 ,
Crte` 18
b) to~ka 6 , taka {to 26  2'    .

15
5. Na crt. 19 e: $% & &' & () & ./ &/  FP (.  FP $&  FP .
Odredi gi dol`inite na otse~kite: %' ')  )/ ako %/   FP .

Crte` 19
Crte` 20

6. Daden e triagolnik $%& , vo koj %' e bisektrisa na agolot % . Odredi gi dol`inite


na otse~kite $' i '& , ako stranite na triagolnikot se dolgi:
$% FP %&   FP i $&  FP .
7. Niz temeto & na paralelogramot $%&' e povle~ena prava S , koja gi se~e
prodol`enijata na stranite $% i $' soodvetno vo to~kite 0 i 1 (crt. 20). Doka`i
deka $%  $0  $'  $1  .
8. Dadena otse~ka $%  FP razdeli ja na dva dela, koi se proporcionalni na otse~kite
D  FP i E  FP .
9. Razdeli ja otse~kata 01  FP na tri delovi, {to se soodvetno proporcionalni na
otse~kite: D  FP E  FP i F  FP .
10.Konstruiraj ja ~etvrtata geometriska proporcionala [ za otse~kite:
a) D  FP E  FP F  FP od proporcijata D  E F  [ .
b) D  FP , E  FP , F FP od proporcijata D  E [  F .
11. Dadeni se tri otse~ki so dol`ini D E F . Konstruiraj otse~ka ~ija dol`ina [ e
DE DF EF
ednakva na: a) [ , b) [ , v) [ .
F E D
12. Dadena e otse~ka so dol`ina D i otse~ka so edine~na dol`ina. Konstruiraj druga
otse~ka ~ija dol`ina e [ D  , koristej}i ja proporcijata [  D D  .

B SLI^NI TRIAGOLNICI

I. 6. SLI^NI FIGURI. SLI^NI TRIAGOLNICI


Na crt. 21, dadeni se dve karti na R. Makedonija vo ist razmer   , a na crt.
22 i edna karta vo drug razmer   .
Figurite (kartite) na koi e pretstavena R. Makedonija vo ist razmer (crt. 21 a, b) se
skladni figuri. Poimot skladnost na figuri vi e poznat od VI oddelenie. Kaj skladnosta
na figurite karakteristi~no e toa {to, za koi bilo dve to~ki ; i < i nivnite sliki
; c i < c va`i ravenstvoto ; c< c ;< .
Skladnite figuri imaat ednakva forma i golemina. Me|utoa, vo praktikata mnogu
~esto sre}avame predmeti i figuri, koi imaat ista forma no razli~ni golemini
(dimenzii). Takvi se, na primer, planovite na ista parcela vo razli~ni razmeri, potoa
snimkite na filmskata lenta i nivnite proekcii na filmskoto platno, fotosnimkite
napraveni od ist negativ vo razli~ni dimenzii (crt. 23), itn.

16
Crte` 22

Crte` 21 a) b) crt. 23

Figurite, koi imaat ista


forma, se vikaat sli~ni
figuri.
Figurite so koi e pretstavena R. Makedonija
na crte` 21a,b i crte` 22, isto taka se sli~ni.
Da vidime {to gi karakterizira sli~nite
figuri.
Ako gi odredime odnosite na rastojanijata me|u slikite na gradovite: a) Skopje i
Bitola, b)Bitola i [tip, i v) [tip i Skopje od kartite na crte` 21 i 22, }e zabele`ime
deka:
6% % > > 6 
 , odnosno 6%  ˜ 6% , % >  ˜ %> i > 6  ˜ >6 .
6% %> >6
Toa ne naveduva na mislata deka sli~nite figuri se soodvetni pri nekoe preslikuvawe, koe go ima
svojstvoto: Za koi bilo dve to~ki ; i < i nivnite sliki ;  i < pri toa preslikuvawe va`i
ravenstvoto ; < N ˜ ;< , kade {to N e nekoj pozitiven realen broj. Toa preslikuvawe se vika
sli~nost, a brojot N se vika koeficient na sli~nosta.
Definicija 1. Figurata ) velime deka e sli~na so figurata ) , ako postoi
preslikuvawe od ) na ) , pri koe za sekoi dve razli~ni to~ki ; i < od ) i nivnite
sliki ;  i < od ) , da va`i ravenstvoto ;  <  N ˜ ; <  kade N e daden
pozitiven realen broj, koj se narekuva koeficient na sli~nost.

1. Za koi dve figuri velime deka se sli~ni? Navedi nekolku primeri na sli~ni figuri.
Ako figurata ) e sli~na so figurata ) , toga{ pi{uvame )  ) , ili koga sakame da
go istakneme i koeficientot na sli~nost.
Od definicijata za sli~ni figuri sleduva deka i skladnite figuri se sli~ni, so koeficient na
sli~nost N  .
17
Relacijata sli~nost na figuri gi ima svojstvata na:
o
1 . Refleksivnost: Sekoja figura e sli~na sama na sebe (so koeficient na sli~nost
1), t.e. .
2o. Simetri~nost: Ako figurata F e sli~na na figurata ) , toga{ i figurata ) e
sli~na na figurata ) , t.e. )  ) Ÿ )  ) .
3o.Tranzitivnost: Ako figurata ) e sli~na na figurata ) i figurata ) e sli~na na
figurata ) , toga{ figurata F e sli~na na figurata F2 t.e.
( )  ) , )  ) ) Ÿ )  ) .
Op{tata definicija za sli~nost na dve figuri se odnesuva i za triagolnici. Me|utoa,
ako dvete figuri se triagolnici, definicijata za sli~nost dobiva poprost vid.
Definicija 2. Dva triagolnika $%& i $ %& velime deka se sli~ni, ako imaat
soodvetno ednakvi agli, a soodvetnite strani im se proporcionalni,
$%
%& $&
t.e. )$ )$  )% )%  )& )& , N.
$ %
%& $&
Vo toj slu~aj pi{uvame D $%& D $ %& , kade {to k pretstavuva koeficient
na sli~nosta na D $%& vo odnos na D $ %&

2. To~no li e deka: a) Sekoi dve skladni figuri se i sli~ni, b) Postojat dve neskladni
figuri koi se sli~ni, v) Sekoi dve sli~ni figuri se i skladni, g) Ne sekoi dve
sli~ni figuri se i skladni?
3. Kako }e go odredi{ koeficientot na sli~nosta na figurata ) vo odnos na figurata
) , ako se poznati dva para soodvetni to~ki, na primer: $ o $ i % o % ?
4. Poznato e deka figurata ) e sli~na na figurata ) so koeficient na sli~nost

N . Kolkav e koeficientot na sli~nost na figurata F2 vo odnos na figurata ) ?

5. Doka`i deka ako figurata ) e sli~na na figurata ) so koeficient na sli~nost

N , toga{ figurata ) e sli~na na figurata ) so koeficient na sli~nost .
N
6. Za dve figuri e poznato deka: i . Za koja vrednost na N toa e mo`no?
7. Dadena e otse~ka 36 . Konstruiraj otse~ka $% , {to e sli~na na otse~kata 36 so
 
koeficient na sli~nost: a) N  , b) N  , v) N  g) N , d) N .
 
8. Kako se promenuvaat (zgolemuvaat ili namaluvaat) rastojanijata me|u to~kite pri
sli~nosta so koeficient N , ako a) N !  , b)   N   ?
9. Dadeni se tri otse~ki $% FP &' FP i () FP . Odredi gi koeficientite
na sli~nost na otse~kite: a) $% i &' , b) &' i EF, v) $% i () .
10. To~ni li se slednite tvrdewa: a)( ) # ) i )  ) ) Ÿ )  ) ,
b) ( )  ) i ) # ) ) Ÿ )  ) , v) ( )  ) i )  ) ) Ÿ )  ) ?
11. Odredete go vozdu{noto rastojanie od Bitola do Skopje koristej}i ja kartata na RM
vo razmer   na crt. 21 b.
12. Nacrtaj dva triagolnika $%& i $ %& so dva para ednakvi agli: )$ )$c i )% )% c .
Proveri dali tie se sli~ni.

18
.I. 7. PRIZNACI ZA SLI^NOST
NA TRIAGOLNICITE

Definicijata za sli~nost na dva triagolnika vo sebevklu~uva sporeduvawe na site


{est osnovni elementi (trite agli i trite strani) na edniot triagolnik so elementite
na drugiot triagolnik. Me|utoa postojat minimum uslovi, koi ako bidat ispolneti,
triagolnicite }e bidat sli~ni. Tie uslovi se vikaat priznaci za sli~nost na
triagolnicite i va`at za koi bilo dva triagolnika.

Prv priznak (AA). Dva triagolnika se sli~ni, ako dva agla od edniot triagolnik
se soodvetno ednakvi na dva agla od drugiot triagolnik.

Dokaz*. Neka se dadeni D $%& i D $ %& (crt. 24) i neka )$ )$ i )% )% . Treba
$% %& $&
da doka`eme: )& )& i , odnosno deka '$%&  '$ %& .
$ % %& $&

Od )$ )$ i )% )% sleduva deka i )& )& . Zo{to?

Neka, na primer, $% ! $ % i $& ! $& . Ako


na stranite $% i $& izbereme po edna to~ka 0
i 1 , taka {to $0 $ % i $1 $& , toga{ }e
dobieme triagolnik $01 , koj e skladen so
triagolnikot $ %& (vo soglasnost so priznakot
za skladnost &$& ).
Pravata $% mo`e da se zeme kako Crte` 24
transferzala na pravite 01 i %& , pa bidej}i
soglasnite agli )0 i )% se ednakvi (Zo{to?),
sleduva deka pravite 01 i %& se paralelni,
t.e. 01 & %& (crt. 24).
A spored posledicata 2 na Talesovata teorema,
$% $& %&
od 01 & %& dobivame deka .
$0 $1 01
Zna~i triagolnicite $%& i $01 se sli~ni.

Bidej}i: '$%&  '$01 i '$01 # '$ %& , zatoa '$%&  '$ %& .

1. Dali se sli~ni sekoi dva ramnostrani triagolnika?


Zo{to?

* Za onie koi sakaat da gi pro{irat svoite znaewa.

19
Vtor priznak(SAS). Dva triagolnika se sli~ni, ako dve strani na edniot
triagolnik se soodvetno proporcionalni na dve strani od drugiot triagolnik
$% $& ½
°
Ÿ '$%&  '$ %&
i aglite, {to se zafateni od niv, se ednakvi, t.e. $ % $& ¾ .
)$ )$ °¿
Tret priznak(SSS). Dva triagolnika se sli~ni, ako trite strani na edniot
triagolnik se proporcionalni na trite strani od drugiot, t.e.
$% %& $&
Ÿ '$%&  '$ %& .
$ % %& $&

Zada~a. Da se konstruira triagolnik $ %& {to e sli~en na daden triagolnik



$%& so koeficient na sli~nost N .

Re{enie. Konstrukcijata na baraniot triagolnik $%& e izvedena na crte` 25.

Prvo e konstruirana stranata $ % - soodvetna na $% pri uslov $ % ˜ $% .

Obrazlo`ete kako e izvr{eno toa. Potoa niz to~kata % e povle~ena prava %& -
paralelna na stranata %& , koja ja se~e stranata $& vo to~kata & .
Izvedenata konstrukcija e baranata, bidej}i triagolnicite $%& i $%& imaat:
a) soodvetno ednakvi agli i b) soodvetno
$ % %& $&  .
proporcionalni strani:  
$% %& $& 
Zabele{ka. Ovaa konstrukcija e izvedena za
slu~aj N   . Postapkata ostanuva ista i koga
N !  , samo {to toga{ treba da gi prodol`ime
stranite $% i $& na dadeniot triagolnik $%& .
2. Eden pravoagolen triagolnik ima eden ostar
agol 27o, a drug pravoagolen triagolnik ima
eden ostar agol 63 o . Dali se sli~ni tie Crte` 25
triagolnici? Zo{to?

3. Katetite na eden pravoagolen triagolnik se proporcionalni so katetite na drug pravoagolen


triagolnik. Dali se sli~ni tie triagolnici? Zo{to?
4. Formuliraj gi priznacite za sli~nost na dva pravoagolni triagolnici.
5. Dva ramnokraki triagolnika imaat pri vrvot: a) ednakvi ostri agli, b) ednakvi pravi agli,
v) ednakvi tapi agli. Dali tie triagolnici se sli~ni? Zo{to?
6. Osnovata i krakot na eden ramnokrak triagolnik se proporcionalni so osnovata i krakot na drug
ramnokrak triagolnik. Dali tie triagolnici se sli~ni? Zo{to?
7. Formuliraj gi priznacite za sli~nost na dva ramnokraki triagolnika.

20
8. Ispitaj dali se sli~ni dva triagolnika so strani:
a) FP FP FP i GP FP GP ,
b) GP FP FP i FP FP FP ,
v) FP FP FP i FP FP FP .
9. Stranite na eden triagolnik se dolgi: FPFP i FP ,
a najgolemata strana na nemu sli~en triagolnik e dolga Crte` 26
FP . Odredi gi dol`inite na drugite dve strani na
vtoriot triagolnik.
10.Kolku para sli~ni triagolnici ima na: a) crt. 26 a, kade {to 01 & $% , b)
crt. 26 b, kade {to (' & $% ? Zapi{i.
11.Vo triagolnikot $%& na crt. 27 e povle~ena otse~ka $' , taka {to
(&$' ($%& . Koi triagolnici na toj crte` se sli~ni? Crte` 27
12.Na crt. 28 nacrtan e triagolnik $%& i povle~eni se pravi ./ & $& i
01 & $% . Odredi koi triagolnici na toj crte` se sli~ni.
13.Neka triagolnikot $%& ima strani 3 cm, 4 cm i  FP i '$ %&  '$%&
so koeficient na sli~nost 3. Presmetaj gi perimetrite / i / na dvata
triagolnika i presmetaj go koli~nikot / : / . [to zabele`uva{?

Crte` 28
I. 8. ODNOS NA PERIMETRITE
NA DVA SLI^NI TRIAGOLNIKA
Kakva vrska postoi me|u perimetrite na dva skladni triagolnika?
Perimetrite na dva skladni triagolnika se ednakvi. Zo{to? Toj fakt mo`eme
da go dobieme i kako specijalen slu~aj od vrskata me|u perimetrite na dva sli~ni
triagolnika. Taa vrska ni ja dava slednata:
Teorema 1. Perimetrite na dva sli~ni triagolnika se odnesuvaat kako nivnite
soodvetni strani.
Dokaz. Neka triagolnicite $%& i $ %& so strani D E F i D  E  F se sli~ni. Treba
/ D E F
da doka`eme:     N .
/ D E F
Od '$ %& DABC sleduva deka D E F N , odnosno D N ˜ D E N ˜ E F N ˜ F .
D E F
Ako gi sobereme soodvetnite strani na ovie ravenstva, dobivame:
D  E  F
D  E  F N D  E  F  N , odnosno
DEF D E F
(bidej}i D  E  F /  D  E  F / N   N   N  ),
D E F
/ D E F
N. (1)
/ D E F
Vo specijalen slu~aj, koga dvata triagolnika se skladni, od toa {to N  i od (1)
sleduva deka / / .

21
Zada~a. Najmalite strani na dva sli~ni triagolnika se dolgi: FP i FP , a zbirot
na nivnite perimetri e FP . Da se odredat perimetrite na tie triagolnici.

Re{enie. Ako so / i / gi ozna~ime perimetrite na triagolnicite, toga{ vo


/ 
soglasnost so gornava teorema imame: . A vrz osnova na svojstvata na proporciite,
/ 
/  /  /     ˜ 
od dobivame: ,t.e. . Ottuka: /  FP ,
/  /  /  
a /   /     FP .

Analogna teorema na teorema 1 va`i za soodvetnite visini, te`i{nite linii i


bisektrisi. Ovde pod (dol`ina na) bisektrisa }e ja podrazbirame dol`inata na delot
od bisektrisata {to le`i vo vnatre{nosta na triagolnikot.

Teorema 2. Soodvetnite a) visini, b) te`i{ni linii, v) bisektrisi, na dva


sli~ni triagolnika $%& i $ %& , se proporcionalni na soodvetnite strani.

Dokaz. a) Da gi razgledame visinite &' i & ' spu{teni od temiwata & i &
(crt. 29). Toga{ D $&'  '$& ' , bidej}i i dvata se pravoagolni i imaat ednakov agol

(&$' (& $ ' , bidej}i D $%&  D $ %& . Od ovde dobivame deka

&'  & ' $&  $& .


b) Da gi razgledame te`i{nite linii &' i & ' povle~eni od temiwata & i &
soodvetno (crt. 30). 
˜ $ %
$& $ % $ '  $ % $&
Neka N . No, toga{ i .
$& $% $'  $% $&
˜ $%


Crte` 29 Crte` 31

Osven toa, (&$' (& $ ' , pa od vtoriot priznak za sli~nost dobivame deka
& ' $&
'$&'  '$& ' . Ottuka pak sleduva deka .
&' $&

22
v) Da gi razgledame bisek-
trisite CD i C1D1 povle~eni
od temiwata C i C1 soodvetno
(crt.31).

Toga{ koristej}i deka


($&% ($& % dobivame Crte` 30
 
($&' ($&% ($& % ($& '
 
Osven toa (&$' (& $ ' , pa od prviot priznak za sli~nost dobivame
D $&'  D $& ' . Ottuka sleduva deka &'  &' $&  $& .

1. Stranite ne eden triagolnik se dolgi:  FP FP i  FP . Odredi gi dol`inite na


stranite na nemu sli~en triagolnik, ~ij perimetar e  GP .
2. Perimetrite na dva sli~ni ramnokraki triagolnika iznesuvaat:  GP i  GP .
Prviot od niv ima osnova dolga  FP . Odredi gi dol`inite na stranite na drugiot
triagolnik.
3. Dve soodvetni strani na dva sli~ni triagolnika se dolgi  FP i  FP , a razlikata
na nivnite perimetri e ednakva na  FP . Odredi gi perimetrite na tie triagolnici.
4. Stranite na eden triagolnik se odnesuvaat kako broevite      , a perimetarot na
nemu sli~en triagolnik iznesuva  FP . Odredi gi dol`inite na stranite na
drugiot triagolnik.
5. Stranite na eden triagolnik se dolgi  FP  FP i  FP . Odredi gi dol`inite na
stranite na drug triagolnik, {to e sli~en na dadeniot, ako negoviot perimetar e

ednakov na od perimetarot na dadeniot triagolnik.

6. Neka $%& i $ %& se dva sli~ni triagolnika pri {to nivnite perimetri se
odnesuvaat kako    . Kolkav e odnosot na zbirot od dol`inite na te`i{nite linii
za dvata triagolnika?
7. Neka $%& i $ %& se dva pravoagolni sli~ni triagolnika. Doka`i deka radiusite
na opi{anite kru`nici se odnesuvaat kako soodvetnite strani. Dali ova tvrdewe
va`i i vo slu~aj koga triagolnicite ne se pravoagolni?
8. Koristej}i deka DKD EKE  3 , doka`i deka triagolnikot so strani KE  KD i KD KE
FKF
KF
e sli~en so triagolnikot so strani D E i F .

23
I.9. ODNOS NA PLO[TINITE
NA DVA SLI^NI TRIAGOLNIKA

Vo prethodnata nastavna edinica vidovme deka odnosot na perimetrite na dva sli~ni


triagolnika e ist so odnosot na soodvetnite strani. Vsu{nost ova va`i za dol`inski
elementi vo triagolnikot. Imeno, takvi se, na primer, visinite, te`i{nite linii,
bisektrisite, kako i radiusite na vpi{anata i opi{anata kru`nica. Slednata teorema
poka`uva deka za odnosot na plo{tinite na dva sli~ni triagolnika va`i drugo pravilo.
Toa e i prirodno da se o~ekuva bidej}i edinicata merka za plo{tina ne e ista so edinicata
merka za dol`ina.
Teorema 1. Odnosot na plo{tinite na dva sli~ni triagolnika e ednakov na
kvadratot na odnosot na soodvetnite strani, odnosno na kvadratot na koeficientot
na sli~nost.
Dokaz. Neka '$ %&  '$%& (crt.
32). Neka & ' i &' se soodvetnite
visini spu{teni od temiwata & i &
soodvetno. So P 1 da ja ozna~ime
plo{tinata na triagolnikot $ %& , a
so P da ja ozna~ime plo{tinata na
Crte` 32 triagolnikot $%& .
 
Toga{ 3 ˜ $ % ˜ & ' i 3 ˜ $% ˜ &' . Spored toa:
 

˜ $ % ˜ & '
3  $ % & ' .
˜
3  $% &'
˜ $% ˜ &'

No vo sli~ni triagolnici odnosot na soodvetnite visini e ednakov na odnosot na

3 $ % $ % § $ % ·
soodvetnite strani, t.e. & ' $ %
N . Zatoa,  ˜ ¨ ¸ N  , i so toa
&' $% 3 $% $% © $% ¹
teoremata e doka`ana.

Zada~a. Edno ve{ta~ko ezero ima forma na triagolnik. Plo{tinata na ezeroto na


geografska karta vo razmer  iznesuva  FP  . Kolkava e plo{tinata na
ezeroto?
Re{enie. Ako so 3 ja ozna~ime plo{tinata na ezeroto, toga{ spored prethodnata

FP  §  ·
teorema va`i ¨ ¸ , 3 FP  ˜  ,
3 ©  ¹

3  ˜  FP  ˜  NP


 
 NP  .
Zabele{ka. Ako ezeroto nema forma na triagolnik, kvadrat ili sli~no, toga{
negovata plo{tina mo`eme da ja izmerime na sledniot na~in. Geografskata karta so
ezeroto ja pokrivame so proyirna milimetarska hartija (hratija podelena na kvadrat~iwa
so dimenzija PP [ PP ). Broime kolku mali kvadrat~iwa so strana  PP se pod voda"
"

24
i na toj na~in pribli`no ja nao|ame plo{tinata na ezeroto na geografskata karta.
Za nao|awe na plo{tinata na ezeroto postapuvame kako vo prethodnata zada~a. Taka i
samite mo`ete da ja opredelite plo{tinata na nekoi od na{ite ezera.

1. Plo{tinite na dva triagolnika se odnesuvaat kako    . Ako najmalata strana na


pomaliot triagolnik e dolga 4 cm, najdi ja najmalata strana na pogolemiot triagolnik.
2. Neka perimetrite na dva sli~ni triagolnika se odnesuvaat kako 7 : 3. Presmetaj kako
se odnesuvaat soodvetnite plo{tini.
3
3. Koristej}i ja formulata U za opredeluvawe na radius na vpi{ana kru`nica vo
V
triagolnik, poka`i deka radiusite na vpi{anite kru`nici na dva sli~ni triagolnika
se odnesuvaat kako i soodvetnite strani na tie dva triagolnika. Pritoa mo`e{ da ja
koristi{ teorema 1 od I. 8. i teorema 1 od I. 9.
4. Poznato e deka za radiusot na opi{anata kru`nica 5 okolu triagolnik va`i
DEF
formulata 5 , kade D E i F se stranite, a 3 plo{tinata na triagolnikot.
3
Koristej}i ja ovaa formula, poka`i deka radiusite na opi{anite kru`nici na dva
sli~ni triagolnika se odnesuvaat kako i soodvetnite strani na tie dva triagolnika.
Pritoa mo`e{ da ja koristi{ teorema 1 od I. 8. i teorema 1 od I. 9.
5. Koristej}i ja Heronovata formula 3 V V  D V  E V  F obidi se da najde{ nov
dokaz na teorema 1 za plo{tinite na sli~ni triagolnici.

I. 10. PRIMENA NA SLI^NOSTA NA TRIAGOLNICI

Svojstvata na sli~nite triagolnici nao|aat mnogu {iroka i raznovidna primena, ne


samo vo zada~ite za presmetuvawe, zada~ite za doka`uvawe, konstruktivnite zada~i,
tuku i vo praktikata. Na nekolku primeri }e ja poka`eme nejzinata primena.

Zada~a 1. Da se odredi visinata na edno drvo spored dol`inata na negovata senka


(crt. 33).
Re{enie. Drvoto, ~ija visina $6
sakame da ja odredime, vo eden moment
neka frla senka dolga $% P . Zema-
me letva dolga P i ja postavuvame da
stoi normalno kon zemjinata povr{ina.
Letvata $6 , isto taka, frla senka, koja
vo istiot moment neka iznesuva, na
primer $ % P .
Crte` 33

25
Pravite 6% i 6 % na crte` 33 ni gi pretstavuvaat son~evite zraci {to pominuvaat niz
vrvnite to~ki 6 i 6 na drvoto i letvata.
Bidej}i son~evite zraci se paralelni i vo ist moment tie pa|aat pod ist agol kon
zemjinata povr{ina, zatoa pravoagolnite triagolnici $%6 i $ %6 }e imaat soodvetno
$6 $%
ednakvi agli. Spored toa, tie se sli~ni, pa }e va`i proporcijata , od kade
$6  $ %
$6  
dobivame: , odnosno $6 ˜  . Zna~i, drvoto e visoko P .
    

Zada~a 2. Da se odredi rastojanieto me|u to~kite $ i % , od koi to~kata % e


nedostapna, na primer, taa neka se nao|a na nekoe ostrov~e (crte` 34).
Re{enie. Prvo, od to~kata $ }e ja trasirame i }e
ja izmerime dol`inata na edna proizvolna otse~ka
$& , a potoa so pomo{ na polski aglomer }e gi
izmerime aglite D i J . Na primer, neka pri toa
najdeme deka $& PD q i J q . Koga e toa
gotovo, na list hartija go crtame triagolnikot $ %&
vo razmer  , koj e sli~en so triagolnikot $%&
$% $& Crte` 34
vo prirodata, pa }e imame:  , odnosno
$ % $&
$%  ˜ $ % . Spored toa, ako od crte`ot ja izmerime dol`inata na otse~kata $ % i
nejziniot meren broj go pomno`ime so koeficientot na sli~nosta -  , }e go dobieme
baranoto rastojanie $% .
Ako $ % FP , toga{ $%  ˜ FP P . Zna~i, rastojanieto me|u
to~kite $ i % e P .
Zada~a 3. Da se doka`e deka presekot na dijagonalite na trapezot gi deli istite
na delovi {to se proporcionalni na osnovite na trapezot.
Dokaz. Neka e daden trapez $%&' , ~ii dijagonali se se~at vo
to~kata 2 (crt. 35). Da gi razgledame triagolnicite $%2 i
&'2 . Bidej}i ( (  i (  (  (Zo{to?), zatoa tie se sli~ni.
Soodvetnite strani na triagolnicite $%2 i &'2 , odnosno
stranite {to le`at nasproti ednakvite agli vo niv se:
$% i &' , %2 i 2' , $2 i 2& .
$% %2 $2
Spored toa, od '$%2  '&'2 sleduva deka ,
Crte` 35 &' 2' 2&
{to treba da se doka`e.
Zada~a 4. Da se konstruira triagolnik $%& , ako se
dadeni te`i{nata linija WF i aglite D i E .
Re{enie. Analiza. Baraniot triagolnik $%& e
sli~en so sekoj triagolnik so dadenite agli D i E .
Zatoa celishodno e prvo da konstruirame nekoj sli~en
triagolnik $ %& na baraniot so proizvolna osnova
$ % i dadenite agli na nea (crt. 36). Crte` 36

26
Prvo ja konstruirame te`i{nata linija &' na toj triagolnik. Gledame, taa ne e
skladna so dadenata te`i{na linija WF . No, ako na polupravata &' , ja naneseme otse~kata
&' WF i niz to~kata ' povle~eme prava $% paralelna so $ % , }e go dobieme baraniot
triagolnik $%& .
Konstrukcijata proizleguva od napravenata analiza na zada~ata.
Dokaz. Triagolnikot $%& gi sodr`i site dadeni elementi po golemina i polo`ba,
pa spored toa toj e re{enie na zada~ata.
Diskusija. Zada~ata ima samo edno re{enie ako D  E  q .
Pri re{avaweto na ovaa zada~a se poslu`ivme so pomo{niot triagolnik $ %& , koj
e sli~en so baraniot triagolnik $%& . Zatoa ovoj metod pri re{avawe na
konstruktivnite zada~i se vika metod na sli~ni figuri.

1. Odredi go rastojanieto me|u to~kite $ i % {to se nao|aat na sprotivnite bregovi


na edna reka (crt. 37), ako e %' & $(  %& P &' P '( P .
2. Kratkiot krak na edna rampa e dolg P , a dolgiot
krak P . Kolku visoko }e se izdigne krajot na
dolgiot krak, ako krajot na kratkiot krak se spu{ti
za 0,5 m.
3. Na geografskata karta rastojanijata me|u tri
to~ki se: FP FP i FP . Najgolemoto od tie
rastojanija vo prirodata e NP . Odredi gi drugite
Crte` 37
dve rastojanija vo prirodata i razmerot na kartata.
4. Vo trapezot $%&' $% & &' , 2 e presek na dijagonalite. Odredi gi dol`inite na
otse~kite %2 i 2' , ako $2 FP 2& FP , i %' FP .
5. Osnovite na trapezot se dolgi FP i FP . Odredi ja visinata na trapezot, ako
prodol`enijata na kracite se se~at na rastojanie FP od gornata osnova.
6. Doka`i deka trite sredni linii na eden triagolnik obrazuvaat triagolnik sli~en
so dadeniot.
7. Doka`i: Ako dve tetivi na kru`nicata se se~at, toga{ proizvodot od dol`inite na
delovite na ednata tetiva e ednakov so prozivodot od dol`inite na delovite na drugata
tetiva.
8. Daden e ramnokrak triagolnik $%& so osnova D FP i soodvetna visina K FP .
Konstruiraj sli~en triagolnik $ %& so soodvetna visina K FP .
9. Konstruiraj ramnostran triagolnik so primena na sli~nosta, ako e dadena negovata
visina.
10.Da se konstruira triagolnik sli~en so daden triagolnik $%& , ako e dadena edna
negova: a) visina, b) te`i{na linija.

27
11.Niz to~kata S {to le`i nadvor od kru`nicata N minuvaat pravi a i E , koi ja se~at
kru`nicata N soodveto vo to~kite $ % i &  ' . Doka`i deka va`i ravenstvoto
6$ ˜ 6% 6& ˜ 6' .
12. Vo pravoagolniot triagolnik $%& (& q povle~ena e visinata &' od temeto
na praviot agol kon hipotenuzata. Kolku i koi parovi sli~ni triagolnici se
obrazuvani na crte`ot?

V PITAGOROVA TEOREMA
I. 11. SLI^NOST VO PRAVOAGOLEN TRIAGOLNIK
(EVKLIDOVI TEOREMI)
Neka $%& e pravoagolen triagolnik so prav agol vo temeto & so kateti
$& E %& D i hipotenuza $% F (crt. 38).
Da ja povle~eme visinata &' K na triagolnikot
od temeto na praviot agol kon hipotenuzata. Pri toa
otse~kite $' EF i %' DF pretstavuvaat proekcii
na katetite E i D vrz hipotenuzata F . Visinata &'
go razdeluva dadeniot pravoagolen triagolnik $%&
na drugi dva pravoagolni triagolnika $&' i %&' ,
od koi sekoj e sli~en so dadeniot t.e.
D $&'  D $%& (agolot D im e zaedni~ki), Crte` 38

D %&'  D %$& (agolot E im e zaedni~ki).


D D E E
Od nivnata sli~nost sleduvaat proporciite: F i F , odnosno ravenstvata:
 D F E F
E  EF ˜ F i D DF ˜ F . (1)
Spored toa va`i slednata teorema.

Teorema 1. Sekoja kateta vo pravoagolniot triagolnik e geometriska sredina me|u


nejzinata proekcija vrz hipotenuzata i samata hipotenuza.

Od D $&'  D $%& i D %&'  D BAC sleduva deka i D $&'  D &%' .


Navistina, (&$' ('&% - kako agli so zaemno normalni kraci (crt. 38).
Od sli~nosta na triagolnicite $&' i &%'
$' &' 
sleduva deka: t.e. &' $' ˜ '% odnosno
&' '%
K  DF ˜ EF . (2)
Spored toa, va`i slednava:

Crte` 39

28
Teorema 2. Visinata K vo pravoagolniot triagolnik, {to e povle~ena kon
h hipotenuzata, e geometriska sredina me|u proekciite DF i EF od katetite vrz
hipotenuzata.

Od nea sleduva slednava va`na:

Posledica. Normalata, {to e povle~ena od koja bilo to~ka na kru`nicata kon


eden nejzin dijametar, e geometriska sredina me|u otse~kite na koi taa go razdeluva
dijametarot.

Dokaz. Neka e dadena kru`nica


N  &  N  $% - dijametar i &' A $%
(crt. 39). Ako to~kata & ja soedinime
so krajnite to~ki na dijametarot $% ,
}e go dobieme triagolnikot $%& vo koj
(& q (Zo{to?).
Vo toj triagolnik normalata &' e
visina, povle~ena kon hipotenuzata
$% , pa spored teoremata }e bide

Crte` 40 &' $' ˜ '% .

Zada~a. Da se konstruira otse~ka so dol`ina [ koja e geometriska sredina na dve


otse~ki so dol`ini a i b.

Na polupravata $0 so po~etok vo $ ja nanesuvame otse~kata $' D , a potoa so


po~etok vo ' ja nanesuvame i otse~kata '% E (crt. 40). Konstruirame polukru`nica
so dijametar D  E nad $% , a od to~kata ' izdignuvame normala na otse~kata $% . Neka
& e prese~nata to~ka na polukru`nicata so normalata. Toga{, otse~kata &' }e ja ima
baranata dol`ina [ DE , vrz osnova na gornata posledica.

Pri re{avaweto na slednive zada~i, elementite na pravoagolniot triagolnik }e gi


ozna~uvame: so D i E katetite, so F - hipotenuzata, so K -visinata kon hipotenuzata,
a so DF i EF - proekciite na katetite D i E vrz hipotenuzata F .

1. Dadeno e: D FP i F FP . Da se odredat: E DF  EF i K .


2. Dadeno e: E FP i EF FP . Da se odredat: D F DF i K .
3. Dadeno e: EF FP i K FP . Da se odredat: D E F i DF .

29
4. Dadeno e: DF FP i EF FP . Da se odredat D E F i K .
5. Odredi gi dol`inite na stranite na pravoagolen triagolnik, ako
F FP i DF FP .
6. Edna od katetite na pravoagolen triagolnik e dolga  FP , a nejzinata proekcija vrz
hipotenuzata e dolga  FP . Odredi go perimetarot na toj triagolnik.
DE D E
7. Doka`ete deka, za sekoj pravoagolen triagolnik va`i: a) K , b) .
F DF EF
8. Da se konstruira pravoagolen triagolnik, ako se dadeni DF i EF .
9. Konstruiraj geometriska sredina na dve otse~ki so dol`ini
a)  FP i  FP , b)  FP i  FP .
10.Nacrtaj proizvolen pravoagolnik, a potoa konstruiraj kvadrat koj }e ima ista
plo{tina kako pravoagolnikot.
DE
11.Konstruiraj otse~ka so dol`ina [ , ako e: a) [ DE , b) [ , v) [ D E  F ,

g) [ D   E  kade D E i F se dadeni dol`ini i D > E .
12.Presmetaj ja hipotenuzata na pravoagolen triagolnik ako negovite kateti se:
a) D FP E FP i b) D FP E FP .

I. 12. PITAGOROVA TEOREMA

Edna od najva`nite i najprimenuvani teoremi vo geometrijata e Pitogorovata


teorema*.

Pitagorova teorema: Plo{tinata na kvadratot nad hipotenuzata kaj sekoj


pravoagolen triagolnik e ednakva na zbirot od plo{tinite na kvadratite nad
katetite.

Dokaz. Spored Evklidovata teorema 1 (crt.


38) znaeme deka
E  EF ˜ F i D  DF ˜ F . 1)
Ako gi sobereme soodvetnite levi i
desni strani na ravenstvoto (1) dobivame
D   E  DF ˜ F  EF ˜ F DF  EF F .
No bidej}i DF  EF F , zatoa va`i
a2 + b2 = c2 (2)
so {to teoremata e doka`ana.
Geometriskata interpretacija na ovoj dokaz
e slednata.

* Pitagora - golem starogr~ki matemati~ar i


Crte` 41 filozof, koj `iveel vo VI vek pr.n.e.

30
Ako &' e visinata spu{tena od temeto & na praviot agol (crt. 41), toga{ ovaa
prava go razdeluva kvadratot $%% $ nad hipotenuzata $% na dva pravoagolnika:
$'' $ i %'' % . Spored Evklidovata teorema imame
3$'' $ EF ˜ F E  3$&& c$c i 3%'' % DF ˜ F D  3%&& % .
     

So sobirawe na ovie ravenstva go dobivame dokazot.


Deka triagolnikot so strani 3, 4 i  e pravoagolen znaele u{te starite Egip}ani.
Na Egip}anite im bil poznat i odnosot na broevite     . Zatoa pravoagolniot
triagolnik so strani 3, 4 i  se vika egipetski triagolnik. Triagolnikot, pak, so strani
5, 12 i  koj, isto taka, e pravoagolen, bil poznat na starite Indijci. Toj triagolnik
go vikame u{te i indiski triagolnik.
Za svojstvoto na egipetskiot i indiskiot triagolnik znaele u{te starite isto~ni
narodi okolu 20 veka pr.n.e. Verojatno e deka Pitagora, koj patuval vo Egipet i Indija,
gi prenel vo Grcija matemati~kite znaewa na tie narodi od toa vreme. Se pretpostavuva
deka dokazot na teoremata e daden od Pitagora vo negovata {kola.
Svojstvoto na egipetskiot i indiskiot triagolnik mo`eme da go koristime i za
trasirawe na prav agol, kako {to toa go pravele i starite Egip}ani. Da zememe edno
ja`e dolgo 12 dm i na nego da go ozna~ime sekoj decimetar so nekoj beleg, na primer, so
vrzan konec! Ako taka podelenoto ja`e go pricvrstime na ramen teren (ili na podot) vo
tretiot i sedmiot podelok od nego, a negovite krai{ta gi soedinime, toga{ agolot
pome|u stranite 3 dm i 4 dm e prav.
Od ravenstvoto (2) sleduva:
a2 = c2 – b2 i b2 = c2 – a2. (3)
Spored toa: Plo{tinata na kvadratot nad ednata kateta vo sekoj pravoagolen
triagolnik e ednakva na razlikata od plo{tinite na kvadratot nad hipotenuzata i
kvadratot nad drugata kateta.

Ako zememe predvid deka od dadenata plo{tina na kvadratot, dol`inata na negovata


strana e ednakva na kvadratniot koren od plo{tinata, toga{ od ravenstvata (2) i (3)
dobivame:
F D   E  D F   E i E F  D . (4)
So pomo{ na formulite (4) lesno mo`e da se odredi dol`inata na koja i da e strana
na pravoagolniot triagolnik, koga se dadeni dol`inite na drugite dve negovi strani.
Za Pitagorovata teorema va`i i nejzinata

Obratna teorema: Ako kvadratot nad najgolemata strana na eden triagolnik e


ednakov na zbirot od kvadartite nad drugite dve strani, toga{ toj triagolnik e
pravoagolen.

Dokaz*: Neka za triagolnikot $%& va`i


  
ravenstvoto $% $&  %& (crt. 42). Da
konstruirame pravoagolen triagolnik
$ %& )& q so kateti $& $& i
%& %& .
* Za onie koi sakaat da gi pro{irat svoite znaewa. Crte` 42

31
Vrz osnova na Pitagorovata teorema za triagolnikot $%& imame:
     
$ % $&  %& $&  %& $% t.e. $ % $% .
Spored toa, konstruiraniot pravoagolen triagolnik $ %& e skladen na dadeniot
triagolnik $%& (vo soglasnost so priznakot za skladnost &&& ), a ottuka sleduva deka
i triagolnikot $%& e pravoagolen. So toa teoremata e doka`ana.

1. Koristej}i se so crt. 43 doka`i ja Pitagorovata teorema vo op{t slu~aj!


2. Razgledaj go crt. 44, nacrtaj, ist takov, izre`i gi ozna~enite delovi od dvata kvadrata
i od niv probaj da sostavi{ eden nov kvadrat so strana ednakva na hipotenuzata na
pravoagolniot triagolnik.

Crte` 43 Crte` 44

3. Kaj Indijcite najden e crt. 45. [to poka`uva toj crte`? Dali toj e vo vrska so
Pitagorovata teorema?
4. Dali mo`at dol`inite na stranite na
pravoagolniot triagolnik da gi imaat
slednite merni broevi, izrazeni so isti
dol`inski edinici: a) 15 , 36, 39,
b) 7, 12, 15, v) 13, 20, 25, g) 9, 12, 15?
5. Odredi ja hipotenuzata (s) na pra-
voagolen triagolnik, ako se poznati
negovite kateti:
Crte` 45
a) a=5 cm, b=12 cm, b) a=7 cm, b=9 cm,
v) a=8 cm, b=15cm, g) a=15,8 m, b=24,2 m.
6. Presmetaj ja ednata kateta, koga e poznata drugata kateta i hiptenuzata na
pravoagolniot triagolnik: a) a=14 cm, c=18 cm,
b) a=24 m, c=26 m, v) b=3,3 cm, c=6,5 cm, g) b=8,5 m, c=12 m.

32
7. Presmetaj go perimetarot i plo{tinata na pra-
voagolen triagolnik, ako se poznati hipotenuzata
i ednata kateta: a) c=65 cm, a=48 cm, b) c=28 cm, b=20 cm,
v) c=17 m, b=12,3 m.
8. Presmetaj go perimetarot na eden pravoagolen
triagolnik, ako e poznata negovata plo{tina
P=54 cm2 i edna negova kateta 12 cm. Crte` 46

9. Presmetaj go perimetarot i plo{tinata na {rafiraniot del od pravoagolnikot ABCD


(crt.46). Dimenziite se dadeni vo santimetri.
10. Skala dolga 6,5 m e potprena na yid. Do koja visina taa go dopira yidot, ako dolniot
nejzin kraj e na rastojanie od 4 m od yidot?
11. Edna od katetite na pravoagolen triagolnik e dolga 3 cm, a nejzinata proekcija vrz
hipotenuzata e dolga 1,8 cm. Odredi go perimetarot na toj triagolnik.
12. Kateta {to le`i sproti agol od 30o e dolga b cm. Odredi go perimetarot na toj
triagolnik!

I. 13. PRIMENA NA PITAGOROVATA TEOREMA


VO KONSTRUKTIVNITE ZADA^I

So primena na Pitagorovata teorema mo`at da se re{at nekoi konstruktivni zada~i:

Zada~a 1. Da se konstruira kvadrat, ~ija plo{tina e ednakva na zbirot od


plo{tinite na dva dadeni kvadrata.

Re{enie: Dadenite kvadarti


neka se: edniot so strana a , a
drugiot so strana b (crt.47).
Da konstruirame pravoagolen
triagolnik, ~ii kateti se a i b
(strani na dadenite kvadrati).
Hipotenuzata c na konstruiraniot
pravoagolen triagolnik }e ni ja
dade stranata na baraniot kvadrat Crte` 47
(crt. 47). Navistina, vo soglasnost
so Pitagorovata teorema va`i c2= a2+ b2.

Zada~a 2. Da se konstruira kvadrat, ~ija plo{tina e ednakva na razlikata od


plo{tinite na dva dadeni kvadrata.

33
Re{enie: Dadenite kvadrati neka se: edniot so strana d, a drugiot so strana e, pri
{to d>e (crt. 48). Ako konstruirame pravoagolen triagolnik ~ija hipotenuza e ednakva
na d (stranata na pogolemiot kvadrat), a ednata kateta da e ednakva na e (stranata na
pomaliot kvadrat); toga{ drugata kateta f na toj triagolnik }e ni ja dade stranata na
baraniot kvadrat (crt. 48). Navistina spored Pitagorovata teorema: f 2= d2- e2.

Crte` 48 Crte` 49

Zada~a 3. Da se konstruira kvadart, ~ija plo{tina e dvapati pomala od plo{tinata


na daden kvadrat so strana c.

Re{enie: Konstruiraj ramnokrak pravoagolen triagolnik, ~ija hipotenuza da e


ednakva na stranata c na dadeniot kvadart! Kvadartot nad sekoja kateta od konstruiraniot
ramnokrak pravoagolen triagolnik e baraniot kvadrat (crt. 49).
Vo soglasnost so Pitagorovata teorema imame:
F
a2+a2=c2, odnosno 2a2=c2, a ottuka D  .


Zada~a 4. Da se konstruiraat otse~ki so dol`ini [ D   E i \ D   E  , kade


{to a i b se dol`ini na dve dadeni otse~ki (a>b).

Re{enie: Konstrukcijata na otse~kata [ D   E  se sveduva na konstrukcija na


pravoagolen triagolnik SAB so kateti 6$ D i 6% E (crt. 50 a). Toga{ hipotenuzata
na triagolnikot SAB }e bide baranata otse~ka x.

 
Vtorata barana otse~ka \ D E ,
pak, e kateta na pravoagolniot triagolnik
ABS (crt. 50 b), ~ija hipotenuza e dadenata
otse~ka a, a drugata kateta e otse~kata b.

Zada~a 5. Da se konstruira otse~ka


dolga Q , kade {to n=2, 3, 4, 5, 6, 7 , ...
Crte` 50

34
Re{enie: Da konstruirame pravoagolen triagolnik ASB, ~ii kateti se dolgi
$6 $%  (crt.51). Hipotenuzata na triagolnikot ASB }e bide otse~ka dolga  . Ako
otse~kata  ja zememe za edna kateta, a otse~kata 1 - za druga kateta na pravoagolniot
triagolnik BSC, toga{ hipotenuzata na triagolnikot BSC }e ima dol`ina  . Na sli~en
na~in se konstruiraat otse~kite: 6'  6(  6)  , itn; s– dodeka ne ja dobieme
baranata otse~ka Q (crt. 51).

Crte` 51 Crte` 52

Otse~kata Q mo`e polesno da se konstruira i direktno vrz osnova posledicata na


teoremata 2 vo I. 11. Od [ Q Q ˜  gledame deka baranata otse~ka [ Q e geometriska
sredina me|u otse~kite so dol`ini n i 1, ~ija konstrukcija e dadena na crt. 52.

1. Nacrtaj tri kvadrati, a potoa konstruiraj nov kvadrat, ~ija plo{tina }e bide ednakva
na zbirot na plo{tinite na prvite tri.
2. Konstruiraj kvadarat so plo{tina 7 cm2. (Vnimavaj: 7=16-9).
3. Dadeni se dve otse~ki so dol`ini a i b. Konstruiraj otse~ki x i y, taka {to
[ D  E  \ D   E .

4. Dadena e otse~ka dolga a. Konstruiraj otse~ka [ D   .

5. Konstruiraj gi otse~kite: a) [  , b) [  , v) [  .

6. Konstruiraj gi otse~kite: a) [ D   DF , b) [ D   DF , kade {to a, b, c se dadeni


otse~ki.
7. Konstruiraj kvadrat {to ima ista plo{tina kako deltoid, ~ii dijagonali se
d1=6 cm i d2=7 cm.

35
I. 14. PRIMENA NA PITAGOROVATA TEOREMA
NA RAMNINSKI GEOMETRISKI FIGURI

So povlekuvaweto na nekoi otse~ki kaj odredeni geometriski figuri , tie }e mo`at


da se razdelat na takvi delovi me|u koi }e ima i pravoagolni triagolnici. Primenata
na Pitagorovata teorema kaj taka sozdadenite pravoagolni triagolnici vo golem broj
slu~ai go ovozmo`uva presmetuvaweto na nekoi nepoznati elementi kaj geometriskite
figuri.
1. So povlekuvawe na edna dijagonala vo pravoagolnikot istiot se razdeluva na dva
skladni pravoagolni triagolnika. So primena na Pitagorovata teorema na eden od tie
triagolnici, lesno ja presmetuvame dijagonalata, ako ni se poznati stranite na
pravoagolnikot. Ako, pak, ni e poznata dijagonalata i edna od stranite na pravoagolnikot
lesno ja odreduvame drugata strana.
Zada~a 1. Da se presmeta plo{tinata na pravoagolnik, ako se poznati
stranata a = 5 dm i dijagonalata d = 6,5 dm.
Re{enie: Prvo ja odreduvame dol`inata na drugata strana na pravoagolnikot:
E G   D          |  GP .
Plo{tinata na pravoagolnikot e: 3 D ˜ E |  ˜    GP  .
2. Na crt. 53 e nacrtan kvadrat ABCD so strana a i okolu nego
opi{ana e kru`nica. Dijagonalata d go deli kvadratot na dva
skladni ramnokraki pravoagolni triagolnici, a taa e hipotenuza
na sekoj od tie triagolnici. So primena na Pitagorovata teorema
dobivame:
G  D   D  D   t.e. G D  . (1)
Spored toa:
Crte` 53 Dol`inata na dijagonalata na kvadratot e ednakva na
proizvodot od dol`inata na negovata strana i brojot  .

Bidej}i radiusot R na opi{anata kru`nica okolu kvadratot e polovina od negovata


dijagonala d, dobivame
D 
5 . (2)

Da ja izrazime sega stranata a na kvadratot kako funkcija od negovata dijagonala d.
2 2 2 2 2 D G
Od pravoagolniot triagolnik ABC dobivame: a +a =d , 2a =d , . A ottuka nao|ame:

G G ˜ G  G 
D , t.e. D . (3)
   
Zada~a 2. Da se odredi stranata na kvadratot, ako negovata dijagonala e dolga 7cm.

G   ˜  
Re{enie: D |  (cm).
  

36
3. Na crte` 54 e nacrtan romb ABCD i se povle~eni
dvete negovi dijagonali. Poznato ti e deka dijagona-
lite na rombot se prepolovuvaat i se normalni edna
na druga. Spored toa, tie go razdeluvaat rombot na
~etiri skladni pravoagolni triagolnici. Kaj sekoj od
dobienite triagolnici hipotenuza e stranata na
rombot, a kateti se polovinkite od dijagonalite. So
Crte` 54
primena na Pitagorovata teorema na eden od dobie-
nite triagolnici, dobivame:
 
 § G · § G  ·
D ¨ ¸ ¨ ¸ . (4)
© ¹ ©  ¹
Zada~a 3. Da se presmeta plo{tinata na eden dvor, {to ima forma na romb, ako se
poznati stranata a=45 m i edna dijagonala d1=54 m.

Re{enie: Plo{tinata na rombot mo`e da se presmeta na dva na~ina: ili so pomo{


na stranata i visinata ili pak so pomo{ na dvete dijagonali na rombot. Baranata
plo{tina nie }e ja odredime na vtoriot na~in. Od ravenstvoto (4) dobivame:
  
§ G ·  § G ·  §  ·  
¨ ¸ D  ¨ ¸   ¨ ¸       
© ¹ © ¹ © ¹
G
A ottuka   , zna~i: d2=36.2=72(m). Zatoa:

G ˜ G   ˜ 
3  ˜   P  .
 
4. Neka e daden ramnokrak triagolnik ABC (crt. 55) so osnova
$% D i krak $& E . Ako ja spu{time visinata h od vrvot C
kon osnovata a, taa }e go razdeli ramnokrakiot triagolnik na
dva skladni pravoagolni triagolnika. Hipotenuzata na sekoj od
tie triagolnici }e bide krakot b, a kateti se spu{tenata visina
h i polovinata od osnovata na ramnokrakiot triagolnik.
Ako se bara visinata, a se poznati osnovata i krakot, so
primena na Pitagorovata teorema na triagolnikot ADC nao|ame:
 
  §D· §D·
Crte` 55 K E  ¨ ¸ , t.e. K E  ¨ ¸ . (5)
© ¹ © ¹
Zada~a 4. Plo{tinata na eden ramnokrak triagolnik e P=60 cm2. Da se odredi krakot
na toj triagolnik, ako osnovata mu e a=10 cm.
DK
Re{enie: Od formulata za plo{tina na triagolnikot 3 , imame

3  ˜ 
K  ˜   FP .
D 
A krakot go odreduvame od ravenstvoto (5):
 
§D· §  ·
E ¨ ¸ K

¨ ¸  

    , a ottuka E FP .

© ¹ © ¹

37
5. Na crte` 56 nacrtan e ramnostran triagolnik ABC so
strana a. Ako ja povle~eme koja i da bilo visina, taa }e go
razdeli ramnostraniot triagolnik na dva skladni pravoagol-
ni triagolnika. Hipotenuzata na sekoj od tie triagolnici
§D·
}e bide stranata a, a kateti }e bidat: polovinata strana ¨ ¸
© ¹
i visinata h na ramnostraniot triagolnik.
So primena na Pitagorovata teorema na pravoagolniot
triagolnik ADC (ili DBC) mo`e da se presmeta visinata na Crte` 56
ramnostraniot triagolnik:

  §D·  D  D   D  D  D  D 
K D ¨ ¸ D  , a ottuka: K .
© ¹     
D
Spored toa: K ˜ . (6)

Visinata na ramnostraniot triagolnik e ednakva na proizvodot od polovinata
na negovata strana i brojot  .

Formulata (6) za visinata na ramnostraniot triagolnik vo geometrijata se koristi


mnogu ~esto, a so pomo{ na nea se izveduvaat i drugi va`ni formuli, kako {to se:
formulata za presmetuvawe na plo{tinata na ramnostran triagolnik, formulite za
radius na vpi{anata i opi{anata kru`nica okolu ramnostraniot triagolnik i dr.
Taka, za plo{tinata na ramnostraniot triagolnik dobivame
D D D D D
3 ˜K ˜ ˜   , t.e. 3 . (7)
    
Plo{tinata na ramnostran triagolnik e ednakva na proizvodot od ~etvrtinkata
od kvadratot na negovata strana i brojot  .

Na crte` 57 nacrtan e ramnostran triagolnik ABC pa vo


nego i okolu nego e vpi{ana i e opi{ana kru`nica.
Znaete deka visinite na ramnostraniot triagolnik
istovremeno se i negovi te`i{ni linii, a tie se se~at vo
edna to~ka koja ja deli sekoja te`i{na linija na delovi vo
odnos 2:1 smetaj}i od temeto.
&2  5 
Spored toa: , t.e. , a ottuka R=2r.
2'  U 
Bidej}i R+r=h, zatoa 2r+r=h, 3 r= h. Ottuka
 
U K a 5 K. (8)
Crte` 57  
§D ·
Ako vo formulite (8) visinata h ja zamenime so izrazot za nea ¨ ˜  ¸ ,
© ¹
  D D   D D
}e dobieme: U K ˜   i 5 K ˜   , t.e.
       
D D
U i5 . (9)
 
38
Zada~a 5. Stranata na ramnostran triagolnik iznesuva a=12 cm. Da se odredi negovata
visina, plo{tinata i radiusite na opi{anata i vpi{anata kru`nica.

Re{enie: Zamenuvaj}i ja dol`inata na stranata a na ramnostraniot triagolnik vo


formulite (6), (7) i (9), nao|ame:
D 
K ˜   |  ˜  |   FP 
 
D 
3   |  ˜    |  FP  
 
D 
5   |  ˜  |  FP 
 
D 
U   |  ˜   |  FP 
 
6. Konstruirajte pravilen {estagolnik ABCDEF, pa
vo nego vpi{ete i okolu nego opi{ete kru`nica (crt.
58). Ako centarot na pravilniot {estagolnik go
soedinime so sekoe teme, istiot se razdeluva na {est
skladni ramnostrani triagolnici. Spored toa,
plo{tinata na pravilniot {estagolnik }e bide:
D D 
3 ˜   , t.e.
 
D 
3 . (10)

Crte` 58
Od konstrukcijata na pravilniot {estagolnik jasno
e deka radiusot na opi{anata kru`nica e ednakov na stranata na pravilniot {estagolnik:
R=a. (11)
Od crte` 58 gledame deka radiusot na vpi{anata kru`nica e ednakov na visinata
na ramnostraniot triagolnik ABO, t.e.
D
U . (12)

Zada~a 6. Perimetarot na pravilen {estagolnik iznesuva 24 cm. Da se presmeta
negovata plo{tina, radiusot na opi{anata i radiusot na vpi{anata kru`nica.

Re{enie: Stranata na pravilniot {estagolnik, koga e poznat negoviot perimetar,


/ 
}e bide: D  FP .
  D   ˜ 
Baranata plo{tina e: 3   |  ˜  ˜   |  , a radiusite na
 
vpi{anata i opi{anata kru`nica se:
D 
U   |  ˜   | FP i R = 4 cm.
 

39
7. Neka e daden ramnokrak trapez ABCD, so osnovi
$% D i &' E i krak $' %& F (crt. 59). Ako
niz temeto D povle~eme prava paralelna so krakot
BC, ramnokrakiot trapez }e se razdeli na romboid
BCDE i eden ramnokrak triagolnik AED. Taka
dobieniot ramnokrak triagolnik AED }e ima osnova
a-b i visina ednakva na visinata na ramnokrakiot
trapez. Visinata DF go deli ramnokrakiot
triagolnik na dva pravoagolni triagolnika. So
primena na Pitagorovata teorema na eden od tie
Crte` 59 triagolnici mo`eme da ja odredime visinata na
ramnokrakiot trapez:
 
§ DE· §D E·
K F  ¨ ¸ , odnosno K F  ¨ ¸ . (13)
©  ¹ ©  ¹

Zada~a 7. Edna niva ima forma na ramnokrak trapez so osnovi a=120 m i b=70 m i krak
c=65 m. Da se odredi nejzinata plo{tina.

Re{enie: Za presmetuvawe na plo{tinata na nivata potrebno e da ja znaeme visinata


na trapezot, zatoa }e ja odredime prvo nea:
 
§ DE· §    ·
K F  ¨ ¸   ¨ ¸         P 
©  ¹ ©  ¹
Baranata plo{tina na nivata }e bide:
DE    
3 ˜K ˜  ˜   ˜   P  .
  
8. Neka e dadena kru`nica k (O, r) i edna nejzina tetiva
$% W (crt.60). Ako centarot O go soedinime so krajnite
to~ki na tetivata AB go dobivame ramnokrakiot triagolnik
ABO, ~ija osnova e dadenata tetiva AB, a kraci se radiusite
OA i OB.
Visinata OD, povle~ena od vrvot O kon osnovata AB, go
razdeluva triagolnikot ABO na dva skladni pravoagolni
triagolnika AOD i BOD. Pritoa, dol`inata na povle~enata
visina OD pretstavuva i centralno rastojanie na tetivata
AB od centarot O na kru`nicata, pa zatoa obi~no ja
ozna~uvame so d, t.e. 2' G .
Crte` 60
So primena na Pitagorovata teorema dobivame:
 
§W· §W·
G 
U  ¨ ¸ , odnosno G

U ¨ ¸ .


© ¹ © ¹
Ako, pak, e poznato centralnoto rastojanie d i radiusot r, toga{ dol`inata na
tetivata, }e bide:

§W·  U  G  .
¨ ¸ U   G  , odnosno W
© ¹

40
Zada~a 8. Vo kru`nica so radius r=4 cm povle~ena e tetiva dolga t=6,4 cm. Da se odredi
centralnoto rastojanie na taa tetiva od centarot na kru`nicata.
 
§W· §   ·
Re{enie: G 
U ¨ ¸ 
 ¨ ¸         FP .
©¹ ©  ¹

1. Najdi ja dijagonalata na pravoagolnik, ~ii strani se:


a) a=7 cm, b=9 cm, b) a=6,2 cm, b=8,5 cm.
2. Vo oblasta na eden prav agol le`i to~ka M, koja od negovite kraci e oddale~ena 4,8
cm i 1,4 cm. Odredi kolku e oddale~ena to~kata M od temeto na praviot agol.
3. Odredi ja stranata na kvadrat, ~ija dijagonala e 3 cm.
4. Presmetaj ja visinata na ramnokrak triagolnik, ako se poznati negovata osnova a=7
cm i krakot b=5,5 cm.
5. Odredi go radiusot na vpi{anata i opi{anata kru`nica na ramnostran triagolnik
so strana a=22,5 cm.
6. Vo pravilen {estagolnik so strana 4 cm vpi{i kru`nica i presmetaj go nejziniot
radius.
7. Presmetaj go perimetarot na pravoagolen trapez, koga se poznati negovite osnovi
a=5,7 cm, b=1,9 cm i visinata h=2,5 cm.
8. Vo kru`nica so radius 6,5 cm povle~ena e tetiva dolga 5 cm. Odredi go nejzinoto
centralno rastojanie.
9. Centrite na dve kru`nici so radius 3,9 cm se na rastojanie eden od drug 7,2 cm.
Odredi ja nivnata zaedni~ka tetiva.
10. Vo krug so radius r=13 cm povle~eni se dve paralelni tetivi so dol`ini t1=10 cm i t2=24 cm.
Presmetaj go rastojanieto me|u tie tetivi, ako tie se nao|aat:
a) na ista strana, b) na razli~ni strani od centarot.
11. Dve kru`nici so radius 6 cm i 10 cm se dopiraat odnadvor. Presmetaj ja dol`inata
na otse~kata od nivnata zaedni~ka tangenta, {to e zaklu~ena me|u to~kite na
dopirawe.
12. Dadeni se dve kru`nici so radius r, od koi sekoja minuva niz centarot na drugata.
Izrazi ja dol`inata na nivnata zaedni~ka tetiva preku r.

41
ZA POVTORUVAWE I
UTVRDUVAWE - I

1. Pomalata od dve otse~ki se sodr`i vo pogolemata 4 pati i ostanuva ostatok koj, pak,
se sodr`i vo pomalata otse~ka to~no 5 pati. Kolku e dolga pogolemata otse~ka, ako
pomalata e dolga 1cm?
2.To~kata 0 ja deli otse~kata AB vo razmer $0  0%    . Odredi gi dol`inite na
otse~kite $% i 0% , ako $% = 3,6 cm.
3. To~kata T ja deli otse~kata $% vo razmer m : n. Odredi gi razmerite $7  $% i 7%  $% .
4. Dadena e otse~ka AB i nejzinata to~ka C, takva {to $&  &%   . Odredi gi razmerite:
a) $%  %& , b) $&  $% , v) $%  $& .
D
F
5. Doka`i: Ako va`i proporcijata , toga{ va`i i proporcijata D  E F  G .
E
G E G
6. Vo ramnokrak triagolnik ABC so osnova $% FP i krak %& FP , bisektrisata na
agolot A ja se~e stranata BC vo to~kata S. Odredi ja dol`inata na otse~kata BS.
7. Vo trapezot ABCD ( $% & &' ) prodol`enijata na kracite AD i BC se se~at vo to~kata
M. Ako: a) $' FP %& FP i &0  FP odredi go '0 ,
b) $'  '0    i %&  FP , odredi go rastojanieto &0 .
8. Za kolku treba da se prodol`i otse~kata $%  FP taka {to da va`i razmerot
$%  %0    ?
9. Niz to~kata M {to le`i na stranata AB od triagolnikot ABC e povle~ena otse~ka MN
paralelna na stranata AC. Odredi ja dol`inata na otse~kata BN, ako $0  0%    ,
a %&  FP .
10.Doka`i: Ako figurata F e sli~na so figurata F1 so koeficient k1, a figurata F1 e
sli~na so figurata F2 so koeficient k2 , toga{ figurata F e sli~na na figurata F2
so koeficient k=k1 . k2
11.Za tri figuri F1, F2 i F3 poznato e deka F1 F2 i F2 F3. Odredi gi koeficientite
na slednite sli~nosti: F2 F1, F3 F2, F1 F3 i F3 F1.
12.Agolot pri vrvot na daden ramnokrak triagolnik ima 36o. Doka`i deka, bisektrisata
na eden agol pri osnovata na toj triagolnik otsekuva od nego triagolnik {to e sli~en
so dadeniot.
13.Stranite na eden triagolnik se odnesuvaat kako broevite 3 : 5 : 9. Najkusata strana na
triagolnik sli~en so nego e dolga 6 cm. Odredete gi dol`inite na drugite dve strani
na vtoriot triagolnik.
14.Vo triagolnikot ABC vpi{an e romb, ~ij eden agol se sovpa|a so agolot A na
triagolnikot ABC. Odredi ja dol`inata na stranata na rombot, ako $% =12 cm,
$& =18 cm.

42
15.Kracite na eden trapez se dolgi 15 cm i 10 cm, a golemata osnova 33 cm. Pomalata
dijagonala go razdeluva trapezot na dva sli~ni triagolnika. Odredi ja dol`inata na
pomalata osnova i dijagonalata.
16.Dadena e kru`nica so radius 8 cm i edna to~ka S {to le`i nadvor od nea. Odredi go
rastojanieto na to~kata S od centarot na kru`nicata, ako dol`inata na tangentnata otse~ka,
{to e povle~ena od to~kata S kon kru`nicata iznesuva 15 cm.
17.Vo kru`nica so radius 3,4 cm povle~eni se dve paralelni tetivi dolgi 6 cm i 3,2 cm.
Odredi go rastojanieto me|u tetivite, ako tie se nao|aat: a) na ista strana, b) na
razli~ni strani od centarot.
18.Daden e romb so strana a=5 cm i ostar agol 60o. Odredi gi dol`inite na negovite
dijagonali.
19.Edna ulica e {iroka 10 m. Na rastojanie 4 m od rabot na ulicata kon sredinata na
istata, postavena e skala dolga 12 m. Ako skalata od toa mesto ja potpreme na zgradata
od edna strana na ulicata, a potoa ja potpreme na zgradata od sprotivnata strana na
ulicata, do koja visina dostiga skalata na tie dve zgradi?
20.Eden top ima doseg 5 km, t.e. mo`e da isfrli granata na dale~ina najmnogu od 5 km. Ako topot e
oddale~en 3 km od eden pravoliniski pat, kolkav del od patot e pod udar na topot?
21.Daden e ramnokrak trapez so osnovi a=5,6 cm, b=1,6 cm i krak c=2,5 cm. Odredete ja
visinata i dijagonalata na trapezot.
22.Ramnostran triagolnik, kvadrat i pravilen {estagolnik vpi{ani se vo kru`nica so
daden radius r. Izrazete gi nivnite strani preku r i doka`ete deka va`i:
  
D D  D .
23.Dadeni se dve otse~ki so dol`ini a cm i b cm i edini~na otse~ka. Konstruiraj otse~ka
D D
so dol`ina x cm, taka {to: a) [ , b) x = ab, v) [ .
E E

ZA SAMOKONTROLA - I

1. To~no li e deka: ako va`i proporcijata a : b = c : d, toga{ va`i i proporcijata:


a) a : c = b : d, b) b : a = d : c ?
2. Dva pravoagolni triagolnika imaat po eden ednakov ostar agol. Dali tie se sli~ni?
Zo{to?
3. Stranite na eden triagolnik se dolgi: 9 cm, 15 cm, i 18 cm. Odredi gi stranite na drug
triagolnik, {to e sli~en so dadeniot, ako najdolgata negova strana e ednakva na
najkusata strana na dadeniot triagolnik.

43
4. Konstruiraj triagolnik koj }e bide sli~en so daden triagolnik ABC so koeficient na

sli~nost .

5. Konstruiraj triagolnik ABC, ako se dadeni aglite a i b i visinata hc .
6. Doka`i deka soodvetnite: a) bisektrisi, b) te`i{ni linii na dva sli~ni triagolnika
se proporcionalni so soodvetnite strani.
7. Nacrtaj proizvolen triagolnik, a potoa konstruiraj kvadrat koj }e ima ista plo{tina
kako triagolnikot.
8. Vo edna kru`nica e vpi{an pravoagolen triagolnik so kateti dolgi 2,4 cm i 7 cm.
Odredi go radiusot na kru`nicata.
9. Od edno pristani{te istovremeno isplovile dva broda i toa edniot vo nasoka na
istok so brzina od 30 km na ~as, a drugiot vo nasoka na sever so brzina 22,5 km na ~as.
Kolku kilometri tie }e bidat oddale~eni eden od drug posle tri ~asa od trgnuvaweto?
10.Vo kvadrat so strana 9 cm edno teme e svrzano so sredinata na edna strana na koja ne
le`i toa teme. Presmetaj ja dol`inata na povle~enata otse~ka. Napravi crte`.
11.Presmetaj go rastojanieto od koordinatniot po~etok od to~kata so koordinati:
a) A (5, 12), b) B (6, 8), v) C (3,–4).
12.Presmetaj gi perimetarot, radiusot na vpi{anata i radiusot na opi{anata kru`nica
na ramnostran triagolnik ~ija visina e 18 cm.

44
A LINEARNI RAVENKI
II. 1. RAVENSTVO. IDENTITET. RAVENKA

Vo prethodnite oddelenija re{avavme ravenki. Sega }e go dopolnime toa znaewe {to


go imame za ravenkite. Za da go definirame poimot za ravenka, prethodno }e go
definirame poop{tiot poim za ravenstvo.
Definicija 1. Dva broja ili dva izraza svrzani so znakot " = " (ednakvi)
obrazuvaat ravenstvo.
Ravenstva se, na primer:
12 = 12, (1)
7 = 15, (1')
4 · (7 – 2) = 8 + 2 · 6, (1'')
5x – 2x = 3x, (2)
4(y + 3) = 4y + 12, (2')
x2 = 3x, (3)
3x = x + 10. (3')
Kaj sekoe ravenstvo razlikuvame dve strani, leva i desna strana. Toa, vsu{nost se,
izrazite {to se nao|aat na levo i na desno od znakot " = " za ravenstvo.
Ravenstvata, kaj koi levata i desnata strana se broevi ili brojni izrazi, se vikaat
brojni ili numeri~ki ravenstva.

Takvi se, na primer ravenstvata (1), (1') i (1").


Brojnite ravenstva vsu{nost, se iskazi zapi{ani so matemati~ki simboli, pa spored
toa tie mo`e da bidat ili to~ni (vistiniti) ili neto~ni (nevistiniti).
Na primer, ravenstvata (1) i (1") se to~ni, a ravenstvoto 7 = 15 e neto~no ravenstvo.

45
Ako dvete strani (ili barem ednata) na ravenstvoto se izrazi so promenlivi, toa se
vika algebarsko ili funkcionalno ravenstvo.
Na primer, takvi se ravenstvata (2), (2'), (3), (3').
Algebarskite ravenstva mo`at da bidat od dva vida, spored toa dali tie preminuvaat
vo to~ni brojni ravenstva za koi bilo vrednosti na promenlivite, ili ne.
Definicija 2. Algebarsko ravenstvo, koe za koi bilo vrednosti na promenlivite
preminuva vo to~no brojno ravenstvo, se vika identitet.

Na primer ravenstvata (2) i (2') se identiteti. Tie se to~ni (vistiniti) za koi bilo
vrednosti na x (odnosno u).
Da navedeme nekoi pova`ni identiteti:
a+0=a a·1=a
a+b=b+a ab = ba
(a + b) + c = a + (b + c) (ab) c = a (bc)
a (b + c) = ab + ac
Tie gi izrazuvaat poznatite svojstva na aritmeti~kite operacii.
To~nite brojni ravenstva, isto taka, se vikaat identiteti. Na primer, ravenstvoto
3 + 4 = 12 - 5 e identitet.

Definicija 3. Sekoe algebarsko ravenstvo {to ne e identitet se vika ravenka.

Na primer, ravenstvata x2 = 3x i 3x = x + 10 za x = 1 preminuvaat vo neto~ni brojni


ravenstva 1 = 3 i 3 = 11. Zna~i, tie ne se identiteti, tuku ravenki.

1. [to e ravenstvo? Kakvi vidovi ravenstva imame?


2. Objasni zo{to slednive ravenstva se identiteti:
a) 2x + 5 = 5 + 2x, b) 3(a + 2) = 3a + 6, v) 5x + 2x = 4x + 3x, g) 0 . x = 0.
3. Doka`i deka slednive ravenstva ne se identiteti:
a) a2 + 1 = 2a, b) 5x - 1 = 4x + 7, v) [ [ .
4. Odredi gi soodvetnite vrednosti na izrazite x2 i 3x za x = 0 i x = 3. Mo`e li da se
tvrdi deka ravenstvoto x2 = 3x e identitet? Doka`i deka toa ne e identitet.
5. Koi od slednite ravenstva se identiteti:
a) x - 2 = 2 - x, b) (x - 2)2 = (2 - x)2, v) D D , g) D D , d) x . 1 = x?
6. Proveri dali brojnite ravenstva se identiteti:
a) 33 + 43 + 53 = 63, b) (1 + 2 + 3 + 4)2 = 13 +23 + 33 + 43.

46
II.2. VIDOVI RAVENKI

Levata ili desnata strana na ravenkite se izrazi so edna, dve ili pove}e promenlivi.
Promenlivite vo ravenkite se vikaat i nepoznati. Tie mo`at da bidat ozna~eni so
koi bilo bukvi, no voobi~aeno e da se ozna~uvaat so poslednite bukvi od latinskata
azbuka x, y, z,...
Spored brojot na nepoznatite, ravenkite mo`at da bidat: ravenki so edna nepoznata,
ravenki so dve nepoznati, ravenki so tri nepoznati, itn.
Na primer, ravenki so edna nepoznata se
 
2x-5=0, |x2-1|=x,   , ...
[ [
ravenki so dve nepoznati se
 
x+2y=5, x2+5y3-7xy=0, [  \  , ...
ravenki so tri nepoznati se
xyz=x+y+z, x2+y2+z2=1, x+yz=5y itn.
Ravenkite so edna nepoznata simboli~ki gi ozna~uvame so
f(x) = g(x), (1)
kade {to f(x) i g(x) se nekoi izrazi od nepoznatata x. Primer na ravenki so edna
nepoznata se ravenkite (3) i (3’) vo II.1.
Znaete deka sekoj od izrazite f(x) i g(x) vo ravenkata (1) ima svoja definiciona
oblast D1 i D2. Tie mo`at da se sovpa|aat ili da ne se sovpa|aat. Zaedni~kiot del
(presekot) od niv se vika definiciona oblast na ravenkata ili oblast na menuvawe na
nepoznatata vo ravenkata i obi~no se ozna~uva so D.
[ 
Na primer, definicionata oblast na ravenkata   e mno`estvo na
[ [

site realni broevi, osven broevite 0 i 2 (za x=0 ne e definiran izrazot , a za x=2 ne e
[ [
definiran izrazot ), t.e. D=R\{0,2}.
[
^estopati levata i desnata strana na edna ravenka se nekoi polinomi, i vo toj slu~aj
mo`eme da zboruvame za stepen na ravenkata. Za stepen na ravenkata se zema pogolemiot
od stepenite na levata i desnata strana na ravenkata.
Na primer, ravenkata xyz=5 ima stepen 3, ravenkata x+x2+2x4=y5 ima stepen 5 itn.
No, prvata ravenka po odnos na sekoja od promenlivite x, y i z ima stepen 1, a vtorata
ravenka po odnos na x ima stepen 4, a po odnos na y ima stepen 5.
Vo ovaa tema }e razgleduvame linearni ravenki, t.e. ravenki so stepen 1, koi imaat
samo edna nepoznata. Takva e na primer ravenkata 2x-5(x+1)=4x+3.

47
1. Kakvi mo`at da bidat ravenkite spored brojot na nepoznatite?
2. [to e definiciona oblast na edna ravenka so edna nepoznata?
3. Napi{i ravenka od tret stepen so dve nepoznati.
4. Napi{i edna ravenka so edna promenliva ~ija definiciona oblast e D=R\{0,1,2}.

II. 3. RE[ENIE NA RAVENKA

Neka e dadeno ravenstvoto: 4x = x + 6. (1)


Za da poka`eme deka ravenstvoto (1) ne e identitet, dovolno e da pronajdeme barem
edna vrednost na x za koja toa preminuva vo neto~no ravenstvo. Gledame, za x = 1 toa
preminuva vo neto~no ravenstvo 4 = 7. Zna~i, ravenstvoto (1) ne e identitet, tuku ravenka
so edna nepoznata.
Da se pra{ame: Postojat li vrednosti na nepoznatata x (ako ima takvi), za koi
ravenkata (1) preminuva vo to~no brojno ravenstvo? Ili so drugi zborovi: Za koi
vrednosti na x soodvetnite vrednosti na izrazite 4x i x + 6 se ednakvi?
Za taa cel za nekoi vrednosti na x ja sostavuvame slednata tablica na soodvetnite
vrednosti na izrazite 4x i x+ 6:

x -3 -2 -1 0 1 2 3 4 5
4x -12 -8 -4 0 4 8 12 16 20
x+ 6 3 4 5 6 7 8 9 10 11

Od tablicava gledame: za x = 2 i dvata izraza 4x i x + 6 dobivaat ednakva vrednost 8.


Spored toa, za x = 2, ravenkata (1) preminuva vo to~no brojno ravenstvo 8 = 8.
Brojot 2 se vika re{enie ili koren na ravenkata (1). U{te velime: brojot 2 ja
zadovoluva ravenkata (1).

Definicija. Re{enie ili koren na edna ravenka so edna nepoznata se vika sekoja
vrednost na nepoznatata za koja ravenkata preminuva vo to~no brojno ravenstvo.

Od gornata tablica najdovme deka brojot 2 e re{enie na ravenkata (1). Me|utoa, od


tablicata ne mo`e da se zaklu~i dali ravenkata (1) nema i drugi re{enija. Zna~i, potpoln
odgovor na toa pra{awe tablicata ne mo`e da ni dade.
Odreduvaweto na site re{enija (koreni) na edna ravenka se vika re{avawe na
ravenkata.
Da se re{i edna ravenka, zna~i da se odredi mno`estvoto na nejzinite re{enija.
Mno`estvoto re{enija na ravenkata mo`e da se sostoi od eden, dva, tri, itn. broevi,

48
a mo`e da bide i prazno mno`estvo ili beskone~no mno`estvo. Da go poka`eme toa na
slednite primeri:
Primer 1. Ravenkata x + 3 = 7 ima edinstveno re{enie 4, bidej}i samo za x = 4 taa
preminuva vo to~no brojno ravenstvo 7 = 7.
Primer 2. Ravenkata (x+3)(x-1)(x-5) = 0 ima tri re{enija:-3, 1 i 5. Toa go zaklu~uvame
ottamu {to: za x = -3 prviot mno`itel (x+3) stanuva ednakov na nula, za x = 1 - vtoriot
mno`itel (x – 1) stanuva ednakov na nula, a za x = 5 tretiot mno`itel (x-5) e ednakov na
nula. A {tom eden od mno`itelite na proizvodot (x+3)(x-1)(x-5) e ednakov na nula, toga{
i toj }e bide nula. No, za sekoja druga vrednost na x nitu eden od trite mno`iteli ne e
nula, pa spored toa i proizvodot ne e nula.
Primer 3. Ravenkata x + 3 = x nema nitu edno re{enie, bidej}i za koja bilo vrednost na
x vrednosta na izrazot x + 3 e za 3 pogolema od samata vrednost na x.
Vo takov slu~aj velime deka mno`estvoto re{enija na ravenkata e prazno mno`estvo.
Primer 4. Ravenkata [ [ ima beskone~no mno`estvo re{enija. Nejzino re{enie e
nulata i sekoj pozitiven realen broj, no ima i broevi koi ne se nejzini re{enija - site
negativni broevi. Spored toa, taa ne e identitet.

1. Proveri koi vrednosti na x od mno`estvoto {-3,-2,0,1,2} se re{enija na ravenkata


x2 = 2- x.
2. Re{i gi slednite ravenki:
[ 
a) 3x - 5 = 7, b)    , v) 3(x - 1)(x - 4) = 0 , g) y3 = y.

3. Sostavi ravenka so edna nepoznata, koja {to ima mno`estvo re{enija:
a) {-2}, b) {-3,-1}, v) {0,1,4}, g) ‡ .
4. Za koi vrednosti na x vrednosta na izrazot 3x -5 e ednakva na: a) 0, b) 4, v) - 2?
Zapi{i go uslovot na zada~ata so ravenka!
5. Ispitaj za koi celi broevi od 1 do 7 se zadovoleni slednite ravenki:
[
a) x(x - 4) = 3x - 10, b) 2x - 3 = x + 1, v) [  ,

[  
g) x(x2 + 11) = 6(x2 + 1), d) [  , |) x2 + 12 = 7x.
 
[   [
6. Proveri dali: a) x = 3 e koren na ravenkata   , b) x = -6 e koren na ravenkata
[   
[     , v) [ e koren na ravenkata 3x - 5 = x - 4, g) x = 0 i x = - 5 se koreni
 
na ravenkata x(x + 7) - 2x = 0.

49
II. 4. EKVIVALENTNI RAVENKI

Da gi razgledame ravenkite:
(x + 2) ( x – 2) = 0, (1)
x(x + 2) ( x – 2) = 0, (2)
x2 = 4. (3)
Ravenkata (1) ima dve re{enija: –2 i 2, a ravenkata (2) ima tri re{enija: 0, –2 i 2.
Gledame: sekoe re{enie na ravenkata (1) e re{enie i na ravenkata (2), no ne sekoe re{enie
na ravenkata (2) e re{enie i na ravenkata ( 1). Na primer, brojot 0 e re{enie na (2) no ne
e re{enie na (1).
Ravenkata (3) ima dve re{enija: –2 i 2, bidej}i samo kvadratite na tie broevi se ednakvi
na brojot 4.
Zabele`uvame: mno`estvata od re{enijata na ravenkite (1) i (3) se sovpa|aat, t.e.
sekoe re{enie na ravenkata (1) e re{enie i na ravenkata (3) i obratno: sekoe re{enie na
ravenkata (3) e re{enie na ravenkata (1). Takvi dve ravenki se vikaat ekvivalentni.
Definicija. Dve ravenki se vikaat ekvivalentni, ako mno`estvata na nivnite
re{enija se sovpa|aat.
I za dve ravenki, koi nemaat re{enija velime deka se ekvivalentni. Na primer,
ravenkite x – 3 = x i 0.x = 3 se ekvivalentni, bidej}i mno`estvoto re{enija i na ednata i na
drugata ravenka e prazno mno`estvo.
Edna od osnovnite zada~i na algebrata e re{avaweto na razli~nite ravenki. Pri
re{avaweto na sekoja ravenka, obi~no istata ja zamenuvame so druga poprosta ravenka,
{to e ekvivalentna na nea. Potoa, taa ravenka ja zamenuvame so treta u{te poprosta,
itn., s– dodeka ne dojdeme do najprostata ravenka, ~ii re{enija se o~igledni.
Koga dadenata ravenka }e ja zamenime so druga poprosta ekvivalentna na nea ravenka,
velime deka dadenata ravenka sme ja transformirale vo poprosta, koja gi ima istite
re{enija kako i dadenata.

1. Za koi dve ravenki velime deka se ekvivalentni?


2. Objasni zo{to ravenkite (x – 1) (x – 2) = 0 i (x – 1) (x – 2) (x – 3) = 0 ne se ekvivalentni.
3. Ispitaj dali se ekvivalentni slednite parovi ravenki:
a) 3x – 5 = 4 i 2x = 6, b) x + 2 = 0 i x(x + 2) = 0,
v) 2x(x – 1) (x – 3) = 0 i 5(x – 1) (x – 3) = 0.
4. Napi{i tri para ekvivalentni ravenki i tri para neekvivalentni ravenki.

50
II. 5. OSNOVNI SVOJSTVA
NA RAVENSTVATA I RAVENKITE
Neka a, b i c se proizvolni racionalni broevi.
Dobro vi se poznati slednive osnovni svojstva na brojnite ravenstva:
1o. Svojstvo na refleksivnost: a=a
2o. Svojstvo na simetri~nost: D EŸE D
o
3 . Svojstvo na tranzitivnost: (D E i E F )Ÿ D F
4o. Svojstvo na monotonost na zbirot: Ako kon dvete strani na edno to~no ravenstvo
a = b dademe eden ist broj c, pak, }e dobieme to~no ravenstvo, t.e.
(D E i F5 )Ÿ D  F E  F .
o
5 . Svojstvo na monotonost na proizvodot: Ako dvete strani na edno to~no ravenstvo
gi pomno`ime so eden ist broj c z 0, pak, }e dobieme to~no ravenstvo: ac = bc, t.e.
D E LF z  Ÿ DF EF .
Vrz osnova na gornite svojstva na brojnite ravenstva lesno gi doka`uvame i slednive
dve va`ni teoremi za transformacija na ravenkite:

Teorema 1. Ako kon dvete strani na edna ravenka


I [ J [ (1)
dodademe eden ist broj k ili ist izraz M [ {to e definiran za sekoja vrednost od h od
definicionata oblast ' na ravenkata (1), }e dobieme nova ravenka:
I [  N J [  N , (2)
odnosno, I [  M [ J [  M [ (2')
koja e ekvivalentna na dadenata.

Dokaz*: Treba da doka`eme deka: sekoe re{enie na ravenkata (1) e re{enie i na


ravenkata (2), odnosno (2'); i obratno: sekoe re{enie na ravenkata (2), odnosno (2') e
re{enie i na ravenkata (1).
a) Neka x0 e re{enie na ravenkata (1). Toga{ }e va`i brojnoto ravenstvo
I [ J [ . (3)
Za x = x0 izrazot M [ dobiva to~no opredelena vrednost, t.e M [ e nekoj broj.
Ako brojot M [ go dodademe kon dvete strani na brojnoto ravenstvo (3), vo soglasnost
so svojstvoto 4o, }e va`i i ravenstvoto
I [  M [ J [  M [ . (4)
Zna~i, x0 e re{enie i na ravenkata (2').
b) Neka x1 e re{enie na ravenkata (2'), t.e. neka va`i brojnoto ravenstvo
I [  M [ J [  M [ . (5)
*Za onie koi sakaat da gi pro{irat svoite znaewa.
51
Ako kon dvete strani na ravenstvoto (5) go dodademe brojot M [ , }e go dobieme
ravenstvoto I [ J [ . (6)
Zna~i, x1 e re{enie i na ravenkata (1). So toa doka`avme deka ravenkata (2') e
ekvivalentna na ravenkata (1). Sli~no se doka`uva deka i ravenkata (2) e ekvivalentna
na ravenkata (1).
Od teorema 1 sleduvaat slednite dve posledici:

Posledica 1. Sekoj ~len na edna ravenka mo`e da se prenese od ednata na drugata


nejzina strana, no so sprotiven znak.

Navistina, ako kon dvete strani na ravenkata I [ J [  M [ dodademe M [ ,


ja dobivame ravenkata I [  M [ J [ .
Da go primenime toa svojstvo na ravenkata 8x-2 = 5x + 7.
So prenesuvawe na ~lenot 5x od desnata strana na ravenkata so sprotiven znak, a
~lenot -2 od levata na desnata strana, isto taka so sprotiven znak, }e ja dobieme ravenkata:
8x - 5x =7 +2, koja e ekvivalentna na dadenata.
Posledica 2. Ako vo dvete strani na ravenkata ima identi~ni (ednakvi) ~lenovi,
tie mo`at da se poni{tat (izostavat).

Primer: Neka e dadena ravenkata x2 + 3x = x2 - 6.


Zebele`uvame, vo dvete strani na ravenkata se sre}ava eden ist izraz [  . So dodavawe
kon dvete strani -h2, ja dobivame ravenkata [  , koja e ekvivalentna na dadenata.

Teorema 2. Ako dvete strani na edna ravenka


I [ J [ (1)
gi pomno`ime so eden ist broj N z  , }e dobieme nova ravenka
I [ ˜ N J [ ˜ N (7)
koja e ekvivalentna na dadenata ravenka (1).

Dokaz*: Neka x0 e re{enie na ravenkata (1), t.e. neka e to~no brojnoto ravenstvo
I [ J [ . (8)
Ako dvete strani na ravenstvoto (8) gi pomno`ime so N z  , vo soglasnost so svojstvoto
5o, }e va`i i ravenstvoto:
I [ ˜ N J [ ˜ N . (9)
Zna~i, h0 e re{enie i na ravenkata (7).
Neka sega, x1 e re{enie na ravenkata (7), t.e. neka
I [ ˜ N J [ ˜ N (10)

e to~no brojno ravenstvo. Bidej}i k z 0, zatoa i z .
N
*Za onie koi sakaat da gi pro{irat svoite znaewa.
52

Ako dvete strani na ravenstvoto (10) gi pomno`ime so brojot z  , }e go dobieme
ravenstvoto I [ J [ . N

Zna~i, x1 e re{enie i na ravenkata (1).


So toa teoremata 2 e doka`ana. Vrz osnova na nea vr{ime transformacija na
ravenkite {to sodr`at drobni ~lenovi vo ravenki bez drobni ~lenovi.
[  [   [
Primer. Neka e dadena ravenkata  .
  
Re{enie. Za da se oslobodime od imenitelite vo ravenkata, dvete nejzini strani
gi mno`ime so brojot 12 (najmal zaedni~ki sodr`atel za imenitelite 4, 2 i 3) i ja
dobivame ravenkata
[  [ [
˜  ˜ ˜   ˜
  
(x +1) · 3 – (x – 3) · 6 = 2x ·4 + 5 ·12
koja e ekvivalentna na dadenata.

Posledica 3. Znacite na site ~lenovi na ravenkata mo`at da se promenat vo


sprotivni, ako dvete strani na ravenkata gi pomno`ime so brojot -1.
Na primer, ravenkata –4x + 2 =-7 e ekvivalentna na ravenkata 4x – 2 = 7.

1. Ako kon dvete strani na edno to~no brojno ravenstvo dodademe eden ist broj,
dali sekoga{ se dobiva pak to~no brojno ravenstvo?
2. Ako kon dvete strani na edno neto~no brojno ravenstvo dodademe eden ist broj,
dali sekoga{ se dobiva pak neto~no brojno ravenstvo?
3. Ako dvete strani na edno to~no brojno ravenstvo gi pomno`ime so eden ist broj,
dali sekoga{ se dobiva pak to~no brojno ravenstvo?
4. Ako dvete strani na edno neto~no brojno ravenstvo gi pomno`ime so eden ist
(koj bilo) broj, dali sekoga{ se dobiva pak neto~no brojno ravenstvo? Navedi primer!
5. Ako dvete strani na edno: a) to~no, b) neto~no brojno ravenstvo gi pomno`ime so
brojot 0, kakvo ravenstvo (to~no ili neto~no) se dobiva?
6. Kakva ravenka se dobiva, ako kon dvete strani na edna ravenka dodademe eden ist
broj ili eden ist polinom po odnos na nepoznatata?
7. Mo`at li oddelni ~lenovi na edna ravenka da preminuvaat od edna na druga nejzina
strana i kako se slu~uva toa ? Poka`i go toa kaj ravenkata 7x – 6 = 2x + 9.

53
8. Ravenkata x2 – 3x + 2 = 7 – 2x transformiraj ja vo ekvivalentna ravenka, vo koja:
a) ~lenovite so nepoznatata da se nao|aat na levata strana, a ~lenovite bez nepoznatata
- na desnata strana na ravenkata,
b) site ~lenovi da se nao|aat na levata strana na ravenkata.
9. Kako se osloboduvame od imenitelite na pomalite ~lenovi vo ravenkata? Poka`i go
toa na eden primer!
10.Transformiraj gi ravenkite vo ekvivalentni ravenki bez dropki:
   [  [   [  [  [   [
a)  [   [ , b)  [  , v)  [   , g)  .
         
11.Dali se ekvivalntni ravenkite:
a) x= 3 i x2 = 3x, b) x = 3 i x + x2 = 3 + x2,
v) 3x(x – 2) = 12 i x(x–2) = 4?
12.Re{i gi slednite prosti ravenki:
a) x – 3 = 8, b) 3x = 15, v) 20 – x = 5, g) 21 = 7x, d) 4 – x = 5, |) 10 = x +6.

II. 6. OP[T VID NA LINEARNA RAVENKA


SO EDNA NEPOZNATA

Neka se dadeni ravenkite so edna nepoznata:

[  [
[ , (1)
 
10x – 2(x – 1) = 3(x + 2), (2)
x(x – 2) = (x – 5) (x +1) . (3)
Ako dvete strani na ravenkata (1) gi pomno`ime so brojot 10 (NZS za 2 i 5), }e ja
dobieme ravenkata 10x – 5(x– 1) = 2(x +2) {to e ekvivalentna na dadenata.
Potoa, po osloboduvawe od zagradite, so prenesuvawe na site ~lenovi na ravenkata
od desnata na levata strana i izvr{uvawe na potrebnite uprostuvawa, ja dobivame
ravenkata:
3x + 1 = 0, (1')
koja e ekvivalentna na ravenkata (1).
Na sli~en na~in i ravenkite (2) i (3) mo`eme da gi transformirame soodvetno vo
ravenkite:
5x – 4 = 0 (2')
i 2x + 5 = 0. (3')
Gledame, sekoja od dadenite ravenki (1), (2) i (3) e ekvivalentna na nekoja ravenka od
vidot ax + b = 0, kade {to a i b se to~no opredeleni realni broevi.

54
Definicija. Sekoja ravenka so edna nepoznata h, koja mo`e da se transformira vo
ekvivalentna ravenka od vidot
ax + b = 0 (4)
kade {to a i b se dadeni broevi, se vika linearna ravenka so edna nepoznata.

Ovoj vid na linearnata ravenka se vika normalen vid na ravenkata.


Brojot a se vika koeficient pred nepoznatata x, a b - sloboden ~len.
Ako vo ravenkata (4) koeficientot a e razli~en od nula, toga{a taa u{te se vika i
ravenka od prv stepen so edna nepoznata. Ako, pak, a = 0, toga{ linearnata ravenka (4) ne
mo`e da se nare~e ravenka od prv stepen.

1. Koi ravenki se vikaat linearni ravenki so edna nepoznata?


2. [to e koeficient, a {to sloboden ~len na linearna ravenka vo normalen vid?
[ [ 
3. Ravenkata a)    [ b) [   [   , dovedi ja vo normalen vid, a potoa
  
opredeli go koeficientot i slobodniot ~len.

II. 7. RE[AVAWE NA LINEARNI RAVENKI


SO EDNA NEPOZNATA
Re{avaweto na linearnite ravenki so edna nepoznata go vr{ime vrz osnova na primenata
na teoremite za transformacija na ravenkite i nivnite posledici. Pritoa, mo`ni se
slednive tri slu~ai:
I slu~aj: a z 0.

[  [  [
Primer 1. Da se re{i ravenkata   .
  
Re{enie: Prvo se osloboduvame od dropkite vo ravenkata. Za taa cel dvete nejzini
strani }e gi pomno`ime so brojot 12. Ja dobivame ekvivalentnata ravenka:
6( x – 2) – 4(x – 3) = 4 ·12 – 3(x – 4).
Se osloboduvame od zagradite, izvr{uvaj}i gi nazna~enite operacii:
6x – 12 – 4x+12 = 48 – 3x + 12.
Potoa, gi prenesuvame site ~lenovi od levata na desnata strana na ravenkata:
6x –12 – 4x +12 – 48 +3x – 12= 0 i vr{ime uprostuvawe na izrazot na desnata strana.
Taka ja dobivame linearnata ravenka 5x – 60 = 0, so koeficient a = 5 i b =–60.

55
Go prenesuvame slobodniot ~len _60 od levata na desnata strana: 5x = 60, a potoa, dvete
strani na dobienata ravenka gi delime so koeficientot 5. Taka ja dobivame najprostata
ravenka x = 12, {to e ekvivalentna na dadenata. Od nea gledame deka brojot 12 e edinstvenoto
nejzino re{enie, a toa e re{enie i na dadenata ravenka.
Vo op{t slu~aj, linearnata ravenka ax + b = 0 (1)
ja re{avame koga prvo, slobodniot ~len b go prenesuvame na desnata strana na ravenkata,
a potoa pri a z 0, dvete strani na dobienata ravenka:
ax = -b (2)
gi delime so koeficientot a i ja dobivame ekvivalentnata ravenka:
E
[  .
D
E
Zna~i: Pri a z 0 linearnata ravenka ax + b = 0 ima edinstveno re{enie [  .
D
II slu~aj: a = 0, b z 0.
Primer 2. Da se re{i ravenkata 2x – 1 = 2(x + 3).
Re{enie: 2x – 1 = 2x + 6,
2x – 1 – 2x – 6 = 0,
0 · x – 7 = 0.
Dobivame linearna ravenka, vo koja a = 0 i b = –7.
Od nea ja dobivame ravenkata 0 · x = 7.
Gledame proizvodot 0 · x za koja bilo vrednost na x e ednakov na nula. Zna~i, nema
vrednost na x za koja da e zadovolena ravenkata 0 · x = 7, t.e. taa nema re{enie. Bidej}i
dadenata ravenka e ekvivalentna na poslednata, zatoa i taa nema re{enie.
Spored toa: Pri a = 0, b z 0 linearnata ravenka ax + b = 0 nema re{enie.
Ravenka koja nema re{enie obi~no se vika protivre~na ravenka.
III slu~aj. a = 0 i b = 0.
Primer 3. Da se re{i ravenkata: 2x + 3 = 2(x – 1) + 5.
Re{enie: Ako izrazot na desnata strana na ravenkata go transformirame, }e
zabele`ime deka toj e identi~en na izrazot {to se nao|a na levata strana na ravenkata
2x +3 = 2x +3.
Ako ja re{avame ravenkata, }e dojdeme do ravenkata
2x – 2x + 3 – 3 = 0, odnosno 0 · x + 0 = 0.
Poslednata ravenka e zadovolena za sekoja vrednost na x, t.e. sekoj broj e re{enie na
taa ravenka. Zna~i, dadenoto ravenstvo ne e ravenka, tuku identitet.

56
Spored toa: Pri a = 0 i b = 0 linearnata ravenka ax + b = 0 e identitet, t.e. nejzinoto
re{enie e sekoj realen broj.
Od razgledanite primeri gledame, deka pri re{avaweto na linearnite ravenki so
edna nepoznata vo praktikata gi izvr{uvame slednite transformacii:

1o. Osloboduvawe od imenitelite na drobnite ~lenovi (ako ima takvi) vo ravenkata.


2o. Osloboduvawe od zagradite (ako ima takvi) vo ravenkata, izvr{uvaj}i gi
nazna~enite operacii.
3o. Gi grupirame ~lenovite {to ja sodr`at nepoznatata na edna strana, a
slobodnite ~lenovi na drugata strana na ravenkata.
4o. Gi sveduvame sli~nite ~lenovi vo ravenkata.
5o. Dvete strani na dobienata ravenka gi delime so koeficientot pred nepoznatata
(ako toj koeficient ne e nula).

Vo nekoi slu~ai ravenkata mo`e da se re{i i polesno, koga }e se promeni ustanoveniot


redosled na gornive transformacii: Na primer: Ravenkata 5(x + 2) = 35 polesno mo`e da se
re{i, ne ako prvo se oslobodime od zagradata, tuku ako prvo gi podelime dvete strani na
ravenkata so 5. Taka dobivame, x + 2 = 7 t. e. x = 5.
Drug primer: Ravenkata 21 = 7x, so delewe na dvete strani so brojot 7, go dobiva vidot
3 = x, {to e isto so x = 3.

Da se re{at ravenkite so edna nepoznata:


1. a) (x – 2) – 9 = 21 – x, b) 3x – (2x – 7) = 11 – 2x,
2
2. a) 2(x + 1)2 – x2 = (x – 1)2 , b) (x– 5) – 20 = (x+ 3)2.
3. a) 5x – 3 = 3x + 28 – (3x – 4), b) 9x – 18 = 4(x – 1).
[ [ [ [
4. a) [     , b)  [ .
   

[ § · § ·
5. a) , b)  ¨ [  ¸  ¨ [  ¸ .
 © ¹ ©  ¹

2 2
6. a) (x – 5) – (x – 1) = 8, b) (x + 3)2 + 35 = (x – 2)2.

57
  [ [     [ [ 
7. a)  , b)  [  .
    

[   [   [      [ [
8. a)  , b)  .
    

II. 8. PRIMENA NA LINEARNA RAVENKA


SO EDNA NEPOZNATA

Re{avaweto na mnogu zada~i od fizikata, hemijata, algebrata, geometrijata i drugite


oblasti na naukata i praktikata se sveduva na sostavuvawe i re{avawe na linearna ravenka
so edna nepoznata. Toa se zada~i, vo koi zavisnostite me|u veli~inite ne se izrazeni so
ravenka, tuku se iska`ani so zborovi - tekstualno. Zatoa, pri nivnoto re{avawe,
neophodno e zavisnostite me|u veli~inite vo zada~ata da se izrazat i prevedat na
matemati~ki jazik so pomo{ na nekoja ravenka, a potoa, istata da se re{i.
Re{avaweto na takvite zada~i }e go poka`eme so primeri.

Zada~a 1. Dvajca velosipedisti istovremeno trgnale na pat. Edniot trgnal od Veles


za Gevgelija, a drugiot od Gevgelija za Veles. Prviot minuval sredno po 15 km na ~as, a
vtoriot, {to trgnal od Gevgelija, minuval sredno po 12, 5 km na ~as. Po kolku ~asa od
trgnuvaweto tie }e se sretnat, ako rastojanieto me|u Veles i Gevgelija e 110 km?

Re{enie. Zada~ata prvo }e ja re{ime aritmeti~ki. Razmisluvame:


Po 1 ~as patuvawe od trgnuvaweto, velosipedistite }e go namalat rastojanieto me|u
sebe za 15 km + 12, 5 km = 27, 5 km. O~igledno e deka odgovor na zada~ata }e dobieme, koga
}e najdeme kolku pati se sodr`at 27, 5 km vo 110 km. Presmetuvaj}i go koli~nikot
110 : 27, 5 nao|ame deka dvajcata velosipedisti }e se sretnat po 4 ~asa od trgnuvaweto.

Crte` 1

Da vidime sega, kako }e ja re{ime zada~ata algebarski so pomo{ na ravenka.


Vo zada~ata se bara da se opredeli samo eden broj, spored toa, samata zada~a n– upatuva:
brojot na ~asovite {to }e gi pominat velosipedistite od trgnuvaweto do sretnuvaweto
da go ozna~ime so x. Potoa razmisluvame: Prviot velosipedist za 1 ~as izminuva 15 km,
a za x ~asa }e izmine (15x) km. Vtoriot velosipedist za 1 ~as izminuva 12, 5 km, a za x ~asa
}e izmine (12, 5 x) km.

58
Vo soglasnost so uslovot na zada~ata, izminatite pati{ta 15x i 12, 5x se delovi od
celiot pat 110 km, pa spored toa, ravenkata {to treba da ja sostavime, }e glasi
15x + 12, 5x = 110.
Pri sostavuvawe na ravenkata vo ovaa zada~a po`elno e da napravime skica kako na
crt. 1.
Potoa, sostavenata ravenka ja re{avame i go nao|ame nejziniot koren x = 4. Zna~i:
dvajcata velosipedisti }e se sretnat po 4 ~asa od trgnuvaweto, vo {to lesno se uveruvame,
ako ja izvr{ime potrebnata proverka.


Zada~a 2. Ako kon imenitelot na dropkata go dodademe brojot 20, koj broj treba da go dodademe

kon nejziniot broitel, taka {to, novodobienata dropka da bide ednakva na recipro~nata vrednost od
dadenata dropka?

Re{enie: Baraniot broj {to treba da go dodademe kon broitelot na dropkata, da go ozna~ime so


x. Ako gi izvr{ime privedenite promeni so broitelot i imenitelot na dropkata , }e ja dobieme

 [  [
dropkata , t.e. .
   

Spored uslovot na zada~ata, taka dobienata dropka treba da bide ednakva na recipro~nata vrednost
 
na dadenata dropka , t.e. da bide ednakva na dropkata .
 
 [ 
Ravenkata {to treba da ja sostavime }e bide: .
 
Ako ja re{ime, }e najdeme deka nejzinoto re{enie e x = 29.

Proveri, dali toe e re{enie i na zada~ata!

Zada~a 3. Zbirot na cifrite na eden dvocifren broj iznesuva 14. Ako kon toj broj dodademe 72,
}e dobieme broj {to e napi{an so istite cifri, no vo obraten red. Koj e toj broj?

Re{enie. Ako cifrata na desetkite na baraniot dvocifren broj ja ozna~ime so x, toga{ negovata
cifra na edinicite, }e bide 14 - x. Bidej}i edna desetka ima deset edinici, a x desetki }e imaat 10 x
edinici, zatoa baraniot broj }e bide 10x + (14 - x), a brojot {to e napi{an so istite cifri, no vo
obraten red, }e bide: 10(14 - x) + x.
Spored uslovot na zada~ata, baraniot broj 10x + (14 - x) koga }e go zgolemime za 72 treba da go
dobieme brojot 10(14 - x) + x. Taka doa|ame do ravenkata
10x +(14 - x) + 72 = 10(14 - x) + x.
Ako taa ravenka ja re{ime, nao|ame deka nejzin koren e x = 3. Spored toa, cifrata na desetkite na
baraniot broj }e bide ednakva na 3, a cifrata na edinicite }e bide ednakva na 14 – 3 = 11.

59
Bidej}i cifrata na edinicite (a, isto taka, i cifrata na desetkite) na koj bilo
dvocifren broj ne smee da bide pogolema od 9, zatoa velime, deka dobienoto re{enie
nema smisla.
Na ovoj primer najubavo mo`eme da se uverime deka i pokraj toa {to ravenkata e
pravilno sostavena i re{ena, sepak, nejzinoto re{enie, mo`e da se slu~i, da ne e mo`no,
t.e. da ne gi zadovoluva uslovite na zada~ata. Vo takov slu~aj velime, deka zada~ata nema
re{enie.

Zada~a 4. Edna majka ima 35 godini, a nejziniot sin ima 11 godini. Po kolku godini
majkata }e bide 4 pati postara od svojot sin?

Re{enie. Baraniot broj godini da go ozna~ime so x, t.e. neka po x godini majkata }e


stane 4 pati postara od svojot sin.
Majkata sega ima 35 godina, a po x godini taa }e ima 35 + x godini.
Sega sinot ima 11 godini, a po h godini toj }e ima 11 + x godini.
Vo soglasnost so uslovot na zada~ata ja sostavuvame ravenkata
35 + x = 4(11 + x).
Koga }e ja re{ime ravenkata, nao|ame deka taa ima re{enie x = – 3. Gledame deka
dobienoto re{enie e negativen broj x = – 3. Toa poka`uva deka zada~ata pri dadenite
uslovi nema re{enie, t.e. majkata ne mo`e da stane 4 pati postara od svojot sin.
Me|utoa, ako zememe predvid deka vremeto vo odnos na nekoj moment mo`e da se izrazi
ili so pozitiven ili so negativen broj, vo zavisnost od toa dali toa se meri po ili pred
toj moment, toga{ dobieniot odogovor x = – 3 godini poka`uva deka majkata pred 3 godini
bila ~etiri pati postara od svojot sin. Navistina, pred tri godini majkata imala
35 – 3 = 32 godini, a nejziniot sin toga{ imal 11 – 3 = 8 godini. Zna~i, pred 3 godini
majkata bila 4 pati postara od sinot.
Spored toa, ako go promenime uslovot na zada~ata da glasi: Edna majka ima 35 godini,
"
a sinot ima 11 godini. Pred kolku godini majkata bila 4 pati postara od svojot sin?",
toga{ }e ja dobieme ravenkata 35 – x = 4(11 – x) i zada~ata }e ima re{enie x = 3 .

Zada~a 5. Eden ramnokrak triagolnik ima perimetar 19 cm, a krakot mu e podolg od


osnovata za 3,5 cm. Da se odredat dol`inite na osnovata i krakot na toj triagolnik.

Re{enie: Vo zada~ata ima dve nepoznati veli~ini: dol`inata na osnovata i dol`inata


na krakot. Ako dol`inata na osnovata ja ozna~ime so x, toga{, spored uslovot na zada~ata,
dol`inata na krakot, }e bide: (x + 3, 5) cm.

60
Perimetarot na koj bilo triagolnik e ednakov na zbirot od dol`inite na negovite
strani, pa spored uslovot na zada~ata, ja sostavuvame ravenkata:
x + 2(x + 3, 5) = 19.
Taa ima re{enie x = 4. Zna~i: Dol`inata na osnovata iznesuva 4 cm, a dol`inata na
krakot 4 + 3,5 = 7,5 (cm).
Od razgledanite primeri gledame, deka pri re{avaweto na ovoj vid zada~i
neophodno e:

1o. Vnimatelno da se pro~ita i razbere tekstot na zada~ata.


2o. Da se utvrdi koja veli~ina }e se zeme za nepoznata i istata da se ozna~i so x ili
so druga bukva.
3o. Da se utvrdat zavisnostite me|u baranata i dadenite veli~ini vo zada~ata.
4o. So pomo{ na zavisnostite da se sostavi ravenkata.
5o. Da se re{i dobienata ravenka.
6o. Da se izvr{i proverka, dali re{enieto na ravenkata gi zadovoluva uslovite na
zada~ata ili ne.

1. Koj broj koga }e go namalime za 49 go dava istiot rezultat, kako i koga }e go podelime
so 8?
2. Koj broj koga }e go podelime so 8 dava koli~nik 15 i ostatok 5?
3. Ako zamisleniot broj go zgolemime 2 pati i od dobieniot proizvod odzememe 15, }e
go dobieme brojot 27. Koj e toj broj?
4. Ako kon zamisleniot broj dodademe 5 i dobieniot zbir go namalime 3 pati, }e go
dobieme brojot 6. Koj broj e zamislen?
5. Zbirot na tri posledovatelni celi broevi iznesuva 39. Koi se tie broevi?
6. Zbirot od polovinata, tretinata i ~etvrtinata od eden broj za 3 e pogolem od samiot
broj. Koj e toj broj?

61
7. Eden od dva komplementni agli e 4 pati pogolem od drugiot agol. Presmetaj ja
goleminata na sekoj od tie agli!
8. Ako dol`inata na stranata na eden kvadrat ja zgolemime za 3 cm, negovata plo{tina
}e se zgolemi za 57 cm2. Odredi ja dol`inata na stranata na dadeniot kvadrat!
9. Vo eden pravoagolen triagolnik ednata kateta e dolga 20 cm, a hipotenuzata e za 8 cm
podolga od drugata kateta. Odredi gi dol`inite na drugata kateta i hipotenuzata!
10. Osnovata na eden ramnokrak triagolnik e dolga 6 cm, a visinata mu e za 1 cm pokratka
od krakot. Kolkav e krakot?
11. Perimetarot na eden ramnokrak triagolnik e 21 cm, a krakot e za 3 cm podolg od
osnovata. Odredi ja osnovata na triagolnikot!
12. Dol`inite na trite strani na eden triagolnik se izrazeni so tri posledovatelni
celi broevi. Odredi gi dol`inite na stranite na triagolnikot, ako negoviot
perimetar iznesuva 24 cm.

B LINEARNI NERAVENKI SO EDNA NEPOZNATA


II.9. NERAVENSTVO I NERAVENKA
Znaeme, sekoj pozitiven realen broj a e pogolem od nulata, a sekoj negativen realen
broj b e pomal od nulata. Toa simboli~ki go zapi{uvame so zapisite (formulite) a > 0 i
b < 0.
Na brojnata oska pozitivnite realni broevi gi pretstavuvame so to~ki nadesno od
po~etokot O, a negativnite realni broevi - so to~ki nalevo od po~etokot O. Znaete,
isto taka, od dva realni broja a i b pogolem e onoj, {to e pretstaven na brojnata oska
nadesno od drugiot.

Definicija 1. Dva broja ili dva izraza (brojni ili algebarski) svrzani so znakot >"
"
(pogolem od) ili <" (pomal od) velime, obrazuvaat neravenstvo.
"

Primeri na neravenstva se slednite simboli~ki zapisi:


8 < 12, 4 · 5 > 52, a > b, x < x + 1, 3x –1 > x + 2, itn.
Kaj sekoe neravenstvo razlikuvame leva i desna strana i znak (ili nasoka)(< ili >)
na neravenstvoto.
Za dve neravenstva so ist znak na neravenstvo (dvete so znak <, ili dvete so znak >)
velime, deka se ednakvo naso~eni, a za dve neravenstva so sprotivni znaci na neraven-
stvoto (ednoto so znak <, a drugoto so znak >) velime deka se sprotivno naso~eni.
Na primer, neravenstvata 2x > 5 i x > 7 se ednakvo naso~eni, a neravenstvata
x < 5 i x > 8 se sprotivno naso~eni.

62
Ako dvete strani na neravenstvoto se broevi ili brojni izrazi, velime deka toa e
brojno ili numeri~ko neravenstvo. Ako, pak, dvete ili barem edna strana na
neravenstvoto e nekoj izraz so promenliva, toga{ toa se vika neravenstvo so promenliva
ili algebarsko neravenstvo.
Brojnite neravenstva, vsu{nost, se iskazi, so sekoe od niv ne{to se tvrdi, za koe
mo`e da se ka`e deka e to~no ili neto~no.
Me|utoa, neravenstvata so promenlivi ne se iskazi, bidej}i za sekoe od niv ne mo`e
da se ka`e deka e to~no ili neto~no. Tie stanuvaat iskazi, ako promenlivite vo niv se
zamenat so nekoi brojni vrednosti. Na primer, neravenstvoto x > 5 ne e iskaz. No za x = 3
toa stanuva nevistinit iskaz 3 > 5, a za x = 7 toa preminuva vo vistinit iskaz 7 > 5.
Promenlivite vo algebarskite neravenstva se vikaat u{te i nepoznati. Za nivno
ozna~uvawe, obi~no gi koristime bukvite h, u, ...
Ako za nekoj izbor na brojni vrednosti na nepoznatite x = x0, y = y0, … dadeno neravenstvo
premine vo to~no brojno neravenstvo, velime, deka toj izbor na brojni vrednosti na
nepoznatite go zadovoluva toa neravenstvo. Na primer, x = 7 go zadovoluva neravenstvoto
x > 5, a x = 3 ne go zadovoluva.
Neravenstvata {to sodr`at nepoznati mo`at da bidat od dva vida, spored toa, dali
tie se zadovoleni za sekoj ili ne za sekoj proizvolen izbor dopu{teni vrednosti na
nepoznatite vo niv.

Definicija 2. Neravenstvo, {to e zadovoleno za sekoj proizvolen izbor na brojni


vrednosti na nepoznatie, se vika identi~no neravenstvo, ili samo kratko neravenstvo.

Usvoeno e i sekoe to~no brojno neravenstvo da se vika identi~no neravenstvo.

Definicija 3. Neravenstvo, {to ne e zadovoleno makar i samo za eden opredelen


izbor na dopu{teni brojni vrednosti na nepoznatite, se vika neravenka.

Na primer, neravenstvoto x + 1 > x e identi~no neravenstvo, bidej}i za koja bilo


vrednost na x levata strana e za 1 pogolema od desnata strana. A neravenstvoto x < 6 za x=8
ne e zadovoleno 8 < 6, pa spored toa, toa e neravenka.
Spored brojot na nepoznatite, neravenkite mo`at da bidat: so edna, so dve, so tri ili
pove}e nepoznati.
Neravenkite so edna nepoznata imaat op{t vid:
f (x) < g (x) ili f (x) > g (x), (1)
a neravenkite so dve nepoznati imaat op{t vid

f (x, y) < g (x, y) ili f (x, y) > g (x, y). (2)

63
Ako vo neravenkite (1) i (2) izrazite f(x) i g(x), odnosno f (x, y), g (x, y) se celi racionalni
(polinomi) po odnos na nepoznatite, toga{ vo zavisnost od stepenot na tie polinomi,
neravenkite (1) i (2) mo`at da bidat: od prv stepen ili linearni neravenki; od vtor
stepen ili kvadratni neravenki, od tret stepen, itn.
Nie }e razgleduvame samo neravenki od prv stepen so edna nepoznata.
Ponekoga{, pome|u stranite na neravenstvoto mo`e da stoi i eden od alternativnite
znaci, koi dopu{taat i ednakvost: d " (pomal ili ednakov na), ili t " (pogolem ili
" "
ednakov na). Na primer, neravenstvata  d  i  d  se vistiniti iskazi,
a neravenstvata  t  i 9 > 9 se nevistiniti izrazi (Zo{to ?).

1. Koj od slednive iskazi e vistinit, a koj nevistinit:


a) –5 > –2, b) 3 > 0, v) 4 < –7, g) 42 – 3 > 5 + 1?

2. Koe od slednive brojni neravenstva e vistinito, a koe nevistinito:


a) 5 – 3 · 2 > 4, b) 6 > –2, v)  t  , g)  d  ?

3. Proveri za koi vrednosti na [  ^   ` neravenstvoto e zadovoleno:


a) x + 4 < 7, b) x2 – 4 < 0.

4. [to mo`e{ da zaklu~i{ za razlikata a – b, ako e to~no neravenstvoto:


a) a < b, b) a > b, v) a = b?

II. 10. SVOJSTVA NA BROJNITE NERAVENSTVA

Vrz osnova na svojstvata na razlikata na dva realni broja, zaklu~uvame deka va`i:
Ako namalenikot e pogolem od namalitelot, toga{ razlikata e pozitiven realen
broj , i obratno.
Ako, pak, namalenikot e pomal od namalitelot, toga{ razlikata e negativen realen
broj, i obratno.

64
Ovie dve svojstva n upatuvaat da gi usvoime slednite dve definicii:

Definicija 1. Brojot a velime deka e pomal od brojot b, ako i samo ako, razlikata
a – b e negativen broj, t.e.
D E œ D E  . (1)

Definicija 2. Brojot a velime deka e pogolem od brojot b, ako i samo ako, razlikata
a – b e pozitiven broj, t.e.
D !E œ DE ! . (2)

Simbolot œ go ~itame: ako i samo ako" ili e ekvivalentno na".


" "
Na primer: 2 < 9, bidej}i 2 – 9 < 0,
–1 > – 6, bidej}i –1 – (–6) = –1 + 6 = 5 > 0.
]e poka`eme deka brojnite neravenstva gi imaat slednive pova`ni svojstva:
1 o . D  E œ E ! D , t.e. ako gi smenime mestata na levata i desnata strana na
neravenstvoto, toga{ treba da se promeni i nasokata na neravenstvoto vo sprotivna.

Dokaz. Od a < b soglasno (1) sleduva deka razlikata a - b e negativna. Toga{ pak razlikata
b - a }e bide pozitivna, t.e. b – a > 0. Ottuka sleduva deka b > a.

2o. D  E L E  F Ÿ D  F (svojstvo na tranzitivnost).

Dokaz. Od a < b i b < c sleduva deka a – b < 0 i b – c < 0. Bidej}i zbirot na dva negativni
broja e isto negativen broj, pa zatoa }e bide (a – b) + (b – c) < 0 . Ottuka dobivame

a – c < 0, pa }e bide a < c.


Neravenstvata a < b i b < c mo`at da se obedinat vo edno neravenstvo a < b < c, koe se
vika dvojno neravenstvo, t.e.

a < b < c Ÿ ( a < b i b < c). (3)

3 o. D  E L P  5 Ÿ D  P  E  P , (4)
t.e. kon dvete strani na neravenstvoto mo`e da se dodade (ili odzeme) eden ist broj.

Na primer, ako kon dvete strani na to~noto neravenstvo –3 < 5 go dodademe brojot 7,
pak, dobivame to~no neravenstvo –3 + 7 < 5 +7, t.e. 4 < 12.

65
Od ova svojstvo sleduvaat slednive dve posledici:

Posledica 1. Ako vo dvete strani na neravenstvoto ima ednakvi ~lenovi, tie mo`at
da se izostavat, t.e. D  P  E  P Ÿ D  E .

Posledica 2. Sekoj ~len na neravenstvoto mo`e da se prenese od edna na druga strana,


samo pritoa, negoviot znak stanuva sprotiven.

Na primer, D!Ÿ D !.

4o. D  E L N !  Ÿ D ˜ N  E ˜ N , (5)
t.e. Ako dvete strani na neravenstvoto gi pomno`ime so eden ist pozitiven broj,
neravenstvoto ne ja menuva svojata nasoka.

 
Na primer, to~noto neravenstvo  so mno`ewe na dvete negovi strani so 4, toa,
 
pak, preminuva vo to~no neravenstvo 3 < 10.

5 o. D  E L N   Ÿ D ˜ N ! E ˜ N , (6)
t.e. Ako dvete strani na neravenstvoto gi pomno`ime so eden ist negativen broj,
neravenstvoto }e ja promeni svojata nasoka vo sprotivna.

Dokaz. Od a < b sleduva a– b < 0. [tom broevite a – b i k se negativni, toga{ nivniot


proizvod }e bide pozitiven, t.e. }e bide (a –b) · k > 0, ak – bk > 0, odnosno ak >bk.
Primer. Ako dvete strani na neravenstvoto –3 < 5 gi pomno`ime so brojot –2, }e
dobieme (–3) · (–2) > 5 · (–2), odnosno 6 > –10.

1. Dali e vistinito neravenstvoto: a)  t  , b)  d  , v) a + k2 > a, g) a – k2 < a?


2. Dali e vistinito neravenstvoto za sekoja vrednost na x :
a) 2x < 3x, b) –x < x , v) x > x – 3, g) x2 > x?
3. Razmeni gi mestata na levata i desnata strana na neravenstvoto:
a) 6 > –x, b) 7 < –2, v) 8 > 2 – x.

66
4. Na koi prosti neravenstva e ekvivalentno dvojnoto neravenstvo –5 < x < 9?
5. Od neravenstvata 2 < 6 i 3 < 5 obrazuvaj dvojno neravenstvo {to mo`e da gi zameni.
6. Kako na brojnata oska go pretstavuvame mno`estvoto na site realni broevi, {to se
nao|aat pome|u broevite –2 i 7?

II. 11. BROJNA OSKA. INTERVALI

1. Poznato vi e {to e brojna oska. Na nea gi pretstavuvame realnite broevi so to~ki.


I brojnite neravenstva mo`at da bidat pretstavuvani na brojnata oska. Na primer, na
brojnata oska neka na brojot a mu soodvetstvuva to~ka A, a na brojot b - to~ka B. Toga{,
geometriskata smisla na neravenstvoto a < b e vo toa, {to to~kata A na brojnata oska
le`i nalevo od to~kata B; a ako e a > b, toga{ to~kata A le`i nadesno od to~kata B.

Na primer, svojstvoto 1 o D  E Ÿ E ! D
(razgledano vo II. 10) geometriski e o~igledno: Ako
to~kata A na brojnata oska le`i nalevo od to~kata
Crte` 2 B, toga{ to~kata B le`i nadesno od to~kata A, i
obratno (crt.2). Ist e slu~ajot i so svojstvoto
o
2 ( D  E i E  F ) Ÿ D  F –(svojstvo na tranzitivnost).
Ova svojstvo geometriski e, isto, o~igledno: Ako to~kata A na brojnata oska le`i nalevo
od to~kata B, a to~kata B le`i nalevo od to~kata C, toga{ to~kata A }e le`i u{te ponalevo
od to~kata C (crt. 2).
2. Pri razgleduvawe na razni pra{awa svrzani so realnite broevi, ~esto go koristime
poimot interval. Nego go definirame:

Definicija 1. Neka a i b se dva realni broja, pri {to a < b. Interval e mno`estvo na
site realni broevi x, koi zadovoluvaat edno od slednive dvojni neravenstva:

D d [ d E ; a < x < b; D d [  E ; D  [ d E .

Realnite broevi a i b se vikaat kraevi na intervalot.


Intervalot opredelen so neravenstvoto D d [ d E gi sodr`i kraevite a i b (crt. 3a).
Toj se vika zatvoren interval ili segment i se ozna~uva simboli~ki so [a,b]. Intervalot
opredelen so neravenstvoto a < x < b ne gi sodr`i kraevite a i b (crt. 3b).Toj se vika
otvoren interval i se ozna~uva so (a, b).
Ako intervalot sodr`i samo eden od kraevite, toj se vika poluotvoren interval i se
ozna~uva so [a, b), odnosno (a, b], a e opredelen so neravenstvoto D d [  E (crt. 3v), odnosno
D  [ d E (crt. 3g).

67
Crte` 3

Razlikata b – a se vika dol`ina na intervalot.


Geometriski, intervalot pretstavuva mno`estvo to~ki na del od brojnata oska, koi
le`at pome|u to~kite a i b. Na crte`ot 3 polnite (odnosno praznite) to~ki poka`uvaat
deka kraevite a i b pripa|aat (odnosno ne pripa|aat) na intervalot (a, b).
Pokraj gornite intervali, razlikuvame i beskone~ni" intervali. Niv gi definirame
"
vaka:

Definicija 2. Mno`estvoto na site realni broevi velime obrazuvaat interval od


– f (minus beskone~nost) do + f (plus beskone~nost) i se ozna~uva so simbolot
(- f ,+ f ).

Definicija 3. Mno`estvoto na site realni broevi x, koi se pogolemi (odnosno


pomali) od nekoj broj a, se vika interval od a do + f (odnosno od – f do a) i se ozna~uva so
(a,+ f ), (odnosno so (– f , a)).

Tie se karakteriziraat so neravenstvata: x > a, odnosno x < a.


Zabele{ka. Simbolite – f ,+ f ne gi smetame za broevi. So niv ne e definirana ni-
edna operacija. Usvoeno e, samo me|u niv i realnite broevi da se voveduvaat relaciite
na neednakvost.
Spored toa, intervalite (– f ,+ f ), (a, + f ) i (– f , a) mo`at da se izrazat so
neravenstvata:
– f < x < + f ; a < x < + f i – f < x < a.

1. Po {to se razlikuva brojnata oska od sekoja druga prava?


2. Koi to~ki od brojnata oska soodvetstvuvaat na:
a) pozitivnite realni broevi, b) negativnite realni broevi?
3. Mo`e li intervalot da se sostoi samo od:
a) celi broevi, b) racionalni broevi, v) iracionalnite broevi?

68
4. Poka`i deka [ \  > [ \ @ . Odredi ja unijata i presekot na tie dva intervala.

5. Dadeni se intervalite (1, 5) i (3, 7). Odredi gi intervalite:


a)  ˆ   , b)  ‰  , v) (1, 5) \ (3, 7).

6. Koi vrednosti na promenlivata [  ^    ` ja zadovoluvaat neravenkata:
a) 2x – 3 < 5, b) x < 9, v) [ t  ?

II. 12. RE[ENIE NA NERAVENKA.


EKVIVALENTNI NERAVENKI

Da ja razgledame neravenkata 4x – 5 < 3x + 1.


Zabele`uvame, za x = 5 taa preminuva vo vistinit iskaz 4 · 5 – 5 < 3 · 5 +1, odnosno
15 < 16; a za x = 10 taa preminuva vo nevistinit iskaz: 4 · 10 – 5 < 3 · 10 +1, odnosno 35 < 31.
Za brojot 5 velime deka e re{enie na dadenata ravenka, a brojot 10 ne e nejzino re{enie.

Definicija 1. Sekoja vrednost na nepoznatata, za koja e zadovolena dadena neravenka,


se vika re{enie na taa neravenka.

Site re{enija na edna neravenka I [ ! J [ obrazuvaat edno mno`estvo, koe se


vika mno`estvo re{enija na neravenkata i obi~no se ozna~uva so M.

Da se re{i edna neravenka, zna~i, da se odredi mno`estvoto re{enija na taa neravenka.

Definicija 2. Dve neravenki velime deka se ekvivalentni vo dadena brojna oblast,


ako nivnite mno`estva re{enija vo taa brojna oblast se sovpa|aat (se ednakvi).

Primer. Neravenkite x – 3 > 5 i x > 5 +3 se ekvivalentni, bidej}i i dvete se


zadovoleni samo za vrednostite na x pogolemi od 8. Vo soglasnost so definicijata,
dve neravenki mo`at da bidat ekvivalentni vo edna brojna oblast, a da ne se

ekvivalentni vo druga brojna oblast. Na primer, neravenkite [ > 2 i x > 2 se


ekvivalentni vo oblasta na pozitivnite realni broevi. No, vo oblasta na site realni
broevi tie ne se ekvivalentni, bidej}i vo taa oblast prvata neravenka e zadovolena,
osven za x > 2, u{te i za mno`estvoto brojni vrednosti na x {to se pomali od – 2.

69
Dve neravenki se ekvivalentni vo dadena brojna oblast u{te i koga dvete nemaat
re{enija vo taa oblast.
Re{avaweto na dadena neravenka go vr{ime, taka {to istata ja transformirame vo
drugi poprosti, no ekvivalentni na nea neravenki, dodeka ne dojdeme do najprostata
neravenka, ~ie mno`estvo re{enija e o~igledno.

1. [to e re{enie na neravenka?


2. Za koi neravenki velime deka se ekvivalentni?
3. Navedi primer na eden par ekvivalentni neravenki i navedi primer na eden par
neravenki koi ne se ekvivalentni.

II. 13. TEOREMI ZA EKVIVALENTNI NERAVENKI

Transformacijata na neravenkite vo poprosti ekvivalentni neravenki ja vr{ime


vrz osnova na slednive teoremi i posledici.
]e se ograni~ime samo na neravenkite od vidot I [ ! J [ , a neravenkite od vidot
I [  J [ , lesno mo`at da se svedat na prviot vid.

Teorema 1. Ako kon dvete strani na dadena neravenka dodademe eden ist racionalen
izraz, koj e definiran za sekoj x od definicionata oblast D na neravenkata, }e dobieme
nova neravenka ekvivalentna na dadenata.

Neka e dadena neravenkata I [ ! J [ (1)

i eden racionalen izraz M [ , koj e definiran za sekoj [  ' . ]e doka`eme deka


neravenkata I [  M [ ! J [  M [ (2)

e ekvivalentna na dadenata neravenka.

Dokaz. a) Neka x = x0 e edno od re{enijata na neravenkata (1), t.e. neka e I [R ! J [R .

Ako kon dvete strani na toa to~no brojno neravenstvo go dodademe brojot M [R -
brojnata vrednost na izrazot M [ za x = x0, vo soglasnost so svojstvoto 3o na brojnite
neravenstva, }e go dobieme neravenstvoto I [R  M [R ! J [R  M [R .
Spored toa, re{enieto na neravenkata (1) x = x0 e re{enie i na neravenkata (2).

70
b) Neka sega x = x1 e edno re{enie na neravenkata (2), t.e. neka va`i:
I [  M [ ! J [  M [ .
Ako od dvete strani na neravenstvoto go izostavime brojot M [ - brojna vrednost
na M [ za x = x1 }e go dobieme neravenstvoto: I [ ! J [ . Zna~i, koe bilo re{enie
na neravenkata (2) e re{enie i na neravenkata (1).
So toa teoremata e doka`ana. Od nea sleduvaat posledicite:

Posledica 1. Ako vo dvete strani na neravenkata ima ednakvi ~lenovi so isti znaci,
tie mo`e da se otfrlat.

Primer. Vo neravenkata 5x – 2x + 3 > 8 – 2x gledame deka vo dvete strani se sre}ava eden


ist ~len (–2h). Ako toj ~len go izostavime od dvete strani, }e dobieme ekvivalentna
neravenka 5x + 3 > 8.

Posledica 2. Sekoj ~len na neravenkata mo`e da se prenese od ednata na drugata


strana, ako pritoa se promeni negoviot znak vo sprotiven.

Primer. Dadena e neravenkata 4x – 5 > 3x – 2.


Po prenesuvaweto na ~lenot –5 so sprotiven znak od levata na desnata strana, a ~lenot
3x od desnata na levata strana, dobivame nova ekvivalentna neravenka 4x – 3x > 5 – 2,
odnosno x > 3.

Teorema 2. Ako dvete strani na dadena neravenka se pomno`at (ili podelat) so eden
ist pozitiven broj k, }e se dobie neravenka ekvivalentna na dadenata.

Dokazot na ovaa teorema e analogen na dokazot na teoremata 1, a so povikuvawe na


svojstvoto 4o na brojnite neravenstva.

Deleweto na dvete strani so pozitivniot broj k, znaeme deka mo`e da se zameni so



mno`ewe na dvete strani na neravenkata so recipro~nata vrednost , t.e.
N
 
I [ ˜ ! J [ ˜ .
N N

Posledica 3. Ako dvete strani na neravenkata imaat zaedni~ki pozitiven


mno`itel, so nego mo`at da se podelat dve strani na neravenkata, pri {to }e se dobie
neravenka ekvivalentna na dadenata.

71
Vo takov slu~aj velime deka neravenkata ja skratuvame.
Primer. Ako dvete strani na neravenkata 3(x – 2) < 15 gi podelime so 3 (ili pomno`ime
so  ), se dobiva ekvivalentna neravenka x – 2 < 5.


Posledica 4. Neravenkata so drobni brojni koeficienti, mo`e da se transformira


vo neravenka so celi brojni koeficienti, ako dvete strani na neravenkata se
pomno`at so pozitivniot najmal zaedni~ki sodr`atel na imenitelite na dropkite.

Takvata transformacija se vika osloboduvawe od imenitelite.

Primer. Dadena e neravenkata  [    [     .


 
Ako dvete nejzini strani gi pomno`ime so NZS na imenitelite, t.e. so brojot 6, }e
dobieme neravenka 2 (2x – 7) < 3 (x + 1) – 5 · 6 ekvivalentna na dadenata.

Teorema 3. Ako dvete strani na dadena neravenka gi pomno`ime (ili podelime) so


eden ist negativen broj N i ako ja promenime nasokata na neravenkata vo sprotivna, }e
dobieme neravenka, koja e ekvivalentna na dadenata, t.e.

N   Ÿ ª¬ I [ ! J [ œ I [ ˜ N  J [ ˜ N ¼º

Dokazot na ovaa teorema e sli~en kako i na teoremite 1 i 2, samo so povikuvawe na


svojstvoto 5o na brojnite neravenstva.

Posledica 5. Ako dvete strani na dadena neravenka gi pomno`ime so brojot –1,


nasokata na neravenkata treba da se promeni vo sprotivna.

Primer. Ako dvete strani na neravenkata –x > 7 gi pomno`ime so –1, taa go dobiva
vidot x < –7.

1. Koi od slednive neravenki se ekvivalentni i zo{to?


a) 6x – 2 > 7 + 3x, b) 6x – 3x > 7 + 2, v) 3x > 9, g) x > 3.

72
2. Najdi neravenka bez imenitel {to }e bide ekvivalentna na neravenkata:
[   [ [  [  [ 
a)   , b)  ! .
     
3. So koristewe na teoremite za ekvivalentnost na neravenkite i nivnite posledici
poka`i deka: a)  [   !  [   œ [ !  , b)  [    [   œ [   .
 
4. Doka`i ja teoremata 2.

II. 14. RE[AVAWE NA LINEARNA NERAVENKA


SO EDNA NEPOZNATA

Linearna neravenka so edna nepoznata e sekoja neravenka koja mo`e da se transformira


vo vidot
ax + b > 0 ili ax + b < 0,
kade {to a i b se koi bilo dadeni realni broevi.

Ako dvete strani na neravenkata 3x – 2 < 0 gi pomno`ime so –1, taa }e premine vo vidot
–3x + 2 > 0. Zatoa, dovolno e da ja prou~ime samo neravenkata
ax + b > 0. (1)
Pri ispituvawe na re{enijata na ovaa neravenka vo zavisnost od vrednostite na a i b,
}e razlikuvame tri slu~ai:
I. Neka a > 0. Ako b go prefrlime na desnata strana, a potoa dvete strani na neraven-
kata gi podelime so a, }e ja dobieme neravenkata [ !  E , a so nea i baranoto mno`estvo
D
re{enija na neravenkata (1). Spored toa:

Pri a > 0, mno`estvoto re{enija na neravenkata (1) se site realni broevi pogolemi
§ E ·
od  E , odnosno 0 ¨  f¸ .
D © D ¹

II. Neka a < 0. So prenesuvawe na b na drugata strana, a potoa so delewe na dvete strani
E
na neravenkata so a, se dobiva neravenkata [   . Spored toa:
D

Pri a<0, mno`estvoto re{enija na neravenkata (1) se site realni broevi pomali od
E § E·
brojot  , t.e. 0 ¨ f  ¸ .
D © D¹

73
III. Neka a = 0. Vo toj slu~aj neravenkata (1) go dobiva vidot
0 · x + b > 0, odnosno b > 0.
Ako b e pozitiven broj, neravenkata (1) e zadovolena za sekoj realen broj x, t.e.
0 f f 5 . A ako b e negativen realen broj, toga{ neravenkata (1) nema re{enie,
t.e. 0 ‡.

Primer 1. Da se re{i neravenkata [     ! [     .


 
Re{enie. Prvo }e se oslobodime od dropkite vo nea, taka {to dvete nejzini strani gi
pomno`ime so 6 (NZS za imenitelite):
2 (x – 3) – 6 > 3 (x – 1) – 12.
Potoa se osloboduvame od zagradite i gi grupirame site ~lenovi {to ja sodr`at
nepoznatata na levata strana, a slobodnite ~lenovi na desnata strana, i ja dobivame
ekvivalentnata neravenka: 2x – 3x > –3 – 12 + 6 + 6, odnosno –x > –3.
So mno`ewe na dvete nejzini strani so –1, dobivame x < 3.
Poslednata neravenka e zadovolena za sekoj realen broj pomal od 3. Zna~i, mno`es-
tvoto re{enija na dadenata neravenka go so~inuvaat site realni broevi od intervalot
(– f , 3).
Mno`estvoto re{enija na neravenkata grafi~ki mo`eme da go pretstavime na broj-
nata oska so to~kite, {to le`at nalevo od to~kata 3 kako {to e poka`ano na crte`ot 4.
Pritoa, delot od brojnata oska na koj le`at to~kite (broevite) za koi ne e zadovolena
neravenkata, obi~no, go precrtuvame (crt. 4).

Crte` 4 Crte` 5

Primer 2. Da se re{i neravenkata (x – 2)2 – 3x < x( x – 3).

Re{enie. x2 – 4x + 4 – 3x < x2 – 3x; –4x < –4.


Gi delime dvete strani so –4 i dobivame x > 1, t.e. mno`estvoto re{enija na neravenkata
go so~inuvaat site realni broevi pogolemi od 1, t.e. M = (1, f ). Toa grafi~ki e pretstaveno
na crt ` 5.

74
Da se re{at neravenkite (1 – 5):
1. a) 3x – 4 > 2 – x, b) 5 – 2y < y + 8.
2. a) x – (2 – x) < 3x + 7, b) (x – 3)2 < x(x + 1).
3. a) 2x – 5 < x, b) 3( x –2) > x –1.

[   [ [  [  [  
4. a)   , b) ! .
    

\  \  \ [ 
5. a)  ! , b)  [   !  .
   

II. 15. PRIMENA NA LINEARNI NERAVENKI


SO EDNA NEPOZNATA

Linearnite neravenki so edna nepoznata nao|aat raznovidna primena vo matematikata


i praktikata. Na primer, re{avaweto na linearnite ravenki i neravenki so parametri
sekoga{ e prosledeno so re{avawe i na nekoja linearna neravenka so edna nepoznata.
Eve nekolku primeri za primenata na linearnite neravenki.
Primer 1. Za koi vrednosti na promenlivata x izrazot  [   e realen broj?

Re{enie. Znaeme, kvadratniot koren e realen broj, samo, ako potkorenoviot broj e
nenegativen realen broj. Zna~i, izrazot  [   }e bide realen broj, samo za vrednostite
na x, za koi va`i neravenkata  [   t  . Zna~i, dadeniot izraz }e bide realen broj za

vrednosti na [ t  , odnosno ako [  ª«  f ·¸ .
 ¬ ¹

Primer 2. Na kolku na~ini brojot 50 mo`e da se razdeli na dva celi pozitivni


sobiroka, ~ija razlika da e pogolema od 20?

Re{enie. Ako edniot sobirok e x, toga{ drugiot sobirok }e bide 50 – x. Vo soglasnost


so uslovot na zada~ata broevite x i 50 – x treba da se celi broevi, ~ija razlika e pogolema
od 20, t.e. treba da va`i neravenkata x – (50 – x ) > 20, kade {to x e priroden broj pomal od 50.
Ako ja re{ime neravenkata, }e vidime deka nejzinoto mno`estvo re{enija e intervalot
(35, 50), t.e. 35 < x < 50.

75
Bidej}i nepoznatata x e priroden broj, taa mo`e da gi zema slednive vrednosti:
x = 31, 32, 33, ..., 49. Niv gi ima to~no 19. Toj broj ni poka`uva na kolku na~ini mo`e da se
izbere prviot sobirok, a na sekoj od niv mu odgovara po eden vtor sobirok 50 – x. Zna~i,
postojat vkupno 19 na~ini.

Primer 3. Da se re{i neravenkata mx – 5 > 3x – m, kade {to x e nepoznata, a m e parametar.

Re{enie. Gi prenesuvame ~lenovite {to ja sodr`at nepoznatata x na desnata strana,


a drugite ~lenovi na levata strana, pa ja dobivame neravenkata mx – 3x > 5 – m, odnosno
(m – 3) x > 5 – m.
Bidej}i koeficientot (m – 3), vo zavisnost od parametarot m, mo`e da bide pozitiven
broj, nula ili negativen broj, zatoa }e razlikuvame tri slu~ai.
1. Ako e m – 3 > 0, t.e. m > 3, toga{ dadenata neravenka }e ima mno`estvo re{enija
P
[! .
P
2. Ako e m – 3 < 0, t.e. m < 3, toga{ dadenata neravenka }e ima mno`estvo re{enija
P
[ .
P
3. Ako e m – 3 = 0, t.e. m = 3, toga{ poslednata neravenka go dobiva vidot 0 · x > 2. Vo takov
slu~aj taa ne e zadovolena ni za edna vrednost na x, t.e. nejzinoto mno`estvo re{enija e
prazno mno`estvo.

1. Za koi vrednosti na x dropkata   [ }e bide pozitivna?



2. Za koi vrednosti na parametarot k re{enieto na ravenkata 2(3 – x) = 3(k – 2x) e:
a) pozitiven broj, b) nula, v) negativen broj?
3. Za koi vrednosti na parametarot m re{enieto na ravenkata
a) 2x – m = m –2, b) 5m = 2x – m, v) x + 3m = 9 e pogolemo od –3?
D  [
4. Re{i gi slednive neravenki so parametri: a)2 (m – x) < mx +5, b)   D .
 
5. Re{i ja neravenkata so parametar: a) 1 + kx < 2k – 3x, b) cx + c2 < 2x – 6.

76
V SISTEM LINEARNI NERAVENKI
SO EDNA NEPOZNATA
II.16. RE[ENIE NA SISTEM LINEARNI
NERAVENKI SO EDNA NEPOZNATA

Mno`estvo od dve ili nekolku linearni neravenki so edna ista nepoznata, za koi se
baraat zaedni~ki re{enija, se vika sistem linearni neravenki so edna nepoznata.
Primer. Ako promenlivata x treba, istovremeno, da gi zadovoluva uslovite:
x > –3, x > 1 i x > 5, toga{ ovie tri neravenki, velime, obrazuvaat sistem linearni
neravenki so edna nepoznata.
Re{enie na ovoj sistem linearni neravenki e sekoja vrednost na x, za koja i trite
neravenki od sistemot se zadovoleni. Ne e te{ko da zaklu~ime, deka re{enie na gorniot
sistem }e bide sekoj realen broj pogolem od 5, a mno`estvoto re{enija – intervalot
M = (5, f ).

Zatoa mo‘eme da ka‘eme deka:

Daden broj x e re{enie na eden sistem linearni ravenki so edna nepoznata, ako ja
zadovoluva sekoja od neravenkite od dadeniot sistem. Mno‘estvoto od site takvi
vrednosti x se narekuva mno‘estvo re{enija na dadeniot sistem.

­ [   ! 
°
Na primer, brojot 3 e re{enie na sistemot ® [   !  , bidej}i 2×3-1=5>4, -3+5=2>0 i
°[  
¯
3×3=9<10. Me|utoa, brojot 0 ne e re{enie, bidej}i prvata neravenka ne e zadovolena, bez
razlika dali ostanatite dve neravenki se zadovoleni.
Daden sistem linearni neravenki mo‘e i da nema re{enie. Takov e, na primer,
­[  
sistemot ® [ !  , bidej}i ne postoi broj koj, istovremeno, e pogolem od 5, a e pomal od 1.
¯

1. Koga eden broj e re{enie na daden sistem linearni neravenki so edna nepoznata?
2. Navedi primer na sistem linearni neravenki so edna nepoznata koj nema re{enie.
3. Navedi primer na sistem linearni neravenki so edna nepoznata za koj edno re{enie
e brojot -1, a brojot 2 ne e re{enie na toj sistem.

77
II.17. RE[AVAWE NA SISTEM LINEARNI
NERAVENKI SO EDNA NEPOZNATA

Za da se re{i daden sistem od dve ili nekolku linearni neravenki so edna nepoznata,
prvo, treba da se re{i oddelno sekoja neravenka od sistemot, a potoa da se izdvoi zaedni~-
kiot del (presekot) od mno`estvata na nivnite re{enija. Ako takov zaedni~ki del ne
postoi, t.e. ako toj e prazno mno`estvo, sistemot na neravenki nema re{enie, ili
sistemot e protivre~en.
­ [   !   [
Primer 1. Da se re{i sistemot neravenki ® [   !  .
¯
Re{enie. Lesno nao|ame deka prvata neravenka e zadovolena za x > 3, a vtorata za x < 8.
Zna~i, re{enija na sistemot se site realni broevi pome|u 3 i 8.
Odreduvaweto na mno`estvoto re{enija na sistemot, zna~itelno se olesnuva, ako
mno`estvata na re{enijata na sekoja oddelna neravenka od sistemot bidat pretstaveni
na ista brojna oska. Toa go izveduvame, taka {to gi precrtuvame onie delovi od brojnata
oska na koi le`at to~kite za koi sekoja oddelna neravenka ne e zadovolena. Vo takov
slu~aj, neprecrtaniot del od brojnata oska (ako ima takov) }e ni go dade mno`estvoto
re{enija na dadeniot sistem neravenki. Kako {to gledame, na{iot sistem neravenki
ima mno`estvo re{enija odredeni so 3 < x < 8 (crt. 6), ili re{enija na sistemot se site
realni broevi od intervalot M =( 3, 8).

Primer 2. Za koi vrednosti na


parametarot k re{enieto na raven-
kata 3x – k = 2 – x }e bide pozitivno i
Crte` 6
pomalo od 5?
N 
Re{enie. Otkako }e ja re{ime dadenata neravenka, dobivame [ . Spored uslo-

N
vot na zada~ata, nejzinoto re{enie treba da ja zadovoluva dvojnata neravenka   ,


koja, pak, e ekvivalentna na sistemot neravenki:

­N  
°°  ! 
®
°N    
°̄ 

78
Prvata neravenka vo dobieniot sistem e zadovolena za k > –2, a vtorata za k < 18.
Zna~i, sistemot neravenki }e ima re{enija odredeni so intervalot –2 < k < 18.
Ottuka zaklu~uvame, re{enieto na dadenata ravenka }e bide pozitivno i pomalo od 5,
koga parametarot k zema vrednosti od intervalot (–2, 18), t.e. koga e ispolnet uslovot
–2 < k < 18.

Re{i gi sistemite linearni neravenki:

­ [   ! [   ­   [   [  
1. a) ® b) ®
¯ [    [   , ¯  [ ! [   .

­[ ­ [     !   [
°  [  °
2. a) ®  b) ® [      
°  [   [   , °
¯ ¯ [ !  [   .

­
° [   [   ­  [   [ [
° °°    !   
3. a) ®  [   ! [   b) ®
° [  °[  [   !   [  
°  [   °̄   .
¯  ,

4. Ispitaj za koi vrednosti na parametarot m ravenkata x –2 = 3(x –m) ima


a) pozitivno re{enie, b) negativno re{enie, v) re{enie x= 0.
5. Za koi vrednosti na parametarot p izrazite a = 2p + 3, b = 4p – 5 i c = p + 13 }e mo`at da
bidat merni broevi na stranite na eden triagolnik?

79
G LINEARNA FUNKCIJA

II. 18. LINEARNA FUNKCIJA

Da go razgledame sledniov:
Primer. Po~etnata temperatura na vodata, {to e stavena vo lonec za zagrevawe, e 7°C.
Sekoja minuta temperaturata na vodata se poka~uvala za 3°C. Kolkava }e bide
temperaturata na vodata vo lonecot po x minuti?
Re{enie: Ako za 1 minuta zagrevawe temperaturata na vodata se poka~uva za 3°C, toga{
za x minuti zagrevawe taa }e se poka~i za (3x)°C. O~igledno e deka vrednosta na
temperaturata y (vo Celziusovi stepeni) na vodata zavisno od vremeto x (vo minuti)
mo`e da se odredi po formulata y = 7 + 3x.
Na sekoja vrednost na x i soodvetstvuva edinstvena vrednost na y. Zna~i, so formulata
y = 7 + 3x e zadadena funkcija.
Zakonot na poka~uvaweto na temperaturata na vodata, {to e izrazen so gornata for-
mula, e to~en samo za ograni~en vremenski interval, bidej}i znaete deka najvisokata
temperatura {to mo`e da ja dostigne vodata pri zagrevawe e 100°C.
Sored toa, funkcijata {to e zadadena so formulata y = 3x+ 7 }e ima definiciona
oblast (domen) D = [ 0; 31].
Odgovorot na zada~ata }e glasi: Baranata temperatura y na vodata }e bide: y = 3x+ 7,
kade {to [  > @ .
Vo praktikata, fizikata i matematikata ~esto se sre}avame so funkcii, {to se
zadadeni so formulata od vidot
y = ax + b,
kade {to x i y se promenlivi, a a i b dadeni fiksni broevi.

Definicija 1. Funkcijata I  5 o 5 {to e zadadena so formulata od vidot


\ D[  E , kade h i u se promenlivi, a a i b - dadeni broevi, se vika linearna funkcija.

Promenlivata x se vika nezavisno promenliva (ili argument), a y - zavisno promen-


liva (ili vrednost na funkcijata). Broevite a i b mo`at da bidat koi bilo realni broe-
vi (vklu~uvaj}i ja i nulata) i se vikaat parametri na linearnata funkcija. Brojot a u{te
se vika i koeficient pred argumentot x, a b - sloboden ~len.
Ako se poznati parametrite a i b, linearnata funkcija e ednozna~no opredelena.

80
Ako linearnata funkcija ne e svrzana so nekoj konkreten problem (kakov {to e slu~ajot
vo razgledaniot primer), toga{ nejzinata definiciona oblast (domen) }e bide
mno`estvoto R na site realni broevi, bidej}i izrazot ax + b ima smisla za sekoj realen
broj x.
Zatoa, obi~no, domenot na linearnata funkcija ne go soop{tuvame, a go podrazbirame;
i ~esto velime linearnata funkcija y = ax + b" namesto funkcija I  5 o 5 zadadena so
"
formulata y = ax + b".
Pri b = 0 funkcijata y = ax + b go dobiva vidot y = ax.
Funkcijata y = ax, kade {to [  5 e specijalen vid na op{tata linearna funkcija
y = ax+b za b = 0 i u{te se vika funkcija na pravata proporcionalnost.

[
1. Funkcijata I  5 o 5 zadadena e so formulata: a) y = – h + 5, b) y = x, v) \ ,

[
g) y = x2 – 3, d) \ . Koi od tie funkcii se linearni?

2. Dadena e funkcijata I so formulata y = 2x – 3, kade {to [  ^  ` .
Odredi go mno`estvoto na vrednostite na taa funkcija!
3. Funkciite I i J se zadadeni so tablicite:
a) b)
x –3 –1 0 2 7 x –5 –2 0 3 4

I (x) 0 2 3 5 10 J (x) 5 2 0 –3 –4

Poka`i deka I i J se linearni funkcii i istite izrazi gi analiti~ki!


4. Dol`inata na `elezna pra~ka pri 0°C e O P . Pri zagrevawe na sekoj 1°C dol`inata
na pra~kata se zgolemuva za 0, 00012 m, a pri ladewe na sekoj 1°C - taa se namaluva za
0, 00012m. Kolkava }e bide dol`inata O na pra~kata na temperatura W q& ? Izrazi ja
zavisnosta na pra~kata od temperaturata! Kakva funkcija e taa zavisnost?
5. Ednata strana na pravoagolnikot e dolga 3 cm, a drugata x cm. Na {to e ednakva
plo{tinata P na pravoagolnikot? Kakva funkcija e zavisnosta na P od h?
6. Eden col ima dol`ina 24, 5 mm. Izrazi ja zavisnosta na dol`inata (y) na otse~kite
(vo mm) od brojot x na colovite. Kakva funkcija e taa zavisnost?

81
7. Zasitenite jaglevodorodi (ili parafini) imaat op{ta formula CnH2n+2, kade {to n
e brojot na jaglerodnite atomi vo molekulot, pri {to Q d  . Izrazi ja zavisnosta
na vodorodnite atomi kako funkcija od brojot na jaglerodnite atomi vo molekulite
na zasitenite jaglevodorodi. [to e domen na taa funkcija?
8. Od 5 toni `elezna ruda se dobivaat 3 toni `elezo. Izrazi ja masata na dobienoto
`elezo (y) kako funkcija od masata na `eleznata ruda (x). Kakva e taa funkcija i
{to e nejzin domen?
9. Eden cm3 bakar ima masa 8,9 g. Izrazi ja masata (y) na edno par~e bakar kako funkcija
od negoviot volumen (x).
10. Eden avion se dvi`i ramnomerno pravoliniski so brzina od 250 km na ~as. Pretstavi
go izminatiot pat na avionot kako funkcija na vremeto! Kakva funkcija se dobiva?

II. 19. GRAFIK NA FUNKCIJATA y = ax


Vo funkcijata y = ax, ~ija definiciona oblast e mno`estvoto na site realni broevi,
parametarot a mo`e da bide pozitiven broj, nula, ili negativen broj.
Neka e dadena funkcijata y = 2x kade {to [  5 i a = 2 > 0. Za da go konstruirame
nejziniot grafikot, ja sostavuvame tablicata:

x –4 – 3,5 –3 – 2,5 – 2 –1 0 1 1, 5 2 3 3, 5 4

y –8 –7 –6 –5 –4 –2 0 2 3 4 6 7 8

Ako sekoj par soodvetni vrednosti na


promenlivite h i u od tablicata gi zememe za
koordinati na to~ka, }e gi dobieme to~kite:
(–4; –8), (–3,5; –7), (–3; –6), (–2,5; –5), (–2; –4),
(–1; –2), (0; 0), (1; 2), (1,5; 3), (2; 4), itn.
Gi konstruirame tie to~ki vo koordinatnata
ramnina (crt. 7). Zabele`uvame deka konstru-
iranite to~ki le`at na edna prava {to minuva
niz koordinatniot po~etok. Taa prava (crt. 8)
}e pretstavuva grafik na funkcijata y = 2x, kade
{to [  5 .
Na ist na~in, mo`eme da go konstruirame i
grafikot na funkcijata y = –2x, kade {to [  5 i a = –2
< 0. Toj grafik (crt. 9), isto taka, pretstavuva prava,
koja minuva niz koordinantniot po~etok.

82
Crte` 7
Crte` 9

Ako vo formulata y = ax e a = 0, toga{ grafikot


na funkcijata y = 0x (odnosno y = 0) vo mno`estvoto
na site realni broevi e samata x - oska, bidej}i
samo to~kite od nea imaat ordinata y = 0.
Spored toa, }e va`i slednava:

Crte` 8

Teorema: Grafikot na funkcijata {to e zadadena so formulata y = ax, [  5 e


prava, koja minuva niz koordinatniot po~etok.

Ako definicionata oblast (domenot) D na funkcijata y = ax e nekoe podmno`estvo od


5 ' Ž 5 , toga{ i nejziniot grafik }e pretstavuva, isto taka, nekoe podmno`estvo od
to~kite na pravata. Na primer: poluprava, otse~ka, ili oddelni to~ki.
Na crte` 10 pretstaveni se graficite na ~etiri funkcii:
 
\ [  \  [ \  [ i \  [.
 
Gledame: od vrednosta na parametarot a zavisi
agolot {to grafikot na funkcijata y = ax go zafa}a
so pozitivnata nasoka na x-oskata. Pri a > 0 toj
agol e ostar, a pri a < 0 toj e tap agol (crt. 10).
Zatoa parametarot a se vika u{te i aglov
koeficient na pravata, {to e grafik na funkcijata
y = ax.

Crte` 10

83

1. Nacrtaj gi graficite na funkciite: a) y = 3x, b) y = –3x, v) y = x, g) y = –x, d) \ [.


2. Niz koi kvadranti minuva grafikot na funkcijata y = ax, ako e: a) a > 0, b) a < 0?
3. Eden bazen se polni od edna cevka, koja dava 12 litri voda vo minuta. a) Izrazi go
koli~estvoto voda (y) vo bazenot kako funkcija od vremeto (x) na polneweto!
b) Dobienata funkcija pretstavi ja grafi~ki i od grafikot pro~itaj kolku litri

voda }e ima vo bazenot po  ~asa od po~etokot na negovoto polnewe?

4. Edno {i{e sobira 0,6 litri pivo. Izrazi go koli~estvoto pivo (y) vo litri, kako
funkcija od brojot na {i{iwata (x). [to e definiciona oblast na taa funkcija?
Funkcijata pretstavi ja grafi~ki!
5. Eden pe{ak minuva po 4 km na ~as. Izminatiot pat izrazi go kako funkcija od vremeto,
potoa dobienata funkcija pretstavi ja grafi~ki!
6. Od 100 kg bra{no se dobivaat 120 kg leb. Izrazi go koli~estvoto dobien leb (y) kako
funkcija od koli~estvoto bra{no (x).
7. Elektri~na struja jaka 1 amper od rastvorot na srebren nitrat oddeluva 1,118 mg
srebro za 1 sekunda. Izrazi go koli~estvoto na oddelenoto srebro (y) kako funkcija
od vremeto (x).

II. 20. GRAFIK NA FUNKCIJATA y = ax + b


Neka funkcijata e zadadena so formulata \ [   . Za da go konstruirame nejziniot

grafik, ja sostavuvame tablicata:

x –5 –4 –3 –2 –1 0 1 2 3 4 5
     
y 1 1 2 2 3 3 4 4 5 5
     

Potoa, vo kordinatnata ramnina gi konstruirame site to~ki, ~ii koordinati se


soodvetnite vrednosti na promenlivite x i y zemeni od tablicata.
Da vidime kade le`at taka konstruiranite to~ki na crte` 11.

84
Za taa cel vo istiot koordinaten sistem
da go nacrtame i grafikot na funkcijata
 , za koj znaeme deka e prava {to minuva
\ [

niz koordinatniot po~etok O(0, 0) i to~kata

A (2,1) (crt. 11).

Bidej}i koja bilo vrednost na izrazot



\ [   sekoga{ e za 3 pogolema od sood-

vetnata vrednost na izrazot  [ , zatoa

ordinatata na sekoja to~ka od grafikot na Crte` 11

funkcijata \ [   }e bide za 3 pogolema
 
od soodvetnata to~ka na grafikot na funkcijata \ [.


Znaeme, grafikot na funkcijata \ [ e

prava, zna~i i grafikot na funkcijata

\ [   e prava. Taa e paralelna na pravata


\ [ i le`i za 3 edinici nad nea vo nasoka

na u - oskata. (crt. 12)

Vo op{t slu~aj va`i slednava:

Crte` 12

Teorema: Grafikot na linearnata funkcija \ D[  E [  5 e prava.

Sli~no kako kaj funkcijata y = ax i tuka va`i: Ako definicionata oblast (domenot)
na linearnata funkcija y = ax + b ne e mno`estvoto 5 na site realni broevi, tuku nekoe
podmno`estvo od 5 , toga{ i nejziniot grafik, isto taka, }e bide nekoe podmno`estvo
od to~kite na pravata, na primer: poluprava, otse~ka, ili oddelni to~ki.
Bidej}i polo`bata na pravata vo ramninata e ednozna~no opredelena so dve razli~ni
to~ki, zatoa za crtawe na grafikot na linearnata funkcija dovolno e da se opredelat
koordinatite samo na dve to~ki od grafikot.

85
Nacrtaj gi graficite na slednite funkcii:
1. a) y = x+1, b) y = 2x-3.
 
2. a) y = -3x+0,5, b) y = x+ .
 

3. a) y = -2x+4, b) y = 4x-3.

II. 21. ZAEMNA POLO@BA NA GRAFICITE


NA NEKOI LINEARNI FUNKCII
Da vidime sega, kako od vrednostite na parametrite a i b zavisi polo`bata na grafi-
kot na funkcijata y = ax + b vo koordinatnata ramnina.
 
Na crte` 13 nacrtani se graficite na linearnite funkcii: \ [   \ [ i
 
 
\ [   . Gledame deka i trite funkcii imaat eden ist koeficient D , a se
 
razlikuvaat samo po slobodniot ~len b.

Nivnite grafici se tri paralelni pravi


koi ja se~at y- oskata soodvetno vo to~kite A(0;
3), O(0; 0) i B(0; –2), ~ii ordinati se ednakvi na
slobodniot ~len b vo formulata na soodvetnata
funkcija.
Zna~i, grafikot na linearnata funkcija
y = ax + b ja se~e y - oskata vo to~kata (0, b),
~ija ordinata e ednakva na b.
Na crte` 14 nacrtani se graficite na
funkciite:

Crte` 13
 
\ [   \  [   \  [   i \  [.
 
Gledame, nivnite grafici ja se~at y - oskata vo ista to~ka A (0, 2) (site imaat slobodni
~lenovi b = 2), a so pozitivnata nasoka na x-oskata zafa}aat razli~ni (ostri ili tapi)
agli.

86
Ako a > 0, grafikot na funkcijata

\ D[  E ( \ [i \  [   ) so pozitivnata

nasoka na h-oskata gradi ostar agol; a ako

D(\  [i \  [   ), toga{ toj agol e tap

(crt. 14).
Koeficientot a se vika aglov koeficient na
pravata, {to e grafik na funkcijata y = ax + b.
Ako a = 0, toga{ formulata na linearnata
Crte` 14
funkcija go dobiva vidot y = 0x + b, t.e. y = b. Toga{
na koja bilo vrednost na argumentot x soodvetstvuva edna ista vrednost na funkcijata
- ednakva na b. Vo takov slu~aj, grafikot na
funkcijata y = b }e bide prava {to e paralelna so
apcisnata oska, a }e ja se~e ordinatnata oska vo
to~ka, ~ija ordinata e ednakva na b. Na primer,
graficite na funkciite y = 4 i y = –3 nacrtani se na
crte` 15.
Ako a = 0 i b = 0, toga{ linearnata funkcija go
dobiva vidot y = 0x + 0, odnosno y = 0.
Vo takov slu~aj, za proizvolni vrednosti na
argumentot x, funkcijata sekoga{ dobiva vrednost
nula. Bidej}i, samo to~kite od apcisnata oska imaat
ordinati nula, zatoa grafikot na funkcijata y = 0,
Crte` 15
}e bide samata apscisna oska (crt. 15).

1. Vo ist koordinaten sistem nacrtaj gi graficite na funkciite:


y = 2x – 3 i y = 2x +1. Kakva zaemna polo`ba imaat graficite na tie funkcii?
2. Nacrtaj gi graficite na funkciite y = 2x – 4 i y = – x + 5 vo ist koordinaten sistem.
Opredeli gi koordinatite na prese~nata to~ka na graficite!
o
3. Nacrtaj go grafikot na funkcijata y=1,5x – 6. Odgovori na pra{awata: 1 . Za koi
vrednosti na x funkcijata dobiva: a) vrednost nula, b) pozitivni vrednosti,
v) negativni vrednosti? 2o. Kako se menuva promenlivata y (raste ili namaluva)
koga x raste? 3o Na koe mno`estvo se preslikuva intervalot [4; 8]?

87
4. Bez da go nacrta{ go grafikot na funkcijata y = –2x + 3 utvrdi koja od slednite to~ki
mu pripa|a na grafikot: A (0; 3), B (–1; 4), C (2, –1), '  .

5. Bez da go nacrta{ go grafikot na funkcijata y = –2x + 4 odredi vo koja to~ka grafikot


ja se~e: a) y - oskata, b) x - oskata.
6. Nacrtaj go grafikot na funkcijata y = –3x +6. Za koja vrednost na x izrazot –3x + 6
dobiva vrednost nula? Odredi go mno`estvoto vrednosti na x, za koi e:
a) –3x +6 > 0, b) –3x + 6 < 0.

II. 22. RASTEWE, OPA\AWE I NULA NA LINEARNA FUNKCIJA

Za brojnite funkcii }e vovedeme eden nov poim - nula na funkcija.


Neka funkcijata I  5o5 e zadadena so formulata \ I [ . Nuli na funkcijata f gi
vikame onie vrednosti na argumentot x za koi vrednosta na funkcijata e nula, t.e. x=a
e nula na funkcijata \ I [ , ako f (a )=0. Zna~i, nuli na funkcijata \ I [ se site
koreni na ravenkata I [ = 0.
E
Linearnata funkcija y = ax + b, a z 0, ima edinstvena nula [  koja e re{enie na
D
ravenkata ax + b z 0. Funkcijata na prava proporcionalnost y = ax, a z 0 ima isto taka
edinstvena nula i taa e x = 0.
Za brojnite funkcii isto taka, se voveduvaat, i poimite za rastewe i opa|awe na
edna funkcija. Me|utoa, ovie poimi nie }e gi definirame samo za linearnite funkcii.

Definicija. Za edna linearna funkcija y=f(x) velime deka e raste~ka, ako od uslovot
x1<x2 sleduva deka f(x1) < f(x2). Za edna linearna funkcija y=f(x) velime deka e opa|a~ka, ako od
uslovot x1<x2 sleduva deka f(x1) > f(x2).

Primer 1. Funkcijata y = 2x-3 e raste~ka, bidej}i od x1<x2 sleduva


f(x2) - f(x1) = (2x2-3) - (2x1-3) = 2x2-2x1 = 2(x2-x1) > 0,
odnosno f(x1) < f(x2).
Primer 2. Funkcijata y = -3x+1 e opa|a~ka, bidej}i od x1<x2 sleduva
f(x2) - f(x1) = (-3x2+1) - (-3x1+1) = -3x2+3x1 = 3(x1-x2) < 0,
odnosno f(x1) > f(x2).
Nacrtaj gi graficite na funkciite od primer 1 i 2. Od niv zabele`uva{ deka za
raste~kata funkcija y = 2x-3 koga x go zgolemuva{ toga{ i vrednosta na funkcijata }e se
zgolemi, dodeka za opa|a~kata funkcija y = -3x+1, koga x go zgolemuva{, toga{ vrednosta
na funkcijata se namaluva.

Teorema 1. Ako koeficientot pred x vo edna linearna funkcija e pozitiven


broj, toga{ funkcijata e raste~ka, a ako koeficientot pred x e negativen broj,
toga{ funkcijata e opa|a~ka.

88
Slu~ajot koga koeficientot pred x vo edna linearna funkcija e nula ne e opfaten so
prethodnata teorema. Za proizvolna vrednost na argumentot konstantnata funkcija prima
ista vrednost, t.e. taa e konstantna. Konstantnite funkcii ne se nitu raste~ki nitu
opa|a~ki, {to mo`e da se sogleda od definicijata za raste~ka odnosno opa|a~ka funkcija.

1. Najdi gi nulite na funkcijata a) y = 3x+4, b) y = -3x+2, v) y = 4x.


2. Dali se raste~ki ili opa|a~ki slednite funkcii
a) y = 7x+3, b) y = -3x, v) y = 5, g) y = -4x+1, d) y = x+7?
3. Doka`i ja teorema 1.

II. 23. GRAFI^KO RE[AVAWE NA LINEARNA RAVENKA


SO EDNA NEPOZNATA

Neka e dadena nekoja ravenka so edna nepoznata:


I [ J [ . (1)
Znaete, izrazite na dvete strani na ravenkata (1) se odredeni funkcii od nepoznatata
x: \ I [ i \ J [ .
Re{avaweto na ravenkite so edna nepoznata (1) mo`e da ja dobie slednava prosta
geometriska interpretacija:
Neka funkciite \ I [ i \ J [ se grafi~ki pretstaveni vo ist koordinaten
sistem xOy. Nivnite grafici mo`at da imaat edna ili pove}e zaedni~ki to~ki ili da
nemaat nitu edna zaedni~ka to~ka. Apscisite na zaedni~kite to~ki (ako ima takvi) na
graficite na dvete funkcii I [ i J [ se onie vrednosti na x za koi nivnite soodvetni
vrednosti se ednakvi. A tie vrednosti na x se tokmu baranite re{enija na ravenkata (1).
Vakviot na~in na odreduvawe na re{enijata na ravenkata (1) se vika nejzino grafi~ko
re{avawe.
Nie, tuka }e se zadr`ime samo na grafi~koto re{avawe na linearnite ravenki so
edna nepoznata.
Neka linearnata ravenka so edna nepoznata e dadena vo op{t vid
ax + b = 0. (2)
Gledame, levata strana na ravenkata (2) pretstavuva linearna funkcija y = ax+b, a,
isto taka, i desnata strana e linearna funkcija y = 0, ~ij grafik e x - oskata.
Spored toa, grafi~ki da se re{i ravenkata (2), zna~i, da se odredi apscisata na
presekot na pravata y = ax+b i x - oskata.
Ako a z 0, grafikot na funkcijata y = ax+b ja se~e x - oskata vo edna edinstvena to~ka so
E
apscisa [  . Taa apscisa, vsu{nost, e i nula na linearnata funkcija, bidej}i za taa
D
vrednost na x vrednosta na funkcijata e ednakva na nula.

89
Zada~a 1. Da se re{i grafi~ki ravenkata –2x + 3 = 0.

Re{enie: Go crtame grafikot na linearnata


funkcija y = –2x + 3. Bidej}i toj e prava, dovolno e da
konstruirame samo dve to~ki od nego. Da zememe,
na primer, to~ki so apscisi x = 0 i x = 2. Za x = 0 imame
y = –2 · 0 + 3 = 3, a za x = 2 imame y = –2 · 2 +3 = –1. Pravata {to
minuva niz to~kite A(0, 3) i B(2, –1) e grafik na
funkcijata y = –2x + 3 (crt.16).
Za da ja re{ime grafi~ki ravenkata –2x + 3 = 0, treba
da ja odredime apscisata na prese~nata to~ka na pravata
AB i x - oskata.
Crte` 16
Od crte`ot 16 gledame deka toa e to~kata S so

apscisa  . Toj broj e baranoto re{enie na dadenata

ravenka.
[
Zada~a 2: Da se re{i grafi~ki ravenkata  [.


Re{enie: Dadenata linearna ravenka mo`e grafi~ki


da se re{i i bez da se doveduva istata vo op{t vid.

Gledame, dvete nejzini strani se dve linearni funkcii


[
\   i y = 2 – x. Gi crtame nivnite grafici vo ist

koordinaten sistem xOy. Od crte`ot 17 gi odreduvame
koordinatite na prese~nata to~ka S na nacrtanite
grafici. Taa ima koordinati S(4, –2) i poka`uva deka za
x = 4 dvete funkcii dobivaat ednakva vrednost y = – 2.
Spored toa, baranoto re{enie na dadenata ravenka e
[  - apscisata na prese~nata to~ka S na dvata grafika. Crte` 17

Zada~a 3: Da se re{i grafi~ki ravenkata 2x + 3 = 2x.

Re{enie: Koga }e gi nacrtame graficite na funkciite y = 2x + 3 i y = 2x, zabele`uvame


deka tie se dve paralelni pravi i nemaat nitu edna zaedni~ka to~ka. Spored toa,
dadenata ravenka nema re{enie.

90
Re{i gi grafi~ki ravenkite:
[
1. a) 2x –3 = 0, b)  , v) x – 1 = 0.

[ 
2. a)     , b) [    , v) –x + 3 = 0.
 
[ 
3. a) 3x = –x +4, b)  [  .
 
4. a) 3x – 2 = 7, b) –2x + 5 = x.
[ [ [ [
5. a)  , b) .
   
§ ·
6. a) –2x + 3 = 2(1 – x), b) [    ¨ [  ¸ .
© ¹

ZA POVTORUVAWE
I UTVRDUVAWE - II

 
1. Proveri dali: a) x = 1 e re{enie na ravenkata  [  [  , b) x = –2 e re{enie na
ravenkata (x + 1) (x +2) = 0.  
 [
2. Re{i gi ravenkite: a) (x + 7) (x + 3) = x (x –11), b) [     ,
v) 3x – 14 – (5x – 8) = 9x – 25.  

3. Ispitaj dali se ekvivalentni ravenkite:


[ 
a)    [ i 5 – (x + 1) = 10x, b) x – 3 = 1 i x2 – 1 = 15.

4. Proizvodot na dva posledovatelni celi broja e za 26 pomal od proizvodot na narednite
dva posledovatelni broja. Koi se tie broevi?

5. Proveri za koi vrednosti na [  ^   ` neravenstvoto 3x – 5 > 6 – 2x


e zadovoleno.
6. Dadeni se intervalite A = (–3, 3), B = [ 0, 8], i C = (– f , 5). Odredi gi intervalite:
a) $ ˆ % , b) $ ‰ % , v) % ˆ & , g) % ‰ & .
[ [
7. Re{i gi neravenkite: a) 3x – 8 < 2 (x – 1) , b)  ! [ .
 
8. Smeni gi mestata na levata i desnata strana na neravenkata:
a) 5 –x > –2x + 1, b) 12 < – 4x, v) 2 > 2x.
9. Re{i gi sistemite linearni neravenki so edna nepoznata:
­  [ ! [ ­[  [   ­ [  
a) ® [    [ , b) ®   [ ! [   , v) ®  [ .
¯ ¯ ¯

91
10. Neka A = { a, b}, B = {1, 2, 3}. a) Odredi gi site funkcii od A vo B, b) site injekcii od A vo B.
11. Nacrtaj go grafikot na funkcijata y = 3x – 2. Od crte`ot odredi za koi vrednosti na
h: a) funkcijata ima vrednost nula, b) funkcijata dobiva vrednost 4.
12. Nacrtaj go grafikot na funkcijata y =–2x +1. Od grafikot odredi za koi vrednosti
na x funkcijata dobiva: a) pozitivni vrednosti, b) negativni vrednosti.
13. Ako na sekoja klupa vo eden del od parkot sedat po 4 deca, za 2 deca }e nema mesto.
No, ako na sekoja klupa sedat po 5 deca, toga{ edna klupa }e ostane prazna. Kolku
deca bile i kolku klupi imalo?
14. Doka`i deka poluperimetarot na triagolnik e pogolem od koja bilo strana na
triagolnikot.
15. Nacrtaj go grafikot na funkcijata y = 4 –x. So pomo{ na grafikot, re{i gi slednive
ravenki i neravenki: a) 4 – x = 0, b) 4 – x = 3, v) 4 – x > 0, g) 4 – x <0.
16. Funkcijata e zadadena so formulata y = –x + 2 na mno`estvoto D = [–2, 5]. Nacrtaj go
grafikot na funkcijata i odredi go mno`estvoto na nejzinite vrednosti.
17. Nacrtaj go grafikot na funkcijata y = 2x – 7. So pomo{ na grafikot utvrdi za kolku
edinici se zgolemuva vrednosta na y, ako vrednosta na x se zgolemi:
a) od 2 do 5 , b) od -3 do 1.
18. Selata A i B se oddale~eni edno od drugo 7 km. Eden selanec trgnal od seloto B vo
nasoka {to e sprotivna na nasokata kon A, i se dvi`el so brzina 4, 5 km na ~as.
a) Izrazi ja oddale~enosta (y) na selanecot od seloto A, kako funkcija na vremeto
(x), b) Nacrtaj go grafikot na dobienata funkcija.
19. Ako brojot na stepenite {to gi poka`uva termometarot so Celziusova skala, go
ozna~ime so x, a brojot na stepenite {to go poka`uva vo toj moment termometarot
so Farenhajtovata skala, go ozna~ime so y, toga{ zavisnosta pome|u tie dva broja

mo`e da se izrazi so slednava formula: \ [   . Grafi~ki pretstavi ja taa

funkcija i od grafikot opredeli: kolku stepeni }e poka`uva termometarot so
Farenhajtovata skala, ako vo toj moment termometarot na Celziusovata skala
poka`uva x = 15°C. Opredeli ja temperaturata na Celziusovata skala, koga
Farenhajtoviot termometar poka`uva 0°F.
20. Edna kada vo koja ima 420 litri voda po~nuva da se prazni preku edna cevka, od koja
istekuvaat po 8 litri voda vo 1 minuta. Izrazi go koli~estvoto na voda vo kadata
kako funkcija od vremeto na prazneweto! Nacrtaj go grafikot na dobienata
funkcija! Od nacrtaniot grafik opredeli kolku litri voda }e ostanat vo kadata po
34 minuti. Pro~itaj od grafikot za koe vreme }e se isprazni kadata!

92
ZA SAMOKONTROLA - II

1. Kolku re{enija ima ravenkata x(x –3)=0 i koi se tie?

[ [ [   [
2. Re{i gi ravenkite: a)   , b)   ,
   
v) 4(y + 3) (3y – 2) + 3(1 – 4y) (y – 1) = 59.
3. Kakva ravenka se dobiva ako: a) kon dvete strani na dadena ravenka dodademe eden ist
broj, b) dvete strani na ravenkata gi pomno`ime so eden ist broj k¹0?


4. Od eden top platno otse~eni se od negovata dol`ina i ostanalo u{te 24 metri.

Kolku metri platno imalo vo topot?
5. Tatkoto sega e 4 pati postar od svojata }erka, a pred 3 godini toj bil 5 pati postar
od nea. Po kolku godini ima sega sekoj od niv?
6. Dadeno e edno neto~no brojno ravenstvo. Kakvo ravenstvo }e dobie{ (to~no ili
neto~no) ako dvete negovi strani gi pomno`i{ so: a) brojot nula, b) broj razli~en
od nula?
7. Dadeni se intervalite A = (–2, 3] i B = [0, 8). Odredi: a) ja nivnata unija, b) go nivniot
presek.
8. Re{i gi neravenkite: a) 4( x + 1) > 12, b) –3x + 2 < –5.
9. Re{i go sistemot linearni neravenki so edna nepoznata

­[ !  ­ [   ­ [ !  ­ [ ! 
a) ® b) ® v) ® g) ®
¯ [ !  , ¯[   , ¯[ !  , ¯[   .

10. Grafikot na funkcijata y = kx + b minuva niz to~kata A(–1, 3) i e paralelen so


pravata x – 3y = 5. Odredi gi koeficientite k i b.
11. Odredi gi koordinatite na to~kata vo koja grafikot na ravenkata 3x – 4y = 12 ja
se~e: a) x –oskata, b) y –oskata.
12. Za koja vrednost na koeficientot a, grafikot na funkcijata y = ax –3 }e minuva
niz to~kata A(–1, –5)? Potoa nacrtaj go grafikot.

93
94
A LINEARNI RAVENKI SO DVE NEPOZNATI

III. 1. EKVIVALENTNI LINEARNI RAVENKI


SO DVE NEPOZNATI
Poimot ekvivalentni ravenki se odnesuva i za ravenkite so dve nepoznati, imeno:

Dve linearni ravenki so dve nepoznati se ekvivalentni ako mno`estvata od


nivnite re{enija se sovpa|aat.

Da zabele`ime deka: za ravenkite so dve nepoznati va`at istite svojstva (teoremi)


{to va`at i za ravenkite so edna nepoznata.
Spored toa, ravenkite:
x – y = 3, x = y + 3, x – y – 3 = 0 se ekvivalentni.
Neka e dadena ravenkata 2x + y = 5. (1)
Edna od nepoznatite, vo nea, da ja izrazime so pomo{ na drugata nepoznata, na primer,
nepoznatata y so pomo{ na x.
Ako ~lenot 2x go preneseme od levata na desnata strana, ja dobivame ravenkata
y = 5 –2x, (1')
koja e ekvivalentna na ravenkata (1).

Ako nepoznatata x ja izrazime so pomo{ na nepoznatata y dobivame ravenka



x =  y+2,5

koja, isto taka, e ekvivalentna na ravenkata (1).
Vo op{t slu~aj ako vo ravenkata ax+by=c e a¹0, toga{ mo‘eme da ja dovedeme do
ekvivalentna ravenka od oblik
x = ky+n, (2)

95
a ako b¹0, toga{ mo‘eme da ja dovedeme do ekvivalentna ravenka od oblik
y = kx+n. (3)
Sega se nametnuva slednoto pra{awe. Ako se dadeni dve ravenki od ist oblik, na
primer, (2), (3) ili, pak, ax+by=c, kako }e znaeme dali tie se ekvivalentni ili ne se
ekvivalentni? Odgovorot ni go davaat slednive teoremi.

Teorema 1. Ravenkite y = kx+n i y = k1x+n1 se ekvivalentni samo ako k=k1 i n=n1.


Dokaz. Da pretpostavime deka ravenkite y = kx+n i y = k1x+n1 se ekvivalentni.
Parot (0,n) e re{enie na prvata ravenka, pa mora da e re{enie i na vtorata ravenka.
Zna~i, mora da va`i n = k1×0+n1, t.e. n = n1. Parot (1,k+n) e re{enie na prvata ravenka, pa
mora da e re{enie i na vtorata ravenka. Zatoa k+n= k1+n1. Koristej}i deka n = n1 dobivame
deka i k = k1, i so toa dokazot e zavr{en.
Analogno se doka‘uva i slednava
Teorema 2. Ravenkite x = py+q i x = p1y+q1 se ekvivalentni samo ako p=p1 i q=q1.
Mo‘e da se doka‘e i slednava
Teorema 3. Ravenkite ax+by = c i a1x+b1y = c1 se ekvivalentni samo ako postoi broj k
takov {to a1=ka, b1=kb i c1=kc.

Pravite ax+by = c i (ka)x+(kb)y = kc imaat ist koeficient na pravec i ist sloboden ~len,
pa zatoa tie se poklopuvaat. Teorema 3 tvrdi deka ako dvete pravi ax+by = c i a1x+b1y = c1 se
sovpa|aat, toga{ nivnite koeficienti se proporcionalni.

1. Koga dve linearni ravenki so dve nepoznati se ekvivalentni?


2. Najdi ~etiri re{enija na ravenkata:
a) 0x + 3y = 12, b) 2x + 0y = 6.
3. Proveri dali ravenkite se ekvivalentni:
a) x + y = 2 i 3x + 3y = 6, b) 2x + y = 8 i 2x + y + 2 = 10.
4. Izrazi ja nepoznatata y so pomo{ na x od ravenkata:
a) 2x + 3y = 5, b) 3x – y = 1, v) 2(x – y) = 3x + y.
5. Izrazi ja nepoznatata x so pomo{ na y od ravenkata:
a) 2x – y = 4, b) y = 3x – 5, v) x + y = 2x – y.
6. Doka`i ja teorema 2.
7. Doka`i ja teorema 3.

96
III. 2. LINEARNA RAVENKA
SO DVE NEPOZNATI
Neka se dadeni ravenkite so dve nepoznati:
[  \ [
 , (1)
 
3(x – 2) – 2(y – 5) = x – y, (2)

(x + 3)2 = x(x + 1) – 2(y – 5). (3)


Ako dvete strani na ravenkata (1) gi pomno`ime so brojot 6 (NZS za 3 i 2) }e ja
dobieme ravenkata 2(2x – y) = 3x – 6.
Potoa, po osloboduvawe od zagradata i grupirawe na ~lenovite {to sodr`at
nepoznati, na levata strana, a na drugite ~lenovi na desnata strana, ja dobivame
slednata ravenka:
x – 2y = –6, (1')
koja e ekvivalentna na ravenkata (1).
Na sli~en na~in i ravenkite (2) i (3) mo`eme da gi transformirame soodvetno vo
ravenkite:
2x – y = –4, (2')
5x + 2y = 1. (3')
Gledame, sekoja od dadenite ravenki (1), (2), (3) e ekvivalentna na nekoja ravenka
od vidot
ax + by = c, (4)
kade {to a, b i c se to~no opredeleni realni broevi.

Definicija. Sekoja ravenka so dve nepoznati x i y, koja mo`e da se transformira


vo ekvivalentna ravenka od vidot:
ax + by = c (4)
kade {to a, b i c se dadeni realni broevi, pri {to barem eden od broevite a i b e
razli~en od nula, se vika linearna ravenka so dve nepoznati.

Broevite a i b se vikaat koeficienti pred nepoznatite, a c – sloboden ~len.


Spored usvoenata definicija, ravenkata 0x + 0y = c ne se smata za linearna, bidej}i
i a = 0 i b = 0.

Re{enie na edna linearna ravenka so dve nepoznati, kako i kaj sekoja ravenka so
dve nepoznati, se vika sekoj par broevi (x0, y0) za koj linearnata ravenka preminuva
vo to~no brojno ravenstvo.

97
Znaete deka: sekoj podreden par broevi [  \ geometriski mo`e da se pretstavi so
edna to~ka na koordinatnata ramnina. Spored toa, sekoe re{enie na linearnata ravenka
(4) geometriski mo`e da se interpretira so edna to~ka od ramninata xOy, a mno`estvoto
re{enija na ravenkata (4) – so nekoe mno`estvo to~ki od istata ramnina.
Mno`estvoto na site to~ki vo koordinatnata ramnina, ~ii koordinati (smetani kako
podredeni parovi broevi) se re{enija na ravenkata (4), se vika grafik na linearnata
ravenka so dve nepoznati.
]e poka`eme deka va`i slednava:

Teorema: Grafikot na sekoja linearna ravenka so dve nepoznati e prava.

Dokaz. Linearnata ravenka so dve nepoznati ax + by = c pri b z 0 mo`e da se transformira


vo vidot
D F
\  [ . (4')
E E
Toa go postignuvame koga ~lenot ax }e go preneseme na desnata strana so sprotiven
znak, a potoa dvete strani na ravenkata }e gi podelime so brojot b z 0.
Koga ravenkata (4) ja transformirame vo vidot (4') velime deka nepoznatata y ja
izrazuvame so pomo{ na x, ili kako funkcija od x.
Znaeme deka, funkcijata {to e zadadena so formulata (ravenkata) (4') e linearna
funkcija, ~ij grafik e prava. Taa prava e grafik i na linearnata ravenka (4).
Sega da pretpostavime deka b = 0, no a z 0. Toga{ ravenkata (4) go dobiva vidot:
ax + 0 · y = c . (5)
F
Pri a z 0 imame [ . Zna~i, ravenkata (5) preminuva vo to~no brojno ravenstvo za
D
F §F · §F · §F · §F ·
sekoj par broevi, vo koj [ , a y – proizvolen broj, na primer, ¨  ¸  ¨   ¸  ¨  ¸  ¨ ¸ itn.
D ©D ¹ ©D ¹ ©D ¹ ©D ¹

Spored toa, grafik na taa ravenka e


mno`estvoto to~ki na koi apcisata im e
F
ednakva na , a ordinatata – proizvolen
D
realen broj. Mno`estvoto od takvite to~ki
obrazuva prava {to minuva niz to~kata
§F ·
0 ¨  ¸ i e paralelna na u –oskata.
©D ¹
Na primer, na crt. 1 e nacrtan grafikot
na linearnata ravenka 2x + 0y = 6.
Crte` 1

98
Zna~i, i pri b = 0, no a z 0 grafikot na linearnata
ravenka (4), isto taka, e prava.
Bidej}i za konstrukcijata na pravata e dovolno
da znaeme najmalku dve nejzini to~ki, zatoa
grafikot na linearnata ravenka mo`eme da go
nacrtame i samo so pomo{ na dve to~ki.

Primer. Da se nacrta grafikot na ravenkata


4x – 2y = 9.

Re{enie. Za x = 0 dobivame –2y = 9 ili \  , a


za y = 0 dobivame 4x = 9 ili [  . Zna~i, parovite

§  · §  ·
broevi ¨   ¸ i ¨   ¸ se dve re{enija na
© ¹ ©  ¹ Crte` 2

dadenata ravenka. Gi konstruirame to~kite


§ · §  ·
$ ¨   ¸ i % ¨   ¸ i niz niv povlekuvame prava. Taa prava e grafik na ravenkata
© ¹ ©  ¹
4x – 2y = 9 (crt. 2).

[   \ 
1. Uprosti gi ravenkite: a) 3(x – 1) – 5(y + 3) = 2x – y, b)  [  \ .
 
2. Najdi nekolku re{enija na ravenkata: a) x – 4y = 7, b) 3x + y = 4.
3. Dadena e ravenkata 2x – 5y = 9. Najdi go ona re{enie, za koe:
a) x = 0, b) y = 0, v) x = –1, g) y = 2.
4. Nacrtaj gi graficite na ravenkite: a) x + 2y = 6, b) x – y = 3, v) 3x – y + 1 = 0.
5. Nacrtaj go grafikot na ravenkata: a) 0x + 2y = 5, b) 3x – 0y = 12, v) 2x + 0y = 0.
6. Nacrtaj go grafikot na ravenkata 2x – y = 5 i od nego odredi gi re{enijata na
ravenkata, za koi: a) x = 0, b) y = 0, v) x = 3, g) y = –1.
7. Na grafikot na ravenkata x + 3y = –2 le`i to~ka M so apcisa –5. Odredi ja
ordinatata na taa to~ka!
8. Vo ravenkata 3x +by = 5 odredi go koeficientot b, taka {to grafikot na
ravenkata da minuva niz to~kata C(–1, 2).
9. Za koja vrednost na c grafikot na ravenkata 2x – y = c }e minuva niz to~kata
T(–1, 5)?
10. Ravenkite 2x – 4y = 6 i x – 2y = 3 se ekvivalentni. [to mo`eme da ka`eme za nivnite
grafici?

99
B SISTEM OD DVE LINEARNI RAVENKI
SO DVE NEPOZNATI
III. 3. SISTEM OD DVE LINEARNI RAVENKI
SO DVE NEPOZNATI
Da ja razgledame slednava zada~a:
Zada~a: Zbirot na dva broja e ednakov na 11, a razlikata na prviot so vtoriot e 3.
Koi se tie broevi?
Nepoznatite broevi da gi ozna~ime so bukvite x i y. Toga{, uslovite {to treba da gi
zadovoluvaat broevite x i y lesno gi izrazuvame so ravenkite
x + y = 11 i x – y = 3. (1)
Znaeme, sekoja od tie dve ravenki ima beskone~no mnogu re{enija, no re{enie na
na{ata zada~a }e bide samo onoj par broevi, za koj i dvete ravenki preminuvaat vo to~ni
brojni ravenstva.
Da go pobarame toj par broevi.
Za taa cel }e najdeme nekoi od re{enijata na sekoja od dvete ravenki.
Za prvata ravenka x + y = 11, odnosno y = 11 – x eve nekoi od niv:

x –1 0 1 2 3 4 5 6 7 8 9 ...

y = 11 – x 12 11 10 9 8 7 6 5 4 3 2 ...

i za vtorata ravenka x – y = 3, t.e. y = x – 3.

x –1 0 1 2 3 4 5 6 7 8 ...

y=x–3 –4 –3 –2 –1 0 1 2 3 4 5 ...

Od tablicite gledame deka dvete ravenki imaat samo edno zaedni~ko re{enie
x = 7, y = 4. Spored toa, baranite broevi vo zada~ata se 7 i 4.
Navistina 7 + 4 = 11 i 7 – 4 = 3.
Ponatamu, }e doka`eme deka ravenkite (1) ne mo`at da imaat drugi zaedni~ki re{enija.
Postojat mnogu zada~i ~ie re{avawe se sveduva na odreduvawe na zaedni~kite re{enija
na dve ravenki so dve nepoznati.
Ako pred nas e postavena zada~a da gi odredime zaedni~kite re{enija na dve ravenki
so edni isti nepoznati, toga{ velime deka tie dve ravenki obrazuvaat sistem ravenki so
dve nepoznati.
Za da ozna~ime deka ravenkite (1) obrazuvaat sistem ravenki gi pi{uvame edna pod
druga i od levata strana postavuvame golema zagrada, na primer:
­ [  \ 
® (1')
¯[  \  .
100
Sistemot ravenki so dve nepoznati simboli~ki go ozna~uvame:
­° I [ \ 
® (2)
°̄ J [ \ 
kade {to I [ \ i J [ \ se dadeni izrazi so dve promenlivi (nepoznati) [ i \ .

Definicija. Sekoj par vrednosti na nepoznatite (x0, y0), za koj i dvete ravenki na
sistemot (2) preminuvaat vo to~ni brojni ravenstva, se vika re{enie na sistemot
ravenki.
Na primer, najdovme deka sistemot ravenki (1') ima edno re{enie: (7, 4).
Da se re{i sistemot ravenki, zna~i, da se odredi mno`estvoto od negovite re{enija.
Mno`estvoto re{enija na ednata ravenka na sistemot (2) da go ozna~ime so M1,
a mno`estvoto re{enija na drugata ravenka – so M2. Toga{ mno`estvoto M od zaedni~kite
elementi na M1 i M2, }e pretstavuva presek od M1 i M2, t. e. 0 0  ˆ 0  .
Ako i dvete ravenki na sistemot (2) se linearni ravenki so dve nepoznati, toga{ toj
u{te se vika i sistem linearni ravenki so dve nepoznati.
Sekoj sistem linearni ravenki so dve nepoznati mo`e da se dovede vo op{t vid:
­D [  E \ F
® (3)
¯D [  E \ F
kade a1, b1, c1, a2, b2, c2 se dadeni broevi.
Broevite a1, b1, a2 i b2 se vikaat koeficienti pred nepoznatite, a c1i c2– slobodni
~lenovi na sistemot (3).

1. [to e re{enie na sistem ravenki?


2. Koj e potreben i dovolen uslov parot (x0, y0) da e re{enie na sistemot ravenki
­° I [ \ 
®
°̄ J [ \  ?
3. Proveri dali parovite broevi: (–3, 2), (3, 9), (0, –2) i (–1, 1) se re{enija na sistemot
­[   \ 
ravenki ®
¯  \  [ D[ . E \ 
­  
4. Sistemot linearni ravenki od oblikot ®D [  E \  se vika homogen sistem. Poka`i
¯  
deka homogeniot sistem sekoga{ ima barem edno re{enie. Koe e toa re{enie?
­[   \ 
5. Ako znae{ deka parot (1, 2) e re{enie na sistemot ravenki ®  [  \  , najdi edno
¯
­ \  [ 
re{enie za sistemot ®
¯ \  [  .
(Upatstvo. Na koj na~in e dobien noviot od po~etniot sistem?)

101
III. 4. GRAFI^KO RE[AVAWE
NA SISTEM LINEARNI RAVENKI
SO DVE NEPOZNATI

Vidovme deka grafikot na sekoja linearna ravenka so dve nepoznati


ax + by = c, (1)
kade {to barem eden od koeficientite a i b e razli~en od nula, e nekoja prava p.
Neka e daden sistemot ravenki
­[  \ 
® (2)
¯ [   \ 
Da gi konstruirame graficite na dvete ravenki vo sistemot (2) vo ist koordinaten
sistem (crt. 3). Gledame deka grafik na prvata ravenka e pravata p {to minuva niz to~kite
§ ·
$ ¨  ¸ i B(7, 0) a grafik na vtorata
© ¹
ravenka e pravata q {to minuva niz
to~kite C(0, –2) i D(4, 4).
Koordinatite na to~kite od pravata
p ni go davaat mno`estvoto na site
re{enija na ravenkata x + 5y = 7, a
koordinatite na to~kite na pravata q ni
go davaat mno`estvoto na site re{enija
na vtorata ravenka 3x – 2y = 4 vo sistemot
(2). Spored toa, koordinatite na
Crte` 3
zaedni~kata to~ka (S) na dvete pravi p i q
}e ni go dadat zaedni~koto re{enie na
dvete ravenki, odnosno re{enieto na dadeniot sistem ravenki (2).
Od crte`ot 3 gledame: pravite p i q imaat samo edna zaedni~ka to~ka S, ~ii koordinati
se x = 2, y = 1.
Taka nao|ame deka dadeniot sistem linearni ravenki (2) ima edinstveno re{enie –
parot broevi (2, 1).
Vakviot na~in na re{avawe na sistemite linearni ravenki so dve nepoznati se vika
nivno grafi~ko re{avawe.
Kako {to gledame grafi~koto re{avawe na sistem linearni ravenki so dve nepoznati
se sostoi vo odreduvawe na koordinatite na zaedni~kite to~ki na dve pravi – grafici na
ravenkite vo sistemot, nacrtani vo ist koordinaten sistem.
Znaete deka dve pravi vo ramninata mo`at ili da se se~at, ili da se paralelni, ili da
se sovpa|aat (specijalen slu~aj na paralelnost). Kaj razgledaniot sistem (2) graficite
na ravenkite bea pravi {to se se~at vo to~kata S(2, 1) (crt. 3). Vidovme, sistemot (2) ima
edinstveno re{enie (2, 1).

102
Da gi razgledame i preostanatite dva slu~aja.
Primer 1. Da se re{i sistemot ravenki:
­ [  \ 
® (3)
¯ [   \ 
Ako od ravenkite nepoznatata y ja izrazime kako funkcija od x, go dobivame sistemot:
­ \ [  
® (3')
¯ \ [   
Gledame, pravite {to se grafici na ravenkite vo
sistemot (3) imaat ednakvi aglovi koeficienti, a razli~ni
slobodni ~lenovi. Toa zna~i, deka tie se paralelni i ja
se~at y -oskata vo to~kite (0, 5) i (0, 2) (crt. 4). Spored toa,
pravite y = 3x + 5 i y = 3x + 2 nemaat zaedni~ki to~ki. Zatoa,
sistemot (3'), odnosno (3) nema re{enie.
Deka dadeniot sistem ravenki (3) nema re{enie,
mo`eme da se uverime i na ovoj na~in: Ako dvete strani na
vtorata ravenka vo sistemot (3) gi podelime so 2, go
dobivame sistemot
­ [  \ 
® (3'')
¯ [  \  
Crte` 4
Ovoj sistem nema re{enie, bidej}i izrazot 3x – y za edni isti vrednosti na x i y ne
mo`e da dobie razli~ni vrednosti –5 i –2. Spored toa i dadeniot sistem ravenki nema
re{enie.
Primer 2. Da se re{i sistemot ravenki
­[   \ 
® (4)
¯ [   \  
Ako dvete strani na vtorata ravenka gi podelime so 3, dadeniot sistem }e go dobie
vidot:

­[   \ 
® (4')
¯[   \ 

Gledame, sistemot se sostoi od dve


ednakvi ravenki. Zatoa, nivnite grafici se
sovpa|aat (crt. 5). Koordinatite na koja
bilo to~ka od nivniot zaedni~ki grafik }e
bide re{enie na sistemot (4). Zna~i,
dadeniot sistem ravenki (4) ima beskone~no
mno`estvo re{enija. Crte` 5

103
Od prethodnite razgleduvawa sleduva deka:

1o. Ako pravite - graficite na dadenite ravenki, se se~at: sistemot ima edinstveno
re{enie.
2o.Ako pravite se paralelni i razli~ni, toga{ sistemot nema nitu edno re{enie, i
3o. Ako pravite se sovpa|aat, toga{ sistemot ima beskone~no mnogu re{enija.

1. Re{i gi grafi~ki sistemite linearni ravenki:


­[   \  ­\ [ ­[   \  
a) ® , b) ® , v) ® .
¯[  \  ¯\  [ ¯  [  \ 
2. So pomo{ na grafici utvrdi koj od slednive sistemi ravenki: ima edno re{enie,
ima beskone~no mnogu re{enija, nema re{enija:

­ [   \  ­[   \  ­[  \ 
a) ® , b) ® , v) ® .
¯[   \  ¯ [   \  ¯  [   \ 

3. Proveri dali pravata 3x – y = 5 minuva niz presekot na pravite:

x + 2y = –3 i 2x + y = 0.

4. Ispitaj grafi~ki dali sistemite od tri linearni ravenki so dve nepoznati imaat
­[  \  ­ [  \ 
° °
[  \  [ \ 
re{enie: a) ® b) ®
°[  \  , ° [   \  .
¯ ¯
5. Sostavi sistem linearni ravenki so dve nepoznati, koj:
a) ima edno re{enie, b) nema re{enie, v) ima beskone~no mnogu re{enija!
­ D[   \ 
6. Za koi vrednosti na a i b sistemot ravenki ® }e ima re{enie (3, –1)?
¯ [  E\ 
7. Napi{i koj bilo sistem linearni ravenki koj }e ima re{enie (–1, 3).
8. Neka e daden sistem linearni ravenki. Dali re{enieto na toj sistem }e se promeni,
ako:
a)prvata ravenka ja pomno`i{ so nekoj broj k z 0,
b) vtorata ravenka ja pomno`i{ so nekoj broj k z 0?

104
III. 5. EKVIVALENTNI SISTEMI

Analogno na poimot ekvivalentni ravenki (vidi II.3) go voveduvame i poimot ekviva-


lentni sistemi ravenki.

Definicija. Dva sistema ravenki se vikaat ekvivalentni, ako mno`estvata na


nivnite re{enija se sovpa|aat.
So drugi zborovi zna~i: Dva sistema ravenki se ekvivalentni, ako sekoe re{enie na
edniot sistem e re{enie i na vtoriot sistem, i obratno, ako sekoe re{enie na vtoriot
sistem e re{enie i na prviot sistem.

Obi~no, i za dva sistema ravenki koi nemaat re{enija velime deka se ekvivalentni.
Pri re{avaweto na sistemite ravenki, kako i pri re{avaweto na ravenkite,
neophodno e da znaeme koi transformacii mo`eme da gi vr{ime so ravenkite na daden
sistem, a pritoa da bideme sigurni deka noviot sistem ravenki e ekvivalenten na dadeniot
sistem.
Transformaciite {to doveduvaat do ekvivalentni sistemi ravenki gi vr{ime vrz
osnova na tri va`ni svojstva (teoremi) na istite ravenki. Tie svojstva tuka }e gi dademe
bez dokaz.

Teorema 1. Ako koja bilo od ravenkite na sistemot ja zamenime so ekvivalentna na


nea ravenka, }e dobieme ekvivalenten sistem.

Na primer: Sistemite ravenki


­[ \
°   ­  [  \ 
®  i ®
°¯ [   \  ¯ [   \ 
se ekvivalentni, bidej}i prvite ravenki im se ekvivalentni, a vtorite ravenki im se
ednakvi.
[ \
Ravenkata 2x – y = 16 ja dobivame koga dvete strani na ravenkata   gi pomno`ime
 
so brojot 4.

Teorema 2. Od koja bilo ravenka na daden sistem edna od nepoznatite mo`eme da ja


izrazime so pomo{ na drugata nepoznata i so dobieniot izraz da ja zamenime istata
nepoznata vo drugata ravenka. Toga{ novata ravenka zaedno so prvata ravenka }e
obrazuvaat sistem {to e ekvivalenten na dadeniot sistem.

­[   \ 
Neka e daden sistemot ravenki: ®[   \  . (1)
¯

105
Od prvata ravenka nepoznatata x da ja izrazime so pomo{ na y. Ja dobivame ravenkata
x = 1 – 2y. Potoa, ako vo vtorata ravenka na sistemot, nepoznatata x ja zamenime so izrazot
1 – 2y, }e ja dobieme ravenkata 3(1 – 2y) – 4y = 13, koja zaedno so ravenkata x = 1 – 2y }e obrazuva
nov sistem: ­° [    \
® (2)

    \   \  .
°̄
Vo sglasnost so teoremata 2 taka dobieniot sistem (2) e ekvivalenten na dadeniot
sistem (1). Sistemot (2) ima re{enie (3,–1). Uveri se deka toa e re{enie i na sistemot (1).

Teorema 3. Ako gi sobereme soodvetnite levi i desni strani, na dvete ravenki vo


sistemot, }e dobieme nova ravenka, koja zaedno so edna od dadenite }e obrazuva nov
sistem ekvivelenten na dadeniot.

­ [   \ 
Neka e daden sistemot ravenki: ® [  \  . (3)
¯
Da gi sobereme pooddelno levite i desnite strani na dvete ravenki na sistemot (3),
pri {to ja dobivame ravenkata 7x + y = 5. Vo soglasnost so teoremata 3, sekoj od sistemite
ravenki:
­ [   \  ­ [  \ 
® i ® (4)
¯ [  \  ¯ [  \ 
e ekvivalenten na dadeniot sistem (3).
Podocna so grafi~ko re{avawe na sistemite (3) i (4) lesno se uveruvame deka sekoj
od niv ima edinstveno re{enie (1, –2). Zna~i tie se ekvivalentni.

1. Dali slednive sistemi ravenki se ekvivalentni:

­ [   \  ­ [   \  ­ [   \  ­° [    [   
a) ® i ® , b) ® i ® ?
¯ [  \  ¯ \ [   ¯ \ [   °̄ \  [  
­ [   \ 
2. Poka`i deka sistemot ravenki ® e ekvivalenten na sekoj od slednive dva
¯ [   \ 
­ [   \  ­ [ 
sistema ravenki ® i ® .
¯  [  ¯ [   \ 
3. Primenuvaj}i transformacii vo soglasnost so teoremite 1, 2 i 3 najdi nekolku
­[  \ 
ekvivalentni sistemi na sistemot ® .
¯ [  \ 
­D [  E \ 
4. Dali sistemot ravenki ® e ekvivalenten so sistemot ravenki
¯D [  E \ 
­ S [  T \ 
® , kade a1, a2, b1, b2, p1, p2, q1,q2 se proizvolni broevi?
¯ S [  T \ 

106
(Upatstvo. Pogledni ja zada~ata 4 od III.3.)
­[   \ 
5. Obidi se da go re{i{ sistemot ®
¯ [  .
­  [   \ 
6. Obidi se da go re{i{ sistemot ®
¯ \  .

III. 6. RE[AVAWE NA SISTEM LINEARNI RAVENKI


SO DVE NEPOZNATI SO METOD NA ZAMENA

Postojat pove}e metodi za re{avawe na sistemite linearni ravenki so dve


nepoznati. Nie }e koristime samo dva metoda: metod na zamena i metod na
sprotivni koeficienti.
Prviot metod da go ilustrirame so primeri.
Primer 1. Da se re{i sistemot linearni ravenki:
­ [  \ 
® (1)
¯ [   \  .
Re{enie. Od prvata ravenka na sistemot, nepoznatata u }e da ja izrazime so pomo{
na x. Go dobivame ekvivalentniot sistem
­ \   [
® (1')
¯ [   \  .
Koga vo vtorata ravenka na sistemot (1') nepoznatata y ja zamenime so izrazot
5 – 3x, vo soglasnost so teoremata 2 dobivame nov sistem
­° \    [
® (1'')
°̄ [      [ 
{to e ekvivalenten na sistemite (1') i (1).
Gledame, vtorata ravenka na sistemot (1'') e so edna nepoznata. Koga }e ja re{ime,
nao|ame x = 1. Zamenuvaj}i ja najdenata vrednost na x vo prvata ravenka na sistemot (1''), ja
nao|ame vrednosta i na y: y = 5 – 3 · 1 = 5 – 3 = 2.
Zna~i sistemot ravenki (1'') ima re{enie x = 1, y = 2. Toa e re{enie i na dadeniot sistem.
Primer 2. Da se re{i sistemot ravenki:
­ [   \ 
® (2)
¯ [   \  .
Re{enie. Od vtorata ravenka da ja izrazime, na primer, nepoznatata x so pomo{ na y.
  \
Pritoa dobivame [ . Potoa dobieniot izraz za x go vnesuvame vo prvata ravenka

na sistemot (2). Taka go dobivame ekvivalentniot sistem:
­   \
°° ˜    \ 
®
°[    \ . (2')
°̄ 

107
Prvata ravenka na sistemot (2’) e so edna nepoznata i nejzinoto re{enie e \  .
Na krajot vrednosta \  ja zamenuvame vo vtorata ravenka na sistemot (2’), pa dobivame:
       
[  Spored toa, baranoto re{enie e [  \  .
  
Od razgleduvanite primeri gledame deka re{avaweto na sistemot linearni
ravenki so dve nepoznati po metodot na zamena se sostoi vo slednoto:
10. Od ednata (koja bilo) ravenka na dadeniot sistem edna od nepoznatite (na
primer y), ja izrazuvame so pomo{ na drugata nepoznata (x).
2o. Dobieniot izraz za nepoznatata y go zamenuvame vo drugata ravenka na sistemot.
Dobivame ravenka so edna nepoznata.
3o. Ja re{avame dobienata ravenka so edna nepoznata x i ja nao|ame vrednosta na taa
nepoznata.
4o. Ja zamenuvame najdenata vrednost x na nepoznatata vo dobieniot izraz za drugata
nepoznata y i ja nao|ame vrednosta i na taa nepoznata.
Taka, dobienite vrednosti na nepoznatite x i y se re{enie na dadeniot sistem.

1. Re{i gi slednive sistemi ravenki:

­  [   \  ­ [  \  ­ [  ­ [   \ 
® ® ® ®
a) ¯ [   \  , b) ¯ ˜ [   \  , v) ¯ [   \  , g) ¯ [   \  .

Po metodot na zamena re{i gi sistemite ravenki:


­ [  \  ­[  \  ­ [  \  ­[  \ 
2. a) ®  [   \  , b) ® \ [ . 3. a) ® [   \  , b) ®  [   \  .
¯ ¯ ¯ ¯

­ [   \  ­[  \  ­[  \  ­  [  \ 
4. a) ®[   \  , b) ® \ [   5. a) ®  [   \   b) ®  [  \  
¯ ¯ . ¯ ¯
6. Dovedi gi, prvo, sistemite ravenki do celi koeficienti, a potoa re{i gi po metodot

­[ \ ­[ \
°°   
 °°   

na zamena: a) ® b) ®
°[  \
 °[  \

°̄   °̄  
7. Dovedi gi, prvo, sistemite ravenki vo op{t vid, a potoa re{i gi:
­° [     \    ­°  [  \   \ 
a) ® [     \     b) ®
°̄ °̄ [     \  [  

108
III.7. RE[AVAWE NA SISTEM LINEARNI RAVENKI
SO DVE NEPOZNATI SO METOD NA SPROTIVNI KOEFICIENTI

Primer 1. Da se re{i sistemot ravenki:


­ [   \ 
® (1)
¯ [   \  
Re{enie. Zabele`uvame, koeficientite pred nepoznatata y vo dvete ravenki se
sprotivni broevi. Zatoa, ako gi sobereme soodvetnite levi i desni strani na dvete
ravenki vo sistemot, ~lenovite {to ja sodr`at nepoznatata y }e se poni{tat i }e dobieme
ravenka so edna nepoznata 7x = 14, t.e. x=2.
Vo soglasnost so teoremata 3 dobienata ravenka zaedno so edna (koja bilo) od ravenkite
na sistemot (1) }e obrazuva nov sistem
­ [ 
® (1')
¯ [   \  
ekvivalenten na dadeniot.
Ako dobienata vrednost na nepoznatata x ja zamenime vo vtorata ravenka na sistemot
(1'), }e ja dobieme vrednosta na drugata nepoznata y, t.e. od 2 · 2 + 3y = 1 dobivame y = –1.
Zna~i, sistemot (1') ima re{enie x = 2, y = –1. Toa e re{enie i na dadeniot sistem.

­° ˜    ˜   ­ 
Proverka: ® odnosno °®
°̄ ˜    ˜    °̄  
Primer 2. Da se re{i sistemot ravenki:
­ [   \ 
® (2)
¯ [   \  
Re{enie: Ako gi sobereme soodvetnite levi i desni strani na ravenkite vo sistemot
(2), pak, }e dobieme ravenka so dve nepoznati. Za da go re{ime ovoj sistem na na~in kako
prethodniot, istiot treba, prvo, da go zamenime so drug, ekvivalenten na nego, sistem,vo
koj koeficientite pred ista nepoznata da se sprotivni broevi. Toa mo`eme da go storime
ili so koeficientite pred nepoznatata x, ili so koeficientite pred nepoznatata y.
Na primer:
Ako dvete strani na prvata ravenka gi pomno`ime so 2, a dvete strani vo vtorata
ravenka – so 3, }e dobieme ekvivalenten sistem
­ [   \ 
® (2')
¯ [   \  
vo koj koeficientite pred nepoznatata y se sprotivni broevi.
Gi sobirame soodvetnite levi i desni strani na ravenkite vo sistemot (2') i ja dobivame
ravenkata 23x = 92, t.e. x = 4.

109
Taa ravenka zaedno so edna (koja bilo) ravenka na dadeniot sistem, na primer, vtorata
ravenka, }e obrazuva nov sistem ekvivalenten na dadeniot
­[ 
® (2'')
¯ [   \  
Od vtorata ravenka, zamenuvaj}i vo nea x = 4, ja dobivame vrednosta i na nepoznatata y,
t.e. od 5 · 4 + 2y = 26 dobivame y = 3.
Zna~i, sistemot ravenki ima re{enie x = 4, y = 3.
Ovoj metod na re{avawe na sistemite linearni ravenki so dve nepoznati se vika
metod na sprotivni koeficienti ili metod na sobirawe.
Od razgledanite primeri gledame deka ovoj metod se sostoi vo slednovo:

1o. Dadeniot sistem ravenki go zamenuvame so drug ekvivalenten sistem vo koj


koeficientite pred edna ista nepoznata (na primer, y) se sprotivni broevi.
2o. Gi sobirame soodvetnite levi i desni strani na dvete ravenki vo dobieniot
sistem. Dobivame ravenka so edna nepoznata x.
3o. Ja re{avame dobienata ravenka so edna nepoznata (po x) i ja odreduvame vrednosta
na taa nepoznata.
4o. Ja zamenuvame dobienata vrednost na nepoznatata x vo edna od ravenkite na
dadeniot sistem, ja re{avame taa ravenka i ja nao|ame vrednosta i na drugata nepoznata y.
Taka odredenite vrednosti na nepoznatite x i y se re{enie na dadeniot sistem
ravenki.

Re{i gi slednive sistemi ravenki po metodot na sprotivni koeficienti:


­ [   \  ­ [   \ 
1. a) ®  [   \  , b) ® [   \  .
¯ ¯

­ [  \  ­  [   \ 
2. a) ® [  \  , b) ® [  \  .
¯ ¯

­ [   \  ­ [   \ 
3. a) ® [   \  , b) ® [   \  .
¯ ¯

­  [   \  ­ [  \ 
4. a) ® [   \  , b) ® [   \  .
¯ ¯
Prvo dovedi gi sistemite ravenki vo op{t vid, a potoa re{i gi:

110
­ [    \   ­[ \
°°    [ \ °°  
5. a) ® b) ®
°[    \  [ \, °[ \
°̄   °̄   .

­ [    \   \  
­[ \  [ 
°°  
°°    
 
6. a) ® b) ®° [   \  
°[  \  [ 
°̄ 
 \  [ .

°̄    ,

III. 8. PRIMENA NA SISTEMITE LINEARNI RAVENKI


SO DVE NEPOZNATI

Re{avaweto na mnogu zada~i od matematikata, fizikata, tehnikata i praktikata {to


se iska`ani tekstualno, se sveduva na sostavuvawe i re{avawe na nekoj sistem linearni
ravenki so dve nepoznati. Nekoi od takvite zada~i, koi porano gi re{avavme so pomo{
na linearni ravenki so edna nepoznata, mnogu polesno se re{avaat so pomo{ na sistem
linearni ravenki so dve nepoznati.
Re{avaweto na takvite zada~i }e go poka`eme na nekolku primeri.

Zada~a 1. Vo dva sada imalo po odredeno koli~estvo voda. Ako od prviot sad vo vtoriot
presipeme 3 litri voda, vo vtoriot sad }e ima 2 pati pove}e voda otkolku vo prviot. No,
ako od vtoriot presipeme 3 litri voda vo prviot sad, toga{ vo dvata sada }e ima po isto
koli~estvo voda. Po kolku litri voda imalo vo sekoj sad?

Re{enie. Neka vo prviot sad ima x litri voda, a vo vtoriot sad y litri voda.
Ako od prviot sad presipeme 3 litri voda vo vtoriot sad, toga{ vo prviot sad }e ima
(x – 3) litri, a vo vtoriot sad }e ima (y + 3) litri .
Vo soglasnost so uslovot vo zada~ata, vo toj slu~aj, vo vtoriot sad }e ima 2 pati
pove}e voda otkolku vo prviot. Taa situacija ja izrazuvame so ravenkata 2(x – 3) = y + 3.
Ako pak, od vtoriot sad presipeme 3 litri voda vo prviot sad, toga{ vo prviot sad }e
ima (x + 3) litri, a vo vtoriot (y – 3) litri. Vo soglasnost so uslovot vo zada~ata, vo toj
slu~aj, vo dvata sada }e ima po isto koli~estvo voda, t.e. }e ja dobieme ravenkata
x+3=y–3.
Spored toa, nepoznatite x i y treba da go zadovoluvaat sistemot ravenki:
­° [   \  
®
°̄ [   \   

111
Dobieniot sistem ravenki e ekvivalenten na sistemot:
­ [  \ 
®
¯ [  \  .
Ako go re{ime toj sistem, dobivame x = 15, y = 21.
Odgovor: Vo prviot sad imalo 15 litri voda, a vo vtoriot 21 litar voda.
Zada~a 2. Zbirot na cifrite na eden dvocifren broj e ednakov na 11. Ako kon nego
dodademe 27 }e dobieme dvocifren broj {to e zapi{an so istite cifri, no vo obraten
red. Koj e toj broj?
Re{enie. Cifrata na desetkite na baraniot broj neka e x, a cifrata na edinicite
neka e y. Toga{ dvocifreniot broj e 10x + y, a brojot {to e zapi{an so istite cifri no vo
obraten red }e bide 10y + x. Vo soglasnost so uslovite na zada~ata go sostavuvame sistemot
ravenki:
­ [  \ 
®
¯ [  \    \  [ 
Koga }e go re{ime toj sistem ravenki, dobivame x = 4, y = 7.
Zna~i, baraniot dvocifren broj e 47.
Navistina: 47 + 27 = 74, brojot 74 e zapi{an so istite cifri no vo obraten red.
Zada~a 3. Zbirot na godinite na majkata i }erkata iznesuva 37. Pred edna godina majkata
bila 6 pati postara od svojata }erka. Sega, kolku godini ima sekoja od niv?
Re{enie. Sega majkata neka ima x godini, a }erkata y godini. Pred 1 godina majkata
imala (x – 1) godini, a }erkata (y – 1) godini. Vo soglasnost so uslovot na zada~ata lesno go
sostavuvame sistemot ravenki:
­° [  \ 
®
°̄ [    \   
Koga }e go re{ime sostaveniot sistem ravenki, nao|ame: x = 31, y = 6.
Najdenoto re{enie gi zadovoluva uslovite na zada~ata, spored toa: Sega majkata ima
31 godina, a }erkata 6 godini.
Zada~a 4. Agolot pri osnovata na eden ramnokrak triagolnik e 2 pati pogolem od
agolot pri vrvot. Odredi ja goleminata na aglite na toj triagolnik!
Re{enie. Znaete, aglite pri osnovata na ramnokrakiot triagolnik imaat ednakva
golemina. Goleminata na agolot pri osnovata da ja ozna~ime so x, a taa na agolot pri
vrvot - so y. Vo soglasnost so uslovot na zada~ata nepoznatite x i y treba da ja zadovoluvaat
ravenkata x = 2y.
Ako zememe predvid deka zbirot na trite vnatre{ni agli na sekoj triagolnik e ednakov
na 180o, nepoznatite x i y treba da ja zadovoluvaat u{te i ravenkata
x + x + y = 180o.

112
­[ \
Koga }e go re{ime sistemot ravenki ® [  \ q dobivame x = 72o, y = 36o.
¯
Spored toa, dvata agla pri osnovata na ramnokrakiot triagolnik imaat po 72o, a agolot
pri vrvot ima 36o.
Zada~a 5. Eden rezervoar, koj sobira 2000 litri voda, }e se napolni od dve slavini za
20 min. Po kolku litri voda dava sekoja slavina vo minuta, ako se znae deka ednata od niv
dava 10 litri pove}e vo minuta, otkolku drugata slavina.
Re{enie. Neka ednata slavina dava x litri vo minuta, a drugata slavina - y litri vo
minuta. Za 20 minuti prvata slavina }e dade (20x) litri, a vtorata - (20y) litri.
Spored uslovite na zada~ata, gi sostavuvame ravenkite:
20x + 20y = 2000 i x = y + 10.
­  [   \ 
Koga }e go re{ime sistemot ravenki ® [ \  
dobivame: x = 55, y = 45.
¯
Zna~i, ednata slavina dava 55 litri vo minuta, a drugata slavina dava 45 litri vo
minuta.

1. Razlikata na dva broja e 22, a nivniot zbir e 130. Koi se tie broevi?
2. Zbirot na dva broja iznesuva 73, a razlikata na prviot so vtoriot broj e 17.
Koi se tie broevi?
3. Eden broj e 5 pati pogolem od drug broj, a nivniot zbir e 78. Koi se tie broevi?
4. Dol`inata na eden pravoagolnik e 2 pati pogolema od negovata {irina. Kolkava e
dol`inata i {irinata na pravoagolnikot, ako negoviot perimetar iznesuva 21cm.
5. Agolot pri vrvot na eden ramnokrak triagolnik e 3 pati pogolem od sekoj agol pri
osnovata. Odredi ja goleminata na aglite na toj triagolnik?
6. Vo eden triagolnik eden od aglite e 5 pati pogolem od drugiot agol, a tretiot agol
e ednakov na zbirot od prvite dva agla. Kolkavi se aglite na toj triagolnik?
7. Vo eden pravoagolen triagolnik ednata kateta e za 4cm pomala od drugata kateta.
Ako sekoja kateta ja zgolemime za 3cm, toga{ negovata plo{tina }e se zgolemi
za 63cm2. Odredi gi dol`inite na katetite na toj triagolnik!
8. Dol`inata na eden pravoagolnik e 2 pati pogolema od negovata {irina. Ako
dol`inata na pravoagolnikot ja namalime za 4cm, a visinata ja zgolemime za
2cm, negoviot perimetar }e bide ednakov na 50cm. Odredi ja dol`inata i {irinata
na pravoagolnikot!
9. Ovo{nata gradina na edna u~ili{na zadruga imala 180 ovo{ni drvja, slivi,
kru{i i jabolka. Slivi imalo 2 pati pove}e otkolku kru{i, a jabolka imalo
tolku kolku {to imalo slivi i kru{i zaedno. Kolku drvja oddelno slivi, kru{i
i jabolka imalo vo gradinata?

113
10. Vo edna fabrika na tri oddelenija rabotat vkupno 460 rabotnici. Vo prvoto oddelenie
rabotat 2 pati pove}e rabotnici otkolku vo vtoroto, a vo tretoto oddelenie rabotat
36 rabotnici pomalku otkolku vo vtoroto oddelenie. Po kolku rabotnici oddelno
rabotat vo sekoe oddelenie?
11. Za 98 denari kupeni se 17 mali i golemi tetratki. Edna mala tetratka ~ini 4 denari,
a edna golema tetratka ~ini 9 denari. Po kolku tetratki od sekoj vid se kupeni?
12. Rasitnuvaweto na edna stodenarka izvr{eno e samo vo 5 - denarki i 2 - denarki, koi
na broj gi imalo 26. Najdi kolku imalo 5 - denarki, a kolku - 2 denarki.
13. Vo eden magacin ima 2 pati pove}e bra{no otkolku vo drug magacin. Ako vo prviot
magacin doneseme u{te 4 toni bra{no, a vo vtoriot magacin doneseme u{te 13 toni,
toga{ vo dvata magacina }e ima isto koli~estvo bra{no. Kolku toni bra{no imalo
vo sekoj magacin pooddelno?

V RABOTA SO PODATOCI

III.9. PRINCIP NA DIRIHLE

Da pretpostavime deka: edinaeset top~iwa treba da gi rasporedime vo deset kutii.


Toga{ jasno e deka, kako bilo da gi rasporeduvame top~iwata, neophodno e vo edna od
kutiite da smestime barem dve top~iwa. Ako, pak, se dadeni sedum top~iwa a samo tri
kutii, {to mo`eme da zaklu~ime? Toga{ }e postoi barem edna kutija vo koja ima barem
tri top~iwa. Navistina, ako vo sekoja kutija stavime najmnogu dve top~iwa, toga{
vkupniot broj na top~iwa bi bil najmnogu 3 × 2=6, a ne sedum. Pritoa, da napomeneme deka
pri smestuvaweto na top~iwata vo kutiite ne e bitno dali imame nekakva strategija pri
smestuvaweto, na primer, dali sakame vo sekoja kutija da stavime pribli`no ist broj na
top~iwa, ili pak, sakame site top~iwa da gi staveime vo edna kutija i sli~no.
Prethodnite zada~i mo`eme da gi obop{time so slednite zaklu~oci: Ako treba n+1
top~e da smestime vo n kutii, toga{ }e postoi barem edna kutija vo koja ima barem dve
top~iwa. Va`i, isto taka, i slednoto mnogu posilno tvrdewe. Neka n, p i k se prirodni
broevi i pritoa nk + p top~iwa, treba da gi rasporedime vo n kutii. Toga{ }e postoi
barem edna kutija vo koja }e ima barem k+1 top~e. Navistina, ako vo sekoja kutija se
smestat po najmnogu k top~iwa, toga{ vkupniot broj na top~iwa }e bide najmnogu nk,
a ne mo`e da bide nk + p kolku {to se pretpostavuva. Zabele`uvame deka ne sme vo sostojba
da tvrdime koja e taa kutija vo koja ima dve ili p+1 top~e, no, samo tvrdime deka postoi
takva kutija.
Prethodnoto tvrdewe e poznato kako princip na Dirihle, vo ~est na germanskiot
matemati~ar Dirihle (P. L. Dirihlet, 1805-1859). Principot na Dirihle go interpretiravme
preku smestuvawe na top~iwa vo kutii, no, kako {to }e vidime so slednive primeri ovoj
princip nao|a ~esta primena vo golem broj na zada~i. ]e navedime nekolku takvi primeri.

114
Primer 1. Sekoj od ~lenovite na edno dru{tvo od osum prijateli edna{ nedelno odi
vo bazen da pliva, na primer Ana odi na bezen sekoja sreda, Dragan odi sekoj petok itn.
Poka`i deka postojat dvajca prijateli koi odat na plivawe vo ist den od nedelata.
Re{enie. Bidej}i ima sedum dena vo nedelata, a prijatelite se vkupno 8 i 8>7,
od principot na Dirihle }e postojat dvajca koi odat na plivawe vo ist den.
Primer 2. Poka`i deka me|u deset prirodni broevi sekoga{ postojat dva broja koi
zapo~nuvaat so ista cifra.
Re{enie. Sekoj od desette broevi zapo~nuva so edna od devette cifri 1,2,3,4,5,6,7,8, 9,
pa spored principot na Dirihle }e postojat dva broja koi zapo~nuvaat na ista cifra.
Da zabele`ime deka takvo tvrdewe ne va`i ako barame poslednata cifra da im bide
ista. Zo{to?
Primer 3. Ivana, Ilinka i Irina berele jabolka i zaedno nabrale 17 kg jabolka.
Poka`i deka barem edna od niv nabrala barem 6 kg jabolka, ako se znae deka sekoja od niv
nabrala cel broj kilogrami.
Re{enie. Da prepostavime deka ne e vistina deka nekoja od niv nabrala barem 6 kg
jabolka. Toga{ sekoja od niv nabrala najmnogu po 5 kg jabolka, pa tie site zaedno nabrale
najmnogu 3×5=15 kg jabolka. Toa protivre~i na uslovot na zada~ata deka tie zaedno nabrale
17 kg jabolka. Zna~i, barem edna od niv nabrala barem 6 kg jabolka. Ovde koristevme deka
17=3×5+2>3×5.
Primer 4. Doka`i deka me|u ~etiri proizvolni prirodni broevi, sekoga{ postojat
dva takvi {to, nivnata razlika e deliva so 3.
Re{enie. Sekoj od dadenite ~etiri prirodni broevi go delime so 3 i go odreduvame
ostatokot pri delewe so 3. Ostatocite mo`at da bidat 0, 1 ili 2. Zatoa, mo`eme da
smetame deka dadenite broevi se 3k1+r1, 3k2+r2, 3k3+r3 i 3k4+r4, kade r1,r2,r3,r4Î{0,1,2}. Zna~i,
sekoj od ~etirite ostatoci r1, r2, r3 i r4 prima edna od trite mo`ni vrednosti. Spored
principot na Dirihle, postojat barem dva broja me|u niv, koi se ednakvi. Neka, na primer,
r2 = r4. Toga{ (3k2+r2) - (3k4+r4) = 3k2+r2 - 3k4 - r4 = 3k2 - 3k4 = 3(k2 - k4), pa ottuka 3|(3k2+r2) - (3k4+r4).

1. Petar vo dvata xeba ima vkupno 101 denar. Poka`i deka vo eden od xebovite Petar
ima barem 51 denar, pri pretpostavka deka vo xebovite nema moneta pomala
od eden denar.
2. Vo eden hotel ima dvokrevetni i trikrevetni sobi. Poka‘i deka vo hotelot ima
barem 34 sobi, ako se znae deka vo hotelot se smesteni 100 gosti.
3. Vo edno u~ili{te u~at 1000 deca. Poka`i deka ima barem tri deca vo toa u~ili{te
rodeni na ist datum (neobavezno ista godina).
4. Mite ima privrzok so ~etiri klu~evi. Bez da gleda so koj klu~ otvora vo temnica,
Mite uspeal da vleze vo svojot stan pri osmiot obid. Poka`i deka postoi klu~ so
koj Mite barem tripati se obiduval da ja otklu~i vratata.

115
III.10. POPULACIJA. PRIMEROK

^estopati vo sekojdnevnata praktika se javuva problem od edno mno‘estvo da se


izdvojat i prebrojat onie elementi, koi imaat nekoe obele‘je, odnosno koi se izdvojuvaat
so nekoe svojstvo. Na primer, ako vo edno oddelenie treba da se prebrojat u~enicite koi
imaat odli~en ili mnogu dobar uspeh zaedno, toa ne e nikakov problem. Isto taka, ne e
problem da se najde razmerot na ovie solidni u~enici sprema vkupniot broj na u~enici
vo oddelenieto. Me|utoa, ako treba da se najde kolkav e razmerot na lu|eto koi se povisoki
od 170 cm sprema vkupniot broj na ‘iteli, situacijata e mnogu poslo‘ena.
To~niot razmer e nevozmo‘no da se opredeli, bidej}i problem e da se opredeli nivniot
to~en broj, a posebno koga se ima predvid deka sekojdnevno brojot na ‘iteli se menuva
(postojano edni umiraat i se ra|aat novi ‘iteli). Me|utoa, kako {to }e vidime, i ovoj
problem mo‘e da se re{i.
Vo slu~aj koga rabotime so golem broj podatoci, velime deka se raboti za statisti~ki
podatoci, a delot od matematikata {to se zanimava so obrabotka na takvite podatoci, se
vika matemati~ka statistika.
Ako od mno‘estvoto S treba da se izdvojat i prebrojat onie elementi so dadeno svojstvo,
toga{ postapuvame na sledniot na~in. Od mno‘estvoto S izbirame edno negovo
podmno‘estvo U, pa se prebrojuvaat onie elementi od U koi go imaat baranoto svojstvo.
Mno‘estvoto S se narekuva populacija, a podmno‘estvoto U se narekuva primerok. Brojot
na elementi na populacijata se narekuva obem na populacijata, a brojot na elementite vo
primerokot se narekuva obem na primerokot.
Primer 1. Da go razgledame ve}e postaveniot problem na odreduvawe na brojot na
‘iteli vo R. Makedonija, koi se povisoki od 170 cm. Mno‘estvoto gra|ani na R.
Makedonija e populacija, i znaeme deka obemot na ovaa populacija e okolu 2 000 000.
Izbirame edno podmno‘estvo - primerok od 1000 gra|ani i }e gi prebroime onie od niv,
koi se povisoki od 170 cm. Neka me|u izbranite 1000 gra|ani ima 530 koi se povisoki od
170 cm. Bidej}i 2 000 000:1000=2000, nie vo primerokot na 2000 gra|ani sme birale po
eden, zatoa o~ekuvame deka brojot na gra|ani koi se povisoki od 170 cm vo celata Republika
da bide 530×2000=1 060 000.
Pritoa, ako sakame podatocite {to }e gi dobieme {to e mo‘no pove}e da odgovaraat
na stvarnosta, treba vo izborot na tie iljada gra|ani - primerokot, da bidat zastapeni
site vozrasti i toa vo onoj procent vo koj tie se zastapeni vo populacijata.
Na toj na~in mo‘eme da ja sostavime tabelava:

Visina povisoki od 170 cm 170 cm ili poniski


Gr|ani od primerokot 530 470
Gra|ani od populacijata 1 060 000 940 000

Primer 2. Vo edna fabrika se proizvedeni 15000 svetilki. Proizvoditelot saka da


go ispita kvalitetot na svetilkite, poto~no da proceni kolku od niv imaat vek na traewe
podolg od 1000 ~asa.

116
Za taa cel proizvolno se izbrani 200 svetilki i se podlo‘eni na ispituvawe, za da
se vidi kolku od niv nema da pregorat po 1000 ~asa svetewe. Se poka‘alo deka 180 svetilki
,,izdr‘ale“ 1000 ~asa, a ostanatite 20 svetilki pregorele pred vreme. Zna~i, 10% od
primerokot e so lo{ kvalitet, a ostanatite 90% se so dobar kvalitet. Zatoa se o~ekuva
10% od populacijata (1500 svetilki) da bidat so lo{ kvalitet, a ostanatite 90% (13500
svetilki) da bidat so dobar kvalitet. Na toj na~in mo‘eme da ja sostavime slednava
tabela:
Vek na traewe pomalku od 1000 ~asa pove}e od 1000 ~asa
Broj na svetilki od primerokot 20 180

Broj na svetilki od populacijata 13500 1500

1. [to e populacija, a {to primerok?


2. Ako obemot na populcijata e 100 000, dali e dovolno obemot na primerokot
da bide 4? Objasni go odgovorot.
3. Na 100 prodadeni televizori vo edna prodavnica stignale 5 reklamacii vo
garantniot rok. Kolku reklamacii se o~ekuvaat na 1200 televizori?
4. Spored podatocite na Svetskata zdravstvena organizacija okolu 3% od lu|eto vo
tekot na ‘ivotot zaboluvaat od epilepsija. Kolkav e toj broj na lu|e vo na{ata
Republika?

III.11. NASTAN. SIGUREN I SLU^AEN NASTAN

1. ^esto velime: prirodata se nao|a vo postojano dvi`ewe. Sekojdnevno se odigruvaat


najrazli~ni pojavi. Mnogu od tie pojavi mo`eme to~no da gi predvidime, ako gi poznavame
site pri~ini za nivnoto slu~uvawe i razvitok. Na primer, vodata vo zavisnost od
temperaturata i atmosferskiot pritisok, znaeme, mo`e da se nao|a vo edna od trite
mo`ni agregatni sostojbi: cvrsta, te~na ili gasovita sostojba. Meteorologijata, vrz
osnova na golem broj na podatoci, e vo sostojba da dade vremenska prognoza za narednite
denovi i koja vo najgolem broj slu~ai se ostvaruva. Takov e slu~ajot i so site prirodni i
op{testveni pojavi. Dvi`eweto vo prirodata se karakterizira i manifestira vo
postojanata promenlivost na sostojbite.
Sekoja promena na edna sostojba vo druga, se vika nastan (slu~ka ili sobitie). Na
primer, nastan e koga no}ta go smeni denot, koga vo proletta razlistuvaat drvjata, koga
}e se rodat bliznaci ili trojka. Nastan za slavewe e koga }e se dobie premijata na
loto, itn.

117
Celiot `ivot se sostoi od oddelni nastani, a uspe{noto snao|awe vo tie nastani
zna~i snao|awe vo `ivotot. Uspe{noto snao|awe, pak, vo nastanite podrazbira nivno
sledewe, prou~uvawe i predviduvawe. Najdobar primer za toa e meteorologijata, pri
{to nejziniot razvoj kako nauka, nesomneno, vodi kon poto~ni vremenski prognozi.
Na prv pogled izgleda deka nastanite ne mo`e da se predviduvaat. Me|utoa, nastanite
imaat nekoi op{ti karakteristikii i postojat zakonomernosti vo nivnoto slu~uvawe.
2. Predmetot na nivno prou~uvawe se naukite verojatnost i statistika.
Razlikuvame nastani koi se mo`ni i nastani koi se nevozmo`ni. Mo`ni nastani se,
na primer:
- uspe{no lansirawe na vselenski brod,
- uspe{en plasman na nekoj turnir,
- dobivka na loto, ako se kupi loz i dr.
Nevozmo`ni nastani se, na primer:
- prestanok na rotacijata na Zemjata okolu Sonceto,
- vo sostavot na vodata da se najde tret element,
- rekite da te~at vo sprotivni nasoki,
- Sonceto da izgree na zapad,
- da se izvle~e sino top~e od kutija vo koja ima samo beli i crni top~iwa, i dr.
Za nas pogolem interes na prou~uvawe se mo`nite nastani. Tie od svoja strana se
delat na sigurni nastani, koi pod odredeni uslovi mora da se slu~at i slu~ajni nastani
ili verojatni nastani, za koi postoi mo`nost da se slu~at, no postoi mo`nost i da ne se
slu~at.
Vo sigurni nastani spa|aat, na primer, slednive:
- po denot doa|a no}ta, a po no}ta povtorno doa|a denot,
- krajot na `ivotot e smrtta,
- sigurno e deka po izvesno vreme }e ima zatemnuvawe na Mese~inata ili na Sonceto,
- sigurno e deka }e se izvle~e belo ili crno top~e od edna kutija vo koja ima samo
beli i crni top~iwa i dr.
Postojat mnogu drugi nastani vo prirodata koi se sigurni. Tie se odnesuvaat pred s–
na nekoi osnovni zakoni koi va`at vo fizikata, hemijata, astronomijata, biologijata
itn., kako i posledici na nekoi logi~ki zakoni. No ponekoga{ ne e mo`no da se predvidat
nekoi nastani. Eve nekolku primeri na slu~ajni nastani:
- nedogovorena sredba so nekoj blizok prijatel na ulica,
- da pregori nova svetilka, verojatno, lo{ primerok,
- na prvi januari slednata godina da padne sneg,
- da se izvle~e belo top~e od kutija vo koja ima 2 beli i 7 crveni top~iwa, i mnogu
drugi.
Nastanite obi~no }e gi ozna~uvame so bukvite A,B,C,...

118
1. Koi nastani se vikaat sigurni nastani? Navedi nekolku primeri.
2. [to se navozmo`ni nastani? Navedi nekolku nevozmo`ni nastani.
3. Koi nastani se vikaat slu~ajni nastani? Navedi nekolku.
4. Kakov nastan e: da se najde detelina so ~etiri lista?

III. 12. SISTEMSKI I SLU^AEN


IZBOR NA PRIMEROK

Matemati~kata statistika e nauka koja se zanimava so obrabotka na golem broj


podatoci (statisti~ki podatoci). Statisti~koto mno`estvo za koe sakame da dobieme
nekoi informacii ili nekoi negovi svojstva se vika statisti~ka masa ili populacija.
Vo princip, toa mno`estvo ima ogromen broj elementi, a elementite mo`at da bidat
site `iteli na eden grad ili na edna dr`ava, site ribi vo edno ezero, site zrna p~enica
vo nekoj region i sli~no. Vo osnova, postoi nekoja zaedni~ka osobina koja gi obedinuva
elementite na toa mno`estvo. Ovaa osobina se vika obele`je ili karakteristika, a za
elementite od populacijata velime deka se od ist vid. Ispituvaweto na celata populacija
e nevozmo`no od slednive pri~ini: 1. Toa mno`estvo, re~isi, sekoga{ ima ogromen broj
elementi {to ne e mo`no vremenski site da se ispitaat i 2. Objektite na toa mno`estvo
~estopati se te{ko dostapni. Na primer, ne sme vo vo sostojba da gi izvadime site ribi
od ezeroto, da gi ispitame i, pak, da gi vratime nazad. Za taa cel, na proizvolen na~in
izbirame edno podmno`estvo ‰ so zna~itelno pomal broj elementi, no za koe }e bideme
vo sostojba da gi ispitame pooddelno site negovi elementi, a potoa da donesuvame zaklu~ok
za celata populacija. Takvoto podmno`estvo ‰ go vikame primerok. Brojot na elementite
na populacijata se vika obem na populacijata, a brojot na elementite na primerokot se
vika obem na primerokot.
Primer 1. Vo edna fabrika za akumulatori proizvedeni se 10 000 akumulatori. Za da
go ispitaat kvalitetot na akumulatorite proizvoditelite na proizvolen na~in izbiraat
50 akumulatori. Ispituvaweto na ovie akumulatori poka`alo deka 47 od niv se ispravni,
a 3 neispravni. Bidej}i 10 000 : 50 = 200, zatoa se o~ekuva deka 3 · 200 = 600 akumulatori se
neispravni. Baraweto na neispravnite akumulatori me|u site 10 000 prakti~no e
neizvodlivo. Zatoa od fabrikata gi pu{taat site akumulatori vo promet, no o~ekuvaat
da dobijat reklamacija za okolu 600 akumulatori.
Izborot na primerokot e od su{tinska va`nost za dobivawe na pravilen zaklu~ok,
pa zatoa vo matemati~kata statistika se dava posebno vnimanie na ovoj problem. Najva`no
e izborot na primerokot da bide slu~aen. Za da bideme sigurni deka izborot na
primerokot navistina, e slu~aen, mo`eme da pristapime kon sistematski izbor na
primerok. Toa }e go ilustrirame na slednive primeri.

119
Primer 2. Vo edna ambulanta se lekuvaat 3 540 gra|ani. So cel, da se izbere eden
primerok od pribli`no edna desetina od ovie gra|ani, nie mo`eme da gi izbereme onie,
~ij reden broj na kartonot zavr{uva na primer na cifra 5.
Primer 3. Ako sakame da anketirame okolu 1 500 gra|ani vo Skopje, toa mo`eme da go
napravime po telefon, yvonej}i, na primer, na site telefonski broevi {to zavr{uvaat
na 37. So toa, vsu{nost, }e se javime kaj sekoj stoti pretplatnik na fiksnata telefonija.

1. Edna medicinska ekipa, so cel, da ispita kolkav procent od gra|anite vo Makedonija


imaat negativna krvna grupa, oti{le vo edno u~ili{te i od 753 deca na{le deka 98
u~enici imaat negativna krvna grupa. Dali primerokot e dobro izbran? Kolku
gra|ani vo Makedonija imaat negativna krvna grupa?
2. Novinarite za da go proverat javnoto mislewe na gra|anite pred nekoj referendum
anketirale 500 polnoletni gra|ani. Potvrden odgovor na postavenoto pra{awe dale
356 ispitanici. Kolku se o~ekuva da bide brojot na gra|anite {to }e odgovorat
pozitivno na referendumot vo nekoj grad so 700 000 polnoletni gra|ani?
3. Frli vo vozduh 100 pati edna para i izbroj kolku pati }e padne broj". Toj broj
Q  "
ozna~i go so n , a potoa presmetaj  .
 

III. 13. VEROJATNOST NA SLU^AEN NASTAN

Igra~ite na loto znaat deka koga }e uplatat loto, imat mo`nost da dobijat sedumka".
"
Iako mo`nosta e mala, taa sepak postoi. Me|utoa, nevozmo`en nastan e nekoj da dobie
pari na loto, ako ne uplatil za igrata. No, ako nekoj gi popolnil site mo`ni kombinacii
(toa e samo teoriski, bidej}i se mnogu), toga{ negovata dobivka sedumka"na loto e
"
siguren nastan. Ako popolnime osum kombinacii, toga{ kolkava e mo`nosta za dobivka?
Poznato e deka od 39 broja mo`e na 15 380 937 na~ini da se izvle~at sedum broja. No,
igra~ot }e dobie samo na 8 izvlekuvawa. Zatoa, velime deka verojatnosta za dobivka na
sedumka" e
" 
|   .

Da rzgledame op{t slu~aj, ako se mo`ni n razli~ni nastani site me|u sebe ednakvo
mo`ni da se slu~at (t.e. ednakvo verojatni). Neka nastanot A se slu~uva, ako se slu~i
eden koj bilo od m nastani, takanare~eni povolni nastani. Pri tie pretpostavki ja davame
slednata definicija.
Definicija. Verojatnosta V(A) da se slu~i eden nastan A se vika razmerot (odnosot)
na brojot m na povolnite nastani i brojot n na site elementarni (povolni i nepovolni)
nastani, t.e.

120
P
9 $ =
. (1)
Q
Bidej}i n e brojot na povolni i nepovolnio nastani, dobivame  d P d Q , a od ovde
imame  d 9 $ d  , (2)
t.e. verojatnosta na eden nastan e broj me|u 0 i 1, vklu~uvaj}i gi 0 i 1. Ako A e siguren
nastan, toga{ m = n, pa V(A) = 1. Ako, pak, A e nevozmo`en nastan, toga{ m = 0, pa V(A) = 0.
Primer 1. Da se odredi verojatnosta deka a) so frlawe na eden zar }e se padne {estka",
"
b) so frlawe na dva zara }e se padnat dve {estki", v) so frlawe na dva zara }e se padnat
"
petka" i {estka".
" "
Re{enie. a) Ako frlame eden zar, mo`ni se {est elementarni nastani: da se padne 1,
2, 3, 4, 5 ili 6. Me|u ovie nastani samo eden e povolen, a toa e ako se padne {estka".
 "
Zna~i n = 6, a m = 1, pa baranata verojatnost e |  .

b) Ako frlame dva zara, mo`ni se slednite 36 elementarni nastani:

(1,1) (1,2) (1, 3) (1, 4) (1, 5) (1, 6)


(2, 1) (2, 2) (2, 3) (2, 4) (2, 5) (2, 6)
(3,1) (3, 2) (3, 3) (3, 4) (3, 5) (3, 6)
(4, 1) (4, 2) (4, 3) (4, 4) (4, 5) (4, 6)
(5, 1) (5, 2) (5, 3) (5, 4) (5, 5) (5, 6)
(6, 1) (6, 2) (6, 3) (6, 4) (6, 5) (6, 6)

Samo eden nastan me|u ovie 36 e povolen, a toa e (6, 6). Zna~i, m = 1, a n = 36, pa baranata

verojatnost e |   .

v) Imaj}i ja predvid diskusijata pod b), imame n = 36, a m = 2 bidej}i imame dve povolni

mo`nosti (5, 6) i (6, 5). Zatoa baranata verojatnost e |   .

Za daden nastan A, so $ go ozna~uvame negoviot sprotiven nastan, koj se slu~uva,
toga{ i samo toga{, koga A ne se slu~uva. So drugi zborovi, povolnite elementarni
nastani za A (gi ima m) se nepovolni za nastanot $ i obratno. Za ovie dva nastana A
i $ velime deka se isklu~uvaat, no i deka se nadopolnuvaat.
Zna~i, za nastanot $ imame n – m povolni slu~ai (nepovolni za A) i m nepovolni
QP
slu~ai (povolni za A). Zna~i , verojatnosta }e bide 9 $ Q
  9 $ .
Zna~i, va`i 9 $  9 $  . (3)
Da se navratime na primer 1 a). Sprotiven nastan na nastanot da se padne {estka"
"
e i nastanot da se padne 1 ili 2 ili 3 ili 4 ili 5. Za verojatnosta na $ dobivame
  
9 $ . Zabele`uvame deka 9 $  9 $   , taka {to formulata (3) va`i.
  
121
1. [to e verojatnost na nekoj slu~aen nastan?
2. Dali mo`e verojatnosta na nekoj nastan da bide: a) – 0,5; b) 0; v) 1; d) 1,1; |) 2?
3. [to e sprotiven nastan na nastanot A? Ako verojatnosta na nastanot A e r,
kolkava e verojatnosta na sprotivniot nastan?
4. Kolkava e verojatnosta deka so frlawe na eden zar }e se padne 3? Potoa, so
golem broj frlawa na zarot proveri go rezultatot.

III.14. EKSPERIMENTALNA PROVERKA NA REZULTATITE

Da se zapra{ame: [to vsu{nost zna~i, deka verojatnosta so edno frlawe na zar da



se padne petka" e ? Odgovorot e sledniot: Ako kockata - zarot ja frlime golem broj
"  
pati, na primer, 1000, toga{ pribli`no ˜ |  pati }e se padne brojot 5. Poto~no,

 
ako se padne 172 pati brojot 5, toga{ razmerot malku }e se razlikuva od . Vo
 
   
slu~ajov |  , a |  pa }e bide   . Ako, namesto, 1000 pati
   
N 
kockata ja frlime 100000 pati i pritoa k pati se padne brojot 5, toga{    }e

bide u{te pomal broj. So drugi zborovi, kolku pove}e obidi pravime, tolku pove}e
N 
razmerot e poblizok do ( n e broj na obidi - frlawa na kockata, a k poka`uva kolku
Q 
pati }e se padne 5).

1. Eden zar frli go 100 pati i broj kolku pati }e padne brojot 5 (k pati). Potoa
N 
presmetaj ja razlikata  . Potoa zemi dva zara i istovremeno frli gi 500 pati.
 
Neka k1 pati se padnat dve {estki zaedno i neka k2 pati se padnat 5 i 6. Potoa presmetaj gi
N  N 
razlikite  i   . [to zabele`uva{?
   
2. Zemi edna 5 - denarka, frli ja vo vozduh 200 pati i broj kolku pati }e se padne
N 
broj". Ako toj broj na pojavuvawa e k, presmetaj ja razlikata  . Zo{to namalitelot
"  

e izbran da bide ? [to zabele`uva{?

Da napomeneme deka definicijata za verojatnost {to ja usvoivme se poka`uva deka e
"zadovoluva~ka" vo golem broj primeri. Me|utoa, so nea ne mo`eme da presmetuvame
verojatnosti vo geometrijata. Strogata definicija na verojatnost bara mnogu

122
dopolnitelni znaewa. Ovde }e izneseme nekoi primeri od t. n. geometriska verojatnost,
koi intuitivno }e gi prifatime.
Primer 2. Da izbereme edna otse~ka AB i edna tretina od nejzinata dol`ina da ja
oboime so crvena boja (crt. 10).

$ %
0 N
Crte` 10

Da izbereme proizvolna to~ka od otse~kata AV. Kolkava e verojatnosta deka }e



izbereme crveno oboena to~ka? Intuitivno e jasno deka taa verojatnost e $0  $% .

Mo`eme eksperimentalno da se uverime vo toa, ako izbereme golem broj na takvi to~ki.
Sekoj izbor na to~ka od otse~kata AV mo`eme da go napravime na sledniot na~in. Edno
lice ramnomerno pravoliniski go dvi`i molivot od A do B, od B do A, od A do B itn, a
vtoroto lice so zatvoreni o~i vo proizvolen moment veli stop". Vo toj moment prvoto
"
lice prestanuva so dvi`eweto i to~kata e izbrana. O~igledno deka vo ovoj slu~aj se
raboti za slu~aen izbor na to~ka od otse~kata AV.
Sledniot primer poka`uva kako eksperimentalno mo`e da se dobie brojot S .
Prepora~uvame da go izvedete toj eksperoment i da ja odredite pribli`nata vrednost na
brojot S .
Primer 3. Zemame edna igla so proizvolna
dol`ina d. Potoa na eden list hartija povlekuvame
golem broj paralelni pravi, koi se na rastojanie od 2d
2d (crt. 11). Matemati~ki se poka`uva deka
verojatnosta na proizvolno frlena igla po 2d

pa|aweto vrz listot da se~e nekoja od pravite e . 2d
S
Zatoa, ako iglata ja frlite golem broj pati, na
Crte` 11
primer, 500 pati, i ako iglata k pati ja se~e nekoja

od pravite, toga{ }e bide broj blizok do S  .
N
*Istoriski bele{ki. Za osnova~ na teorijata na verojatnost se smeta francuskiot
matemati~ar - amater od Tuluz, Pjer Ferma (1601 - 1665), koj po profesija bil pravnik.
Kon sredinata na XVII vek toj zaedno so francuskiot matemati~ar Blez Paskal (1623 -
1662) i Holan|anecot Kristijan Hajgens (1629 -1695) gi postavile osnovite na
verojatnosta. Glaven pottiknuva~ za razvojot na ovaa disciplina bile hazardnite igri,
no imalo i drugi pottiknuva~i na toj razvoj: osiguritelnite zavodi, demografskata
statistika, kako i teorijata i metodite za obrabotka na podatoci dobieni od nabquduvawe.
Podocna teorijata na verojatnosta bila nadgraduvana od golem broj matemati~ari.
*Za onie koi sakaat da gi pro{irat svoite znaewa.
123
3. Vo edna kutija se nao|aat 25 sini, 35 zeleni, i 40 crveni top~iwa. Koja e verojatnosta
da se izvle~e so edno izvlekuvawe: a) sino, b) zeleno, v) crveno top~e?
4. Koja e verojatnosta frlena kocka da go poka`e brojot 2? Proverete go rezultatot
eksperimentalno.
5. Sme ja zaboravile poslednata cifra na nekoj telefonski broj, pa treba da ja pogodime.
Koja e verojatnosta da se pogodi to~niot telefonski broj so edno vrtewe?
6. Koja e verojatnosta dve frleni kocki za igrawe da poka`at zbir od 7? Proverete go
odgovorot eksperimentalno.
7. Edna to~ka se dvi`i so konstantna brzina v dol` edna kru`nica, postavena vo
koordinaten sistem, taka {to koordinatniot po~etok e centar na kru`nicata.
Kakva e verojatnosta deka vo proizvolen moment to~kata }e se najde vo prviot
kvadrant?

ZA POVTORUVAWE I
UTVRDUVAWE - III

1. Dadena e ravenkata 5h – 2u = 1. Najdi dva para broevi koi se re{enija na dadenata


ravenka i dva para broevi koi ne se re{enija na dadenata ravenka.
2. Najdi sistem linearni ravenki ~ie re{enie e parot (2, –1).
3. Ako prvata i vtorata ravenka na eden sistem si gi promenat mestata, dali
novodobieniot sistem }e bide ekvivalenten so po~etniot?
­ [   \ 
4. Daden e sistem linearni ravenki ® [   \  . Re{i go ovoj sistem so: a) metod na
¯
zamena; b) metod na sprotivni koeficienti; v) grafi~ki metod.
­ [   \ 
5. Dali e dobro sistemot ® [   \  da go re{avame grafi~ki? Zo{to?
¯
6. Koj sistem e polesno da go re{avame so metodot na zamena, otkolku so metodot na
sprotivni koeficienti?
­ [  \ 
7. Re{i go sistemot linearni ravenki ® [   \  . [to zabele`uva{?
¯

124
­ [  \ 
8. Re{i go sistemot linearni ravenki ® [   \  . [to zabele`uva{?
¯
­ [   \ 
9. Re{i go sistemot linearni ravenki ®  [   \  .
¯
­ D[  E\ DE
10. Najdi edno re{enie na sistemot ®F[  G\ F  G , kade a, b, c, d se proizvolni broevi.
¯
­ D[  \ 
11. Odredi go koeficientot a, taka {to sistemot ® [   \  da ima re{enie od oblik (c, c).
¯
­D[  \ 
12. Opredeli gi koeficientite a i b, taka {to sistemot ravenki ® da ima
re{enie (-1, 1). ¯ [  E\ 
­° [     \  [ 
13. Re{i go sistemot ® .
°̄ [   \   \   
14. Vo eden magacin imalo 2350 kg bra{no pove}e otkolku vo drug magacin. Sekoj den od
sekoj magacin se iznesuvalo po 280 kg bra{no, pa po pet dena vo prviot magacin
ostanalo 2 pati pove}e bra{no, otkolku vo vtoriot magacin. Po kolku kg bra{no
imalo vo sekoj magacin na po~etokot?
15. Dve deca zaedni~ki kupile edna kniga, koja ~inela 45 denari. Ednoto dete dalo 2
pati pove}e otkolku drugoto. Po kolku denari dalo sekoe dete?
16. Edna banknota od 50 denari rasitneta e vo 13 moneti: 5 denarki i 2 denarki. Kolku
imalo 5-denarki, a kolku 2-denarki?
17. Vo eden restoran ru~ale 15 lu|e, ma`i i `eni. Sekoj ma` platil po 400 denari, a
sekoja `ena po 325 denari za ru~ekot. Kolku bile ma`i, a kolku `eni, ako site
vkupno platile 5550 denari?
18. Vo dve osnovni u~ili{ta ima vkupno 800 u~enici. Vo ednoto u~ili{te imalo 66
u~enici pove}e otkolku vo drugoto. Po kolku u~enici imalo vo sekoe u~ili{te?
19. Eden avion niz veter leta so brzina od 190 km na ~as, a sproti veterot leta so brzina
od 160 km na ~as. Da se opredeli brzinata na veterot i sopstvenata brzina na avionot!
20. Tatkoto sega e postar od sinot za 32 godini, a po 7 godini }e bide 3 pati postar od
sinot. Kolku godini ima tatkoto, a kolku sinot?
21. Tatkoto e 4 pati postar od sinot, a sinot e 27 godini pomlad od tatkoto. Kolku
godini ima sinot, a kolku tatkoto?
22. Vo dva sada imalo 36 litri te~nost. Ako od prviot sad preleeme vo vtoriot 2 litra,
vo dvata sada }e ima isto koli~estvo te~nost. Po kolku litri te~nost imalo vo
sekoj sad?
23. Imame dva rastvora od 30% i 55% solna kiselnia. Kolku litri treba da zememe od
prviot i vtoriot rastvor za da dobieme 50 litri rastvor od 50% solna kiselina?
24. Majstorot i negoviot u~enik trebalo vo edna smena da izrabotat 130 detali.
Majstorot ja natfrlil normata za 10%, a u~enikot - za 20%, pa taka, tie izrabotile
148 detali. Kolkava bila normata za majstorot, a kolkava - za u~enikot?

125
25. Koj e sprotivniot nastan na sledniot nastan: Pri frlawe na zar da se padne broj
pogolem od 2? Presmetaj ja negovata vrednost.
26. Vo eden klas ima 32 u~enika. Poka`i deka postojat barem dva u~enika ~ii prezimiwa
po~nuvaat so ista bukva i postojat barem dva u~enika rodeni na ist datum, no ne
mora ist mesec i ista godina.
27. Kolkava e verojatnosta so frlawe na zar da se padne neparen broj? Koj e sprotivniot
nastan i koja e negovata verojatnost?
28. Kolkava e verojatnosta od {pil so 52 karti so slu~aen izbor na edna karta da
izvle~e{: a) desetka, b) srce?

ZA SAMOKONTROLA - III

1. Najdi gi site parovi prirodni broevi, {to se re{enija na ravenkata:


a) x + y = 8, b) xy = 12.
2. Za koi vrednosti na s sistemot linearni ravenki nema re{enie:
­[   \  ­[   \ 
a) ® b) ®
¯F[   \  , ¯ [  F\  ?
3. Za koi vrednosti na s sistemot linearni ravenki ima beskone~no mno`estvo
­[   \  ­[  \ 
re{enija: a) ® b) ®
¯ [   \ F , ¯F[   \  ?
4. Za koi vrednosti na a i b pravata y = ax + b minuva niz to~kite A (–1, 2) i V (2, 5)?
5. Odredi ja to~kata vo koja grafikot na ravenkata 3x – 4y = 12 ja se~e:
a) x - oskata, b) y - oskata.
6. Stara kineska zada~a: Vo eden kafez imalo pitomi zajci i fazani, koi zaedno imale
100 noze i 36 glavi. Kolku zajaci, a kolku fazani imalo vo kafezot?
7. Stara kineska zada~a: Edna prava trska, ~ija visina e 9 stopi, prekr{ena e taka {to
so vrvot da ja dopira zemjata na rastojanie 3 stopi od dolniot kraj na trskata.
Na koja viso~ina e prekr{ena trskata?
8. Zbirot na dva broja e ednakov na 299. Prviot broj pretstavuva 15% od vtoriot.
Koi se tie broevi?
9. Kolku e sega ~asot, ako izminatiot del od denot pretstavuva 20 % od denot {to
preostanuva?
10. Poka`i deka me|u 15 lu|e postojat trojca koi se rodeni na ist den vo nedelata.
11. Petar zamislil eden broj pome|u 51 i 100. Kolkava e verojatnosta deka Petar zamislil
prost broj?
12. Vo edna prodavnica od 30 prodadeni ma{ini za perewe, vo garantniot rok dobile
dve reklamacii. Vo prodavnicata imalo nara~ano 2 500 ma{ini. Kolku reklamacii
na ovie ma{ini se o~ekuvaat da pristignat po nivnata proda`ba?

126
A TO^KA, PRAVA I RAMNINA VO PROSTOROT

IV.1. ZAEMNI POLO@BI NA TO^KI, PRAVI


I RAMNINI VO PROSTOROT

Dve osnovni granki na geometrijata se: planimetrija i stereometrija. Planimetrija


e delot od geometrijata {to gi prou~uva ramninskite figuri, dodeka stereometrija e
delot od geometrijata {to gi prou~uva prostornite figuri. Vo izminatite godini se
sre}avavme so ramninski figuri (triagolnik, ~etiriagolnik, mnoguagolnik, kru`nica
i dr.), a vo ovaa tema cel na prou~uvawe }e bidat geometriskite tela kako prostorni
figuri (kocka, kvadar, piramida, topka i dr.). Za taa cel, neophodno e najprvo da se
zapoznaeme so osnovnite elementi od stereometrijata (to~ka, prava i ramnina) i so
nivnata zaemna polo`ba vo prostorot.
Znaeme deka to~ka, prava i ramnina se osnovni figuri vo geometrijata i niv ne gi
definirame. So nivna pomo{ mo`eme da definirame novi poimi i da formulirame
tvrdewa, odnosno teoremi. Izvesen broj elementarni tvrdewa (svojstva) ne gi doka`uva-
me i gi vikame aksiomi, a so nivna pomo{ mo`eme da gi doka`uvame ostanatite tvrdewa.
Od aksiomite edna ve}e ti e poznata, a toa e aksiomata za paralelnost, koja glasi:
Niz dadena to~ka {to ne le`i na dadena prava, minuva edinstvena prava koja e paralelna
so dadenata prava.
Ovde }e se zapoznaeme so nekolku novi aksiomi i teoremi. Se dogovarame to~kite da
gi bele`ime so golemite latinski bukvi (A,B,C,...), pravite so malite latinski bukvi
(a,b,c,...) a ramninite so golemite gr~ki bukvi ( 6 3 ).

127
IV. 1. 1. ZAEMNA POLO@BA NA TO^KA I PRAVA

Sekoja prava }e ja sfa}ame kako edno mno`estvo ~ii{to elementi se to~ki. Zatoa za
edna to~ka $ i edna prava D to~na e samo edna od dvete mo`nosti:
- $  D i vo toj slu~aj velime, deka to~kata $ le`i na pravata D , odnosno pravata D
minuva niz to~kata $
(crt. 1a),
- $  D i vo toj slu-
~aj velime, deka to~kata
$ ne le`i na pravata D ,
Crte` 1
odnosno pravata D ne
minuva niz to~kata $ (crt. 1b).
Me|utoa, ne sekoe mno`estvo od to~ki vo prostorot pretstavuva prava. Za pravite gi
prifa}ame slednive aksiomi.

A1: Na sekoja prava le`at bezbroj to~ki.


A2: Postojat barem tri to~ki koi ne le`at na ista prava.
A3: Niz koi bilo dve razli~ni to~ki minuva edinstvena prava.

Od aksiomata A3 proizleguva deka, ako sekoja od pravite D


i E minuva niz to~kite $ i % , toga{ pravite D i E se
sovpa|aat. Zatoa, za proizvolni pravi D i E to~na e samo edna
od mo`nostite:
I) D ˆ E = ‡ ,
II) D i E imaat samo edna zaedni~ka to~ka (crt. 2) i vo toj
slu~aj velime deka pravite D i E se se~at, Crte` 2
III) D = E , t.e. pravite se sovpa|aat.
Za tri ili pove}e to~ki velime deka se kolinearni, ako postoi prava koja minuva niz
tie to~ki. Zatoa aksiomata A2 mo`e da se formulira i vaka:
Postojat barem tri nekolinearni to~ki.
1. Objasni zo{to mno`estvoto ^ $` ne mo`e da bide prava. Od koja aksioma proizleguva
toa?
2. Objasni zo{to mno`estvoto od site to~ki vo prostorot ne pretstavuva prava. Od
koja aksioma proizleguva toa?

3. Nacrtaj dve kru`nici {to se se~at, a potoa objasni zo{to kru`nicite ne mo`eme
da gi smetame kako pravi. Od koja aksioma proizleguva toa?

128
IV. 1.2. ZAEMNA POLO@BA NA TO^KA I RAMNINA

Sekoja ramnina }e ja sfa}ame kako edno mno`estvo ~ii{to elementi se to~ki. Za


nagledno pretstavuvawe na ramninata, obi~no crtame paralelogram (crt.3), no toa ne
zna~i, deka ramninata se sostoi od to~kite vo toj paralelogram. Problemot doa|a ottamu,
{to se obiduvame to~kite od prostorot da gi preneseme vo ramninata na listot. Za edna
to~ka $ i ramnina 6 to~na e samo edna od dvete mo`nosti:
- $ 6 i vo toj slu~aj velime, deka to~kata $ le`i na ramninata 6 , odnosno ram-
ninata 6 minuva niz to~kata $ (crt. 3a).
- $ 6 i vo toj slu~aj veli-
me, deka to~kata $ ne le`i na
ramninata 6 , odnosno ramni-
nata 6 ne minuva niz to~kata
$ (crt. 3b).
Ne sekoe mno`estvo od Crte` 3
to~ki vo prostorot pretstavu-
va ramnina. Za ramninite gi prifa}ame slednite aksiomi:

A4: Koi bilo tri nekolinearni to~ki opredeluvaat edna edinstvena ramnina,
koja minuva niz niv.
A5: Postojat barem ~etiri to~ki vo prostorot, koi ne le`at vo ista ramnina.

Za ~etiri ili pove}e to~ki vo prostorot velime, deka se komplanarni ako postoi
ramnina koja minuva niz tie to~ki. Zatoa aksiomata A5 mo`e da se formulira i vaka:

Postojat barem ~etiri nekomplanarni to~ki vo prostorot.


4. Listovite vo edna tetratka se pricvrsteni na dve mesta za koricata. Kolku
razli~ni polo`bi mo`e{ da napravi{ na eden list od tetratkata bez da go
otcepi{ od koricata? Kolku ramnini minuvaat niz dve razli~ni to~ki vo
prostorot?
5. Objasni zo{to mno`estvoto to~ki od prostorot ne mo`e da pretstavuva
ramnina. Od koja aksioma proizleguva toa?

IV. 1.3. ZAEMNA POLO@BA NA PRAVA I RAMNINA

Za zaemnata polo`ba na prava i ramnina ja prifa}ame slednava aksioma.


A6: Ako dve razli~ni to~ki $ i % , od edna prava le`at vo ramninata 6 , toga{ i
sekoja to~ka od pravata a le`i vo ramninata 6 .
Vrz osnova na ovaa aksioma, lesno }e opredelime {to se mo`e da bide presekot
D ˆ 6 na pravata D i ramninata 6 . Spored A6, ako dve to~ki od pravata D le`at vo

129
ramninata 6 , toga{ D Ž 6 . Zatoa, za proizvolna prava D i proizvolna ramnina 6 to~na
e samo edna od mo`nostite:
- D i 6 nemaat zaedni~ki to~ki, t.e. D ˆ 6 ‡ i vo toj slu~aj velime, deka
pravata a e paralelna so ramninata 6 i pi{uvame D & 6 (crt.4a)

Crte` 4

- D i 6 imaat samo edna zaedni~ka to~ka R, t.e. D ˆ 6 ^3` i vo toj slu~aj velime
pravata a ja proboduva ramninata 6 (crt. 4b), a to~kata R se vika probod.
- sekoja to~ka od pravata D pripa|a na ramninata 6 , t.e. D Ž 6 i vo toj slu~aj velime,
deka pravata D le`i vo ramninata 6 (crt. 4v). Ako D le`i vo 6 toga{, isto taka,
usvojuvame deka D i 6 se paralelni, t.e. D & 6 .
Spored toa, edna prava ili e paralelna so ramninata 6 (ako D ˆ 6 ‡ ili D Ž 6 ),
ili pravata a ja proboduva ramninata 6 (ako D ˆ 6 ^3` ).

Teorema 1. Prava D i to~ka $  D opredeluvaat edinstvena ramnina 6 .

Dokaz*. Neka se dadeni prava a i to~ka $  D . Na pravata D izbirame proizvolni


razli~ni to~ki B i S (crt. 5). Toga{ to~kite $ , B i & ne le`at na ista prava, pa spored
A4 postoi ramnina 6 koja minuva niz $ , % i C. Spored A6 va`i D Ž 6 .
Zna~i, poka`avme deka postoi (barem edna)
ramnina 6 , koja minuva niz A i D Ž 6 .
Dali mo`e da postoi i druga ramina 6 1 so istoto
svojstvo? Ne mo`e, bidej}i ako A  6 1 i D Ž 6 1, toga{
A,B,C  6 1. Osven toa A,B,C  6 , pa spored aksiomata
A4 mora da va`i 6 1= 6 . So toa dokazot e zavr{en.
Crte` 5

IV. 1.4. ZAEMNA POLO@BA NA DVE PRAVI VO PROSTOR

Vo IV.1.1 vidovme deka za dve pravi a i b va`i samo edna od mo`nostite: a i b nemaat
zaedni~ki to~ki; a i b se se~at, t.e. imaat edna zaedni~ka to~ka; ili tie se sovpa|aat,
odnosno a = b. Analogno na teoremata 1 va`i slednata teorema.

*Za onie koi sakaat da gi pro{irat svoite znaewa.

130
Teorema 2. Ako pravite a i b se se~at, toga{ postoi edinstvena ramnina S takva
{to a Ž S i b Ž S.

Dokaz*. Neka pravite a i b se se~at vo to~ka S (crt.6). Na pravata a izbirame to~ka A


razli~na od S, a na pravata b izbirame to~ka V razli~na od S. To~kite A, V i S ne se
kolinearni, pa spored A4 postoi ramnina S koja minuva niz to~kite A, V i S. Od A6
sleduva a Ž S (bidej}i A, S  S) i b Ž 6 (bidej}i V, S  S). Zna~i, takva ramnina S postoi.
Ako S1, isto taka, e ramnina za koja a Ž S1 i b Ž S1, toga{ A, V, S  S1. Od edinstvenosta vo
A4 sleduva deka S1= S. Zna~i ramninata S e edinstvena i so toa dokazot e zavr{en.
Neka ABCDA1B1C1D1 e kocka (crt. 7). Pravite a =
AV i b = SS1 ne le`at vo ista ramnina. Navistina,
koga tie bi le`ele vo ista ramnina S, toga{ i
to~kite A,B,C i C1 }e le`at vo S {to o~igledno e
kontradikcija. Zna~i, postojat pravi {to ne le`at
vo ista ramnina i za takvite pravi velime deka se
razminuva~ki. Crte` 6
6. Na crte` 7 najdi nekolku parovi razminuva~ki pravi.

Definicija. Dve pravi a i b {to le`at na ista ramnina i nemaat zaedni~ki to~ki,
ili se sovpa|aat, se vikaat paralelni pravi i pi{uvame a & b.

7. Na crte` 7 najdi nekolku parovi paralelni pravi.


Vrz osnova na prethodnata diskusija gledame deka za dve pravi
va`i samo edna od mo`nostite:

- pravite se se~at (crt. 8a)

- pravite se paralelni
(ili se sovpa|aat vo specijalen slu~aj)(crt. 8b)
Crte` 7
- pravite se razminuvaat (crt. 8v).

Crte` 8

*Za onie koi sakaat da gi pro{irat svoite znaewa.

131
IV. 1.5. ZAEMNA POLO@BA NA DVE RAMNINI
Za zaemnata polo`ba na dve ramnini ja
prifa}ame slednata aksioma.
A7: Ako dve ramnini imaat edna zaedni~ka
to~ka, toga{ tie imaat barem u{te edna
zaedni~ka to~ka.
Od A7 neposredno zaklu~uvame deka ako S1 i
S2 se razli~ni ramnini koi imaat zaedni~ka to~ka
A, toga{ tie imaat barem u{te edna zaedni~ka
to~ka V, pa spored toa imaat i zaedni~ka prava
Crte` 9 AV. (crt. 9). No, osven pravata AV tie ne mo`at
da imaat u{te nekoja zaedni~ka to~ka. Navistina, ako pokraj pravata AV imaat i
zaedni~ka to~ka S {to ne pripa|a na taa prava, toga{ spored teorema 1 pravata AV
i to~kata S opredeluvaat edinstvena ramnina koja gi sodr`i pravata i to~kata.
Zna~i, mora da e S1 = S2, a ova protivre~i na na{ata pretpostavka deka S1 z S2.
8. Navedi nekolku para ramnini na crte` 7, koi {to se se~at.
Ako dve ramnini se ednakvi kako mno`estva to~ki, t.e.
ako S1 = S2, za niv velime deka se sovpa|aat. Za niv ~esto
pi{uvame S1 { S2. Za dve ramnini da se sovpa|aat dovolno e
tie da imaat tri zaedni~ki nekolinearni to~ki (crt. 10).
Crte` 10

Definicija. Za dve ramnini S1 i S2 velime deka se paralelni (S1 & S2) ako tie
nemaat zaedni~ka to~ka ili se sovpa|aat.

9. Navedi nekolku para paralelni ramnini sprema crte` 7.


Spored prethodno ka`anoto, gledame deka dve ramnini
S1 i S2 imaat samo edna od slednite dve zaemni polo`bi:
- tie se paralelni - crte` 11 (ili se sovpa|aat vo speci-
jalen slu~aj - (crt. 10)
- tie se se~at - crte` 9.
Crte` 11

10. Kolku pravi minuvaat niz: a) edna to~ka, b) dve razli~ni to~ki vo prostorot?
11. Kolku pravi opredeluvaat tri razli~ni to~ki {to ne le`at na ista prava?
12. Kolku pravi opredeluvaat ~etiri to~ki vo prostorot, takvi {to koi bilo tri
ne le`at na ista prava?
13. Doka`i deka ramnina i edna prava {to ne le`i na nea ne mo`at da imaat pove}e
od edna zaedni~ka to~ka.

132
14. Poznato e deka r ˆ S={M} i a & S. Kakva zaemna polo`ba mo`at da imaat pravite
r i a?
15. Mo`e li presekot na dve ramnini da se sostoi to~no od 5 to~ki?
16. Dali e dovolno samo da se ka`e:
a) Ramninata e opredelena so tri to~ki, b) Ramninata e ednozna~no opredelena
so dve pravi, v) Ramninata e ednozna~no opredelena so prava i to~ka? Koj uslov
treba da se istakne?
17. Neka ABCDA'B'C'D' e kvadar. Skiciraj gi ramninite opredeleni so: a) pravata
AB i to~kata C', to~kite A, D i B', v) pravite BD i B'D'.
18. Pravata r ja proboduva ramninata S. Dali mo`e na ramninata S da le`i prava
{to }e bide paralelna na pravata r?
19. Kakvi zaemni polo`bi mo`at da imaat dve pravi: a) vo ramnina, b) vo prostorot?
20. Kolku pravi mo`at da se povle~at od dadena to~ka, koi se normalni na dadena
prava?

IV. 2. PARALELNO PROEKTIRAWE I ORTOGONALNO


PROEKTIRAWE
IV. 2.1. NORMALA NA RAMNINA

Definicija. Za pravata a }e velime deka e normalna na ramninata S, ako pravata


a e normalna na sekoja prava koja minuva niz probodot na pravata i ramninata
(crt. 12) i pi{uvame a A S.
No, za da proverime dali edna prava a e normalna
na S, dali ima potreba da proveruvame deka pravata a e
normalna na sekoja prava vo S {to minuva niz probodot
P (a takvi pravi ima bezbroj), }e go napravime sledniot
obid. Na karton postavuvame dva pravoagolni tria-
golnika taka {to }e pripoime dve kateti od triagol-
nicite, a ostanatite dve kateti neka le`at na kartonot
(crt. 13) pri {to zafa}aat agol razli~en od 0o i 180o.
Crte` 12
Pripoenite kateti opredelu-
vaat prava r, a drugite dve kateti
neka opredeluvaat pravi a i b.
O~igledno r A a i r A b. Ako sega
tret pravoagolen triagolnik go
postavime taka {to ednata kateta
se pripoi kon dvete sovpadnati
kateti, }e zabele`ime deka vto-
rata kateta celosno }e le`i na
kartonot. Ako ovaa kateta oprede-
luva prava s (crt. 13), obidot poka- Crte` 13
`uva s A r.

133
Vsu{nost, obidot poka`uva deka va`i slednata teorema koja nema da ja doka`uvame.

Teorema 1. Ako edna prava p ja proboduva dadena ramnina 6 i e normalna na dve


razli~ni pravi od taa ramnina koi minuvaat niz probodot p ˆ 6 , toga{ pravata p
e normalna na ramninata 6 .

Ovaa teorema ima golema prakti~na primena. Na primer, vo grade`ni{tvoto, yidarot


so pomo{ na dve merewa mo`e da proveri dali nekoja cevka e normalna na nekoja ramnina.
]e navedeme u{te dve teoremi bez dokaz.

Teorema 2. Ako dve pravi a i b se normalni na edna ramnina 6 , toga{ pravite a i b


se paralelni.
Teorema 3. Niz dadena to~ka A mo`e da se povle~e samo edna prava koja e normalna na
dadena ramnina.

Vo teorema 3 lesno se poka`uva deka ne e mo`no od dadena


to~ka A da se povle~at dve normali kon dadena ramnina 6 .
Navistina, ako mo`e da se povle~at dve normali, toga{ tie
neka ja se~at ramninata 6 vo to~ki X i Y. Triagolnikot AXY
ima dva pravi agla: vo to~ka X i vo to~ka Y, {to e kontradikcija.
Neka A e proizvolna to~ka, a 6 proizvolna ramnina i A 6 .
Da povle~eme normala od to~kata A kon ramninata 6 . Neka A'
Crte` 14
e probodot na normalata so ramninata 6 . Dol`inata $$ ' se
narekuva rastojanie od to~kata A do ramninata 6 .

IV.2.2. PARALELNO PROEKTIRAWE

Zemete eden predmet, na primer, penkalo,


i nabquduvajte ja negovata senka vrz ramen list
hartija. So vrtewe na listot }e zabele`ite
deka dol`inata na senkata na penkaloto mo`e
da bide pogolema, ednakva ili pomala od
dol`inata na penkaloto. Vo sekoja fiksna
polo`ba na listot, vsu{nost, stanuva zbor za
paralelno proektirawe od pri~ina {to
son~evite zraci mo`eme da gi smetame za
Crte` 15
paralelni.
Paralelnoto proektirawe go definirame na sledniot na~in. Neka e dadena ramnina
6 i prava r koja ne e paralelna so 6 (crt. 15). Niz proizvolna to~ka A od prostorot
povlekuvame prava paralelna so r i neka taa prava ja proboduva ramninata 6 vo to~ka A'.
Vo specijalen slu~aj, ako V  6 , toga{ V' se sovpa|a so V. So toa e definirano
preslikuvawe od mno`estvoto na to~ki vo prostorot na ramninata 6 koe se narekuva
paralelno proektirawe.

134
To~kata A' se vika proekcija na to~kata A vrz ramninata 6 . Pravecot opredelen so
pravata r se narekuva proektira~ki pravec, a ramninata 6 se narekuva proekciona
ramnina.
1. Ako r e proektira~ki pravec za edna paralelna proekcija, koj e potreben i dovolen
uslov da to~kite A i V se proektiraat vo ista to~ka?

IV.2.3. ORTOGONALNO PROEKTIRAWE

Vo specijalen slu~aj ako pri paralelnoto proek-


tirawe proektira~kiot pravec e normalen na ramni-
nata 6 , dobivame ortogonalno proektirawe. Zatoa
ortogonalnoto proektirawe se zadava samo so proek-
cionata ramnina 6 . Ako to~kata A se proektira vo
to~ka A', toga{ za to~kata A' velime deka e ortogo-
nalna proekcija na to~kata A vrz ramninata 6
(crt.16).
Crte` 16
Ako paralelnoto proektirawe ne e ortogonalno,
toga{ velime deka e koso proektirawe.
Ako e dadena figura F, toga{ nejzina proekcija vrz ramninata 6 e mno`estvo to~ki
{to se proekcii na to~kite od figurata F. Na crt. 17 a), b), v) dadeni se ortogonalnite

Crte` 17
proekcii na otse~ka AV. Zabele`uvame deka, ako AV & 6 , toga{ otse~kata A'V' e para-
lelna i ednakva na AB, a ako AV A 6 , toga{ proekcijata pretstavuva to~ka.

Na crte` 18 a),
b), v) dadeni se
ortogonalni
proekcii na prava a.
Zabele`uvame deka
nejzinata proekcija e
prava (crt. 18 a) ili
to~ka (crt. 18b).

Crte` 18

135
Na crte` 19 a), b) dadeni se proekcii na triagolnik. Zabele`uvame deka proekcijata
mo`e da bide triagolnik (crt. 19a), no, mo`e da bide i otse~ka (crt. 19b) dokolku A',V' i
S' se kolinearni to~ki.

Crte` 19

2. Vo proekcionata ramnina izberi: a) otse~ka, b) prava, v) triagolnik a potoa nacrtaj


gi soodvetnite ortogonalni proekcii.
Za ortogonalnite proekcii va`at slednive dve teoremi.

Teorema 4. Za proizvolna otse~ka AB i nejzinata ortogonalna proekcija A'B' va`i


neravenstvoto $% t $c%c .
Teorema 5. Neka to~kata A ne le`i vo ramninata 6 i neka A' e ortogonalna proekcija
na to~kata A vrz 6 , a B e proizvolna to~ka od 6 razli~na od A'. Toga{ va`i $$c  $% .

I dvete teoremi se posledica na faktot deka hipotenuzata e pogolema od katetite.


Od teoremata 5 proizleguva deka rastojanieto od to~kata A do ramninata 6 e najkratkoto
od site rastojanija $% , kade V  6 .

3. Kolku ramnini minuvaat niz dadena to~ka, koi{to se normalni na dadena prava r?
4. Ako a i b se paralelni pravi, {to mo`e{ da ka`e{ za nivnite sliki a' i b' pri
ortogonalna proekcija?
5. Ako a i b se normalni pravi, dali mora i nivnite sliki a' i b' da bidat normalni
pravi pri: a) paralelna proekcija b) ortogonalna proekcija?
6. [to pretstavuva proekcijata na ~etiriagolnik pri ortogonalno proektirawe? Dali
mo`e da bide triagolnik?

136
7. Neka A, V i S se tri kolinearni to~ki pri {to V le`i me|u A i S. Dali pri
ortogonalna proekcija A', V' i S' se kolinearni i dali mora V' da le`i me|u A' i S'?
8. Doka`i ja teoremata 4. Koga va`i ravenstvoto?
9. Doka`i ja teoremata 5.
10. Dali otse~ki so ednakvi dol`ini pri ortogonalna proekcija se preslikuvaat vo
otse~ki so ednakvi dol`ini?

IV.3. AGLI ME\U PRAVI I RAMNINI*

Neka a i b se neparalelni pravi vo prostorot. Toga{ tie ili se se~at ili se


razminuva~ki. Ako a i b se se~at, toga{ tie le`at vo edna ramnina 6 . Pod agol me|u
pravite a i b go podrazbirame agolot {to tie go obrazuvaat vo ramninata 6 . Ako
pravite se razminuva~ki,
izbirame proizvolna to~ka O i
niz nea povlekuvame pravi a' & a
i b' & b. (crt. 20) Pravite a' i b' se
se~at, pa, pod agol me|u razmi-
nuva~kite pravi a i b go pod-
razbirame agolot pome|u a' i b'.
Zna~i, vo sekoj slu~aj ago-
lot me|u dve pravi se sveduva na
Crte` 20
agol me|u dve pravi {to se
se~at. Da napomeneme deka, dve
pravi {to se se~at tie
obrazuvaat dva naporedni agla a nie se dogovarame da go zememe pomaliot od niv. Zatoa
ako M e agol me|u dve pravi, toga{ 0° d M d 90°. Ako pravite a i b se paralelni, toga{
usvojuvame deka agolot me|u niv e 0°.
1. Nacrtaj kvadar a potoa izberi nekolku pravi i opredeli gi aglite me|u niv.

Ako agolot me|u dve pravi e 90°, za pravite


velime deka se normalni. Neka se dadeni prava
a i ramnina 6 . Agol me|u pravata a i ramninata
6 e agolot pome|u pravata a i nejzinata
ortogonalna proekcija a' vrz ramninata 6 (crt.
21) i ozna~uvame ( (a, 6 ). Vo slu~aj koga prava-
ta a e normalna na ramninata 6 , proekcijata a' e
to~ka i toga{ dopolnitelno definirame deka
Crte` 21 ( (a, 6 ) = 90°.

*Za onie koi sakaat da gi pro{irat svoite znaewa.

137
2. Neka a Ž 6 . Opredeli go agolot ) (a, 6 ).
Na kraj da zememe dve ramnini 6 1 i 6 2 koi ne se paralelni. Toga{ tie se se~at vdol`
nekoja prava p. Postavuvame ramnina P koja e normalna na prese~nata prava p (crt. 22).
Ramninata P gi se~e ramninite 6 1 i 6 2 vdol` pravite V i V soodvetno.

Crte` 22

Pravite V i V obrazuvaat dva naporedni agla D i E . Pomaliot od ovie dva agla go


vikame agol me|u ramninite 6 1 i 6 2. Zna~i, za agolot M me|u dve ramnini va`i
0° d M d 90°. Ako ramninite se paralelni, toga{ usvojuvame deka agolot me|u niv e 0°.
Ako agolot me|u dve ramnini e 90°, za ramninite velime deka se zaemno normalni.
Normalni se, na primer, ramninata na podot i ramninata na nekoj yid vo soba. Mo`e da
se doka`e tvrdeweto deka, ramninite 6 i P se normalni ako vo ramninata 6 postoi
prava a koja{to e normalna na ramninata P.
3. Otvori edna kniga taka {to dvete korici obrazuvaat nekoj agol. So pomo{ na
aglomer izmeri go toj agol.

4. Neka pravata p ja proboduva ramninata 6 . Razgledaj go agolot me|u pravata p i


proizvolna prava vo ramninata 6 . Koj od tie agli e najmal? (Mo`e{ da izbere{
list od hartija za ramnina 6 , a penkalo za prava p).
5. Neka ramninite 6 1 i 6 2 se se~at vdol` pravata p. Niz proizvolna to~ka od pravata
p povle~i pravi s1i s2 vo ramninite 6 1 i 6 2 koi se normalni na pravata p.
Dali ) (s1, s2) = ) ( 6 1, 6 2)?
6. Koj uslov e potreben i dovolen za proekcijata na triagolnikot AVS vrz ramninata
6 da bide otse~ka?

138
7. Neka 6 1 i 6 2 se dve ramnini, a s1i s2 se pravi takvi {to s1 A ¦ i s2 A ¦  .
Poka`i deka ) ( 6 1, 6 2) = ) (s1,s2).
8. Neka ramninite 6 1 i 6 2 se normalni. Ako niz proizvolna to~ka od prese~nata prava
izdigneme normala s na ramninata 6 1, kade }e le`i pravata s?
9. Od to~kata M, {to le`i nadvor od ramninata 6 , povle~ena e navednata otse~ka
0$ =12 FP , koja so ramninata 6 obrazuva agol od: a) 30°, b) 45°, v) 60°. Odredi go
rastojanieto od to~kata M do ramninata 6 .
10. Ednata kateta na eden ramnokrak pravoagolen triagolnik le`i na ramninata 6 .
Hipotenuzata na toj triagolnik so ramninata 6 zafa}a agol od 30°. Da se odredi
agolot me|u drugata kateta i hipotenuzata.
11. Otse~kata AB so dol`ina $%  FP e paralelna na dadena ramnina 6 . Otse~kata
VA', kade A' e ortogonalnata proekcija na A vrz 6 , so 6 obrazuva agol od 60°. Da se
odredi dol`inata na otse~kata VA' i rastojanieto na otse~kata AB do ramninata 6 .

IV.4. GEOMETRISKI TELA

Predmetite koi{to n– opkru`uvaat okolu nas se razli~ni vidovi na fizi~ki tela.


Za site niv e osnovno {to tie se del (podmno`estvo) od prostorot. Fizi~kite tela se
karakteriziraat i spored materijata od koja se sostaveni. Od geometriska gledna to~ka
nas ne n– interesira materijata od koja se sostaveni, pa zatoa apstrahiraj}i go ova svojstvo
na fizi~kite tela gi dobivame geometriskite tela.
Dve svojstva se karakteristi~ni za geometriskite tela. Geometriskoto telo mora da
e ograni~eno od site strani. Toa zna~i, deka vo koja bilo nasoka vo prostorot, toa ne
mo`e da se protega do beskrajnost. Ednostavno, }e velime deka toa e ograni~eno. Vtoroto
svojstvo e povrzano so povr{inata na teloto. Granicata, koja go odvojuva delot od
prostorot koj go zafa}a geometriskoto telo od ostanatiot del nadvor od nego, se narekuva
povr{ina na teloto.
Povr{inata na geometriskoto telo go deli prostorot na vnatre{na i nadvore{na
oblast. Unijata na vnatre{nata oblast i granicata go so~inuvaat teloto. Sega mo`eme
da definirame:
Geometriskoto telo e ograni~en del (podmno`estvo) od prostorot, koj svojata granica
(povr{ina na teloto) ja sodr`i kako podmno`estvo.
Primeri za telo se: kocka, kvadar, topka, konus, cilindar, piramida i dr. Da ja
razgledame topkata kako geometrisko telo. Taa se definira kako mno`estvo to~ki vo
prostorot koi se nao|aat na rastojanie ne pogolemo od R od nekoja fiksna to~ka O,
odnosno {X| 2; d R} (crt. 23a). Granicata (negovata povr{ina) vo ovoj slu~aj e sferata
{ ; 2; = R} i toa e podmno`estvo na topkata.

139
Me|utoa, mno`estvoto {X | 2; < R} (crt. 23b) ne e geometrisko telo, bidej}i vo sebe
ne ja sodr`i granicata.

Povr{inata na geometriskoto
telo mo`e da bide sostavena od
ramni delovi (mnoguagolnici) i od
zakriveni povr{ini. Vo zavisnost
od toa, kakva e povr{inata, razli-
kuvame rabesti i val~esti tela.
Crte` 23

Geometriskoto telo {to e zagradeno samo od ramni povr{ini (mnoguagolnici), se


vika rabesto telo.
Geometriskoto telo {to e zagradeno samo so kriva povr{ina ili so krivi i ramni
povr{ini se vika val~esto telo.

Od rabestite tela, ponatamu }e gi izu~uvame prizmata i piramidata, a od val~estite


tela }e gi izu~uvame cilindarot, konusot i topkata. Rabesto telo, ~ija granica se sostoi
od kone~en broj mnoguagolnici, se vika poliedar. Mnoguagolnicite {to ja obrazuvaat
granicata na poliedarot se vikaat yidovi, a nivnite strani se narekuvaat rabovi na
poliedarot. Temiwata na mnoguagolnicite, isto taka, gi vikame temiwa na poliedarot.
1. Kolku a) yidovi, b) rabovi, v) temiwa, ima eden kvadar?
Pri crtaweto na geometriskite tela, nie, vsu{nost, gi crtame nivnite proekcii vrz
nekoja ramnina vo zavisnost od proektira~kiot pravec, odnosno nasokata od koja nie go
nabquduvame teloto. Bidej}i dol`inite na proekciite na otse~kite, vo op{t slu~aj, ne
se ednakvi na vistinskata golemina,
zatoa, pri crtawe na geometriskite tela,
ne sekoga{, se zapazuvaat dol`inite na
rabovite. Zatoa, stranite na kvadratot gi
crtame kako paralelogrami. Voobi~aeno
e geometriskite tela da gi crtame kako
da gi gledame odnapred, malku odozgora i
malku od levo ili od desno. Na crte` 24a
Crte` 24 nacrtana e kocka gledana od leva strana,
a na crte` 24b gledana od desna strana.
Koga kockata ja gledame, samo odnapred (nitu malku odozgora nitu od levo nitu od
desno) crte`ot bi bil samo kvadrat i toa ne dava pretstava za kockata. Koga od istata

140
nasoka crtame konus ili cilindar bi dobile triagolnik i pravoagolnik, a toa ne dava
vizuelna pretstava za soodvetnoto telo. Koga bi gi gledale konusot i cilindarot malku
od visoko }e go dobieme crte` 25 a, b.
Zabele`uvame deka osnovite na
konusot i cilindarot ne gi crtame kako
kru`nici, tuku kako elipsi.
Voobi~aeno e nevidlivite rabovi
ili krivi linii da gi crtame so ispreki-
nati linii.

Crte` 25

2. [to e geometrisko telo?


3. [to e a) rabesto telo, b) val~esto telo?
4. [to se a) yidovi, b) rabovi, v) temiwa na poliedarot?
5. Dali od edno teme na poliedarot mo`e da izleguvaat samo
a) 2 raba, b) 3 rabovi, v) 4 rabovi?
6. Cilindar i konus se postaveni na horizontalna ramnina S. Nacrtaj kako }e izgledaat
ovie dve tela gledani odozgora.
7. Dadena e kocka ABCDA1B1C1D1. Nacrtaj ja ovaa kocka ako taa se nabquduva od pravec
opredelen so dijagonalata AC1.

B PRIZMA
PRIZMA

IV.5. POIM I ELEMENTI NA PRIZMATA

Neka p i p1 se dve paralelni ramnini, a p - edna prava {to gi se~e ramninite p i p1 (crt.
26). Na ramninata p da zememe eden koj i da bilo konveksen mnoguagolnik, na primer,
petagolnikot ABCDE i niz negovite temiwa da povle~eme pravi - paralelni na pravata p.
Taka povle~enite pravi }e ja probodat ramninata p1 vo pet to~no odredeni to~ki
A1B1C1D1E1, koi opredeluvaat drug petagolnik A1B1C1D1E1 vo ramninata p1 (crt. 26).

141
JJJJG
O~igledno e deka so translacija za vektor $ $ petagolnikot A1B1C1D1E1 }e se sovpadne
so petagolnikot ABCDE. Spored toa, tie se skladni: ABCDE # A1B1C1D1E1.
^etiriagolnicite ABB1A1, BCC1B1, CDD1C1, itn. se paralelogrami (crt.26) (Zo{to?).
Povr{inata, koja{to e sostavena od
dvata skladni petagolnika ABCDE i
A1B1C1D1E1 i pette paralelogrami ABB1A1,
BCC 1B 1,... (crt. 26b) go razbiva mno-
`estvoto to~ki od prostorot, {to ne —
pripa|at na dve oblasti: vnatre{na i
nadvore{na. Unijata od taa povr{ina i
vnatre{nata oblast pretstavuva edna
zatvorena i ograni~ena prostorna oblast,
t.e. edno geometrisko telo i toa poliedar.
Crte` 26

Toj poliedar (crt. 26b) go vikame petagolna prizma.


Vo ramninata S , ako namesto petagolnik zememe drug mnoguagolnik, na primer:
triagolnik, ~etiriagolnik, odnosno {estagolnik, na sli~en na~in, }e dobieme i drugi
poliedri, koi se vikaat triagolna, ~etiriagolna, odnosno {estagolna prizma (crt. 27).
Site tie so zaedni~ko ime se vikaat prizmi.

Crte` 27

Definicija: Eden poliedar se vika prizma, ako dva negovi yida se dva skladni
mnoguagolnika, {to le`at vo dve paralelni ramnini, a drugite yidovi se paralelogrami
{to imaat po edna zaedni~ka strana so sekoj od spomenatite yidovi.

Dvata skladni mnoguagolnika se vikaat u{te i osnovi (ili bazi) na prizmata, a


paralelogramite - bo~ni (ili okolni) yidovi na prizmata. Site bo~ni yidovi na prizmata
velime deka ja obrazuvaat bo~nata povr{ina na prizmata.

142
Stranite na osnovite gi vikame osnovni rabovi na prizmata (AB,BC,...,A1B1,...-crt. 27),
a site drugi rabovi - bo~ni rabovi na prizmata (AA1,BB1,CC1,…-crt. 27). Temiwata na
osnovite gi vikame temiwa na prizmata.
Otse~kite {to svrzuvaat dve temiwa na edna prizma, koi ne le`at na ist yid, se
vikaat prostorni dijagonali ili samo dijagonali na taa prizma.
Rastojanieto od proizvolna to~ka O1 od ednata osnova do drugata osnova, t.e.
rastojanieto od to~kata O1 do nejzinata ortogonalna proekcija O, se vika visina na
prizmata i ja ozna~uvame so N, t.e. 22 + (crt. 27).
Od definicijata na prizmata sleduvaat slednive dve va`ni svojstva na prizmata:

1°Site bo~ni rabovi na prizmata se paralelni i ednakvi me|u sebe, kako otse~ki od
paralelni pravi zaklu~eni me|u dve paralelni ramnini.
2° Sekoi dva osnovni raba {to le`at na ist bo~en yid na prizmata, se paralelni i
ednakvi.

1. [to e prizma, {to se bo~ni yidovi a {to e bo~na povr{ina na prizmata?


2. Koi se osnovni, a koi bo~ni rabovi na prizmata?
3. Nacrtaj edna ~etiriagolna prizma, a potoa povle~i gi site nejzini prostorni
dijagonali.
4. Koi svojstva gi imaat bo~nite rabovi na prizmata, a koi osnovnite rabovi {to
le`at na eden ist bo~en yid?
5. Kolku osnovni, a kolku bo~ni rabovi ima: a) triagolna, b)~etiriagolna, v) petagolna
prizma?
6. [to e visina na prizma?

IV.6. VIDOVI I PRESECI NA PRIZMITE

Vo zavisnost od toa dali osnovata na prizmata e triagolnik, ~etiriagolnik,


petagolnik, itn., prizmata soodvetno ja vikame triagolna, ~etiriagolna, petagolna, itn.
Razgledajte gi crte`ite 26 i 27! Na niv se nacrtani po edna triagolna, ~etiriagolna,
petagolna i {estagolna prizma. Kako {to gledate, so neprekinati linii gi crtame site
vidlivi rabovi, a so isprekinati liniii - site nevidlivi rabovi.

143
Ako bo~nite rabovi na edna prizma se normalni na nejzinite osnovi, toga{ nea ja
vikame prava prizma. Vo sprotiven slu~aj, prizmata ja vikame kosa. Nie }e gi razgle-
duvame samo pravite prizmi. Kaj pravite prizmi visinata e ednakva na dol`inata na
bo~nite rabovi, a site bo~ni yidovi se pravoagolnici (Zo{to?).

Definicija: Prava prizma, na koja osnovite se pravilni mnoguagolnici, se vika


pravilna prizma.

Za pravilnite prizmi karakteristi~no e toa {to: Site osnovni rabovi se ednakvi


me|u sebe, a site bo~ni yidovi se skladni pravoagolnici.

Ramninata {to minuva niz edna dijagonala na osnovata na prizmata i niz eden nejzin
bo~en rab, se vika dijagonalna ramnina; a ramninskata figura {to se dobiva koga prizmata
ja prese~eme so dijagonalnata ramnina se vika
dijagonalen presek na prizmata. Na crte` 28
paralelogramot AA1C1C e eden dijagonalen presek na
nacrtanata prizma.
O~igledno e deka: sekoj dijagonalen presek na
prizmata e paralelogram, a kaj pravata prizma toj e
pravoagolnik.
Razlikuvame u{te presek {to se dobiva koga
prizmata ja prese~eme so ramnina paralelna na
osnovite na prizmata, nare~en paralelen presek
(crt. 29). O~igledno e deka kaj sekoja prizma para-
Crte` 28 Crte` 29 lelniot presek e figura skladna na osnovite na
prizmata.

1. [to pretstavuvaat bo~nite yidovi na kosata prizma? Dali mo`e me|u niv da ima i
pravoagolnik?
2. Ako site bo~ni yidovi na edna prizma se pravoagolnici, kakva e taa prizma?
3. Zo{to bo~nite yidovi na prizmata se paralelogrami?
4. Koi preseci na prizmata se vikaat dijagonalni? Koi prizmi nemaat dijagonalni
preseci?
5. [to pretstavuvaat dijagonalnite preseci na pravite prizmi?

144
6. Kolku dijagonalni preseci mo`at da se napravat niz eden bo~en rab kaj:
a) triagolnata, b) ~etiriagolnata, v) petagolnata, g) n - agolnata prizma?
7. Kolku dijagonali, a kolku dijagonalni preseci mo`at da se napravat niz eden bo~en
rab kaj: a) ~etiriagolna, b) petagolna, v) {estagolna prizma?

IV.7. PARALELOPIPED

Definicija 1. Prizma, ~ii{to osnovi se paralelogrami, ja vikame paralelopiped.

Zna~i, site yidovi na paralelopipedot se paralelogrami.


Paralelopipedite, kako i sekoja prizma mo`at da bidat pravi ili kosi (crt. 30).

Definicija 2. Prav paralelopiped, na koj osnovite mu se pravoagolnici, se vika u{te


i pravoagolen paralelopiped ili kvadar (crt. 30b).

Kvadarot vi e dobro poznat na site. Site negovi yidovi se pravoagolnici, pa zatoa


koi i da bilo tri raba {to izleguvaat od edno negovo teme se dva po dva zaemno normalni
(crt. 30).
Dol`inite na tri
rabovi {to izleguvaat
od edno isto teme na
kvadarot se vikaat di-
menzii (dol`ina, {i-
rina i visina) na
kvadarot (crt. 30).
O~igledno e deka:
kvadarot {to ima dve Crte` 30
ednakvi dimenzii pretstavuva pravilna ~etiriagolna prizma, a kvadar na koj trite
dimenzii mu se ednakvi, se vika kocka (crt. 30 v).
Paralelopipedot ima ~etiri dijagonali AC1 ,DB1 ,BD1 i CA1 (crt. 31).

Teorema 1. Dijagonalite na paralelopipedot se se~at vo edna to~ka i se prepo-


lovuvaat od nea.

145
Dokaz*: Da gi razgledame dijagonalite BD1 i CA1 (crt. 31). Tie le`at vo dijagonalniot
presek BCD1A1. Bidej}i ~etiriagolnicite ABCD i ADD1A1 se paralelogrami, zatoa i
dijagonalniot presek BCD 1A 1 e paralelogram. Zna~i, dijagonalite BD1 i CA1 na
paralelopipedot, kako dijagonali na paralelogramot BCD1 A1 se se~at vo to~kata O i se
prepolovuvaat od nea. Se doka`uva deka i drugite dve dijagonali AC 1 i DB 1 na
paralelopipedot se se~at vo istata to~ka O i se prepolovuvaat od nea.
Yidovite na paralelopipedot {to nemaat zaedni~ki temiwa se vikaat sprotivni yidovi.

Teorema 2. Sprotivnite yidovi na paralelopipedot se paralelni i skladni.

Dokaz*: Da gi razgledame sprotivnite yidovi ABB1A1 i


DCC1D1 na paralelopipedot (crt. 31). Bidej}i site yidovi na
paralelopipedot se paralelogrami, zatoa: AB & DC, AA1 & DD1.
Spored toa, yidovite ABB1A1 i DCC1D1 na paralelopipedot se
paralelni.
Od toa {to site yidovi na paralelopipedot se parale-
logrami sleduva deka rabovite AD, BC, B1C1 i A1D1 se paralelni
JJJG
i ednakvi. Spored toa, pri translacija za vektor $' yidot
ABB1A1 }e se sovpadne so sprotivniot yid DCC1D1 na parale-
lopipedot (crt.31). Zna~i, tie se skladni.
Crte` 31

Teorema 3. Kvadratot od dijagonalata na pravoagolniot paralelopiped (t.e.


kvadarot) e ednakov na zbirot od kvadratite na trite negovi dimenzii.

Dokaz: Dimenziite na kvadarot (na crt. 32) neka se


$% D $' E i $$ F a dol`inite na dijagonalata na
kvadarot i dijagonalata na negovata osnova da gi ozna~ime
so d i d1, t.e. neka &$ G i $& G (crt. 32). Znaeme deka,
kaj kvadarot bo~niot rab e normalen na ramninata na negovata
osnova. Zna~i, triagolnikot A1AC e pravoagolen. Od nego
imame: d2 = d12 + c2 . A od pravoagolniot triagolnik ABC
(AB A BC) imame: d12=a2+ b2.
Ottuka dobivame: d2=a2+ b2+ c2.

Crte` 32

Posledica: Dijagonalite na kvadarot se ednakvi. (Zo{to?).

*Za onie koi sakaat da gi pro{irat svoite znaewa.

146
Bidej}i trite dimenzii na kockata se ednakvi a = b = c, zatoa, vo soglasnost so gornava
teorema za nejzinata dijagonala d, dobivame:

d2=a2+ a2+ a2 ili d2=3a2, odnosno G D .


Na primer, ako kockata ima rab a = 8 cm, nejzinata dijagonala }e bide dolga:

G D    |  ˜    (cm).

1. Koja prizma se vika paralelopiped? Vo {to e razlikata me|u prav i pravoagolen


paralelopiped?
2. Kakva figura e presekot na edna kocka i ramnina {to e normalna na edna nejzina
dijagonala?
3. Dali mo`e kockata da se prese~e so edna ramnina, taka {to, da se dobie presek:
a) ramnostran triagolnik, b) ramnokrak triagolnik, v) pravoagolnik, g) kvadrat,
d) trapez?
4. Odredi ja dol`inata na dijagonalata na kvadar, ako negovite dimenzii se:
a) 3 cm, 4 cm, 12 cm, b) 2 cm, 3 cm, 6 cm!
5. Odredi ja dol`inata na dijagonalata na kocka, ~ij rab e 18 cm!
6. Odredi go rastojanieto od temeto na kocka do negovata dijagonala, ako nejziniot
rab e e dolg 1 m!
7. Poznati se dve dimenzii na eden kvadar 12 cm i 21 cm i dol`ina na dijagonalata
29 cm. Odredi ja negovata treta dimenzija!

IV. 8. MRE@A NA PRIZMA

Povr{inata so koja e ograni~ena


prizmata, da zamislime, deka e
razre`ana po eden bo~en rab i po
site (osven po eden) osnovni rabovi
na nejzinite osnovi. Ako potoa site
nejzini yidovi gi soborime (leg-
neme) vo edna ramnina, }e dobieme
figura, koja se vika mre`a na priz-
mata (crt. 33).
Crte` 33

147
O~igledno e deka bo~nata povr{ina na sekoja prava prizma se razviva vo eden op{t
pravoagolnik ABCD, {to e sostaven od tolku pravoagolnici, kolku {to prizmata ima
bo~ni yidovi. Toj pravoagolnik ima dol`ina ednakva na perimetarot na osnovata na
prizmata i visina ednakva na visinata na prizmata (crt. 33).

Zada~a: Da ja nacrtame mre`ata na edna pravilna triagolna prizma so osnoven rab


a = 1,5 cm i visina H = 2 cm.

Bo~nata povr{ina na pravilnata triagolna prizma


se sostoi od tri skladni pravoagolnici, ~ii strani se a
i H, a nejzinite osnovi se ramnostrani triagolnici so
strana a. Koga toa go znaeme, crtaweto na mre`ata stanuva
vaka: prvo crtame eden do drug tri pravoagolnici so
strani a = 1,5 cm i H = 2 cm, a potoa nad sprotivnite strani
na koj i da bilo od niv crtame u{te dva ramnostrani
triagolnika so strana a = 1,5 cm. Taka ja dobivame baranata
mre`a na pravilnata triagolna prizma (crt. 34).
Zo{to ni e potrebno crtaweto mre`i na geometri-
skite tela? Crte` 34

Mre`ite gi koristime za pravewe (sklopuvawe) modeli na telata, a, isto taka, tie


ni pomagaat i za presmetuvawe na plo{tinata na povr{inite na tie tela.
Za da napravime (sklopime) model na prizmata potrebno e prvo, da ja nacrtame
nejzinata mre`a na karton, a potoa da ja ise~eme i kartonot da go previtkame po
isprekinatite linii. Ako sakame taka sklopeniot model na prizmata da ostane sos-
taven, potrebno e so hartija za lepewe (selotejp) da gi pokrieme (zalepime) yidovite na
prizmata {to se zdru`uvaat.

Crte` 35 Crte` 36

Na crte` 35 i 36 nacrtani se mre`ite na edna pravilna {estagolna prizma i edna


prava ~etiriagolna prizma, ~ija osnova e ramnokrak trapez. Napravete modeli na tie
prizmi.

148
1. Nacrtaj mre`a na pravilna petagolna prizma, ~ij osnoven rab e a = 1,5 cm i visinata
H = 5 cm.
2. Nacrtaj mre`a na {estagolna prizma, ~ij osnoven rab e 1,2 cm, a visinata 4 cm.
3. Nacrtaj ja mre`ata na kocka, ~ij rab e a = 2cm!
4. Nacrtaj ja mre`ata i napravi model na pravilna {estagolna prizma, ~ij osnoven
rab e a = 2cm i visinata H = 5 cm.
5. Kolku oski na simetrija ima mre`ata na pravilna triagolna prizma {to e nacrtana
na crte` 34?
6. Pravoagolnik so dol`ina 9 cm i {irina 6 cm pretstavuva bo~na povr{ina na edna
pravilna triagolna prizma. Nacrtaj ja mre`ata na taa prizma! (Vnimavaj, mo`ni se
dve re{enija.)
7. Nacrtaj ja mre`ata i napravi model na pravilna {estagolna prizma kaj koja
pogolemiot dijagonalen presek pretstavuva kvadrat. Ako osnovniot rab na prizmata
e 2 cm, kolakava e visinata na taa prizma?
8. Nacrtaj mre`a i napravi model na kvadar, koj ima dol`ina a = 2cm i {irina b = 3cm,
a dijagonalniot presek mu pretstavuva kvadrat. Kolkava e visinata na toj kvadar?
9. Nacrtaj mre`a na kvadar so strani a = 2cm, b = 3cm i c = 4cm, a potoa presmetaj ja
negovata plo{tina.

IV.9. PLO[TINA NA PRIZMA

Poimot plo{tina na geometriskite sliki (zatvoreni i ograni~eni ramninski


oblasti) ni e poznat od minatata godina.
Granicata na sekoe geometrisko telo pretstavuva nekoja povr{ina, koja ja vikame
povr{ina na geometriskoto telo.
Povr{inata na prizmata, znaete, se sostoi od dva skladni mnoguagolnika - osnovi na
prizmata i odreden broj paralelogrami - bo~na povr{ina na prizmata.

Definicija: Pod plo{tina na povr{inata na edna prizma, ili kratko, plo{tina


na prizmata, }e go podrazbirame zbirot od plo{tinite na dvete nejzini osnovi i
plo{tinata na nejzinata bo~na povr{ina.

149
Ako plo{tinata na prizmata ja ozna~ime so R, a plo{tinata na nejzinata osnova
(baza) so V, a plo{tinata na bo~nata povr{ina - so M, toga{, vo soglasnost so gornava
definicija, }e va`i formulata:
P = 2B + M
Toa e op{ta formula za presmetuvawe na plo{tinata na koja i da bilo prizma.
Ako gi razgledame crte`ite 33, 34, 35, 36 zabele`uvame, deka bo~nata povr{ina na
sekoja prava prizma vsu{nost, pretstavuva, eden pravoagolnik, ~ija dol`ina e ednakva
na perimetarot na osnovata na prizmata, a visinata mu e ednakva na visinata na prizmata.
Spored toa, va`i:

Teorema: Plo{tinata na bo~nata povr{ina na prava prizma e ednakva na proizvodot


od perimetarot na osnovata i visinata na prizmata, t.e.
M = L ·H

IV.9.1. PLO[TINA NA KVADAR I KOCKA

Kvadarot e prava ~etiriagolna prizma ~ii osnovni i bo~ni yidovi se pravoagolnici.


Ako dimenziite na kvadarot gi ozna~ime so a, b, c (pri {to c e visina), toga{ plo{tinata
na osnovata e B = ab, a bo~nata plo{tina }e bide
M = (2a + 2b)c = 2ac + 2bc. Spored toa, formulata za plo{tina glasi:
P = 2B + M = 2ab + 2ac + 2bc, t.e. P = 2(ab + ac + bc).
Kockata e ograni~ena so {est skladni kvadrati. Ako dol`inata na rabot na kockata
ja ozna~ime so a, toga{ plo{tinata na sekoj od {este kvadrati }e bide ednakva na a2, pa
formulata za plo{tina na kockata glasi:
P = 6 a2.

Zada~a 1. Eden kufer ~ii dimenzii se: dol`ina a = 1 m, {irina b = 6 dm i visina c = 25


cm treba da se obvie so platno. Kolku m2 platno e potrebno?

Re{enie: Zada~ata se sveduva na odreduvawe na plo{tinata na kuferot, ~ii dimenzii


se poznati. Pred da gi zamenime mernite broevi na dimenziite vo formulata za plo{tina
na kvadar, tie dimenzii treba da gi izrazime vo isti edinici merki. Vo na{iot slu~aj,
ako sakame baranata plo{tina na kuferot da ja dobieme vo m2, toga{ negovite dimenzii
}e gi izrazime vo metri: a = 1m, b = 6dm = 0,6 m, c = 25cm = 0,25m, pa }e dobieme:
P = 2(ab + ac + bc) = 2(1 · 0,6 + 1 · 0,25 + 0,6 · 0,25) = 2 (m2).
Zna~i, za obvivawe na kuferot potrebno e 2 m2 platno.

150
Zada~a 2: Da se presmeta rabot na kocka, ~ija plo{tina iznesuva P = 384 cm2.

Re{enie: Bidej}i povr{inata na kockata ja so~inuvaat {est skladni kvadrati, zatoa


3 
plo{tinata na sekoj od tie kvadrati }e bide: D

 FP  , a ottuka ja odreduvame
 
i dol`inata na rabot na kockata D   FP .

IV.9.2. PLO[TINI NA NEKOI PRAVILNI


I NEKOI PRAVI PRIZMI

So primeri }e go poka`eme presmetuvaweto na plo{tinata na nekoi pravilni i


pravi prizmi.
.
Zada~a 3. Da se presmeta plo{tinata na pravilna ~etiriagolna prizma so osnoven
rab a = 5 cm i visina H = 12 cm.

Re{enie: Pravilnata ~etiriagolna prizma za osnova ima kvadrat. Plo{tinata na


nejzinata osnova e B = a2, a plo{tinata na bo~nata povr{ina }e bide M = 4aH.
Zna~i, plo{tinata na pravilna ~etiriagolna prizma }e bide:
P = 2B + M = 2a2 + 4aH, odnosno P = 2a(a + 2H).
Ako vo dobienata formula zamenime a = 5 cm i H = 12 cm, dobivame:
P = 2a(a + 2H) = 2 · 5 (5 + 2 · 12) = 10 · 29 = 290 (cm2).

Zada~a 4. Da se presmeta plo{tinata na pravilna triagolna prizma, ~ij osnoven rab


e a = 6 cm, a visinata e H = 10 cm.

Re{enie: Kaj pravilnata triagolna prizma osnovite se ramnostrani triagolnici


§ D  ·
¨¨ % ¸
©  ¸¹ , a bo~nata povr{ina se sostoi od tri skladni pravoagolnici (M = 3aH).

Spored toa, formulata za plo{tina na pravilnata triagolna prizma glasi:


D  §D  ·
3 %  0 ˜  D+ D ¨¨  + ¸¸ .
 ©  ¹
§  ·
Ottuka dobivame: 3  ¨¨   ˜  ¸¸ |   ˜      ˜   FP  .
©  ¹

Zada~a 5. Da se presmeta plo{tinata na pravilna {estagolna prizma, ~ij osnoven


rab e a = 2 cm, a visinata e H = 7,5 cm.

D  
Re{enie: Kaj pravilnata {estagolna prizma e % , a M = 6aH.


151
Spored toa, formulata za plo{tina na pravilnata {estagolna prizma }e glasi:
D  
3 %  0 ˜

 D+ D    D+
D D    + .
Ottuka dobivame:

3 D D    + |  ˜  ˜  ˜   ˜   ˜     ˜ ,46=110,76
t.e. P | 110,76 (cm ). 2

Zada~a 6. Da se presmeta plo{tinata na prava triagolna prizma, ~ija visina e


H = 8 cm, a za osnova ima pravoagolen triagolnik so kateti a = 2,4 cm i b = 3,2 cm.

Re{enie: ]e ja presmetame prvo plo{tinata na osnovata:


DE   ˜  
%   ˜    FP  .
 
Za presmetuvawe na bo~nata plo{tina potrebno e da se znae i hipotenuzata c na
osnovata. So primena na Pitagorovata teorema nao|ame deka:

F D   E             FP .

Potoa ja presmetuvame bo~nata plo{tina:


M = (a +b +c) H = (2, 4 + 3, 2 + 4) · 8 = 9,6 · 8 = 76,8 (cm2).
Baranata plo{tina na dadenata prizma e
P = 2B + M = 2 · 3,84 + 76,8 = 7,68 + 76,8 = 84,48, t.e. P = 84,48 (cm2).

Zada~a 7. Da se presmeta plo{tinata na prava ~etiriagolna prizma, ~ija visina e


H = 12 cm, a osnovata e romb so dijagonali d1 = 8 cm i d2 = 6 cm.

Re{enie: Prvo ja presmetuvame plo{tinata na osnovata na prizmata:


G ˜ G ˜
%  FP .
 
Za presmetuvawe na plo{tinata na bo~nata povr{ina potrebno e da ja opredelime
dol`inata na osnovniot rab na prizmata. So primena na Pitagorovata teorema, nao|ame
deka:
   
§ G · § G · §· §·
D ¨  ¸ ¨  ¸ ¨ ¸ ¨ ¸      FP .
© ¹ © ¹ ©¹ ©¹

Bo~nata plo{tina e: M = 4aH = 4 · 5 · 12 = 240 (cm2), a plo{tinata na prizmata


P = 2B + M = 2 · 24 + 240 = 288 (cm2).

Zada~a 8. Da se presmeta plo{tinata na prava ~etiriagolna prizma ~ija visina e


H = 14 cm, a osnovata e ramnokrak trapez, ~ii paralelni strani se dolgi a = 9 cm, b = 4 cm,
a krakot c = 6,5 cm.

152
Re{enie. Za presmetuvawe na plo{tinata na trapezot (osnovata na prizmata) potrebno
e da ja opredelime visinata na toj trapez (crt. 36). So primena na Pitagorovata teorema
visinata na ramnokrakiot trapez }e bide:
 
§DE· § ·
K F  ¨ ¸    ¨ ¸       FP .
©  ¹ ©  ¹

D  E K    ˜ 
Potoa nao|ame deka: %  ˜   FP  ,
 
a M = (a + b + 2c) · H = (9 + 4 + 2 · 6, 5) · 14 = 26 · 14 = 364 (cm2).
Na krajot lesno ja nao|ame i baranata plo{tina na dadenata prizma, taa }e bide:
P = 2B + M = 2 · 39 + 364 = 78 + 364 = 442 (cm2).

1. Presmetaj ja plo{tinata na kvadar ~ii dimenzii se:


a) a = 6,4 cm, b = 5 cm, c = 3,5 cm, b) a = 1,8 dm, b = 1 dm, c = 7,5 cm.
2. Plo{tinata na edna kocka e 150 cm2. Opredeli go nejziniot rab.
3. Izrazi ja plo{tinata na kockata kako funkcija, od nejzinata dijagonala!
4. Presmetaj ja dijagonalata na kocka, ~ija plo{tina e 150 cm2.
5. Bo~nata plo{tina na edna pravilna ~etiriagolna prizma iznesuva 240 cm2, a visinata
1 dm. Opredeli ja plo{tinata na taa prizma.
6. Presmetaj ja plo{tinata i dol`inata na dijagonalata na pravilna ~etiriagolna
prizma so osnoven rab a = 11cm i visina 2,5 dm.
7. Plo{tinata na pravilna ~etiriagolna prizma iznesuva 288 cm2, a, samo nejzinata
bo~na plo{tina iznesuva 169 cm2. Opredeli ja visinata na prizmata!
8. Da se opredeli plo{tinata na pravilna {estagolna prizma, ako se poznati nejzinata
visina 5 cm, i dol`inata na nejzinata najgolema dijagonala d = 13 cm.
9. Vo salata na eden restoran se nao|aat 4 stolbovi, koi imaat forma na pravilna
{estagolna prizma so osnoven rab 25 cm i visina 5,5 m. Kolku m2 platno e potrebno
za nivno obvivawe?
10. Prava prizma so visina 12 cm, za osnova ima ramnokrak trapez, ~ii paralelni strani
se dolgi 9 cm i 3 cm i visina 4 cm. Odredi ja plo{tinata na taa prizma.

153
11. Prava prizma so visina 8,5 cm za osnova ima ramnokrak triagolnik. Presmetaj ja
plo{tinata na prizmata ako ramnokrakiot triagolnik ima osnova a = 5 cm i krak
b = 6,5 cm.
12. Osnovata na prava prizma e romb so dijagonali 3 cm i 4 cm, a dijagonalata na bo~niot
yid e dolga 6,5 cm. Opredeli ja plo{tinata na prizmata!

IV.10. VOLUMEN NA PRIZMA


IV.10.1. OP[TO ZA VOLUMEN NA TELATA

Na crte` 37 pretstavena e pravilna ~etiriagolna prizma ABCDA1B1C1D1. So nejziniot


dijagonalen presek ACC1A1 dobivame dve pravi triagolni prizmi, koi imaat skladni osnovi
i skladni visini. Tie dve triagolni prizmi, gledame, imaat zaedni~ki yid - dijagonalniot
presek ACC1A1, no nemaat nitu edna zaedni~ka vnatre{na to~ka. Za niv velime, deka se
dve sosedni prizmi, a za sekoja od niv velime deka e del (t.e. podmno`estvo) od pravilnata
~etiriagolna prizma.
Da si zamislime: ednata triagolna prizma, na primer, ABCA1B1C1 da ja dvi`ime na
nekoj na~in (so rotacija i translacija) taka {to, taa da se sovpadne so drugata triagolna
prizma CDAC1D1A1. Toa }e go postigneme ako temiwata A,B,C,A1,B1,C1 na prvata triagolna
prizma gi dovedeme da se sovpadnat soodvetno so temiwata C,D,A, C1,D1,A1, na vtorata
triagolna prizma (crt. 37). O~igledno e deka, pri toa, povr{inata (granicata) i
vnatre{nosta na ednata }e se sovpadne so povr{inata i vnatre{nosta na drugata.
Za dve tela, koi na nekoj na~in mo`at da se sovpadnat vo
site svoi delovi, velime deka se skladni (kongruentni). Na ' &
primer, dvete triagolni prizmi na crte` 37 se skladni.
Poimot volumen na geometriskite tela go voveduvame, $ %
na sli~en na~in, kako i poimot plo{tina na geometriskite
sliki.
Volumenot na edno geometrisko telo T, {to simboli~ki
go ozna~uvame so V (T) ili samo so V, go razgleduvame kako ' &
veli~ina od poseben vid, koja gi karakterizira
geometriskite tela i gi ima slednive svojstva (aksiomi za $ %
volumen): Crte` 37

1°. Sekoe geometrisko telo ima to~no opredelen pozitiven volumen, t.e. V (T)>0.
2°. Ako telata T1 i T2 se skladni, toga{ V (T1 ) = V (T2 ).

154
3°. Ako teloto T e sostaveno od delovi (sosedni tela) T1 i T2 , {to nemaat zaedni~ki
vnatre{ni to~ki, toga{ V (T) = V (T1) + V (T2 ) .
4°.Kocka so rab 1cm se zema deka ima volumen 1cm3 .

Volumenot na teloto, kako i sekoja druga veli~ina, mo`e da se meri i izrazuva so


broevi. Da se izmeri volumenot na edno telo zna~i, da se odredi kolku pati se sodr`i vo
nego volumenot na drugo telo {to e primen za edinica. Brojot, koj poka`uva edinicata
merka kolku pati se sodr`i vo (ili kakov del e od nea) volumenot {to go merime, se vika
meren broj na toj volumen.
Pri izbiraweto na edinicite merki za volumen prakti~no se poka`alo tie da bidat
usoglaseni so edinicite merki za dol`ina na otse~kite. Na primer, od dol`inskite
edinici: m, dm, cm i mm, se izvedeni merki za volumen: metar kuben (m3), decimetar kuben
(dm3), santimetar kuben (cm3) i milimetar kuben (mm3).

Eden metar kuben (m3) e volumenot na kocka so rab dolg 1m.

Dve tela, {to imaat ednakvi


volumeni, se vikaat ednakvo
golemi (ili ekvivalentni).
Od svojstvata 2° i 3° sleduva
deka:
Ednakvo golemi se: a) sekoi
dve skladni tela, b) sekoi dve te-
la koi se sostaveni od soodvetno
skladni delovi (crt. 38), v) sekoi Crte` 38
dve tela, koi mo`at da se dopol-
nat so skladni delovi do dve
skladni tela.

IV.10.2. VOLUMEN NA KVADAR I KOCKA

Teorema: Volumenot na kvadarot e ednakov na proizvodot od negovite tri dimenzii,


t.e.
V=a·b·c

155
Crte` 39

Dokaz: Vo zavisnost od toa, dali dimenziite a, b i c na kvadarot se izrazeni vo pri-


rodni, racionalni ili iracionalni broevi, }e razlikuvame tri slu~ai:

a) Dimenziite na kvadarot neka se izrazeni so prirodni broevi, na primer a = 7 cm,


b = 4 cm i c = 5 cm (crt. 39).
Osnovata na kvadarot mo`e da se razbie na B = ab = (7 · 4) cm2 = 28 cm2. Nad sekoj od tie
ab = 28 cm2 od osnovata mo`eme da postavime po 1 cm3. Zna~i, na celata osnova na kvadarot
mo`eme da postavime vkupno 28 cm3. Na toj na~in dobivame eden plast (sloj) od kocki
visok 1 cm, {to ja pokriva osnovata na kvadarot. Takvi plastovi vo celiot kvadar mo`at
da se postavat 5, bidej}i visinata na kvadarot ima c = 5 cm. Spored toa, volumenot na
dadeniot kvadar }e bide V = abc = (7 · 4 · 5) cm3 = 140 cm3 .
b) Dimenziite na kvadarot neka se izrazeni so racionalni broevi, na primer,

  
D P E P i F P.
  

156
Racionalnite broevi a, b i c mo`eme da gi dovedeme na zaedni~ki imenitel i dobivame

S T U   
D ,E iF (vo na{iot slu~aj D P, E P i F m).
Q Q Q   

§  ·
Ako delot ¨ ¸ od metarot go zememe za nova dol`inska edinica, toga{ o~igledno
Q ©  ¹
e deka taa edinica }e se sodr`i vo dimenziite a, b i c na kvadarot, soodvetno p, q i r pati
(vo na{iot slu~aj 15, 10 i 8 pati). Spored toa, dimenziite na dadeniot kvadar vo novite
edinici }e bidat izrazeni so prirodnite broevi p, q i r; pa negoviot volumen }e bide
ednakov na pqr novi kubni edinici.
O~igledno e deka, starata kubna edinica (metar kuben) }e sodr`i n3 (203 = 8 000) novi
kubni edinici. Zatoa, za da go odredime volumenot na dadeniot kvadar so starite kubni
edinici, proizvodot p q r treba da go podelime so brojot n3. Taka dobivame:

STU S T U
9 ili 9 ˜ ˜ .
Q Q Q Q

S T U
No, bidej}i D E, F , zatoa i vo ovoj slu~aj }e bide V = abc.
Q Q Q

  
Vo konkretniov slu~aj, kvadarot so dimenzii D P E Pi F P ima volumen
  
   
9 ˜ ˜ P .
   
v) Mo`e da se doka`e deka formulata za volumen na kvadarot V = abc va`i i koga
negovite dimenzii se izrazeni so iracionalni broevi, no za toa se potrebni
dopolnitelni znaewa.
Bidej}i kockata e specijalen slu~aj na kvadar, kaj kogo site dimenzii se ednakvi, t.e.
b = c = a , zatoa formulata za presmetuvawe na volumenot na kockata glasi:
V = a · a · a, t.e. V = a3. Zna~i:

Volumenot na kockata e ednakov na kubot od dol`inata na nejziniot rab.

Na primer, kocka so rab a = 9 cm, }e ima volumen: V = a3 = 93 = 729 (cm3).

Ako so B i H gi ozna~ime soodvetno plo{tinata na osnovata i dol`inata na visinata


na kvadarot, toga{ formulata V = abc, koga }e zememe predvid deka ab = B i c = H mo`e da se
zapi{e u{te i vo sledniov vid:
V = BH

157
IV.10.3. VOLUMEN NA PRAVA PRIZMA

Neka e daden kvadar ABCDA1B1C1D1 (crt. 40). O~igledno e deka


so dijagonalniot presek BDD1B1 dadeniot kvadar se razdeluva
na dve skladni triagolni prizmi ABDA1B1D1 i BCDB1C1D1, bidej}i
osnovite im se skladni pravoagolni triagolnici (DABD @ DBCD)
i imaat ednakvi visini. Spored toa, tie se i ednakvo golemi,
t.e imaat ednakvi volumeni. Toa, pak, zna~i deka volumenot na
sekoja od dobienite triagolni prizmi e ednakov na polovina
od volumenot na dadeniot kvadar.
Dadeniot kvadar so osnovni rabovi a i b i visina H ima
volumen ednakov na abH. Ako volumenot na ednata triagolna
prizma go ozna~ime so V, toj }e bide ednakov na: Crte` 40

DE+ DE
9 ˜+ .
 
Bidej}i osnovata na triagolnata prizma e pravoagolen triagolnik so kateti a i b,
DE
zatoa izrazot vsu{nost, pretstavuva, plo{tina na nejzinata osnova. Ako izrazot
DE 
go ozna~ime so B, ja dobivame formulata:

V = BH.
Zna~i, volumenot na prava prizma, ~ija osnova e
pravoagolen triagolnik, e ednakov na proizvodot od
plo{tinata na osnovata i dol`inata na nejzinata
visina.
Da zememe sega edna, koja i da bilo, prava tri-
agolna prizma ABCA1B1C1 (crt. 41). Da ja prese~eme
taa prizma so edna ramnina, koja minuva niz rabot
Crte` 41 CC1 i e normalna na bo~niot yid ABB1A1. Toga{ }e bide:
CD A AB i C1D1 A A1B1(crt. 41).
'
Gledame, dadenata triagolna prizma se razdeluva na dve drugi
( &
triagolni prizmi, ~ii osnovi se pravoagolni triagolnici. Ako so
B, B1 i B2 gi ozna~ime soodvetno plo{tinite na osnovite na dadenata
$ %
prizma i dobienite od nea dve drugi triagolni prizmi, a so H - nivnata
zaedni~ka visina; toga{ volumenot na dadenata triagolna prizma
}e bide: V = B1H + B2 H = (B1 + B2)H = BH, bidej}i e B1 + B2 = B.
Na krajot, da zememe i edna, koja i da bilo, prava prizma. Neka '
toa bide, na primer, petagolna prizma na crte` 42. Gledame deka so ( &
dijagonalnite preseci ACC1A1 i ADD1A1 {to minuvaat niz bo~niot
rab AA1, dadenata petagolna prizma se razdeluva na tri triagolni $ %
prizmi.
Crte` 42

158
Ako plo{tinata na osnovite na dobienite triagolni prizmi gi ozna~ime so B1,B2 i B3
a nivnite volumeni so V1, V2,V3 toga{, }e imame:
V1 = B1H, V2 = B2H, V3 = B3H.
Bidej}i volumenot (V) na dadenata prizma e ednakov na zbirot na volumenite na
dobienite triagolni prizmi, zatoa toj }e bide:
V = V1 + V2 + V3 = B1H + B2H + B3H = (B1 + B2 + B3)H.
No bidej}i zbirot od plo{tinite na osnovite na triagolnite prizmi B1 + B2 + B3 e
ednakov na plo{tinata na osnovata na dadenata petagolna prizma (B), dobivame deka:
V = BH.
Na ist na~in, i sekoja druga prizma mo`eme da ja podelime na odreden broj triagolni
prizmi, pa spored toa va`i slednava:

Teorema: Volumenot na sekoja prava prizma e ednakov na proizvodot od plo{tinata


na osnovata i visinata, t.e.
V = BH.

Pravilnata ~etiriagolna prizma za osnova ima kvadrat, zatoa plo{tinata na nejzinata


osnova }e bide B = a2, a volumenot: V = a2H.
Kaj pravilnata triagolna prizma osnovata e ramnostran triagolnik, ~ija plo{tina
D  D 
e: % , pa spored toa, formulata za volumen glasi: 9 ˜+ .
 
Osnovata na pravilnata {estagolna prizma e pravilen {estagolnik, ~ija plo{tina
D   D  
e: % , pa spored toa, formulata za volumen glasi 9 ˜+ .
 
Zada~a 1. Da se presmeta volumenot na prava prizma, ~ija visina e H = 9, 5 cm, ako
za osnova ima ramnokrak triagolnik so osnova a = 5 cm i krak b = 6, 5 cm.

Re{enie: Za presmetuvawe na plo{tinata na ramnokrakiot triagolnik (osnova na


prizmata) potrebno e, prvo, da ja opredelime visinata na toj triagolnik. So primena na
Pitagorovata teorema, nao|ame:

 
§D· §·
K E  ¨ ¸    ¨ ¸        FP .
©¹ ©¹
DK ˜
Potoa nao|ame: %  FP  , a volumenot e:V = BH = 15 · 9, 5 = 142, 5 (cm3).
 

159
Zada~a 2. Vo eden sad {to ima forma na prava prizma so osnova ramnokrak trapez,
staveno e 8,4 dm3 voda. Do koja visina }e se napolni sadot, ako ramnokrakiot trapez ima
paralelni strani a = 34 cm, b = 16 cm i krak c = 15 cm?

Re{enie: Bidej}i vodata vo sadot }e zazeme ista forma kako i sadot, zatoa zada~ata
se sveduva na opredeluvawe na visinata na prizmata, koga se poznati nejziniot volumen
i rabovite pri osnovata.
9
Bidej}i volumenot na prizmata e V = BH, zatoa + .
%
Za da ja re{ime zada~ata, prvo, treba da ja presmetame plo{tinata na osnovata, t.e.
plo{tinata na ramnokrakiot trapez. So primena na Pitagorovata teorema, prvo, ja
opredeluvame visinata na trapezot:
 
§DE· §    ·
K F  ¨ ¸   ¨ ¸      FP .
©  ¹ ©  ¹

D  E K    ˜ 
Zna~i: %  ˜   FP  .
 
9 
Bidej}i V = 8, 4 dm3 = 8 400 cm3, zatoa +  FP .
% 

Spored toa, sadot }e se napolni do visinata H = 28 cm.

Zada~a 3. Eden mermeren stolb ima forma na pravilna {estagolna prizma so osnoven
rab a = 32 cm i visina H = 4 m. Da se odredi masata na toj stolb vo kilogrami.

Re{enie: Poimot masa na edno materijalno telo vi e poznat od fizikata. Taa se meri
vo gramovi (g), kilogrami (kg) ili toni (t). Znaete deka, masata na 1 cm3 od nekoja materija
izrazena vo gramovi se vika gustina na taa materija, koja se ozna~uva so s. Sekoja materija
ima opredelena gustina. Na primer, destiliranata voda na 4°S ima gustina 1, `elezoto
ima gustina s = 7,8; bakarot - s = 8,9; mermerot s = 2,7 itn.
Koga }e ka`eme, deka gustinata na mermerot e s = 2,7, toa, zna~i, deka 1 cm3 mermer ima
masa 2,7 g. Ottuka e jasno, deka baranata masa M na mermerniot stolb vo na{ata zada~a }e
bide ednakva na proizvodot od negoviot volumen i gustinata s = 2,7, na mermerot t.e.
M = Vs.
Spored toa, prvo }e go presmetame volumenot na stolbot:

D  +   ˜  ˜  ˜ 
9 |  ˜  ˜  ˜   |  FP .
 
Toga{ }e bide: M = Vs = 1063000 · 2,7 = 287010 (g) .
Zna~i, mermerniot stolb ima masa | 287 kg.

160
1. Presmetaj go volumenot na pravilna ~etiriagolna prizma so osnoven rab 4 cm i
visina 13,5 cm.

2. Presmetaj go volumenot na pravilna triagolna prizma so perimetar na osnovata 16,5 cm


i visina 10 cm.

3. Presmetaj go volumenot na kocka, ~ija plo{tina iznesuva 96 cm2.

4. Da se opredeli dijagonalata na kocka, ~ij volumen e 64 dm3.

5. Kolku kutii, dolgi 3 dm so {irina 2,5 dm i debelina 8 cm, mo`at da se smestat vo eden
sandak so dimenzii 1,5 m, 1 m i 0,8 m?

6. Dimenziite na eden kvadar se: 30cm, 1m i 9m. Odredi go rabot na kocka {to ima ednakov
volumen so toj kvadar!

7. Rezervoar vo forma na kvadar ima dimenzii 9 m, 4 m i 2,5 m. Za kolku vreme }e se


napolni toj, ako vo nego se vlevaat po 5 litri vo sekoja sekunda?

8. Kako }e se promeni volumenot na kockata, ako nejziniot rab:


a) se zgolemi 3 pati, b) se namali 2 pati?

9. Od tri metalni kocki so dol`ini na rabovite 3 cm, 4 cm i 5 cm izliena e edna kocka.


Odredi go rabot na taa kocka.

10. Eden bazen ima forma na pravilna ~etiriagolna prizma so osnoven rab vnatre 1,4 m.
Kolku litri voda treba da se ispu{ti od bazenot, taka {to nivoto na vodata vo nego
da se spu{ti za 18 cm?

11. Kolkava sobna povr{ina mo`eme da pokrieme so 0, 64 m3 {tici ako tie se debeli 2,5 cm?

12. Kolkava e masata na eden list cinkova lamarina {to ima forma na kvadar so dol`ina
2 m, {irina 1,2 m i debelina 2 mm? Cinkot ima gustina 7,1.

13. Treba da se napravi cisterna vo forma na pravilna ~etiriagolna prizma so osnoven


rab 1,5 m. Kolku treba taa da e dlaboka, za da sobira 90 hektolitri?

161
V PIRAMIDA

IV. 11. POIM I ELEMENTI NA PIRAMIDATA

Vo ramninata S neka e daden eden konveksen mnogua-


golnik, na primer, petagolnikot ABCDE, a nadvor od ram-
ninata S da zememe edna proizvolna to~ka S (crt. 43).
Ako to~kata S ja soedinime so sekoe teme od petagol-
nikot ABCDE, }e dobieme i pet triagolnici: ABS, BCS, CDS,
DES i EAS.
Povr{inata, {to e sostavena od petagolnikot ABCDE
i pette triagolnici koi imaat edno zaedni~ko teme S,
go razbiva mno`estvoto to~ki od prostorot na dva dela:
vnatre{en i nadvore{en. Unijata od taa povr{ina i
vnatre{niot del pretstavuva edna zatvorena i
ograni~ena prostorna oblast, t.e. edno geometrisko telo,
Crte` 43 i toa poliedar. Toj poliedar go vikame petagolna
piramida (crt. 43).
Ako vo ramninata S , namesto petagolnik zemevme drug mnoguagolnik, na primer
triagolnik, ~etiriagolnik, odnosno {estagolnik, na sli~en na~in, }e dobievme drugi
poliedri, koi se vikaat: triagolna, ~etiriagolna, odnosno {estagolna piramida (crt.
44). Site tie so zaedni~ko ime se vikaat piramidi.

Definicija: Eden poliedar se vika piramida, ako eden yid e mnoguagolnik, a drugite
yidovi se triagolnici so edno zaedni~ko teme i po edna zaedni~ka strana so
mnoguagolnikot.

Crte` 44

Mnoguagolnikot go vikame osnova (ili baza) na piramidata; triagolnicite {to imaat


zaedni~ko teme S, gi vikame bo~ni yidovi na piramidata, a to~kata (temeto) S - vrv na
piramidata.
Site bo~ni yidovi na piramidata velime deka ja obrazuvaat bo~nata povr{ina na
piramidata.

162
Stranite na osnovata gi vikame osnovni rabovi na piramidata (AB, BC, CD... na crt.
43), a site drugi rabovi - bo~ni rabovi na piramidata (AS, BS, CS, ... na crt. 43).
Piramidite gi crtame kako na crte`ite 43 i 44. So neprekinati linii gi crtame
site vidlivi rabovi, a so isprekinati linii - site nevidlivi rabovi.
Visina na piramidata se vika otse~kata od normalata {to e povle~ena od vrvot kon
ramninata na osnovata na piramidata (SO na crt. 44). Visina na piramidata ja vikame
u{te i dol`inata na taa otse~ka, odnosno rastojanieto od vrvot do ramninata na osnovata
na piramidata i ja ozna~uvame so H.

1. [to e toa osnova, bo~na povr{ina, vrv, visina na piramidata?


2. Dali mo`e visinata na piramidata da se sovpadne so eden bo~en rab na
piramidata?
3. Dali mo`at nekoi bo~ni yidovi na piramidata da bidat normalni na osnovata?
Ako mo`e, kolku najmnogu takvi bo~ni yidovi mo`e da ima piramidata?
4. Kolkav najmal broj na: a) yidovi, b) temiwa, v) rabovi mo`e da ima eden poliedar
i koj e toj poliedar?

IV. 12. VIDOVI I SVOJSTVA NA PIRAMIDITE.


DIJAGONALEN PRESEK

Zavisno od brojot na stranite na osnovata, piramidite mo`at da bidat: triagolni,


~etiriagolni, petagolni, itn.
Triagolnata piramida u{te ja vikame i tetraedar. Bidej}i site yidovi na tetraedarot
se triagolnici, zatoa, koj i da bilo od niv mo`e da se zeme za negova osnova.
Kaj nekoi piramidi mo`e da se slu~i site bo~ni rabovi da se
ednakvi eden na drug, a kaj drugi da ne e takov slu~aj.
Da razgledame edna piramida so ednakvi bo~ni rabovi, na primer,
neka e takva ~etiriagolnata piramida SABCD na crte` 45. Neka O e
ortogonalna proekcija na vrvot S vrz ramninata na osnovata na
piramidata SABCD kaj koja e 6$ 6% 6& 6' (crt. 45).
Ako to~kata O ja soedinime so temiwata A, B, C i D }e gi dobieme
pravoagolnite triagolnici SOA,SOB, SOC i SOD. Tie triagolnici se
skladni, bidej}i imaat skladni hipotenuzi i edna zaedni~ka kateta
OS. Od nivnata skladnost, sleduva deka i drugite kateti im se ednakvi,
t.e. deka 2$ 2% 2& 2' .
Crte` 45

163
Toa zna~i, deka okolu osnovata na razgleduvanata piramida mo`e da se opi{e
kru`nica, ~ij centar e vo podno`jeto na visinata na taa piramida.
Ako razgledame i druga, koja i da bilo, piramida so ednakvi bo~ni rabovi, o~igledno
e deka }e dojdeme do ist zaklu~ok. Zna~i, va`i: Osnovata na koja i da bilo piramida so
ednakvi bo~ni rabovi e tetiven mnoguagolnik, ~ij centar na opi{anata kru`nica e vo
podno`jeto na visinata na taa piramida.

Definicija: Piramida so ednakvi bo~ni rabovi, ~ija osnova e pravilen mnoguagolnik,


se vika pravilna piramida.

Za pravilnite piramidi karakteristi~no e toa {to: site osnovni rabovi se ednakvi


me|u sebe, a site bo~ni yidovi se skladni ramnokraki triagolnici.
Visinata na koj i da bilo bo~en yid na pravilnata piramida, {to e povle~ena kon
soodvetniot osnoven rab, se vika apotema na piramidata (SM na crt. 46).

Crte` 46
Apotemite na pravilnata piramida se ednakvi (Zo{to?).
Pravilna triagolna piramida, kaj koja bo~niot rab e ednakov na osnovniot rab, se
vika u{te i pravilen tetraedar.
Na crte` 46 nacrtani se pravilna triagolna, ~etiriagolna i pravilna {estagolna
piramida, na ~ie razgleduvawe posebno }e se zadr`ime. Za niv }e gi upotrebuvame slednite
oznaki:
Dol`inata na osnovniot rab }e ja ozna~uvame so a, na bo~niot rab - so s, na visinata
- so H i na apotemata - so h.
]e poka`eme deka gornive ~etiri elementi a, s, H i h na pravilnite piramidi ne se
nezavisni eden od drug, odnosno ako se poznati koi i da bilo dva od niv, lesno mo`at da
se odredat i drugite dva elementa.
Od crte` 46 gledate deka ortogonalnite proekcii na bo~niot rab SA i apotemata SM
vrz ramninata na osnovata na pravilnite piramidi, vsu{nost, pretstavuvaat, radiusi R i
r na opi{anata i vpi{anata kru`nica na pravilniot mnoguagolnik {to e osnova na
pravilnata piramida.

164
Dol`inite na tie radiusi R i r, kako {to ni e poznato, se funkcii od dol`inata na
stranata a na pravilniot mnoguagolnik i toa:
D  D 
- kaj ramnostraniot triagolnik: 5 U
 
D  D
- kaj kvadratot: 5 U
 
D 
- kaj pravilniot {estagolnik: 5 D U .

Zna~i, so pomo{ na tie formuli lesno gi odreduvame radiusite R i r koga e poznata
dol`inata na osnovniot rab a na soodvetnata pravilna piramida.
Od druga strana pak, od pravoagolnite triagolnici SAO, SOM i SMC so primena na
Pitagorovata teorema imame
- DSAO: s2 = H2 + R2, odnosno H2 = s2 – H2
- DSOM: h2 = H2 + r2, odnosno H2 = h2 – r2
 
 §D·
 §D·
- DSMC: V K  ¨ ¸ , odnosno K  V  ¨ ¸ .
© ¹ © ¹

Zada~a: Pravilna ~etiriagolna piramida ima osnoven rab a = 14 cm i bo~en rab


s = 25 cm. Da se odredat visinata i apotemata na piramidata.

Re{enie: Od pravoagolniot triagolnik SAO (crt. 46b) imame:



 
§D ·
 D
  
+ V 5 V  ¨¨ ¸¸ V   
©  ¹  

      |  FP .

Apotemata h mo`eme da ja odredime ili od pravoagolniot triagolnik SOM ili od

DSMC (crt. 46b). Od DSMC dobivame:


 
§D· §  ·
K 
V ¨ ¸ 
  ¨ ¸      FP .
©¹ ©¹
Zna~i, dadenata piramida ima visina H | 23 cm i apotema h = 24 cm.
Presekot {to se dobiva, koga piramidata }e se prese~e so ramnina, koja minuva niz
vrvot i edna koja i da bilo dijagonala na osnovata, se vika dijagonalen presek na
piramidata.

165
1. Piramidata ~ija osnova e: a) ramnokrak triagolnik, b) pravoagolen triagolnik,
v) romb, g) romboid, d) ramnokrak trapez, |) deltoid, dali mo`e da bide so ednakvi
bo~ni rabovi? Zo{to?
2. Koi piramidi gi vikame pravilni? Koj e neophoden i dovolen uslov za da bide edna
piramida pravilna?
3. Ako osnovata na edna piramida e pravilen mnoguagolnik, dali taa mora da e
pravilna?
4. Ako bo~nite rabovi na edna piramida se ednakvi eden na drug, dali taa mora da e
pravilna?
5. Dali kaj pravilnata piramida postoi to~ka {to e ednakvo oddale~ena od site
temiwa i od vrvot na piramidata?
6. Edna pravilna ~etiriagolna piramida ima osnoven rab 10 cm i apotema 13 cm. Odredi
ja visinata na piramidata!
7. Pravilna {estagolna piramida ima osnoven rab 4 cm i visina 5, 5 cm. Odredi ja
nejzinata apotema!
8. Pravilna triagolna piramida ima osnoven rab 6 cm i bo~en rab 4 cm. Odredi ja
visinata i apotemata na taa piramida!
9. Pravilna ~etiriagolna piramida so osnoven rab a = 10 cm ima visina H = 12 cm.
Odredi go bo~niot rab i apotemata na piramidata!
10. Odredi ja visinata na pravilen tetraedar, ~ij rab e a = 8 cm!
11. Dali mo`e dijagonalniot presek na piramidata da bide: a) ramnokrak triagolnik,
b) pravoagolen triagolnik, v) ramnostran triagolnik?
12. Ako eden pravilen tetraedar go ise~eme v dol` trite bo~ni rabovi i taka dobienata
figura ja rasklopime i postavime vo ramnina, kakva figura }e dobieme?

IV.13. MRE@A NA PIRAMIDA

Ako osnovata i bo~nite yidovi na piramidata gi rasklopime i postavime site da


legnat vo edna ramnina, }e ja dobieme mre`ata na piramidata.
Da ja nacrtame mre`ata na pravilnata triagolna piramida so osnoven rab a i bo~en
rab s.
Baranata mre`a na pravilnata triagolna piramida ja crtame vaka: Od nekoja to~ka S
opi{uvame kru`en lak so radius {to e ednakov na bo~niot rab s na piramidata
(crt. 47a). Potoa, po opi{aniot kru`en lak, so {estar tri pati go prenesuvame osnov-
niot rab a na piramidata kako tetiva na toj lak. Soedinuvaj}i gi dobienite to~ki na

166
kru`niot lak so to~ka-
D ta S, a potoa i po red edna
6 E
so druga, }e dobieme tri
V V ramnokraki triagol-
nici, {to ni gi pret-
D D D stavuvaat bo~nite yido-
D vi na pravilnata tria-
V D D golna piramida. Na
D D krajot nad eden osnoven
V rab go konstruirame i
Crte` 47 ramnostraniot triagol-
nik - osnova na pirami-
data. Taka ja dobivame baranata mre`a na pravilnata triagolna piramida (crt. 47a).
Mre`ata na pravilna triagolna piramida mo`e da se nacrta i na drug na~in, kako na
crte` 47b. Tuka, prvo, nacrtana e osnovata na piramidata, a potoa nad sekoj nejzin rab e
konstruiran po eden ramnokrak triagolnik so krak, {to e ednakov na bo~niot rab s na
piramidata.
Na sli~en
na~in ja crta-
me i mre`ata
na pravilna
~etiriagolna
i pravilna {e-
stagolna pira-
mida.
Na crt. 48a
i b nacrtana e
na dva na~ina
Crte` 48
mre`ata na ed-
na ~etiriagol-
na piramida so ednakvi bo~ni rabovi, ~ija osnova e pravoagolnik so dimenzii a = 2 cm i
b = 1 cm, a bo~niot rab na piramidata e s = 2, 5 cm. Objasni ja postapkata kako e nacrtana
taa.

1. Nacrtaj ja mre`ata i napravi model na pravilna triagolna piramida, ~ij osnoven


rab e a = 5 cm i bo~en rab c = 8 cm.
2. Kolku oski na simetrija ima mre`ata na pravilna triagolna piramida, {to e
nacrtana na: crt. 47a, b) crt. 47b?
3. Nacrtaj ja mre`ata na pravilna ~etiriagolna piramida, ~ij osnoven rab e a = 3 cm i
bo~en rab s = 7, 5 cm.

167
4. Nacrtaj ja mre`ata i napravi model na ~etiriagolna piramida so ednakvi bo~ni
rabovi s = 7 cm, a osnovata na piramidata e pravoagolnik so dimenzii a = 5, 5 cm i
b = 3 cm.
5. Nacrtaj ja mre`ata i napravi model na pravilen tetraedar, ~ij rab e a = 6 cm.
6. Nacrtaj ja mre`ata i napravi model na triagolna piramida so ednakvi bo~ni rabovi
s = 7cm, ~ii osnovni rabovi se a = 3 cm, b = 4, 5 cm i c = 6 cm.
7. Nacrtaj ja mre`ata na triagolna piramida so ednakvi bo~ni rabovi c = 5 cm, a
osnovata — e ramnokrak triagolnik so osnova a = 3 cm i krak b = 4 cm.
8. Nacrtaj ja mre`ata na edna triagolna piramida so ednakvi bo~ni rabovi c = 7 cm,
a osnovata — e pravoagolen triagolnik so kateti a = 3 cm i b = 4 cm.
9. Nacrtaj mre`a na edna piramida, presmetaj gi plo{tinite na site nejzini delovi,
a potoa soberi gi tie plo{tini.

IV.14. PLO[TINA NA PIRAMIDA

Da se opredeli plo{tinata na edna piramida, zna~i da se najde zbirot od plo{tinite


na site nejzini yidovi. Povr{inata (granicata) na piramidata se sostoi od eden
mnoguagolnik - bo~na povr{ina na piramidata, i opredelen broj triagolnici - bo~na
povr{ina na piramidata. Ako plo{tinata na osnovata na piramidata ja ozna~ime so B, a
plo{tinata na bo~nata povr{ina - so M, toga{ za plo{tinata P na piramidata va`i
formulata:
P=B+M
Plo{tinata na piramidata e ednakva na zbirot od plo{tinite na nejzinata osnova i
nejzinata bo~na povr{ina.
Od site vidovi piramidi, nie }e se zadr`ime, samo na presmetuvaweto na plo{tinata na
pravilnite piramidi i plo{tinata na nekoi piramidi so ednakvi bo~ni rabovi.
]e poka`eme deka va`i slednata:
Teorema: Plo{tinata na bo~nata povr{ina na sekoja pravilna piramida e ednakva
na poluproizvodot od perimetarot (L) na osnovata i dol`inata na apotemata (h) na
/K
piramidata, t.e. 0 .


Dokaz: Plo{tinata na eden bo~en yid (ramnokrak triagolnik) na pravilnata n - agolna


D˜K
piramida e ednakva na , kade {to a e dol`inata na osnovniot rab, a h - dol`inata na

apotemata na piramidata. No, bidej}i bo~nata povr{ina na pravilnata n- agolna piramida
se sostoi od n skladni triagolnici, zatoa nejzinata plo{tina }e bide:
DK QDK
0 Q˜ .
 
Ako zememe predvid deka na = L, kade L e perimetar na osnovata, toga{ dobivame:
/K
0 .

168
Od op{tata formula za plo{tinata na piramidata i doka`anata teorema lesno gi
dobivame i formulite za plo{tina na pravilnite: triagolna, ~etiriagolna i {estagolna
piramida.
D  DK
Za pravilnata triagolna piramida, bidej}i e % ,a 0 zatoa formu-
 
lata za plo{tina glasi:
D   DK D§D  ·
3 %0  , odnosno 3  ¨¨   K ¸¸ .
  © ¹
DK
Za pravilnata ~etiriagolna piramida, bidej}i e B=a2 , a 0 DK zatoa

P = B + M = a2 + 2 ah, odnosno P = a (a + 2h).
D   0 DK DK
Za pravilnata {estagolna piramida, bidej}i e % , , zatoa:
 
D   §D  ·
3 %0  DK , odnosno 3 D ¨¨  K ¸¸ .
 ©  ¹
Pravilniot tetraedar, bidej}i e ograni~en so ~etiri skladni ramnostrani tria-
golnici so strana a, zatoa formulata za negovata plo{tina glasi:
D 
3 ˜ D ˜  .

Plo{tinata na nepravilnite piramidi ja nao|ame, koga, prvo, }e gi najdeme pooddelno
plo{tinite na site nejzini yidovi, a potoa istite gi sobereme.
Zada~a 1. Da se presmeta plo{tinata na pravilnata triagolna piramida, koja ima
osnoven rab a = 6 cm i visina H = 8 cm.
Re{enie: Prvo treba da ja presmetame apotemata na piramidata. Nea ja dobivame so
primena na Pitagorovata teorema za pravoagolniot triagolnik SOM (crt. 46a), od kade
{to:
D 
K +   U  , a bidej}i U 
, toga{
 
§D ·

§ ·
K +  ¨¨ ¸¸   ¨¨ ¸¸     |  FP .
©  ¹ ©  ¹
Baranata plo{tina na piramidata }e bide:

D§D  · §  ·
3 ¨¨  K ¸¸ ¨¨   ˜   ¸¸ |   ˜    
©  ¹ ©  ¹
      ˜    , t.e. 3 | FP  .
Zada~a 2. Da se presmeta plo{tinata na pravilna ~etiriagolna piramida, ~ij osnoven
rab e a = 10 cm, a bo~niot rab s = 13 cm.

169
Re{enie: Prvo, }e ja presmetame dol`inata na apotemata na piramidata:

§D·
K 
V ¨ ¸      FP 
©¹
Potoa, ja presmetuvame plo{tinata na piramidata:
P = a(a + 2h) = 10(10 + 2 · 12) = 10 · 34 = 340 (cm2).

Zada~a 3. Da se presmeta plo{tinata na pravilna {estagolna piramida, ~ija visina


e H = 8 cm, a bo~niot rab s = 10 cm.

Re{enie: So primena na Pitagorovata teorema na pravoagolniot triagolnik AOS


(crt. 46 v) prvo, go opredeluvame, radiusot na opi{anata kru`nica R okolu osnovata.
Toj }e bide:

5 V  +          FP .
Bidej}i R = a, toa zna~i, deka osnovniot rab na dadenata piramida }e bide a = 6 cm.
Potoa ja opredeluvame i dol`inata na apotemata na piramidata od pravoagolniot
triagolnik SOM (crt. 46 v), pa dobivame:
 
§D· §·
K V  ¨ ¸   ¨ ¸     |  FP .
©¹ ©¹
Baranata plo{tina na piramidata e:
§D  · §  ˜  ·
3 D ¨¨  K ¸¸ |  ˜  ¨   ¸  ˜     ,
©  ¹ ©  ¹
t.e. P | 265,14 cm2.

Zada~a 4. ^etiriagolna piramida so skladni bo~ni rabovi i visina H = 6 cm za osnova


ima pravoagolnik, ~ii dimenzii se: a = 16 cm i b = 5 cm. Da se presmeta plo{tinata na taa
piramida.

Re{enie: Kako izgleda edna takva piramida i nejzinata mre`a, mo`e da se vidi na
crte` 49. Plo{tinata na nejzinata osnova }e bide B = ab, a bo~nata povr{ina ja so~inuvaat
~etiri ramnokraki triagolnici, od koi dva po dva se skladni.

Ako visinite na neskladnite bo~ni yidovi gi ozna~ime so ha i hb, toga{ plo{tinata


na bo~nata povr{ina }e bide:
DKD EK
0 ˜  ˜ E DKD  EKE .
 
Spored toa formulata za plo{tina na ~etiriagolna piramida so skladni bo~ni rabovi
i osnova pravoagolnik, glasi:

P = ab + aha + bhb.

170
Crte` 49

Prvo, }e gi opredelime bo~nite visini ha i hb so pomo{ na Pitagorovata teorema od


pravoagolnite triagolnici SOM i SON (crt. 49a).

§D·
KD 
+ ¨ ¸           FP ,
©¹

§E·
KE + ¨ ¸ 
        FP .

© ¹
Potoa ja opredeluvame plo{tinata na dadenata piramida:
P = ab + aha + bhb = 16 · 5 + 16 · 16, 5 + 5 · 10 = 80 + 104 + 50 = 234 (cm)2.

1. Presmetaj ja plo{tinata na pravilna triagolna piramida so osnoven rab 5 cm i bo~en


rab 6 cm.
2. Presmetaj ja plo{tinata na pravilna ~etiriagolna piramida so osnoven rab 3 cm i
bo~en rab 7,8 cm.
3. Presmetaj ja plo{tinata na pravilna {estagolna piramida so osnoven rab a = 14 cm i
bo~en rab s = 2, 5 dm.
4. Presmetaj ja plo{tinata na pravilen tetraedar so rab 12 cm.
5. Mre`ata od pravilen tetraedar nacrtana e vo forma na ramnostran triagolnik so
strana 12 cm. Odredi ja visinata na tetraedarot.
6. Kolku }eramidi se potrebni za pokrivawe na edna ku}a, ~ij pokriv ima forma na
pravilna ~etiriagolna piramida so osnoven rab 9 m i bo~en rab 7,5 m, ako za
pokrivaweto na 1 m2 potrebni se 15 }eramidi.

171
7. Pravilna ~etiriagolna piramida so osnoven rab a = 8 cm ima plo{tina 448 cm2.
Opredeli ja visinata i bo~niot rab na piramidata.
8. Pravoagolen triagolnik so kateti a = 3 cm i b = 4 cm pretstavuva osnova na edna
triagolna piramida so ednakvi bo~ni rabovi dolgi po 8 cm. Presmetaj ja plo{tinata
na taa piramida.
9. Bo~nata plo{tina na edna pravilna triagolna piramida iznesuva M = 144 cm2. Odredi
go osnovniot rab na piramidata, ako nejzinata apotema e dolga 12 cm.
10. Apotemite na pravilna ~etiriagolna piramida se ednakvi na osnovniot rab na
piramidata. Najdi formula po koja }e se presmetuva plo{tinata na takvite pravilni
~etiriagolni piramidi, a potoa presmetaj ja plo{tinata na edna takva piramida,
~ij rab e dolg 12 cm.
11. Pravilna ~etiriagolna piramida so osnoven rab a = 11 cm ima plo{tina 792cm2.
Odredi go bo~niot rab i visinata na taa piramida.
12. Plo{tinata na edna pravilna ~etiriagolna piramida, iznesuva 90 cm2, a plo{tinata
na nejzinata bo~na povr{ina e 65 cm2. Presmetaj gi dol`inite na osnovniot rab i
visinata na taa piramida.

IV.15. VOLUMEN NA PIRAMIDA

Od podebel karton (ili lamarina) da


napravime modeli na pravilna ~eti-
riagolna piramida i pravilna ~etiri-
agolna prizma, takvi {to, tie da imaat
skladni osnovi i ednakvi visini.
Od modelot na prizmata da ja otfrlime
ednata negova osnova, a modelot na pira-
midata neka e bez osnova (crt. 50), taka
{to, tie da stanat dve kutii.
Crte` 50
Ako modelot - piramida go napolnime
so siten pesok (ili voda), a potoa pesokot go presipeme vo modelot - prizma, }e
zabele`ime deka modelot - prizma }e se napolni do gore so pesok duri po tretoto
presipuvawe.
Ako napravime sli~en obid i so nekoi drugi modeli na piramida i prizma, koi imaat
skladni osnovi i ednakvi visini, }e dojdeme do ist zaklu~ok, imeno deka:

Volumenot na piramidata e 3 pati pomal (pretstavuva ) od volumenot na prizmata,

koja ima osnovi i visina ednakvi na osnovata i visinata na piramidata.
Bidej}i volumenot na prizmata e ednakov na proizvodot od plo{tinata na osnovata i
dol`inata na visinata, t.e. na BH, od gornite obidi nasetuvame deka }e va`i slednata
teorema.

172

Teorema: Volumenot na piramidata e ednakov na od proizvodot na plo{tinata


na nejzinata osnova i dol`inata na visinata, t.e. 9 ˜ %+


Od gornava teorema lesno gi dobivame i specijalnite formuli za plo{tina na


pravilnite piramidi, i toa:
 D  D + 
- za pravilna triagolna piramida 9 ˜ ˜ + , t.e. 9 ,
  
  D +
- za pravilna ~etiriagolna piramida 9 ˜ D ˜ + , t.e. 9 ,
 
 D   D + 
- za pravilna {estagolna piramida 9 ˜ ˜ + , t.e. 9 .
  
Zada~a 1. Da se opredeli volumenot na pravilnata ~etiriagolna piramida, ~ij bo~en
rab e ednakov na osnovniot rab a = 8 cm.

Re{enie: So primena na Pitagorovata teorema, prvo, }e ja odredime visinata na


piramidata. Od pravoagolniot triagolnik SAO (crt. 46 b) pri s = a = 8 cm, imame:
 
§D · §  ·
+ V 5  
D  ¨¨
¸¸

  ¨¨ ¸¸     |  FP .
©  ¹ ©  ¹
Volumenot na piramidata, }e bide:
D  +  ˜  
9 |  ˜   , t.e. V | 121, 6 cm3.
 

Zada~a 2. Kolkava masa ima edna pravilna triagolna piramida od bakar so osnoven
rab a = 6 cm i bo~en rab s = 10 cm, ako znaeme deka gustinata na bakarot e 8,9?

Re{enie: Masata na piramidata }e ja dobieme, koga nejziniot volumen }e go pomno-


`ime so gustinata na bakarot. Zatoa, prvo, go presmetuvame volumenot, no za pres-
metuvawe na volumenot potrebno e da ja znaeme visinata na piramidata.
Visinata na piramidata }e ja dobieme so primena na Pitagorovata teorema na
pravoagolniot triagolnik SOA (crt.46 a).
 

§D · 
§ ·
+ V  ¨¨ ¸¸   ¨¨ ¸¸     |   FP .
©  ¹ ©  ¹
D  +   ˜   ˜ 
Spored toa, 9 |  ˜   ˜    , t.e. V | 48, 8 cm3.
 
Baranata masa M, }e bide: M = V · s = 48, 8 · 8, 9 = 434, 32 (g).

173
Zada~a 3. Da se presmeta volumenot na pravilna {estagolna piramida so osnoven rab
a = 4 cm, ako plo{tinata na nejziniot najgolem dijagonalen presek iznesuva 26 cm2.
Re{enie: Najgolemiot dijagonalen presek kaj pravilnata {estagolna piramida
pretstavuva ramnokrak triagolnik, ~ija osnova e 2R, a visinata vo isto vreme mu e i
visina na piramidata. Spored toa, formulata za plo{tina na toj presek }e glasi:
5 ˜ +
3 d.pr. 5+ .


Bidej}i, pak, Rd.pr. = 26, a R = a = 4 cm, zatoa od ravenkata 26 = 4H nao|ame +  FP .

Baraniot volumen na piramidata }e bide:
D +   ˜  ˜ 
9  ˜  ˜   , t.e. V | 90 cm3.
 

1. Presmetaj go volumenot na pravilna ~etiriagolna piramida so osnoven rab a = 6 cm


i bo~en rab c = 9 cm.
2. Keopsovata piramida vo Egipet ima za osnova kvadrat so strana a = 234 m.
Piramidata e visoka 148 m, presmetaj go nejziniot volumen.
3. Volumenot na edna pravilna ~etiriagolna piramida e B = 405 cm3, a visinata H = 15cm.
Odredi go osnovniot rab na piramidata.
4. Pravilna ~etiriagolna piramida so osnoven rab 8 cm ima volumen 576 cm3. Opredeli
gi visinata i plo{tinata na taa piramida.
5. Osnovniot rab na edna pravilna {estagolna piramida e 4,5 cm, a plo{tinata na
pogolemiot dijagonalen presek e 72 cm3. Presmetaj go volumenot na piramidata!
6. Visinata na edna pravilna ~etiriagolna piramida e H = 7 cm, a plo{tinata na
dijagonalniot presek e 19, 73 cm2. Presmetaj go volumenot na piramidata.
7. Plo{tinata na bo~nata povr{ina na edna pravilna ~etiriagolna piramida iznesuva
65 cm2, a osnovniot rab e a = 5 cm. Opredeli go volumenot na piramidata.
8. Presmetaj go volumenot na pravilna triagolna piramida, ~ij osnoven rab e a = 9 cm,
a bo~niot rab e c = 15 cm.
9. Izvedi formula za presmetuvawe volumen na pravilen tetraedar so rab a.
10. Presmetaj go volumenot na pravilen tetraedar so rab 1 dm.
11. Masivna metalna kocka so rab 6 cm pretopena e vo pravilna ~etiriagolna piramida,
~ij osnoven rab e ednakov na rabot na kockata. Kolkava }e bide visinata na
piramidata?
12. Kako }e se promeni volumenot na edna pravilna piramida, ako: osnovniot rab go
zgolemime 2 pati, a visinata ja namalime 2 pati, b) osnovniot rab go zgolemime
3 pati, a visinata ostane nepromeneta?

174
G CILINDAR
IV.16. POIM I ELEMENTI NA CILINDAROT.
PRESECI NA CILINDAR

]e poka`eme deka nekoi povr{ini


mo`at da bidat obrazuvani so dvi`ewe
na dadena linija vo prostorot.
Neka s e proizvolna ramninska kriva,
koja vo specijalen slu~aj mo`e da bide i
prava, a p - edna nepodvi`na prava nare-
~ena oska, koja so krivata s le`i na ista
ramnina (crt. 51)
Da zamislime deka krivata s se vrti,
(rotira) vo prostorot okolu dadenata ne-
podvi`na prava p. Pri toa, vrtewe pret-
postavuvame deka krivata {to rotira i
oskata na vrteweto se taka zaemno cvrsto
svrzani, {to proizvolnata to~ka 0  V
sekoga{ obrazuva kru`nica k so centar Crte` 51
2  S , ~ija ramnina e normalna na oskata
p (crt. 51). Pritoa, se obrazuva edna povr{ina koja se vika rotaciona povr{ina.
Zna~i, rotaciona povr{ina se vika povr{inata, koja se obrazuva pri vrteweto
(rotacijata) na edna kriva, okolu dadena nepodvi`na prava, nare~ena oska na vrteweto.
Krivata s {to ja obrazuva rotacionata povr{ina se vika nejzina generatrisa.
Ako generatrisata e prava paralelna na oskata na vrteweto, toga{ dobienata rotaciona
povr{ina se vika u{te i kru`na cilindri~na povr{ina. Rotacionata i kru`nata
cilindri~na povr{ina se zakriveni (val~esti) povr{ini. Toa, zna~i, deka nieden niven
del ne mo`e da se sovpadne so del od ramnina.
Od gornoto sleduva deka presekot na koja i da bilo rotaciona povr{ina i edna ramnina
{to e normalna na oskata na vrteweto, sekoga{ e kru`nica.

Definicija 1. Telo, koe e nastanoto so vrtewe na nekoja figura okolu nekoja


nepodvi`na oska na vrteweto, se vika rotaciono telo.

Da go razgledame vrteweto na pravoagolnikot ABCD okolu edna negova strana, na


primer, okolu stranata CD (crt. 52). O~igledno e deka pravata CD, okolu koja se vrti
pravoagolnikot ABCD, }e pretstavuva oska - na vrteweto. Spored toa, site to~ki od
stranata CD ostanuvaat nepodvi`ni. Sprotivnata strana AB na pravoagolnikot, {to
e paralelna na oskata na vrteweto, }e obrazuva povr{ina koja e del od nekoja kru`na
cilindri~na povr{ina. Drugite dve strani, pak, AD i BC na pravoagolnikot, }e obrazuvaat
dva skladni paralelni kruga so centri vo to~kite D i C i radius U $' %& .

175
Povr{inata, koja e sostavena od kru`nata cilindri~na
povr{ina {to e obrazuvana od otse~kata AB i dvata skladni
paralelni kruga {to se obrazuvani od otse~kite AD i BC (crt. 52),
go razbiva mno`estvoto to~ki od prostorot ({to ne — pripa|aat)
na dve oblasti: vnatre{na i nadvore{na. Unijata od taa povr{ina
i vnatre{nata oblast pretstavuva edna zatvorena i ograni~ena
prostorna oblast, t.e. geometrisko telo, koe go vikame prav kru`en
cilindar.
C
Postojat i drugi vidovi cilindri, no bidej}i nie }e razgleduvame
samo pravi kru`ni cilindri, zatoa prosto }e gi vikame samo
cilindri.

Definicija 2. Cilindar se vika teloto {to e ograni~eno so edna cilindri~na


povr{ina i dva skladni paralelni kruga.

Dvata skladni paralelni kruga se vikaat osnovi (ili bazi) na cilindarot, a delot od
kru`nata cilindri~na povr{ina zaklu~en me|u osnovite - bo~na povr{ina na cilindarot.
Radiusot na sekoja osnova se vika radius na cilindarot, a otse~kata AB vo sekoja nejzina
polo`ba pri vrteweto, se vika generatrisa na cilindarot.
Pravata {to minuva niz centrite na osnovite na cilindarot se
vika oska na cilindarot, a otse~kata od nea me|u centrite na osnovite 3 4
- visina na cilindarot. 2

Cilindarot go crtame kako na crte` 53. Prvo, go crtame pravo-


agolnikot MNQP so osnova MN - dijametarot na cilindarot i visina
MP - visinata na cilindarot (crt. 53). Osnovite na cilindarot gi
crtame kako elipsi i toa, zadniot del na elipsata pri dolnata osnova
na cilindarot go crtame so isprekinati linii. 0 1
2
1. Pravoagolnikot ABCD se vrti okolu stranata AB. [to obrazuva
HWY
pri toa vrtewe: a) to~kata C, b) to~kata D, v) stranata BC, g) stranata
CD, d) stranata AB?

Od na~inot na koj nastanuva rotacionoto telo cilindar, sleduva deka:

- Site generatrisi na cilindarot se ednakvi i paralelni me|u sebe.


- Sekoja generatrisa e normalna na ramninite na osnovite na cilindarot.
- Visinata na cilindarot e ednakva i paralelna na sekoja generatrisa na
cilindarot.

Dol`inite na radiusot, visinata i generatrisata na cilindarot }e gi ozna~uvame


soodvetno so bukvite R, H i s.

176
Kaj cilindarot }e gi izdvoime slednite preseci:

1°. Presekot {to se dobiva koga cilindarot }e se prese~e so ramnina, koja e paralel-
na na ramninite na osnovite na cilindarot, se vika paralelen presek (crt. 54 a). Toj
sekoga{ e krug skladen na osnovite.
2°. Presekot {to minuva niz oskata na cilindarot se vika oskin presek. Toj sekoga{
e pravoagolnik, ~ii strani se: dijametarot na osnovata i generatrisata na cilindarot
(crt. 54 b). Vo specijalen slu~aj oskiniot presek na cilindarot mo`e da bide i kvadrat.
Toa }e bide koga generatrisata na cilindarot e ednakva na dijametarot na osnovata.
Cilindar, na koj oskiniot presek e kvadrat, se vika ramnostran cilindar.
2. Koj presek kaj cilindarot se vika: a) paralelen presek, b) oskin presek?
Kakva figura pretstavuva sekoj od niv?

3. Navedi nekolku primeri na predmeti koi imaat forma na cilindar.


4. Opredeli go radiusot i generatrisata na cilindarot, {to se dobiva koga pravoa-
golnikot so strani a = 7 cm i b = 4 cm se vrti okolu:
a) stranata a, b) stranata b,
v) simetralata na stranata a, g) simetralata na stranata b!
5. Dali kaj prav cilindar postoi to~ka, {to e ednakvo oddale~ena od sekoja to~ka na
kru`nicite na dvete negovi osnovi? Koja e taa to~ka?
6. Presmetaj ja plo{tinata na oskiniot presek na cilindar, ~ii radius i visina se:
a) R = 4 cm, H = 7 cm, b) R = 1, 8 cm, H = 5 cm.
7. Opredeli ja dol`inata na dijagonalata na oskiniot presek na cilindar so radius
1,5 dm i visina 4 dm.
8. Kolkava plo{tina ima oskiniot presek na ramnostran cilindar so radius R = 1 dm?
9. Plo{tinata na oskiniot presek na eden ramnostran cilindar iznesuva 2,25 dm2.
Opredeli gi radiusot i visinata na cilindarot.
10. Plo{tinata na oskiniot presek na eden cilindar, so visina 32 cm iznesuva 416 cm2.
Odredi go radiusot na cilindarot.
11. Vo cilindar so visina 15 cm postavena e ramnina paralelna na oskata na cilin-
darot na rastojanie 4 cm od nea. Ramninata otsekuva od kru`nicata na osnovata na
cilindarot kru`en lak 60o. Najdi ja plo{tinata na dobieniot presek.
12. Ako bo~nata povr{ina na cilindarot ja ise~eme dol` edna generatrisa, toga{ kakva
figura dobivame?

177
IV.17. CRTAWE MRE@A NA CILINDAR

Da zememe edna kutija od kon-


zerva, {to ima forma na cilin-
dar, i da ja izre`eme po kru`-
nicite na osnovite i po edna
generatrisa, kako {to e poka-
`ano na crte` 55a. Ako potoa
kutijata ja razvieme, taka {to
nejzinata bo~na povr{ina, zaed-
no so osnovite da gi postavime
da legnat vo edna ramnina, }e ja
dobieme mre`ata na cilindarot
(crt. 55b). Kako {to gledame:
Crte` 55

Mre`ata na cilindarot se sostoi od dva skladni kruga i eden pravoagolnik.

Pravoagolnikot }e ima dol`ina ednakva na perimetarot na osnovata (L = 2pR) i visina


ednakva na visinata na cilindarot.

Zada~a: Da se nacrta mre`ata na cilindarot so radius R = 0,8 cm i visina H = 2,5 cm.

Re{enie: Prvo, ja crtame razvienata bo~na povr{ina na cilindarot, t.e.


pravoagolnikot so dol`ina 2pR i visina H = 2,5 cm. Dol`inata 2pR koga ni e poznat
radiusot, lesno mo`e da se presmeta. Bidej}i e R = 0,8 cm, nao|ame deka: 2pR | 2 · 3,14 · 0,8 =
5,024 (cm) | 5 (cm).
Dol`inata pR mo`e prib-
li`no da se opredeli i kons-
truktivno kako hipotenuza na
pravoagolen triagolnik so
kateti R i 3R (crt. 56a). Ako
dol`inata pR e opredelena,
toga{ lesno ja opredeluvame i
baranata dol`ina 2pR.
Koga s– toa e gotovo, go cr-
tame pravoagolnikot so dol-
`ina 2pR = 5 cm i visina
H = 2,5 cm. Na krajot od dvete
strani na toj pravoagolnik crta-
Crte` 56

178
me po eden krug so radius R = 0,8 cm, koi se dopiraat do nego. Taka ja dobivame baranata mre`a
na cilindarot (crt. 56b).
Model na cilindar mo`eme da napravime na sledniov na~in:
Na par~e karton (ili lamarina) prvo ja crtame mre`ata na cilindarot po dadenite
negovi dimenzii, potoa nea ja se~eme; pravoagolnikot go svitkuvame da stane bo~na povr-
{ina na cilindarot, a krugovite gi preklopuvame kako na crte` 55a. Sklopeniot model
za da ostane trajno sostaven, potrebno e krugovite i bo~nata plo{tina, kade {to se
zdru`uvaat, da gi zalepime.

1. Nacrtaj ja mre`ata na cilindar, ~ii dimenzii se:


a) R = 1 cm i H = 3,5 cm, b) R = 1,5 cm, i H = 2,5 cm.
2. Nacrtaj ja mre`ata i napravi model na ramnostran cilindar, ako e:
a) R = 1,4 cm, b) H = 5 cm.
3. Nacrtaj ja mre`ata na polucilindar (telo {to se dobiva koga cilindarot }e se
prepolovi so eden negov oskin presek), ako: R = 2 cm i H = 6,5 cm.
Potoa, napravi model na polucilindarot od karton!
4. Nacrtaj ja mre`ata na bure {to ima forma na cilindar so radius na osnovata
4 dm i visina 1,2 m vo razmer 1 : 20.
5. Nacrtaj mre`a na eden cilindar, a potoa presmetaj gi plo{tinite na ramninskite
figuri.

IV.18. PLO[TINA NA CILINDAR

Od definicijata i mre`ata na cilindarot gledame deka negovata povr{ina (grani-


ca) se sostoi od dva skladni kruga i edna kru`na cilindri~na povr{ina - bo~na povr{ina
na cilindarot. Spored toa:
Plo{tinata na cilindarot e ednakva na zbirot na plo{tinite na dvata skladni kruga
- osnovite i negovata bo~na plo{tina, t.e.
P = 2B + M,
kade {to B e plo{tina na ednata osnova (krug), a M - plo{tina na bo~nata povr{ina na
cilindarot.
Znaete deka plo{tinata na krugot ja presmetuvame po formulata B = pR2. Bo~nata
povr{ina na cilindarot e kriva povr{ina, no koga taa }e se razvie vo ramninata od nea
se dobiva eden pravoagolnik, ~ija dol`ina e ednakva na dol`inata na kru`nicata na
osnovata, a visinata e ednakva na visinata na cilindarot (crt. 55 b). Spored toa, }e va`i
slednava:

179
Teorema: Bo~nata plo{tina na cilindarot e ednakva na proizvodot od dol`ina-
ta na kru`nicata na osnovata i visinata na cilindarot, t.e.
M = 2 S R · H.
Spored toa, plo{tinata na cilindarot }e bide:
P = 2B + M = 2 · S R2 + 2 S R · H, t.e. P = 2 S R(R + H).
Kaj ramnostraniot cilindar visinata e ednakva na dijametarot na osnovata, t.e.
H = 2R. Spored toa, formulata za plo{tina na ramnostraniot cilindar, glasi:
P = 2 S R(R + H) = 2 S R(R + 2R) = 2 S R · 3R, t.e. P = 6 S R2.
Zada~a 1. Da se presmeta plo{tinata na cilindar, ~ij radius na osnovata e 18 cm,
a visinata mu e 5 dm.
Re{enie: Ako sakame plo{tinata na cilindarot, {to treba da ja presmetame, da bide
izrazena vo dm2, toga{ i dol`inata na radiusot treba da bide izrazena vo dm, t.e.
R = 18 cm = 1,8 dm.
So zamena na vrednostite na R i H vo formulata dobivame:
P = 2 S R (R + H) | 2 · 3,14 · 1,8 · (1,8 + 5) = 6,28 · 1,8 · 6,8 = 76,8672 (dm2).
Zada~a 2. Bo~nata plo{tina na eden ramnostran cilindar e M = 72 dm2. Kolkava e
negovata plo{tina?
Re{enie: Bo~nata plo{tina na ramnostraniot cilindar e ednakva na:
M = 2 S RH = 2 S R ·2R odnosno M = 4 S R2. Bidej}i bo~nata plo{tina (M) e poznata,
0 
od nea mo`eme da ja opredelime vrednosta na S R2, t.e. S 5   .
 
Zamenuvaj}i ja najdenata vrednost na pR2 vo formulata za plo{tina na ramnostraniot
cilindar, dobivame: P = 6pR2 = 6 · 18 = 108 (dm2).

1. Presmetaj ja plo{tinata na cilindar, ~ij radius i visina se:


a) R = 3, 8 cm, H = 2, 5 cm, b) R = 7 cm, H = 4 dm.
2. Bo~nata plo{tina na ramnostran cilindar e M = 9 dm2. Presmetaj ja plo{tinata
na toj cilindar!
3. Presmetaj ja plo{tinata na ramnostran cilindar, ako e dadeno:
a) R = 1 dm, b) R = 15 cm, v) H = 1 m.
4. Plo{tinata na eden ramnostran cilindar e ednakva na 1701 cm2. Odredi go
radiusot na cilindarot.
5. Presmetaj ja plo{tinata na teloto, {to se dobiva, koga kvadrat so strana
a =7 cm se vrti okolu edna svoja strana!
6. Eden stolb za lepewe oglasi ima dijametar 8 dm, a e visok 2,3 m. Kolku m2
oglasi mo`at da se zalepat na nego?
7. Kolku metri kvadratni lamarina e potrebno za izrabotka na 25 ednakvi }unci
za pe~ka, ~ija dol`ina e 0,8 m, a dijametarot na otvorot iznesuva 12 cm?

180
8. Bo~nata plo{tina na eden cilindar iznesuva 188,4 cm2. Presmetaj ja plo{tinata na
toj cilindar, ako negovata visina e 7,5 cm!
9. Plo{tinata na bo~na povr{inata na eden cilindar iznesuva 141,3 cm2, a radiusot na
osnovata mu e 4,5 cm. Presmetaj ja plo{tinata na toj cilindar!
10. Edna kocka od drvo so rab 15 cm treba da se izdelka vo najgolem mo`en cilindar.
Odredi ja plo{tinata na dobieniot cilindar!
11. Eden cilindar od drvo so dijametar 14 cm i visina 2 dm treba da se izdelka vo mo`no
najgolema pravilna ~etiriagolna prizma. Odredi ja plo{tinata na taa prizma!
12. Odredi go odnosot od bo~nata plo{tina na cilindarot i plo{tinata na negoviot
oskin presek.

IV.19. VOLUMEN NA CILINDAR

Za edna prizma velime deka e vpi{ana vo daden cilindar, ako


mnoguagolnicite na osnovite na prizmata se vpi{ani vo krugovite
na osnovite na cilindarot, a bo~nite rabovi na prizmata se genera-
trisi na cilindarot (crt. 57).
Na crte` 58 pretstaveni se tri skladni cilindri, vo koi se
vpi{ani po edna pravilna prizma i toa, vo prviot e vpi{ana pra-
vilna triagolna, vo vtoriot - pravilna {estagolna, a vo tretiot -
pravilna dvanaesetagolna prizma.

Crte` 57

Gledate deka osnovite na vpi-


{anite pravilni prizmi vo trite
skladni cilindri na crte` 58 se
pravilni mnoguagolnici {to se
vpi{ani vo skladni krugovi.

Crte` 58

Ako vo daden krug (ili vo skladni krugovi) vpi{u-


vame pravilni mnoguagolnici so pogolem i s– pogolem
broj strani (crt. 59), o~igledno e deka perimetrite na
tie mnoguagolnici }e rastat i s– pove}e }e se pribli-
`uvaat do perimetarot na krugot; a, isto taka, i nivni-
te plo{tini }e rastat i s– pove}e }e se pribli`uvaat
do plo{tinata na dadeniot krug. Crte` 59

181
Poradi toa, razbirlivo e deka i volumenite na vpi{anite pravilni prizmi vo
cilindarot, koga brojot na nivnite bo~ni yidovi stanuva s pogolem i pogolem, isto
taka, }e rastat i s pove}e }e se pribli`uvaat do volumenot na cilindarot vo koj se
vpi{ani.
Toa ne upatuva na zaklu~ok deka za presmetuvawe na volumenot na cilindarot mo`eme
da ja koristime istata formula {to ja izvedovme za volumen na prizmite:
V = BH.
Spored toa, }e va`i slednava:
Teorema: Volumenot na cilindarot e ednakov na proizvodot od plo{tinata na
osnovata i dol`inata na visinata, t.e.
V = BH.

No bidej}i B =pR2, zatoa V = pR2H.


Kaj ramnostraniot cilindar, znaeme deka H = 2R, pa spored toa, formulata za negoviot
volumen, glasi: V = pR2H = pR2 · 2R, odnosno
V = 2pR3.

Zada~a 1. Kolku litri voda sobira edno bure {to ima forma na cilindar so radius na
osnovata 3 dm i visina H = 10 dm?

Re{enie: Znaete deka litarot e edinica merka za volumen na te~nosti, koja e ednakva
(ekvivalentna) na volumenot od 1 dm3. Zna~i, volumenot na bureto izrazen vo litri e
ednakov na negoviot volumen izrazen vo dm3. Volumenot na bureto vo dm3 }e bide:
V = pR2H | 3,14 · 32 · 10 = 3,14 · 90 = 282, 6 (dm3).
Zna~i, bureto sobira 282, 6 l.

Zada~a 2. Bakarna `ica dolga 50 m ima masa 200 g. Presmetaj go dijametarot na `icata,
ako gustinata na bakarot e s = 8,9 g / cm3.

Re{enie: Bidej}i masata e M = Vs, zatoa volumenot na `icata }e bide:


0  
9   FP  .
V   

9 5 9
Od V = pR2H dobivame deka 5  , ,t.e.
S+ S+

9  9  
5 ˜  ˜   ˜   |   FP .
S+ S + 

Zna~i, dijametarot na `icata e: G 5  ˜     FP   PP | PP .


182
Zada~a 3. Pravoagolnik so strani a = 7, 5 cm i b = 3 cm se vrti okolu stranata a.
Da se presmeta volumenot na cilindarot {to pritoa se dobiva.

Re{enie: Stranata a }e bide visina na cilindarot, a stranata b }e bide radius na


osnovata. Volumenot na cilindarot }e bide:
V = pb2a | 3,14 · 32 · 7,5 = 211,95 | 212 (cm3).

1. Presmetaj go volumenot na cilindar, ~ij radius i visina se:


a) R = 2,6 cm, H = 7,2 cm, b) R = 4 cm, H = 1,2 dm.
2. Presmetaj go volumenot na ramnostran cilindar, ako: a) R = 6 cm, b) H = 20 cm.
3. Presmetaj kolku litri sobira eden lonec so dijametar na otvorot 5 dm i visina 5 dm.
4. Eden bazen {to ima forma na cilindar sobira 70,65 m3 voda. Opredeli ja dlabo~inata
na bazenot, ako dijametarot mu e 6 m.
5. Iskopan e bunar so dijametar na otvorot 1,5 m i pritoa e isfrleno 14,13 m3 zemja.
Presmetaj ja dlabo~inata na bunarot!
6. Presmetaj go volumenot na cilindar {to e visok 5 cm, a perimetarot na osnovata mu
e 12,56 cm.
7. Kolku treba da e visok eden kazan, za da sobira 125 litri, a da ima dijametar na
dnoto 5 dm?
8. Edna cisterna za benzin, {to ima forma na cilindar, dolga e 7,5 m i ima dijametar
na osnovata 1,6 m. Kolku litri sobira taa?
9. Edna bakarna `ica so dijametar na presekot 2 mm ima masa 280 kg. Opredeli ja
dol`inata na `icata, ako s = 8,9.
10. Cilindar visok 2 dm ima volumen V = 14,13 dm3. Presmetaj ja negovata plo{tina.
11. Vo poluprazen cilindri~en sad so dijametar na osnovata 9,2 cm potopuvame eden kamen.
Opredeli go volumenot na kamenot, ako nivoto na vodata vo sadot se poka~i za 33 cm.
12. Presmetaj ja masata na eden `elezen detaq, {to e
prika`an na crte` 60. Dimenziite se dadeni vo
cm, a s = 7,8.

Crte` 60

183
D KONUS

IV. 20. POIM I ELEMENTI NA KONUSOT.


OSKIN PRESEK

Vidovme deka kru`nata cilindri~na povr{ina e rotaciona povr{ina {to se dobiva


koga nejzinata generatrisa e prava paralelna na oskata na vrteweto. Sega }e se zapoz-
naeme so eden drug vid na rotaciona povr{ina, ~ija generatrisa e prava {to ja se~e
oskata na vrteweto.
Neka p e edna proizvolna nepodvi`na prava {to }e ja zememe
za oska na vrteweto, a s - nekoja prava {to ja se~e oskata p pod
ostar agol a vo nekoja to~ka S (crt. 61). Da zamislime deka
pravata s se vrti (rotira) vo prostorot okolu nepodvi`nata
prava p, taka {to postojano ja se~e pravata p vo to~kata S pod
ist agol a. Pri toa vrtewe na generatrisata s okolu oskata p,
o~igledno e deka normalata MO, {to e spu{tena od proizvolnata
to~ka 0  V i 0  S kon pravata p, obrazuva krug so centar O
i ramnina normalna na oskata p, a to~kata M obrazuva kru`nica
{to go ograni~uva toj krug.
Rotacionata povr{ina, {to ja obrazuva generatrisata s pri
ova vrtewe, se vika kru`na konusna povr{ina. Kru`nata
konusna povr{ina isto kako i cilindri~nata povr{ina e
Crte` 61
zakrivena (val~esta) povr{ina. To~kata S se vika vrv na
konusnata povr{ina.
Da go razgledame vrteweto na eden pravoagolen triagolnik
AOS (AO A SO) okolu edna negova kateta, na primer, okolu katetata
OS (crt. 62). O~igledno e deka: pravata OS, okolu koja se vrti
pravoagolniot triagolnik AOS }e pretstavuva oska na vrteweto.
Spored toa, site to~ki od stranata OS ostanuvaat nepodvi`ni.
Hipotenuzata AS na pravoagolniot triagolnik, {to ja se~e koso
oskata na vrteweto, }e obrazuva povr{ina {to e del od nekoja
kru`na konusna povr{ina, ~ij vrv e vo to~kata S. Drugata kateta,
pak, AO na triagolnikot, bidej}i e normalna na oskata na
vrteweto, }e obrazuva krug so centar O i radius 5 2$ (crt. 62).
Povr{inata, koja e sostavena od delot na kru`nata konusna
povr{ina so vrv S, {to e obrazuvana od hipotenuzata AS i krugot
K( 2 2$ ) {to e obrazuvan od katetata OA (crt. 62) go razbiva Crte` 62

mno`estvoto to~ki od prostorot ({to ne — pripa|aat) na dve


oblasti: vnatre{na i nadvore{na. Unijata od taa povr{ina i vnatre{nata oblast
pretstavuva edna zatvorena i ograni~ena prostorna oblast, t.e. edno geometrisko telo,
koe go vikame prav kru`en konus.

184
Postojat i drugi vidovi konusi, no, bidej}i nie }e razgledame, samo pravi kru`ni
konusi, zatoa }e gi vikame samo konusi.

Definicija: Konus se vika telo {to e ograni~eno so del od edna kru`na konusna
povr{ina so vrv S i eden krug.

Vrvot na konusnata povr{ina se vika vrv na konusot, krugot se vika osnova (ili
baza) na konusot, a delot od konusnata povr{ina zaklu~en me|u vrvot i osnovata - bo~na
povr{ina na konusot.
Radiusot na osnovata se vika radius na konusot, a otse~kata SA koja go svrzuva vrvot S
so koja i da bilo to~ka od kru`nicata na osnovata se vika generatrisa na konusot.
Pravata {to pominuva niz vrvot S i centarot na osnovata se vika oska na konusot,
a otse~kata SO od oskata me|u vrvot i centarot na osnovata - visina na konusot.
Od na~inot na koj nastanuva rotacionoto telo-konus, sleduva deka: site generatrisi
na konusot imaat ista dol`ina.
1. Pravoagolen triagolnik ABC so prav agol vo temeto B se vrti
okolu katetata BC. [to obrazuva pri toa vrtewe:
a) temeto A, b) stranata AB, v) stranata AC, g) pravoagolniot
triagolnik ABC?
Dol`inite na radiusot, visinata i generatrisata na konusot
}e gi ozna~uvame soodvetno so bukvite R, H i s. Vsu{nost, toa se
dol`ini na stranite na pravoagolniot triagolnik AOS (crt. 62),
pa spored toa, }e go zadovoluvaat ravenstvoto s2 = R2 + H2.
Konusot go crtame kako na crte` 63. Prvo, go crtame ramnokra-
kiot triagolnik ABS so osnova AB - dijametarot na konusot i krakot
AS - generatrisa na konusot (crt. 63). Kru`nicata na osnovata na Crte` 63
konusot ja crtame kako elipsa, pri {to eden nejzin del {to e ne-
vidliv go crtame so isprekinata linija.
Ako edna ramnina {to go se~e konusot minuva niz oskata
na konusot, toga{ presekot se vika i oskin presek na konusot.
Oskiniot presek na konusot e ramnokrak triagolnik (DSAC na
crt. 64), ~ii strani se: dve generatrisi i dijametarot na
osnovata na konusot.
Vo specijalen slu~aj oskiniot presek na konusot mo`e da
bide i ramnostran triagolnik. Toa }e bide, ako generatrisata
na konusot e ednakva na dijametarot na osnovata, t.e. ako s = 2R.

Konus, na koj oskiniot presek e ramnostran triagolnik,


Crte` 64
se vika ramnostran konus.

185
2. Kakva figura e presekot na eden prav konus so ramnina, koja e: a) normalna na oskata,
b) mine niz oskata? Koj od tie preseci se vika oskin presek?
3. Nabroj nekolku primeri na predmeti koi imaat forma na konus.
4. Opredeli gi dol`inite na radiusot i generatrisata na konusot {to se dobiva koga
ramnokrak triagolnik so osnova a = 6 cm i krak b = 5 cm se vrti okolu simetralata na
osnovata?
5. Dali vo prav konus postoi to~ka, {to e ednakvo oddale~ena od sekoja to~ka na
kru`nicata na osnovata i vrvot na konusot? Ako postoi, koja e taa to~ka?
6. Dali mo`e dol`inata na generatrisata na konusot da bide ednakva na: a) negovata
visina, b) radiusot na osnovata? Obrazlo`i go odgovorot!
Cr
7. Presmetaj ja plo{tinata na oskiniot presek na konus, ~ija generatrisa i radius na
osnovata se: a) s = 13 cm, R = 5 cm, b) s = 8cm, R = 4,8 cm, v) s = 4,5 cm, R = 2,7 cm.
8. Opredeli ja plo{tinata na oskiniot presek na ramnostran konus so radius na
osnovata R = 6 cm!
9. Presmetaj ja plo{tinata na oskiniot presek na konus ~ija visina e 15 cm,
a generatrisata s = 17 cm.
10. Plo{tinata na oskiniot presek na eden konus, ~ija visina e 9,6 cm, iznesuva 69,12 cm2.
Opredeli go radiusot i generatrisata na konusot!
11. Odredi ja plo{tinata na oskiniot presek na konus, ~ija generatrisa s = 6 cm e
navednata sprema ramninata na osnovata pod agol a) 60o, b) 45o, v) 30o.
12. Ako bo~nata povr{ina na eden konus ja ise~eme dol` edna generatrisa, toga{ kakva
figura }e dobieme?

IV.21. MRE@A NA KONUS

Da zememe eden konus od karton ili lamarina, pa da go izre`eme po kru`nicata na


osnovata i po edna generatrisa, kako {to toa e poka`ano na crte` 65 a. Ako potoa osnovata
i bo~nata povr{ina na konusot gi razvieme, taka {to da mo`eme niv da gi postavime da
le`at vo edna ramnina, }e ja dobieme mre`ata na konusot (crt. 65 b).
Zabele`uvame deka, bo~nata povr{ina na konusot se razviva vo eden kru`en ise~ok
(sektor) ~ij radius e ednakov na generatrisata (s) na konusot, a dol`inata na lakot - na
dol`inata na kru`nicata na osnovata (2pR). Spored toa, mre`ata na konusot se sostoi
od eden krug i eden kru`en ise~ok (crt. 65 b).
Zada~a: Da se nacrta mre`ata na konus, ~ij radius na osnovata e R = 1,2 cm, a
generatrisata s = 4 cm.

186
Re{enie: Prvo, }e ja nacrtame raz-
vienata bo~na povr{ina na konusot,
t.e. kru`niot ise~ok A1A2S, ~ij radius
e ednakov na s = 4 cm, a dol`inata na
lakot mu e ednakva so dol`inata na
kru`nicata (2 S R) na osnovata na
konusot. No, za da go nacrtame toj
kru`en ise~ok, potrebno e prethodno
da go odredime negoviot pripaden
centralen agol a.
Poznato vi e deka dol`inata na
kru`niot lak ja presmetuvame spo-
Crte` 65
S 5D
red formulata O , kade {to R

e radius na kru`nicata ~ij del e toj lak. Vo ovoj slu~aj, kaj na{iot kru`en ise~ok, {to
rte` 43 sakame da go nacrtame, radius e generatrisata s na konusot, a dol`inata, pak, na lakot
treba da bide ednakva u{te i so perimetarot na osnovata na konusot, t.e. treba da e 2 S R.
S VD
Spored toa, treba da postoi odnosot S 5 od kade {to baraniot centralen agol na

na{iot kru`en ise~ok }e bide:
q 5 q ˜ 
D q .
V 
Sega, so pomo{ na aglomer, go crtame
centralniot agol D ($6$ q , pa
okolu negovoto teme S opi{uvame kru`en
lak so radius s = 4 cm (crt. 66). Taka go
dobivame kru`niot ise~ok A1A2S. Potoa
koj i da bilo radius na kru`niot ise~ok
go prodol`uvame (nadvor od lakot A1A2)
za otse~kata BO, koja e ednakva na radiusot
na konusot R = 1,2 cm. Na krajot od to~kata
O opi{uvame kru`nica so radius na ko-
nusot. Taka ja dobivame mre`ata na dade-
niot konus.
Namesto generatrisata mo`e da bide
dadena visinata na konusot. Vo toj slu~aj,
generatrisata s na konusot ja opredeluvame
konstruktivno, kako hipotenuza na pra- Crte` 66
voagolniot triagolnik, ~ija edna kateta e
radiusot na osnovata (R), a drugata kateta e visinata (H) na konusot.
Ako sakame da napravime model na konus od karton, toga{, prvo, ja crtame negovata mre`a
na par~e karton, a potoa ja se~eme. Koga kru`niot ise~ok }e go svitkame taka {to da stane
bo~na povr{ina na konusot, a krugot da go preklopime }e go dobieme baraniot model na
konusot.

187
1. Nacrtaj ja mre`ata na konus, ~ii dimenzii se:
a) R = 1,4 cm, s = 3,5 cm, b) R = 3 cm i H = 4 cm.
2. Nacrtaj ja mre`ata na ramnostran konus, ako e: a) R = 4, 2 cm, b) s = 7 cm. Poka`i deka
razvienata bo~na povr{ina na ramnostraniot konus e polukrug so radius s!
3. Nacrtaj mre`a na polukonus (telo {to se dobiva koga konusot se prepolovi so eden
negov oskin presek), ako e R = 2,7 cm i s = 7 cm. Potoa napravi model na polukonusot!
4. Kolkav centralen agol ima kru`niot ise~ok, {to se dobiva so razvivawe na bo~nata
povr{ina na konus so radius 3 dm i generatrisa 1 dm.
5. Kolkav centralen agol ima kru`niot ise~ok, {to se dobiva so razvivawe na bo~nata
povr{ina na konus so radius 6 cm i visina 8 cm.
6. Kru`niot ise~ok, ~ij radius e 6 cm, ima centralen agol 150o. Toj kru`en ise~ok e
svitkan vo bo~na povr{ina na konus. Odredi go radiusot na dobieniot konus?
7. Polukrug so radius 7,2 cm e svitkan vo bo~na povr{ina na konus.
Kolkav e radiusot na osnovata na toj konus?
8. Visinata na eden konus e ednakva na radiusot na osnovata na konusot.
Kakov triagolnik e oskiniot presek na toj konus?
9. Vo koi slu~ai, agolot pri vrvot na oskiniot presek na eden prav konus }e bide:
a) ostar, b) prav, v) tap?
10. Kolkava e plo{tinata na kru`en ise~ok so radius r i dol`ina l na soodvetniot lak?

IV.22. PLO[TINA NA KONUS

Kako {to vidovme, povr{inata (granicata) na konusot se sostoi od edna osnova (krug)
i edna kru`na konusna povr{ina - bo~na povr{ina na konusot. Spored toa:

Plo{tinata na konusot e ednakva na zbirot na plo{tinata na osnovata i plo{tinata


na negovata bo~na povr{ina, t.e.
P = B + M,

kade {to B e plo{tina na osnovata, a M - plo{tina na bo~nata povr{ina na konusot.


Osnovata na konusot e krug, pa zatoa B = pR2.
Bo~nata povr{ina na konusot e kriva povr{ina, no koga taa }e se razvie vo ramnina,
od nea se dobiva eden kru`en ise~ok (sektor), ~ij radius e ednakov na generatrisata (s)
na konusot, a dol`inata na lakot - na dol`inata na kru`nicata (2pR) na osnovata na
konusot.

188
Poznato ni e deka, plo{tinata na kru`en ise~ok e ednakva na poluproizvodot od
dol`inata na kru`niot lak i negoviot radius. Spored toa, }e va`i slednava:

Teorema: Bo~nata plo{tina na konusot e ednakva na poluproizvodot od dol`inata


na kru`nicata na osnovata i dol`inata na negovata generatrisa:
S 5 ˜ V
0 , t.e. M = pRs.

Ottuka sleduva deka, formulata za plo{tina na konusot, }e glasi:
P = B + M = pR2 + pRs, odnosno P = pR (R + s).
Kaj ramnostraniot konus, bidej}i e s = 2R formulata za negovata plo{tina }e glasi:
P = pR (R + 2R) = pR · 3R = 3pR2.
Zada~a 1. Pokrivot na edna kula ima forma na konus so generatrisa s = 4,5 m i dijametar
na osnovata d = 6 m. Kolku metri kvadratni lamarina e potrebno za pokrivawe na kulata?
Re{enie: Tuka doa|a predvid samo plo{tinata na bo~nata povr{ina na konusot, koja
ja presmetuvame spored formulata: M = pRs | 3,14 · 3 · 4,5 = 42,39.
Zna~i, za pokrivawe na kulata potrebno e 42,39 m2 lamarina.

Zada~a 2. Pravoagolen triagolnik ABC so kateti a = 4 cm, b = 7,5 cm se vrti okolu


katetata b. Da se presmeta plo{tinata na dobienoto rotaciono telo.

Re{enie: Pri toa vrtewe }e se dobie konus so radius R = a = 4 cm i visina H = b = 7,5 cm.
Za presmetuvawe na plo{tinata na dobieniot konus potrebno e da ja znaeme negovata
generatrisa.
Generatrisata na konusot e ednakva na hipotenuzata na pravoagolniot triagolnik
ABC, a }e ja opredelime so primena na Pitagorovata teorema:
V U  +             FP .
Potoa, so zamena na R = 4 cm i s = 8, 5 cm vo formulata za plo{tina na konusot, dobivame:
P = pR (R + s) | 3, 14 · 4 · (4 + 8,5) = 3, 14 · 4 · 12,5 = 3, 14 · 50 = 157 (cm2).

1. Presmetaj ja plo{tinata na konus, ~ij radius i generatrisa se:


a) R = 3, 5 cm, s = 1 dm, b) R = 8 cm, s = 2 dm.
2. Presmetaj ja plo{tinata na konus, ako e poznato:
V
a) R = 3, 5 cm, H = 12 cm, b) R = H = 5 cm, v) s = 18 cm, + .

3. Kru`en ise~ok, ~ij radius e 1,2 cm i centralen agol 150o, e svitkan vo bo~na povr{ina
na konus. Presmetaj ja plo{tinata i visinata na toj konus.

189
4. Presmetaj ja plo{tinata na ramnostran konus, ako e: a) s = 7 cm, b) R = 4 cm.
5. Polukrug so radius 8 cm e svitkan vo bo~na povr{ina na konus. Presmetaj ja
plo{tinata na toj konus!
6. Plo{tinata na oskiniot presek na eden konus e 192 cm2, a visinata mu e 16 cm.
Presmetaj ja plo{tinata na konusot!
7. Eden stolb vo forma na cilindar so dijametar na osnovata 9 dm i visina 2,5 m, zavr{uva
na vrvot so ramnostran konus, {to ima osnova kako i cilindarot. Presmetaj ja
plo{tinata na toj stolb!
8. Odredi go odnosot od plo{tinata na osnovata (B) i:
a) bo~nata plo{tinata (M), b) plo{tinata (P) na eden ramnostran konus.
9. Plo{tinata na osnovata na eden ramnostran konus e B = 157 cm2. Odredi ja bo~nata
plo{tina i plo{tinata na toj konus.
10. Plo{tinata na eden ramnostran konus e P = 129 dm2. Opredeli go negoviot radius.
11. Ramnokrak triagolnik so osnova 12 cm i visina 8 cm se vrti okolu svojata osnova.
Presmetaj ja plo{tinata na dobienoto rotaciono telo!
12. Kvadrat so strana 6 cm se vrti okolu ednata svoja dijagonala. Presmetaj ja plo{tinata
na dobienoto rotaciono telo!

IV.23. VOLUMEN NA KONUS

Za edna piramida velime deka e vpi{ana vo daden konus, ako


nejzinata osnova e mnoguagolnik - vpi{an vo kru`nicata na osnovata
na konusot, a vrvot i se sovpa|a so vrvot na konusot (crt. 67).
O~igledno e deka, bo~nite rabovi na piramidata, {to e vpi{ana vo
konus, se generatrisi na konusot.
Za edna piramida, pak, velime deka e opi{ana okolu daden konus,
ako nejzinata osnova e mnoguagolnik - opi{an okolu kru`nicata na
osnovata na konusot, a vrvot se sovpa|a so vrvot na konusot. Vo toj
slu~aj, bo~nite yidovi na opi{anata piramida }e ja dopiraat bo~nata
povr{ina na konusot. Crte` 67

Na crte` 68 pretstaveni
se tri skladni konusi, vo koi
se vpi{ani po edna pravilna
piramida i toa vo prviot,
e vpi{ana pravilna triagol-
na, vo vtoriot - pravilna
{estagolna, a vo tretiot -
pravilna dvanaesetagolna
piramida.
Crte` 68

190
Ako vo daden konus (ili vo skladni konusi) vpi{uvame pravilni piramidi so pogolem
i s pogolem broj strani (yidovi) (crt. 68), o~igledno e deka, plo{tinite na osnovite na
tie piramidi }e rastat i s– pove}e }e se pribli`uvaat do plo{tinata na osnovata na
konusot, a, isto taka, i volumenite na taka vpi{anite pravilni piramidi }e rastat i s–
pove}e }e se pribli`uvaat do volumenot na dadeniot konus.
Toa n upatuva na zaklu~ok, deka za presmetuvawe na volumenot na konusot mo`eme da
%+
ja polzuvame istata formula so koja go presmetuvame i volumen na piramidite 9 .

Spored toa }e va`i slednava:

Teorema: Volumenot na konusot e ednakov na od proizvodot na plo{tinata na

negovata osnova i dol`inata na visinata, t.e.
%+ 5 +
9 , no bidej}i B = pR2, zatoa 9 .
 

Za opredeluvawe na volumenot na ramnostran konus (s = 2R) prvo potrebno e da ja


opredelime negovata visina. So primena na Pitagorovata teorema imame:

H2 = (2R)2 – R2 = 4R2 – R2 = 3R2, a ottuka + 5  5 .


Spored toa, formulata za volumen na ramnostraniot konus glasi:
S 5+ S 5 5  5 
9 , t.e. 9 .
  
Zada~a 1. Eden kup pesok ima forma na konus so generatrisa s = 2,5 m i perimetar na
osnovata L = 12,56 m. Da se opredeli volumenot na pesokot?

Re{enie: Od poznatiot perimetar na osnovata L = 2pR, prvo, go nao|ame radiusot:


/  
5 |  P ,
S  ˜   
a potoa so primena na Pitagorovata teorema - visinata na konusot:

+ V   5           P .
Volumenot na pesokot }e bide:

S 5  +  ˜  ˜
9 |  ˜  ˜   ˜    , t.e. V | 6, 28 m3.
 
Zada~a 2. Kolkava masa ima oloven ramnostran konus so dijametar d = 8 cm, koga se
znae deka gustinata na olovoto e 11,4?

Re{enie: Prvo go presmetuvame volumenot na ramnostraniot konus:


S 5    ˜  ˜
9 |  , t.e V | 115, 9 cm3
 
Masata na konusot }e bide: M = Vs = 115, 9 · 11, 4 = 1321, 26 (g), t.e. M | 1, 32 kg.

191
1. Presmetaj go volumenot na konus, ako se dadeni:
a) r = 7 cm, H = 6 cm, b) r = 3 cm, s = 5 cm, v) H = 3 dm, s = 3, 4 dm, g) s = 2 dm, H = R.
2. Presmetaj go volumenot na ramnostran konus, ako e dadeno:
a) R = 6 cm, b) s = 14 cm, v) H = 5 cm!
3. Visinata na eden konus e 4 m, a perimetarot na osnovata mu e 15,7 m.
Presmetaj go negoviot volumen!
4. Odredi kolku cm3 te~nost sobira edna ~a{a vo forma na konus so visina 7 cm i
dijametar na otvorot 9 cm!
5. Volumenot na konusot V = 10p cm3, a visinata mu e H = 12 cm.
Odredi go radiusot na konusot!
6. Plo{tinata na oskiniot presek na eden konus e 43 cm2, a radiusot mu e 4,3 cm. Presmetaj
go volumenot na toj konus!
7. Edno par~e lamarina, vo forma na kru`en ise~ok so radius 6 dm i centralen agol
216o e svitkano vo konus. Odredi go volumenot na dobieniot konus!
8. Da se presmeta volumenot na konus, ako plo{tinata na bo~nata povr{ina mu e
M = 15p cm2, a plo{tinata na osnovata B = 9p cm2.
9. Eden ramnostran cilindar i eden ramnostran konus se slepeni so svoite skladni
osnovi, ~ij dijametar e d = 6 cm. Da se odredi plo{tinata i volumenot na taka
kombiniranoto telo.
10. Ramnokrak triagolnik so osnova a = 2,8 dm i krak b =5 dm rotira okolu visinata.
Odredi go volumenot na dobienoto telo!
11. Kvadrat so dijagonala d = 8 cm rotira okolu edna svoja dijagonala. Da se presmetaat
plo{tinata i volumenot na dobienoto telo!
12. Romb so strana a = 5 cm i visina h = 4 cm rotira okolu edna svoja strana. Da se odredat
plo{tinata i volumenot na dobienoto rotaciono telo.

192
\ TOPKA
TOPKA

IV.24. POIM ZA SFERA I TOPKA.


ELEMENTI

Neka O e proizvolna to~ka od prostorot i R koj i da bilo pozitiven realen broj.

Definicija 1. Mno`estvoto od site to~ki od prostorot, ~ie{to rastojanie od


dadena fiksna to~ka O e ednakvo na R, go vikame sfera i go ozna~uvame so S (O, R).

To~kata O ja vikame centar na sferata, a brojot R radius na sferata. Otse~kata {to


go soedinuva centarot na sferata so koja i da bilo to~ka M od nea, isto taka, ja vikame
radius na sferata, bidej}i 20 5 . Otse~kata, pak, koja soedinuva dve proizvolni to~ki
od sferata, se vika nejzina tetiva. A tetiva na sferata {to minuva i niz nejziniot
centar se vika dijametar na sferata. O~igledno e deka dol`inata na sekoj dijametar e
ednakva na dva radiusa, t.e. d = 2R.
Krajnite to~ki na koj i da bilo dijametar gi vikame dijametralno sprotivni to~ki
na sferata.
Potsetete se na definicijata na kru`nicata! Vo {to e razlikata me|u kru`nicata i
sferata? Vo {to se razlikuvaat definiciite na tie dve figuri (mno`estva od to~ki)?
Znaete deka kru`nicata e kriva zatvorena linija vo ramninata, a kakva figura e
sferata?
Zamislete si deka edna polukru`nica so centar O i radius R
rotira okolu svojot dijametar AB (crt. 69). O~igledno e deka
pri toa vrtewe sekoja to~ka M od obrazuvanata rotaciona povr-
{ina e na rastojanie R od centarot na polukru`nicata. Zna~i,
taa rotaciona povr{ina e edna sfera S(O, R).
Spored toa, sferata mo`eme da ja razgleduvame kako rotaci-
ona povr{ina {to se dobiva pri vrteweto na edna polukru`nica
okolu svojot dijametar.
O~igledno e deka sferata S(O, R) go razbiva mno`estvoto
to~ki od prostorot ({to ne i pripa|aat) na dve oblasti: vnat-
re{na i nadvore{na. Unijata na sferata i nejzinata vnatre{na
Crte` 69
oblast pretstavuva edna zatvorena i ograni~ena prostorna oblast,
t.e. edno telo, koe go vikame topka.
Zna~i: Topka se vika teloto {to e ograni~eno so edna sfera.
Potsetete se na definicijata na krug! Topkata, sli~no na definicijata na krugot,
obi~no ja definirame vaka:
Definicija: 2. Topka e mno`estvo na site to~ki od prostorot, ~ie{to rastojanie
od edna fiksna to~ka O ne e pogolemo od R i ja ozna~uvame so T [O, R].

193
Mo`eme da zabele`ime deka sferata S(O, R) e podmno`estvo od topkata T [O, R] i
pretstavuva negova granica.
Centarot, radiusot, tetivata i dijametarot na sferata S(O, R), isto taka, se vikaat:
centar, radius, tetiva i dijametar i na topkata T [O,R].
Niz centarot O na edna topka T [O, R] da postavime ramnina S. Presekot SÇT [O, R] e ramninska
figura koja se sostoi od site to~ki, ~ie rastojanie do to~kata O ÎS ne e pogolemo od R.
Zatoa, ovoj presek }e pretstavuva krug so centar vo O i radius R i se narekuva golem
krug.
Zna~i, golem krug e presekot na edna topka i ramnina, koja minuva niz centarot na
topkata.
Presekot na edna sfera S(O, R) so proizvolna
ramnina, koja minuva niz centarot na sferata, e
kru`nica so centar vo O i radius R i se narekuva
golema kru`nica.
Ako, pak, sferata S(O,R) ja prese~eme so ram- o
nina S za koja rastojanieto od O do S e c, pri
{to 0 < c < R , (crt 70), }e poka`eme deka presekot
}e bide kru`nica, no so pomal radius. Neka O' e
ortogonalna proekcija na to~kata O vrz ramni-
nata S. Koristej}i deka 20 5 i 22c F
Crte` 70
(crt. 70), od pravoagolniot triagolnik OO'M
dobivame
 
U 02c 20  22c 5  F  5
i r }e bide radius na baranata kru`nica. Ovaa kru`nica se narekuva mala kru`nica.
Ako namesto presekot na sferata S(O, R) i ramninata S barame presek na topkata T(O, R)
i ramninata S, baraniot presek }e bide krug so istiot radius r so istiot centar O'.
Ovoj krug se narekuva mal krug.
Ako barame presek na topka T[O,R] ili sfera S(O, R) so
ramninata S pri {to to~kata O se nao|a na rastojanie R do
S, toga{ presekot }e bide edna to~ka T (crt.71). Vo toj slu~aj
za S velime deka e tangentna ramnina za topkata T[O, R],
odnosno sferata S(O, R), a T se narekuva dopirna to~ka. Ako,
pak, rastojanieto od to~kata O do ramninata S e pogolemo
od R, toga{ ramninata S i topkata T[O, R], odnosno sferata
S(O, R) nemaat zaedni~ki to~ki.
Na primer, ako Zemjata kako telo ja tretirame kako top-
ka, toga{ ekvatorot pretstavuva golema kru`nica. Isto taka,
proizvolno izbran meridijan e polukru`nica kako del od
golema kru`nica. Sekoja paralela, razli~na od ekvatorot Crte` 71
pretstavuva mala kru`nica.

194
1. Kakva figura e mno`estvoto to~ki vo prostorot, koi od dadena to~ka T se oddale-
~eni 4,5 cm?
2. Kakva figura pretstavuva presekot na topka i ramnina?
3. Topka so radius 3,4 cm e prese~ena so ramnina, koja e na rastojanie 1,6 cm od centarot.
Odredi gi radiusot, perimetarot i plo{tinata na dobieniot presek!
4. Edna topka e prese~ena so ramnina, koja e na rastojanie 3 cm od centarot. Opredeli
go radiusot na topkata, ako dobieniot presek ima plo{tina 50,24 cm2.
5. Radiusot na edna topka e 1 dm. Presmetaj gi perimetarot i plo{tinata na presekot,
{to go prepolovuva radiusot na topkata?
6. Eden grad se nao|a na 45o severna geografska {irina. Odredi ja dol`inata na para-
lelata na koja se nao|a toj grad, ako znaeme deka radiusot na Zemjata e 6370 km.
7. Presmetaj go radiusot na topka, ako perimetarot na golemiot krug na topkata
e 18,84 cm.
8. Presmetaj go radiusot na topka, ako plo{tinata na eden nejzin golem krug e 78,5 cm2.
9. Niz koi dve to~ki od sferata mo`at da se povle~at pove}e golemi kru`nici?
10. Tri paralelni ramnini go rasekuvaat dijametarot na topkata na ~etiri ednakvi
delovi. Odredi gi plo{tinite na dobienite preseci na topkata so tie ramnini, ako
topkata ima radius R= 12 cm.

IV.25. PLO[TINA NA SFERA

Sferata e kriva povr{ina isto kako i bo~nite povr{ini na cilindarot i konusot.


Vidovme deka i cilindri~nata i konusnata povr{ina uspeavme da gi razvieme" vo edna
"
ramnina, t.e. da gi napravime ramni. Me|utoa, sferata, za razlika od niv, po nikakov
na~in ne mo`eme da ja razvieme" (so site svoi delovi da ja postavime) vo edna ramnina.
"
Zatoa, nemame mo`nosti da ja dobieme mre`ata na sferata i od nea da sogledame kako }e
ja presmetuvame nejzinata plo{tina.
Za plo{tinata na sferata va`i slednata teorema, koja }e ja prifatime bez dokaz.

Teorema: Plo{tinata na sferata e ednakva na ~etirikratnata plo{tina na


nejziniot golem krug, t.e.
P = 4 · pR2, (1)
kade {to R e radius na sferata, odnosno topkata.

195
Ako ne e poznata plo{tinata na sferata, a treba da se opredeli nejziniot radius,
toga{ od formulata (1) lesno nao|ame deka:

3  3 
5 , odnosno 5 ˜ , kade {to |  .
S S  S

Posledica. Plo{tinite na dve sferi se odnesuvaat kako kvadratite na nivnite


radiusi:  
3 S 5 5
.
3 S 5  5 

Zada~a 1. Okolu kocka so rab a = 6 cm opi{ana e sfera. Da se presmeta plo{tinata na


taa sfera.

Re{enie. Bidej}i site temiwa na kockata moraat da le`at i na opi{anata sfera


okolu nea, zatoa centarot na sferata }e se nao|a vo onaa to~ka od kockata, koja e ednakvo
oddale~ena od site nejzini temiwa. Taa to~ka e presekot na dijagonalite na kockata.
Spored toa, opi{anata sfera okolu kockata }e ima radius ednakov na polovina od
dol`inata na dijagonalata na kockata, t.e.
G D   
5   FP .
  
Baranata plo{tina }e bide:



3 S 5  S   S ˜  S FP .

Zada~a 2. Perimetarot na golemiot krug na edna topka e 9p dm. Da se presmeta


plo{tinata na topkata.

Re{enie: Od perimetarot na golemiot krug mo`eme da go odredime radiusot na


topkata, t.e. od L = 2pR }e imame:
/ S
5  GP .
S S
Baranata plo{tina na topkata, toga{ }e bide:

P = 4pR2 = 4p · 4, 52 = 4p · 20, 25 = 81p | 254, 34 (dm2).

196
1. Presmetaj ja plo{tinata na topka, ~ij radius e: a) 3,5 cm, b) 1 dm, v) 14 cm.
2. Plo{tinata na topka e 452,16 cm2. Odredi ja plo{tinata na golemiot krug na topkata!
3. Plo{tinata na golemiot krug na edna topka e 9p cm2. Presmetaj ja plo{tinata na
topkata!
4. Presmetaj ja plo{tinata na povr{inata na Zemjata, koga znae{ deka dol`inata na
ekvatorot e 40 000 km.
5. Kako se odnesuvaat plo{tinite na dve sferi, ~ii radiusi se:
R1 = 2 cm i R2 = 3 cm?
6. Kako se odnesuvaat radiusite na dve sferi, ako nivnite plo{tini se odnesuvaat:
a) 1 : 4, b) 4 : 9, v) 4 : 25?
7. Kolku pati treba da se zgolemi dol`inata na dijametarot na sferata, taka {to
nejzinata plo{tina da se zgolemi: a) 4 pati, b) 9 pati, v) 25 pati, g) 100 pati?
8. Radiusite na dve sferi se 6 cm i 5 cm. Opredeli go radiusot na sferata ~ija plo{tina
e ednakva na: a) zbirot, b) razlikata od plo{tinite na dadenite sferi.
9. Kupolata na edna yvezdarnica ima forma na polusfera so dijametar 15 m. Kolku m2
bakarna lamarina e potrebno za pokrivawe na taa kupola, ako na otpadoci se smeta,
u{te 18%?
10. Radiusot na Mars, pribli`no, e ednakov na polovina od radiusot na Zemjata, a radi-
usot na Jupiter e 11 pati pogolem od radiusot na Zemjata. Kolku pati plo{tinata na
povr{inata na Jupiter e pogolema od plo{tinata na: a) Zemjata, b) Mars?
11. Dadena e kocka so rab a = 9 cm. Odredi ja plo{tinata na topka, {to e:
a) vpi{ana vo kockata, b) opi{ana okolu kockata.
12. Doka`i deka: ramnostran konus i polutopka, koi imaat skladni osnovi, imaat ednakvi
plo{tini!

IV.26. VOLUMEN NA TOPKA

Da zememe dve ~a{i od koi ednata ima forma na polutopka, a drugata forma na konus.
^a{ite neka imaat ednakvi radiusi na otvorite, a dlabo~inite neka im bidat isto kolku
{to e radiusot na otvorite (crt. 72)
Ako imame dve takvi ~a{i, toga{ lesno se uveruvame
deka prvata polutop~esta ~a{a se napolnuva to~no so dve
polni konusni ~a{i voda. Toa ni poka`uva deka volumenot
na polutopkata e dva pati pogolem od volumenot na konusot,
{to ima radius i visina ednakvi na radiusot na polu-
topkata; a volumenot na celata topka }e bide ~etiri pati
pogolem od volumenot na toj konus.
Crte` 72

197
Bidej}i volumenot na konus, ~ija visina e H = R e ednakov na:
S 5 ˜ + S 5 ˜ 5 S 5  5
9 , zatoa volumenot na topkata, }e bide: 9 .
   

 5
Teorema 1: Volumenot na topka so radius R e ednakov na 9 . (1)

5
Zabele`uvame deka volumenot na topkata mo`e da se zapi{e vo oblik 9 ˜3

kade R=4 S R2 e plo{tina na topkata.

Zada~a 1. Da se odredi masata na edno |ule (`elezna topka), ~ij dijametar e 14 cm,
ako s = 7,8.

Re{enie: Prvo, }e go presmetame volumenot na |uleto:

S 5   ˜  ˜ 
9 |  ˜   t.e. V | 1437 cm3.
 
Masata na |uleto }e bide: M = Vs = 1437 · 7, 8 = 11208, 6 (g) | 11, 2 (kg).

Zada~a 2. Odredi go volumenot na topka, ~ija plo{tina e 100p cm2.

Re{enie: Od formulata P = 4pR2 }e go odredime radiusot na topkata:


3 S
5   FP .
S S
Baraniot volumen na topkata, koga se poznati nejzinata plo{tina i radius, }e bide:
5 
9 3˜ S ˜ |  FP .
 

1. Presmetaj go volumenot na topka, ~ij radius e: a) 2,5 cm, b) 4 cm, v)12 cm.
2. Da se presmeta plo{tinata i volumenot na topka, ~ij dijametar e:
a) 9 cm, b) 2,6 dm, v) 3 cm.
3. Kolku metri kubni staklena masa e potrebno za da se napravat 500 stakleni lusteri
vo forma na topka, so vnatre{en dijametar 32 cm, a nadvore{en dijametar 33,4 cm?

198
4. Imame dve ~a{i, od koi ednata ima forma na polutopka so radius R, a drugata for-
ma na konus so radius na osnovata R i visina 2R. Koja ~a{a sobira pove}e?
5. Vo edna kocka so rab a = 7 cm e vpi{ana topka. Presmetaj ja plo{tinata i volumenot
na taa topka?
6. [upliva metalna topka so nadvore{en dijametar 4 dm i debelina na yidot 5 cm, treba
da se pretopi vo masivna topka. Odredi go radiusot na dobienata topka!
7. Radiusite na dve metalni topki se: 7,5 cm i 4,5 cm. Odredi go radiusot na topkata {to
se dobiva koga dvete topki }e se pretopat vo edna. (Koristi deka 83 = 512 | 513).
8. Presmetaj go volumenot na atmosferata, ako znae{ deka taa e debela 200 km, a radiusot
na Zemjata e 6370 km.
9. Edna masivna topka od bakar ima masa 10 kg. Odredi go radiusot na topkata, ako
gustinata na bakarot e 8,9.
10. Tri metalni topki so radius 3 cm, 4 cm i 5 cm treba da se pretopat vo edna topka.
Odredi go radiusot na novata topka! (Koristi deka 63 = 216).
11. Eden rezervoar za voda se sostoi od polutopka so radius 4,5 m i eden cilindar so ist
tolkav radius na osnovata (crt. 73). Kolku hl voda sobira toj rezervoar, ako visinata
na cilindri~niot del od nego iznesuva H = 3 m?

Crte` 73
Crte` 74

12. Na crte` 74 nacrtani se: cilindar, konus i topka. Radiusite na osnovite na cilin-
darot i konusot se ednakvi na radiusot na topkata, a visinite na cilindarot i konusot
se ednakvi na dijametarot na topkata. Poka`i deka volumenot na cilindarot e ednakov
na zbirot od volumenite na konusot i topkata.

ZA POVTORUVAWE I
UTVRDUVAWE - IV

1. Kolku pravi opredeluvaat 4 razli~ni to~ki A, B, C i D takvi {to A, B i C se koline-


arni, a D ne e kolinearna so niv?
2. Kolku ramnini se opredeleni so 4 nekomplanarni to~ki vo prostorot?

199
3. Pravite a, b i c dve po dve se se~at. Ako pravite b i c le`at vo ramninata S, dali i
pravata a le`i vo ramninata S?
4. Ramninata e opredelena so dve razli~ni paralelni pravi. Doka`i.
5. Doka`i deka, ako pravata p e paralelna so nekoja prava q koja le`i vo ramninata S,
toga{ S 6 .
6. Presmetaj ja dol`inata $% , ako e poznato deka $c%c FP , $$c FP i %%c FP ,
kade A' i B' se ortogonalnite proekcii na to~kite A i B vrz ramninata S.
7. Plo{tinata na bo~nata povr{ina na edna pravilna triagolna prizma iznesuva
390 cm2, a nejzinata visina e 2 dm. Opredeli go volumenot na prizmata.
8. Prava prizma so visina 10 cm za osnova ima romb, ~ija strana e a = 5 cm i edna dijagonala
d1 = 6 cm. Presmetaj gi plo{tinata i volumenot na taa prizma.
9. Mo`e li metalna pra~ka, dolga 16, 5 cm da se smesti vo kutija koja ima forma na
kvadar ~ii dimenzii se: 12 cm, 9 cm i 8 cm?
10. Prava prizma so visina 8, 5 cm za osnova ima ramnokrak pravoagolen triagolnik so
hipotenuza od 4 cm. Presmetaj gi plo{tinata i volumenot na prizmata.
11. Kibritna kutija ima dimenzii 5 cm, 3 cm i 1,5 cm. Kolku takvi kutii treba da se
slo`at vo najmal paket {to }e ima forma na kocka? Presmetaj go volumenot na
takov paket.
12. Pravilen tetraedar ima plo{tina 62,28 cm2. Presmetaj gi negoviot rab i visinata.
13. Prava triagolna piramida so bo~en rab s = 9 cm za osnova ima ramnokrak triagolnik,
~ija osnova e a = 5 cm i krak b = 6,5 cm. Presmetaj ja plo{tinata na taa piramida.
14. Visinata na edna pravilna triagolna piramida e ednakva na polovinata od nejziniot
osnoven rab. Presmetaj ja plo{tinata na piramidata, ako a = 6 cm.
15. Pravilna ~etiriagolna piramida so osnoven rab a = 10 cm za dijagonalen presek ima
ramnostran triagolnik. Presmetaj gi plo{tinata i volumenot na taa piramida.
16. Radiusot na opi{anata kru`nica okolu osnovata na pravilna triagolna piramida e
R = 3 cm, a visinata na piramidata e H = 4 cm. Presmetaj gi plo{tinata i volumenot na
piramidata.
17. Kolku litri voda istekuva vo 1 sekunda od edna vodovodna cevka so dijametar na
otvorot 7 cm, ako brzinata na te~eweto na vodata e 3 m/s ?
18. Presmetaj ja plo{tinata na eden cilindar, ako plo{tinata na osnovata iznesuva
50, 24 cm2, a plo{tinata na oskiniot presek e 88 cm2.
19. Dijagonalata na oskiniot presek na eden cilindar, {to e visok 8 cm, ednakva e na
10 cm. Presmetaj gi plo{tinata i volumenot na cilindarot.
20. Rezervoarot za benzin na eden avtomobil ima forma na cilindar so vnatre{en

dijametar 0,3 m i visina 0,6 m. Rezervoarot e napolnet od negovata visina. So toa

koli~estvo na benzin dali mo`e avtomobilot da izmine 180 km pat, ako na 100 km
tro{i 14 l benzin. Ako mo`e, kolku litri benzin }e mu ostanat?

200
21. Plo{tinata na bo~nata povr{ina na eden prav konus iznesuva M = 36p cm2, a perimetarot
na osnovata mu e 9p cm. Presmetaj gi plo{tinata i volumenot na toj konus.
22. Romb so dijagonali d1 = 6, 6 cm i d2 = 4,2 cm se vrti okolu dijagonalata d1. Presmetaj gi
plo{tinata i volumenot na dobienoto telo.
23. Plo{tinata i volumenot na ramnostran konus izrazi gi kako funkcii od negovata
visina.
24. Ramnokrak trapez so osnovi a = 13 cm i b = 7 cm se vrti okolu: a) osnovata a,
b) osnovata b. Presmetaj gi plo{tinata i volumenot na dobienite tela.
25. Eden ramnokrak pravoagolen triagolnik so plo{tina 18 cm2 se vrti a) okolu
hipotenuzata, b) okolu edna kateta. Presmetaj gi plo{tinite i volumenite na
dobienite rotacioni tela.
26. Kolku grama zlato e potrebno za pozlatuvawe na 200 ednakvi top~iwa so dijametar 2 cm,
ako gi pokrivame so sloj debel 0,2 mm. Specifi~nata te`ina na zlatoto e s = 9,4.
27. Ako olovna topka so radius 3 cm ja stavime vo cilindri~en sad so voda, ~ij radius na
osnovata e 4 cm, za kolku }e se izdigne nivoto na vodata vo sadot?
28. Olovna topka so radius 6 cm treba da se pretopi vo top~enca so radius 1 cm. Kolku
takvi top~enca }e se dobijat?
29. Od bakarna topka so radius 10 cm istegnata e `ica so dijametar 2 mm. Najdi ja
dol`inata na `icata.
30. Od drvena kocka so rab a = 8 cm izdelkana e mo`no najgolema topka. Presmetaj kolku
procenti od materijalot odi vo otpadoci.

ZA SAMOKONTROLA - IV

1. Kolku pravi mo`at da opredeluvaat ~etiri to~ki vo prostorot koi ne le`at vo ista
ramnina?
2. Za to~kite A, B i C vo prostorot neka va`i $% FP , $& FP i %& FP . Neka
A', B' i C' se ortogonalni proekcii na to~kite A, B i C vrz ramninata S. [to mo`e da
se ka`e za to~kite A', B' i C'?
3. Plo{tinata na edna pravilna ~etiriagolna prizma, so osnoven rab a = 2,5 dm, iznesuva
92,5 dm2. Opredeli ja visinata na prizmata.
4. Presmetaj go volumenot na pravilna {estagolna prizma so osnoven rab 3 cm, ako
plo{tinata na pogolemiot dijagonalen presek iznesuva 72 cm2.
5. Plo{tinata na bo~nata povr{ina na edna pravilna {estagolna piramida iznesuva
M = 168 cm 2. Presmetaj ja visinata na piramidata, ako osnovniot rab — e a = 6 cm.

201
6. Bo~nite rabovi na edna pravilna triagolna piramida se zaemno normalni i sekoj od
niv e dolg po 12 cm. Presmetaj gi volumenot i plo{tinata na piramidata.
7. Opredeli ja visinata na cilindar, ako radiusot na osnovata mu e R = 1,5 dm, a plo{tinata
P = 61,23 dm2.
8. Pravoagolnik so strani a i b se vrti edna{ okolu stranata a, a potoa okolu stranata
b. Odredi go odnosnot od volumenite na dobienite dve rotacioni tela.
9. Visinata na konusot e 12 cm, a agolot pri vrvot na negoviot oskin presek e 120o.
Opredeli ja plo{tinata na konusot.
10. Volumenot na eden konus e V = 27p cm3, a radiusot mu e R = 3 cm. Opredeli ja visinata na
toj konus.
11. Presmetaj ja plo{tinata na povr{inata na Mese~inata, koga znae{ deka nejziniot
radius e 1740 km.
12. Kako se odnesuvaat volumenite na dve topki, ~ii radiusi se R1 i R2?

202
TEMA I. SLI^NOST NA TRIAGOLNICI

I. 1 str. 6

2. a) 2, b) 1, v) 8, g) 4. 3.. 4. a) 3: 2, b) 3, v) 25 : 3. 5. a) x = 15, y = 5, b) x = 3, y = 6.

6. Upatstvo. a) x = 2a, y = a i z = 4a. Odg. x = 6, y = 3, z = 12. 7. a) x = 5, b) x = 7, 5.
8. x = 6.

I. 2 str. 8

1. , ne. 2. 7, 5 cm. 3. Ne. 4. a) $%  &' , b) $% &' , v) $%  &' . 5. a) ne, b) ne.

      
6.   . 7. a) , b) , v) , g) 2, d) . 8. 1 : 2. 9. $%  01 56  34 56  FP .
      
10. a) da (15 : 8 = 7, 5 : 4), b) ne, v) da (8 : 4 = 12 : 6). 11. $%  FP 01  FP .

I. 3 str. 10
3. Upatstvo. Prvo podeli ja otse~kata na 5 ednakvi dela. 4. Upatstvo. Otse~kata
MN podeli ja na 4 ednakvi dela, a potoa na 5 dela. 6. Upatstvo. Otse~kata PQ
prvo podeli ja na 9 ednakvi dela. 7. Upatstvo. Otse~kata AB razdeli ja na
3 ( = 5 - 2) ednakvi dela, a potoa prodol`i ja na 2 takvi dela na kaj to~kata B.
9. Upatstvo. Otse~kata so dol`ina L razdeli ja na polovina i edniot del razdeli
go vo odnos 5 : 3. 10. Upatstvo. Otse~kata AB razdeli ja vo odnos 3 : 2.

I. 4 str. 13
 
1. a) FP , b) FP . 2. a) Da, b) ne. 3. a) D F , E G , b) D & E D & G  E & F F &G .
 
5. a) 2&  2' $&  %' , b) $&  &' , v) 2%  2' .

203
I. 5 str. 15

1. $ % FP %&  FP . 2. 2 : 3. 3. 1, 6 dm. 5. %' FP ')   FP )/   FP .


6. '&  FP $'  FP . 7. Upatstvo. Koristi deka $0  $0 1&  10
i $'  $1 &0  01 . 11. Baranata otse~ka }e bide ~etvrta geometriska
proporcionala vo proporcijata: a) a : c = x : b, b) a : b = x : c, v) a : b = c : x.

I. 6 str. 18

2. a) da, b) da, v) ne, g) da. $ %  $% . 4. . 6. Samo za k = 1. 8. a) se
3. N

   
zgolemuvaat (k pati), b) se namaluvaat pati. 9. a) N , b) N , v) N .
N   
10. Site tvrdewa se to~ni. 11. H | 107 km.

I. 7 str. 20
1. Da. 2. Da. 3. Sli~ni se vosoglasnost so vtoriot priznak za sli~nost. 5. Da.
6. Sli~ni se vo soglasnost so tretiot priznak za sli~nost. 8. a) da, b) da, v) ne.
9. 15 cm i 18 cm. 10. Tri para: DMNC ~DABC, DMEC ~DADC i DENC ~ D DBC, b)
DABC ~ DEDC, DAMC ~ DENC i DMBC ~ DNDC. 11.DADC ~ DBAC. 12. DABC ~
DMNC ~ DKBL ~ DSNL. 13. L = 12 cm, L1 = 36 cm, L1: L = 3.

I. 8 str. 23
1. 7, 5 cm, b1 = 12, 5 cm i c1 = 15 cm. 2. 9 cm, 13, 5 cm i 13 , 5 cm. 3. L1 = 10 cm,

L = 25 cm. 4. 6,2 cm, 9,3 cm i 12, 4cm. 5. 6 cm, 10 cm, 12 cm. 6. 2:3. 7. Upatstvo.
F KK KF
Koristi deka 5 . 8. hb : ha = a : b bidej}i aha = bhb, KD  D E KD ˜ KD  KF EF,
 KF KD KE
KD KE
bidej}i bhb = chc. Zna~i, KE  KD  DEF .
KF

I. 9 str. 25
1. 6, 37 cm. 2. 49 : 9. 3. Ako koeficientot na sli~nost e k, toga{ e s1 = ks,
P1 = k2P pa za radiusot na vpi{anata kru`nica na sli~niot triagolnik dobivame
3 N3 3
U N˜ NU . 4. Ako k e koeficientot na sli~nost, toga{ a1 = ka,
V NV V
b1 = kb, c1 = kc i P1 = k2P, pa za radiusot na opi{anata kru`nica na sli~niot
DEF ND ˜ NE ˜ NF NDEF
triagolnik dobivame: 5 N5 . 5. Ako k e koeficient na
 3 N  3 3
sli~nost, toga{ a1 = ka, b1 = kb, c1 = kc, s1 = ks, pa za plo{tinata na sli~niot
triagolnik dobivame:

204
3 V V  D V  E V  F NV NV  ND NV  NE NV  NF N  V V  D V  E V  F

N  V V  D V  E V  F N 3 .

I. 10 str. 27

1. 30 m. 2. 3 m. 3. 22, 5 km, a razmerot e 1 : 250 000. 4. %'  FP 2'  FP .

5.   cm. Upatstvo. Koristi go tretiot priznak za sli~nost. Koeficientot na sli~nost



e 1 : 2. 7. Neka tetivite se AB i CD a prese~nata to~ka e O. Toga{ )$&' )$%' kako
periferni agli nad lakot p $' i )&$% )&'% kako periferni agli nad lakot %& p.
Zatoa DAOC ~ DDOB, pa ottuka 2$  2' 2&  2% , odnosno 2$ ˜ 2% 2& ˜ 2' .
8. Upatstvo. Prodol`i ja visinata CD na triagolnikot ABC preku to~kata D do to~kata
D1 pri {to &' FP , a potoa niz D1 povle~i prava p & AB. 11. DSBC ~ DSDA bidej}i
)6%& )6'$ kako periferni agli nad lakot p $& , a osven toa agolot kaj temeto S im e
zaedni~ki. Od sli~nosta sleduva deka 6%  6' 6&  6$ , odnosno 6$ ˜ 6% 6& ˜ 6' . 12. Tri
para: DABC ~ DACD, DABC ~ DCBD i DACD ~ DCBD.

I. 11 str. 29
  
1. b = 12 cm, DF FP EF FP K FP . 2. a = 8 cm, c = 10 cm, ac = 6, 4 cm, h = 4, 8 cm.
  
3. a | 16, 2 cm, b | 10, 8 cm, c = 19, 5 cm, ac = 13, 5 cm. 4. a = 12 cm, E  FP , c = 24 cm, K  FP .
5. a | 6, 4 cm, b | 8 cm. 6. 12 cm. 7. Neka ABC e pravoagolen triagolnik so prav agol
vo temeto C i $% F %& D &$ E &' K e visinata spu{tena od temeto C.
DE
a) Od DBCD ~ DBAC sleduva h : b = a : c, K  . b) Koristej}i deka a 2 = a c · c i
F
D E
b = bc · c dobivame
2 F . 10. Upatstvo. Ako a i b se stranite na pravoagolnikot,
DF EF
toga{ [ DE e stranata na kvadratot. 11. Upatstvo. x e geometriska sredina na
D
otse~kite: a) 2a i b, b) i b, v) a i b + c. 12. a) 13 cm, b) 5 cm.


I. 12 str. 32

4. a) da, b) ne, v) ne, g) da. 5. a) 13 cm, b) 11,4 cm, v) 17 cm, g) 28,9 m. 6. a)   cm,
b) 10 m, v) 5,6 cm, g) 8,47 m. 7. a) L | 156,83 cm, P | 1052 cm2, b) 67,6 cm, P | 196 cm2,
v) L | 41 m, P | 72,2 m2. 8. 36 cm. 9. L | 52,81 cm, P = 104 cm2. 10. 5,12 m.

11. 12 cm. 12. (3 +  )b | 4,73b.

205
I. 13 str. 35

1. Upatstvo. Najprvo konstruiraj kvadrat ~ija plo{tina e ednakva na zbirot od


plo{tinite na prvite dva kvadrata. 5. a) [    , b) [    , v) [    .
   
§ F· §F· § F· §F·
6. Upatstvo. a) [ ¨ D  ¸  ¨ ¸ , b) [ ¨D  ¸ ¨ ¸ .
© ¹ © ¹ © ¹ © ¹

I. 14 str. 41

1. a) 11, 4 cm, b) 10, 5 cm. 2. 5 cm. 3. 2, 1 cm. 4. 4, 2 cm. 5. r | 6,5 cm, R | 13 cm.
6. 3, 5 cm. 7. 14, 6 cm. 8. %)  FP , &)   FP , &' = 13 cm. 9. 6 cm. 10. 3 cm.
11. a) 7 cm, b) 19 cm. 12. $% U  .

Zada~i za povtoruvawe i utvrduvawe - I - str. 42

1. 4, 2 cm. 2. 0%  FP $%   FP . 3. $7  $% P  P  Q , 7%  $% Q  P  Q .


D F D F DE FG
4. a) 10 :7, b) 3 : 10, v) 10 : 3. 5. Ÿ   Ÿ . 6. 3, 6 cm.
E G E G E G
  
7. a) 4,5 cm, b) 6 cm. 8. 6 cm. 9. 4, 5 cm. 11. N  N ,N  , N . 12. Upatstvo.
   
DABD ~ DCAB bidej}i i dvata triagolnika imaat agli 36 o , 72 o , 72 o .

13. 10 cm, 18cm. 14. 4, 8 cm. 15. Pomalata osnova e dolga  cm, a dijagonalata 22

cm. 16. 17 cm. 17. Upatstvo. Odredi gi prvo rastojanijata d1 i d2 na sekoja tetiva
od centarot. Odgovor. a) 1, 4 cm, b) 4, 6 cm. 18. 5 cm i   cm. 19. 11, 3 m i 10, 4 m.
20. 8 km. 21. h = 1, 5 m, d = 3, 9 m. 22. D U  D U  i a6 = r, pa a26 + a26 = 2r2 + r2 =
3r2 = a23. 23. Upatstvo. Baranata otse~ka x cm konstruiraj ja vrz osnova na
proporcijata: a) b : a = a : x, b) a : 1 = x : b, v) a :b = x : 1.

Zada~i za samokontrola - I - str. 43

1. a) da, b) da. 2. Da. 3. 4,5 cm, 7,5 cm i 9 cm. 6. Upatstvo. Neka DABC~ DA'B'C'.
a) Ako AA1 i A'A'1 se bisektrisi, koristej}i go prviot priznak za sli~nost prvo
doka`i deka DABA1 ~ DA'B'A'1. b) Ako AA1 i A'A'1 se te`i{ni linii, koristej}i
go vtoriot priznak za sli~nost prvo doka`i deka DABA1 ~ DA'B'A'1. 7. Ako a i b
DK D
se osnovata i visinata na triagolnikot, toga{ [ ˜ K e stranata na
 
baraniot kvadrat. 8. 3,7 cm. 9. 112,5 km. 10. 10,06 cm. 11. a) 13, b) 10, v) 5.
12. /   cm, r = 6 cm, R = 12 cm.

206
TEMA II. LINEARNA RAVENKA I LINEARNA NERAVENKA.
LINEARNA FUNKCIJA.

II. 1 - str. 46
3. a) Za a = 3 dobivame nevistinit iskaz 9 + 1 = 6, b) za x = 1 imame 4 = 11,
v) za x = –2 toa preminuva vo nevistinit iskaz ç–2 ç= –2, odnosno 2 = –2. 4. Za x
= 0 imame 0 = 0; za x = 3 imame 9 = 9, no za x = 2 ravenstvoto x2 = 3x preminuva vo
nevistinit iskaz 4 = 6. 5. Identiteti se b), v) i d). 6. Tie se identiteti.

II. 3 - str. 48
1. Re{enija se –2 i 1. 2. a) 4, b) 3, v) 1 i 4, g) 0, – 1 i 1. 3. Na primer: a) x = – 2, b)(x
+ 3)(x – 1) = 0, v) x(x – 1)(x – 4) = 0, g) x + 1 = x, 4. a) 3x – 5 = 0, b) 3x – 5 = 4, v) 3x – 5
= –2. 5. Za a) 2 i 5, b) 4, v) 3, g) 1, 2 i 3, d) 1, 2, 3, 4, 5, 6 i 7, |) 3 i 4. 6. a), b), v), g) Da.

II. 4 - str. 50
2. x = 3 e re{enie na vtorata ravenka, no ne i na prvata ravenka. 3. a) da, b), v) ne
se ekvivalentni.
II. 5 - str. 53
1. Da. 2. Da. 3. Da. 4. Ne. Na primer, ako go pomno`ime so 0, dobivame to~no
ravenstvo. 5. Se dobiva: a), b), to~no brojno ravenstvo. 6. Se dobiva ekvivalentna
ravenka. 7. Mo`at. 8. a) x2 - 3x + 2x = 7 – 2, b) x2 – 3x + 2 –7 + 2x = 0. 10. a) 18x –
2 = 5 – 6x, b) 2 + 2x = 4x –1, v) 3(2x – 3) – 2(x – 1) = 12x + 12, g) 2(x – 1) – 3(x – 2) = x +
3. 11. a) Ne se, b) se ekvivalentni, v) se ekvivalentni. 12. a) 11, b) 4, v) 15, g) 3,
d) –1, |) 4.
II. 6. - str. 55
3. a) –5h+6=0, a=–5, b=6, b) 6x+4=0, a=6, b=4.

II. 7 - str. 57

1. a) 16, b) –1, 2. a)   , b)   . 3. a) 7, b) . 4. a) 12, b) 4. 5. a) 9, b)   , 6. a) 2,
   

b) – 4. 7. a) 5, b) . 8. a) –6, b) 4.

II. 8 - str. 61

1. 56. 2. 125. 3. 125. 4. 13. 5. 12, 13, 14. 6. 36. 7. 18o i 72o. 8. a = 8 cm. 9. 21 cm i 29 cm.
10. 5 cm. 11. a = 5 cm. 12. a = 7 cm, b = 8 cm, c = 9 cm.

II. 9 - str. 64

1. a), v) nevistinit, b, g) vistinit. 2. a) nevistinit. b), v), g) vistinit.


3. a) Zadovoleno e za xÎ{–2, –1,0, 1,2}, b) za xÎ{ –1, 0, 1}. 4. a) a – b < 0, b) a – b > 0,
v) a – b = 0.

207
II. 10 - str. 66

1. a), b), v), g) Vistinito. 2. a), b) vistiniti se za sekoj pozitiven realen broj, a
nevistiniti se za sekoj nepozitiven realen broj. v) vistinito za sekoj realen broj, g)
vistinito za sekoj realen broj, osven za x = 0. 3. a) –x < 6, b) –2 > 7, v) 2 – x < 8.
4. Ekvivalentno e na neravenstvata –5 < x i x > 9. 5. Upatstvo. Dvete strani na prvoto
neravenstvo podeli gi so brojot 2. Odgovor. Od 1 < 3 i 3 < 5 sleduva dvojnoto neravenstvo
1 < 3 < 5.

II. 11 - str. 68

1. Po toa {to na brojnata oska se izbrani dve fiksni to~ki {to im odgovaraat na
broevite 0 i 1. 2. a) {to se nadesno od po~etokot 0, b) nalevo od po~etokot 0. 4. (x, y) È
[x, y] = [x, y], (x, y) Ç [x, y] = (x, y). 5. a) (3, 5), b) (1, 7), v) (1, 3). 6. a), b) Za sekoj x Î { -3, -2, -1,
1, 2, 3}, v) za x Î {0, 1, 2, 3}.

II. 13 - str. 72

1. Ekvivalentni se: a), b), v), g). 2. a) x – 2 – 2x < x + 1, b) 3x – 4 > x – 5. 3. a) 6x – 3 > 2x + 5 œ


[  [
6x – 2x >5 + 3 œ 4x > 8 œ x > 2, b)    œ   [    [   œ  [   [     œ [   .
 
II. 14 - str. 75

1. a) 0 §  · , b) M = ( –1, f ). 2. a) M = (–9, f ), b) § ·
¨ f¸ 0 ¨ f¸ . 3. a) M = (– f , 5),
© ¹ © ¹
§  · §  · §  ·
b) 0 ¨   f ¸ . 4. a) M = Æ, b) M = Æ. 5. a) 0 ¨   f ¸ , b) 0 ¨  f¸ .
©  ¹ ©  ¹ ©  ¹
II. 15 - str. 76

1. a) Za x Î (– f , 2), 2. a) k Î (2, f ), b) k = 2, v) k Î (– f , 2). 3. Za a) m Î (–2, f ), b) m Î (–1, f ),


  D  
b) Î (– f ,4). 4. a) Pri m t –2, [ ! ; a pri m < –2, [  , b) [  , x Î R.
P P 
N   N   F   F  
5. Pri k t – 3, [  , a pri k < –3, [ ! , b) pri c d 2, [ ! , a pri c > 2, [  .
N  N  F F

II. 17 - str. 79

1. a) 0 §  · , b) M = Æ. 2. a) M = (–2, f ), b) M = Æ. 3. a) M = (4, f ), b) §  ·
¨  f¸ 0 ¨ ¸ .
©  ¹ ©  ¹
§  · 
4. Za a) P  §   f · , b) P  ¨ f ¸ , v) P . 5. Upatstvo. Izrazite za a, b i c treba da
¨ ¸ ©  ¹ 
© ¹
go zadovoluvaat dvojnoto neravenstvo a – c < b < a + c, odnosno
 §  ·
(2p + 3) – (p + 13) < 4p – 5)<(2p + 3) + (p + 13). Odgovor. Za   S   , odnosno za S  ¨   ¸ .
 ©  ¹

208
II. 18. - str. 81

1. Linearni funkcii se: a), b), v) i d). 2. f(D) = { –7, –5, –3, –1, 1, 3, 7}. 3. f(x) = x + 3, g(x)= –x.
4. l = 2 + 0, 00012 to. Linearna funkcija. P = 3x. Linearna funkcija. 6. y = 24, 5 · x. Linearna

funkcija. 7. y = 2n + 2, n d 60. 8. \ ˜ [ . 9. y = 8,9x. 10. S = 250 t. Linearna funkcija.

II. 19. - str. 84

2. a) I i III, b) II i IV kvadrant. 3. y = 12x, b) 2 700 l. 4. y = 0, 6 x, D = N. 5. S = 4 t.



6. \ ˜ [   [ . 7. y = 1, 118x.

II. 21. - str. 87
1. Graficite se dve paralelni pravi. 2. S(3, 2). 3. 1o. a) Za x = 4, b) za x Î (4, f ), v) za x Î
(– f ,4). 2o. y raste koga x raste. 3o. na intervalot [0, 6]. 4. Na grafikot mu pripa|aat
to~kite A(0, 3) i C(2, –1). 5. To~kata: a) (0, 4), b) (2, 0). 6. Dobiva vrednost nula za
x = 2, a) x Î(– f ,6), b) x Î (6, f ).
II. 22. - str. 89
 
1. a)  , b) , v) 0. 2. a) raste~ka, b) opa|a~ka, v) konstantna, g) opa|a~ka, d) raste~ka.
 

II. 23. - str. 91


  
1. [  , b) x = –15, v) x = 1. 2. a) x = 6, b) [  , v) x = 3. 3. a) x = 1, b) x = –3. 4. x = 3, b) [ .
  
5. a) nema re{enie, b) dvata grafika se sovpa|aat, t.e. ravenkata e identitet. 6. a) nema
re{enie, b) ravenkata e identitet, t. e. sekoj realen broj e nejzino re{enie.

Zada~i za povtoruvawe i utvrduvawe - II - str. 91



1. a), b) e re{enie. 2. a) x = -1, b) nema re{enie, v) [ . 3. a) tie se ekvivalentni,

b) ne se ekvivalentni. 4. 5 i 6. 5. Zadovoleno e za x = 3, x = 4 i x = 5. 6. a) [0, 3), b) (–3, 8],
v) [0, 5), g) (– f , 8]. 7. a) M = (– f , 6), b) M = (– f , 2). 8. a) –2x + 1 < 5 – x, b) –4x > 12, v) 2x < 2.
§ ·
9. a) 0 ¨ f ¸ , b) M = (1, f ), v) M = Æ. 10. a) {(a, 1), (b, 1)}, {(a, 2), (b, 2)}, {(a, 3), (b, 3)}, {(a, 1),
© ¹

(b, 2)}, {(a, 1), (b, 3)}, {(a, 2), (b, 3)}, b) {(a, 1), (b, 2)}, {(a, 1), (b, 3)} i {(a, 2), (b, 3)}. 11. a) Za [ , b) za

§ · § ·
x = 2. 12. Za [  ¨ f ¸ , b) za [  ¨  f ¸ . 13. Imalo 30 deca i 7 klupi. 14. Dokaz. Poznato
© ¹ © ¹
e deka va`i neravenstvoto a + b > c. Kon dvete negovi strani dodaj c, pa }e dobie{ a + b + c
DEF
> 2c, odnosno ! F , {td. 15. a) x = 4, b) x = 1, v) x < 4, g) x > 4. 16. y Î [–1, 4]. 17.

Vrednosta na y }e se zgolemi: a) od –3 do 3, b) od –13 do –5. 18. a) y = 4, 5x + 7. 19. y = 77Fo,

209

[  ˜ | q& . 20. y = 420 – 8t. Posle 34 minuti vo kadata }e ostanat 148 litri voda.

Kadata }e se isprazni za 52,5 minuti.

Zada~i za samokontrola - II - str. 93



1. Dve re{enija x = 0 i x = 3. 2. a) x = 12, b) [
, v) y = 2. 3. a), b) ekvivalentna ravenka.

  
4. Upatstvo. Ako se otse~eni , toga{ ostanalo u{te od topot. Zna~i ˜ [  .
  
Odgovor. 40 m platno. 5. ]erkata ima 12 godini, a tatkoto 48 godini. 6. ]e dobie{: a)
to~no, b) neto~no ravenstvo. 7. a) A È B = (–2, f ), b) A ÇB = [0, 3]. 8. a) M = (2, f ), b)
§  ·  
0 ¨  f¸ . 9. a) M = (5, f ), b) M = (– f , 3), v) M = Æ, g) M = (–2, 0). 10. N E .
©  ¹  
11. Vo to~kata: a) A(4; 0), b) B(0; -3). 12. Za a = 2.

TEMA III. SISTEM LINEARNI RAVENKI

III. 1. - str. 96
2. a) (0,4), (1, 4), (2, 4), (3, 4), b) (3, 0 ), (3, 1), (3, 2), (3, 4). 3. Ravenkite se ekvivalentni.
     
4. a) \  ˜ [  , b) y = 3x – 1, v) \  [ . 5. a) [ \   , b) [ \  , v) x = 2y.
     

III. 2. - str. 99

§ · §  · § · §  ·
3. a) ¨   ¸ , b) ¨   ¸ , v) ¨   ¸ , g) ¨    ¸ .
1. a) x – 4y = 18, b) 3y – 4x = 13,
© ¹ ©  ¹ © ¹ ©  ¹
7. y =1. 8. 4. 9. c = –7. 10. Graficite se paralelni.

III. 3. - str. 101

2. I [  \  i J [  \  . 3. Re{enie e samo parot (0,–2). 4. (0, 0) e baranoto re{enie.


5. (2; 1).
III. 4. - str. 104

§  ·
1. a) (3, 1), b) ¨  ¸ , v) (–3, –1). 2. a) ima edno re{enie, b) nema re{enie, v) ima
©  ¹
beskone~no mnogu re{enija. 3. Minuva. 4. a), b) ima re{enie. 6. a = 2, b = 2. 8. Ne.

210
III. 5. - str. 105

1. a) Da, b) da. 4. Sistemite ne se ekvivalentni, bidej}i (0, 0) e re{enie na edniot


§ · §  ·
sistem, a ne e re{enie na drugiot sistem. 5. ¨  ¸ . 6. ¨   ¸ .
© 
¹ ©  ¹
III. 6. - str. 108
§ ·
1. a) (8, 2), b) (3, 3), v) (–3, –5), g) (–2, –1). 2. a) ¨  ¸ , b) (1, 3). 3. a) (2, 1),
© ¹
b) (5, –1). 4. a) (–1, 2), b) nema re{enie. 5. a) (6, 2), b) (3, 2). 6. a) (12, 20), b) (16, 15).
§   ·
7. a) ¨  ¸ , b) (2, 3).
©   ¹
III. 7. - str. 110
§ · §  ·
1. ¨ ¸ , b) ¨   ¸ . 2. a) (10, 7), b) (1, 2). 3. a) (–2, 1), b) (3, 2). 4. a) (4, 1), b) (2, 1).
© ¹ ©   ¹
5. a) (2, 1), b) (7, 8). 6. a) (3, 4), b) (7, 2).
III. 8. - str. 113

1. 76 i 54. 2. 45 i 28. 3. 13 i 65. 4. 7 cm i 3, 5 cm. 5. 36o, 36o i 108o. 6. 15o, 75o, 90o. 7. 21, 5 cm
i 17, 5 cm. 8. 18 cm i 9 cm. 9. Slivi imalo 60 drvja, kru{i imalo 30 drvja, a jabolka imalo
90 drvja. 10. I – 248, II – 124. III – 88 rabotnici. 11. Mali tetratki imalo 11, a golemi 6.
12. 5 - denarki imalo 16, a 2 - denarki 10. 13. 8 toni i 9 toni.

III. 9. - str. 115


1. Ako vo sekoj od xebovite ima pomalku od 51 denar, zna~i najmnogu 50, toga{ vo dvata
xeba ima vkupno ne pove}e od 2.50=100 denari, {to protivre~i na uslovot na zada~ata.
2. Ako vo hotelot ima najmnogu 33 sobi, toga{ vkupno mo`at da se smestat najmnogu
33. 3=99 lu|e, {to protivre~i na uslovot na zada~ata. 3. Upatstvo. 2 . 365<1000. 4. Mite
imal 7 neuspe{ni obidi so 3 klu~a nesoodvetni za taa brava. Bidej}i 7=3 . 2+1, zatoa
postoi klu~ so koj Mite 3 pati se obiduval da otklu~i.

III.10. - str. 117


2. Ne e dovolen. Brojot 4 e mal. 3. 60. 4. | 60000.
III. 11. - str. 119
4. Slu~aen nastan.
III. 12. - str. 120
1. Primerokot e dobro izbran, bidej}i krvnata grupa kaj ~ovek ne se menuva do krajot na
`ivotot. Potoa pretpostavuvame deka toj faktor ne se menuva od edno do drugo naseleno
mesto. Vo R. Makedonija o~ekuvame da ima okolu 260 000 `iteli so negativna krvna
grupa. 2. 500 000.

211
III. 13. - str. 122
4. a) ne, b), v), g) da, d), |) ne. 5. Verojatnosta na sprotivniot nastan e 1 – p. 6. 0, 33. 7. 0, 25.

III. 14. - str. 124


  
3. a) 9  , b) 9  , v) 9  . 4. Ima samo eden povolen nastan,
  

t.e. m=1, a brojot na site nastani e 6. Odg. a) 9 . 5. Zaboravenata cifra e


xÎ{0,1,2,3,4,5,6,7,8,9}. Odg. 9  . 6. Mno‘estvoto na site elementarni nastani za

mo‘nite zbirovi i+j (1£ i, j £ 6) e 6 × 6=36, a od niv povolni nastani se samo 6: 1+6, 2+5, 3+4,
 
4+3, 5+2, 6+1. Odg. 9 . 7. 0,25.
 

Zada~i za povtoruvawe i utvrduvawe - III - str. 124


1. Re{enija se, na primer, (1, 3) i (3, 7), a ne se re{enija (0, 0), (1, 2). 2. Takov e sistemot
2 x + y = 3, x + y = 1. 3. Da. 4. (2, –1). 5. Ne e dobro, bidej}i pravite se se~at pod mal agol.
6. Vo slu~aj koga postoi koeficient pred x ili y ednakov na 1 ili –1. 7. Sistemot nema
§ ·
re{enie. 8. Re{enie e sekoj par broevi (x, 2x + 1). 9. (1, 1). 11. a = 0. 12. a = –3, b = 5. 13. ¨  ¸ .
© ¹
14. 6 100 kg i 3750 kg. 15. 30 g i 15 g. 16. 5 – denarki imalo 8, a 2 - denarki imalo 5. 17. Ma`i
bile 9, a `eni 6. 18. 367 i 433 u~enici. 19. Brzinata na avionot e 173 km /~as, a na veterot
15 km / ~as. 20. Tatkoto ima 41 godina, a sinot 9 godini. 21. Sinot ima 9 godini, a
tatkoto 36 godini. 22. 20 l i 16 l. 23. Od 30% kiselina treba da se zemat 10 l, a od 55%
kiselina da se zemat 40 l. 24. Normata na majstorot bila 80 detali, a na u~enikot 50
detali. 25. Sprotivniot nastan e "Pri frlawe na zar da se padne eden ili dva", a negovata
 
verojatnost e . 26. Verojatnosta e . Sprotiven nastan e da se padne paren broj, a
 
    
negovata verojatnost isto taka e . 27. a) . b) . 28. a) b) .
    

Zada~i za samokontrola - III - str. 126

1. a) (1, 7), (2, 6), (3, 5), (4, 4), (5, 3), (6, 2) i (7, 1), b) (1, 12), (2, 6), (3, 4), (4, 3), (6, 2), (12,
1). 2. a) Za c = 9, b) za c = 20. 3. a) Za c = 9, b) za c = 6. 4. Upatstvo. Re{i go sistemot
­D  E 
ravenki ®D  E  . Odgovor. za a = 1, b = 3. 5. Vo to~kata a) A(4, 0), b) B(0,–3).
¯
­  [   \ 
6. Upatstvo. Re{i go sistemot ravenki ® . Odgovor. 14 zajci i 22 fazani. 7.
¯ [  \ 
­ [  \ 
°
Na visina 4 stopi. 8. 39 i 260. 9. Upatstvo. Re{i go sistemot ravenki ® [  \ .
°̄ 

Odgovor. Sega e 4 ~asot. 11. . 12. 167.


212
TEMA IV - GEOMETRISKI TELA

IV. 1 - str. 132


1. Sleduva od A1. 2. Sleduva od A2. 3. Sleduva od A3. 4. Niz dve to~ki od prostorot
minuvaat bezbroj ramnini. 5. Sleduva od A5. 10. a) bezbroj, b) edna. 11. 3. 12. 6. 14.
Se se~at ili se razminuvaat. 15. Ne. 16. Ne. a) trite to~ki da ne se kolinearni, b)
pravite treba da se se~at ili da se paralelni, v) to~kite da ne le`at na pravata. 18. Ne.
19. a) da se paralelni ili da se se~at, b) da se paralelni, ili da se se~at, ili da se
razminuvaat. 20. Edna.
IV. 2 - str. 136
1. p & AB. 3. Edna ramnina. 4. a' & b'. 5. a) ne, b) ne. 6. ^etiriagolnik ili otse~ka, a
triagolnik ne mo`e da bide. 7. A', B' i C' se kolinearni i pri toa B' le`i pome|u A' i C'.
8. Ravenstvoto va`i ako AB & S. 10. Ne sekoga{.

IV. 3. - str. 138


2. 0o. 4. Najmal e agolot {to pravata p go zafa}a so nejzinata ortogonalna proekcija vrz
ramninata S. 5. Ravenstvoto va`i. 6. Potreben i dovolen uslov e ramninata na
triagolnikot ABC da e normalna na ramninata S. 8. s Í S2. 9. a) 6cm, b) 8, 46 cm, v) 10, 38 cm.
10. 45o. 11. %$c  FP , a rastojanieto od otse~kata AB do ramninata S e   |  FP .

IV. 4 - str. 141


1. a) 6, b) 12, v) 8. 5. a) ne, b) da, v) da. 7.

IV. 5 - str. 143

5. a) osnovni rabovi ima 6, a bo~ni 3, b) osnovni rabovi ima 8, a bo~ni 4, v) osnovni


rabovi ima 10, a bo~ni 5.
IV. 6 - str. 144
1. Bo~nite rabovi se paralelogrami, a me|u niv mo`e da ima i pravoagolnik. 2. Prava
prizma. 3. Triagolnite prizmi nemaat dijagonali. 5. Pravoagolnici. 6. a) 0, b) 1, v) 2,
g) n – 3. 7. a) nema dijagonali i dijagonalni preseci, b) dijagonali ima 10, a dijagonalni
preseci 5, v) dijagonali ima 18, a dijagonalni preseci ima 9.

IV. 7 - str. 147


2. Ramnostran triagolnik ili {estagolnik. 3. a) da, b), v) da, g), d) da. 4. a) 13 cm,
b) 7 cm. 5. 31,18 cm. 6. Neka dijagonalata e AC1 i go barame rastojanieto od C do AC1.
Stranite na pravoagolniot triagolnik ACC1 se   i  . Za visinata vo pravoagolen
DE 
triagolnik va`i K (bidej}i ch = 2P = ab), pa baranoto rastojanie e K | P .
F 
7. 16 cm.

213
IV. 8 - str. 149
5. Dve. 7. Visinata e 4 cm. 8. Visinata e 8 cm. 9. 52 cm2.

IV. 9 - str. 153



§ G ·
1. a) 143, 8 cm , b) 7, 8 dm . 2. 5 cm. 3. 3  ¨ ¸ =2d2. 4. 8, 66 cm2. 5. 312 cm2. 6. P= 1342 cm2, d = 29,
2 2

© ¹
78 cm. 7. 5, 48 cm. 8. 304, 77 cm2. 9. 33 m2. 10. 312 cm2. 11. 183 cm2, 12. 72 cm2.
IV. 10 - str. 161
1. 216 cm3. 2. 130, 83 cm3. 3. 64 cm3. 4. 6, 93 dm. 5. 200 kutii. 6. 30 cm. 7. Upatstvo. Presmetaj
go volumenot vo litri. Odgovor. 5 ~asa. 8. a) }e se zgolemi 27 pati, b) }e se namali 8
pati. 9. 6 cm. 10. 353 l. 11. 25, 6 m2. 12. 34, 08 kg. 13. 4 m.

IV. 11 - str. 163


2. Mo`e. 3. Mo`e najmnogu dva bo~ni yida da bidat normalni na osnovata. 4. a) Najmaliot
broj na yidovi e 4 i toa e mo`no kaj tetraedarot, b) najmaliot broj na temiwa e 4 i toa e
mo`no kaj tetraedarot, v) najmaliot broj na rabovi e 6 i toa e mo`no kaj tetraedarot.

IV. 12 - str. 166


1. a), b) mo`e, v), g) ne mo`e, d) mo`e, |) ne mo`e. 3. Ne mora. 4. Ne mora. 5. Da.
6.  |  FP . 7. 4,27 cm. 8. 2 cm. 9. Bo~niot rab ima dol`ina  |  FP , apotemata
ima dol`ina 13 cm. 10. 6,53 cm. 11. a) mo`e, b) mo`e, v) mo`e. 12. Ramnostran triagolnik.

IV. 13 - str. 167


2. a) edna oska, b) tri oski.
IV. 14 - str. 171
1. 51,72 cm2. 2. 54,93 cm2 3. 1514,22 cm2. 4. 249 cm2. 5. 5,7 cm. 6. 1620 keramidi. 7. Upatstvo.
Prvo opredeli ja apotemata na piramidata. Odgovor H | 23,7 cm, s = 24,3 cm. 8. 52,6 cm2 9. 8
cm. 10. P = 3a2. Za a = 12 cm se dobiva P = 432 cm2. 11. Upatstvo. Opredeli ja prvo apotemata
na piramidata. Odgovor. s | 31 cm, H = 30 cm. 12. a = 5 cm, H = 6 cm.

IV. 15 - str. 174


1. 95 cm3. 2. 2701 m3. 3. 9 cm. 4. H = 27 cm, P | 500 cm2. 5. 280, 6 cm3. 6. 37, 1 cm3. 7. 50 cm3. 8. 214, 3 cm3.
D 
9. 9 . 10. 0, 12 dm3. 11. 18 cm. 12. a) }e se zgolemi 2 pati, b) }e se zgolemi 9 pati.

IV. 16 - str. 177
1. a) kru`nica, b) kru`nica, v) krug, g) bo~na povr{ina, d) otse~ka. 4. a) R = 4 cm,
s = 7 cm, b) R = 7 cm, s = 4 cm, v) R = 3,5 cm, s = 4 cm, g) R = 2 cm, s = 7 cm. 5. Takva to~ka postoi.
Toa e sredinata na visinata. 6. a) 56 cm 2, b) 18 cm 2. 7. 5 dm. 8. 4 dm2. R = 7, 5 cm,

214
H = 15 cm. 10. 6, 5 cm. 11. 69, 28 cm2. 12. Pravoagolnik.

IV. 18 - str. 180


1. a) 6, 87 dm2, b) 20, 68 dm2. 2. 13, 5 dm2. 3. a) 18, 9 dm2, b) 42, 4 dm2, v) 4, 7 m2. 4. 9, 5 cm. 5. 616 cm2.
6. 5, 8 m2. 7. 7, 54 m2 lim. 8. 289 cm2. 9. 268, 5 cm2. 10. 10, 6 dm2. 11. 988 cm2. 12. p.

IV. 19 - str. 183


1. a) 153 cm3, b) 603 cm3. 2. 1356, 5 cm3, b) 6, 28 dm3. 3. 137 l. 4. 2, 5 m. 5. 8 m. 6. 62, 8 cm3. 7. 6, 37 dm.
8. 15072 l. 9. | 10 km. 10. 33 dm2. 11. | 2, 2 dm3. 12. | 85 g.

IV. 20 - str. 186


1. a) kru`nica, b) krug, v) konusna bo~na povr{ina, g) konus. 2. a) krug, b) ramnokrak
triagolnik i toa e oskiniot presek. 4. R = 3 cm, s = 5 cm. 5. Postoi i taa mo`e da se dobie
kako centar na opi{anata kru`nica okolu koj bilo oskin presek. 6. a), b) ne mo`e. 7. a)
60cm2, b) 30, 72cm2, v) 9, 72 cm2. 8. 62,35 cm2. 10. R = 7,2 cm. 11. a) 15,59 cm2, b) 18 cm2, v) 15, 59 cm2.

IV. 21 - str. 188


4. 108o. 5. 216o. 6. Upatstvo. Generatrisata na konusot ima dol`ina 6 cm. Odgovor. 2, 5 cm.
OU
7. 3, 6 cm. 8. Pravoagolen triagolnik. 9. a) ako R < H, b) ako R = H, v) ako R > H. 10. 3 .


IV. 22 - str. 189


1. a) 148, 5 cm2, b) 7, 03 dm2. 2. a) 176 cm2, b) 20 cm2, v) 174 cm2. 3. Upatstvo. Radiusot na kru`niot
ise~ok pretstavuva generatrisa na konusot. Opredeli go prvo radiusot na konusot.
Odgovor. P | 2, 67 dm2, H | 1, 09 dm. 4. a)115, 5 cm2, b) 151 cm2. 5. 151 cm2. 6. 1206 cm2. 7. 8, 34 m2.
8. a) 1 : 2, b) 1 : 3. 9. M = 314 cm2, P = 471 cm2. 10. 3, 7 dm. 11. 502, 4 cm2. 12. 160 cm2.

IV. 23 - str. 192


1. a) 307, 72 cm3, b) 37, 68 cm3, v) 8 dm3, g) 2,96 cm3. 2. a) 391 cm3, b) 1865 cm3, v) 154 cm3. 3. 26, 17m3.
4. 148,5 dm3. 5. 1, 58 cm. 6. 193, 5 cm3. 7. Upatstvo. Najdi go prvo radiusot na dobieniot
konus. Odgovor. 65, 1 dm3. 8. 37, 68 cm3. 9. P | 198 cm2, V | 218,45 cm3. 10. 9, 85 dm3. 11. P | 142
cm2, V | 67cm3. 12. Upatstvo. Dobienoto telo pretstavuva cilindar na kogo od ednata
strana mu e prilepen konus, a od drugata strana e izvle~en ist takov konus. Odgovor.
P = 80p | 251, 2 cm2, V = 80p | 251,2 cm3.

IV. 24 - str. 195


1. Sfera S(T, 4, 5 cm). 2. Krug, to~ka ili prazno mno`estvo. 3. r = 3 cm, L = 18,84 cm, P = 28,
26 cm2. 4. R = 5cm. 5. L = 44, 4 cm, P | 157 cm2. 6. 28 287 km. 7. 3 cm. 8. 5 cm. 9. Niz dijametralno
sprotivni to~ki. 10. 339,12 cm2; 452,16 cm2; 339,12 cm2.

215
IV. 25 - str. 197

1. a) 153, 86 cm2, b) 12, 56 dm2, v) 2461, 76 cm2. 2. 113, 04 cm2. 3. 113, 04 cm2. 4. 509554140 km2. 5. 4 : 9. 6.
a) 1 : 2, b) 2 : 3, v) 2 : 5. 7. a) 2 pati, b) 3 pati, v) 5 pati, g) 10 pati. 8. a) 7, 81 cm, b) 3, 32 cm.
9. 416, 8 m2. 10. a) 121 pati, b) 484 pati. 11. a) 254, 34 cm2, b) 763, 02 cm2. 12. Ako radiusot na
osnovata e R, toga{ dvete tela imaat plo{tina 3pR2. Zabele{ka. Za plo{tina na polutopka
zemi ja predvid i plo{tinata na golemiot krug.

IV. 26 - str. 198

1. a) 65, 42 cm3, b) 268 cm3, v) 7234,56 cm3. 2. a) P = 254, 34 cm2, V = 381, 5 cm3, b) P = 21, 23 dm2, V = 9,
2 dm3, v) P = 28, 26 dm2, V = 14, 13 dm3. 3. 1, 175 m3. 4. ^a{ite sobiraat podednakvo. 5. P = 154 cm2,
V = 180 cm3. 6. 1, 67 dm. 7. Upatstvo. Dobienata topka }e ima volumen ednakov na zbirot
od volumenite na dvete topki. Odgovor. 8 cm. 8. 105 163 000 000 km3. 9. 6, 45 cm. 10. 6 cm.
S 5  S 5  S 5 
11. 20983 hl. 12. 9.  97  S 5  9& .
  

Zada~i za povtoruvawe i utvrduvawe - IV - str. 199

1. 4. 2. 4. 3. Da. 4. Upatstvo. Izberi dve to~ki od ednata prava i edna to~ka od vtorata
prava. Tie se nekolinearni i opredeluvaat edna ramnina. Toa e baranata ramnina. Poka`i
deka taa ramnina e edinstvena. 5. Ako p ne e paralelna so S, toga{ p ja proboduva ramninata
S vo nekoja to~ka P. No, paralelnite pravi p i q opredeluvaat ramnina P i pritoa P Î P.
Pravata q minuva niz P (zo{to?), pa dobivame deka pravite p i q se se~at vo to~ka
P - kontradikcija. 6. 8, 06 cm. 7. 365, 5 cm3. 8. P = 248 cm2, V = 240 cm3. 9. Mo`e.
10. P | 89, 6 cm2, V = 34 cm3. 11. Paketot }e sodr`i 150 kutii a negoviot volumen }e bide
3375 cm3. 12. a = 6 cm, H = 4,9 cm3. 13. 91cm2. 14. 47 cm2. 15. P | 360 cm2, V | 400 cm3.
16. P | 45 cm2, V | 15, 6 cm3. 17. 11, 5 litri vo sekunda. 18. 376, 8 cm2. 19. P | 208 cm2, V | 226 cm3.
20. Mo`e da pomine, i }e mu ostanat u{te 3,1 litri benzin. 21. P | 177 cm3, V | 140 cm3.

22. P | 51,4 cm2, V | 30,5cm3. 23. P = pH2, 9 S +  . 24. a) P | 301 cm2, V | 452 cm3, b) P | 452 cm2,

V | 553 cm3. 25. a) P | 160 cm2, V | 160 cm3, b) P | 272 cm2, V | 226 cm3. 26. 975 g. 27. 2, 25 cm.

28. 216.29.  km. 30. | 48%.


Zada~i za samokontrola - IV - str. 201

1. 6. 2. A', B' i C' se kolinearni i B' le`i pome|u A' i C'. 3. 8 dm, 4. 280, 6 cm3. 5. 9, 2 cm.
6. V = 288 cm3, P = 341 cm2. 7. 5 dm. 8. V1 : V2 = (pb2a) : (pa2b) = b : a. 9. 2922,8 cm2. 10. 9 cm. 11. 38 026
656 km2. 12. R31 : R32.

216
AZBU^NIK NA POIMITE

- ravenki 50
Agol me|u - neravenki 69
- dve pravi 137 - sistemi ravenki 105
- dve ramnini 138
- prava i ramnina 137 Identitet 46
Aksioma za paralelnost 127 Interval 67
Apotema na piramida 164 -, beskone~en 68
Brojna oska 67 -, zatvoren 67
Bo~na povr{ina -, otvoren 67
- na konus 185 -, poluotvoren 67
- na piramida 162 Yidovi
- na prizma 142 -, bo~ni
Bo~ni yidovi - na paralelopiped 140
-na piramida 163 Kvadar 155
- na prizma 143 Kocka 155
Bo~ni rabovi Koeficient na proporcionalnost 7
- na piramida 163 Kolinearni to~ki 128
- na prizma 143 Komplanarni to~ki 129
Visina na Konus
- konus 185 -, prav kru`en 184
- piramida 163 -, osnova na 185
- prizma 143 -, vrv na 185
- cilindar 176 -, radius na 185
Verojatnost 120 -, mre`a na 187
- na nastan 120 -, oskin presek na 185
Volumen na Linearna ravenka
- kvadar 155 - so edna nepoznata 55
- kocka 155 - so dve nepoznati 97
- piramida 172 -, normalen vid na 55
- prava prizma 153 -, grafik na 98
- telo 154 Linearna neravenka
Generatrisa na - so edna nepoznata 73
- konus 185 Linearna funkcija 80
- cilindar 176 -, grafik na 82
Geometrisko telo 139 Mre`a na prizma 147
-, val~esto 140 - na piramida 167
-, rabesto 140 - na konus i cilindar 187, 178
-, yidovi na 140 Nastan
-, rabovi na 140 -, nevozmo`en 118
-, povr{ina na 140 -, slu~aen 118
-, temiwa na 140 -, povolen 120
Grafik na -, sprotiven 121
- linearna ravenka 98 Neravenka
- funkcija 82 - so edna nepoznata 63
Dijagonala na - so dve nepoznati 63
- kvadar 146 -, re{enie na 69
- kocka 147 -, linearna 64
- paralelopiped 145 Neravenstvo 62
- prizma 143 -, brojno 63
Dijagonalna ramnina 144 - so promenliva 63
Dijagonalen presek na -, algebarsko 63
- prizma 144 -, dvojno 65
- piramida 165 Odnos na perimetrite na dva sli~ni
Dimenzii na kvadar 145 triagolnika 21
Dijagram Odnos na plo{tinite na dva sli~ni
Evklidova teorema 28 triagolnika 24
Ekvivalentni Osnova na
- prizma 142

217
- piramida 168
Rastojanie od to~ka do ramnina 140
- konus 191
Ravenstvo 45
- cilindar 182
Ramnini 127
Paralelopiped 151
-, paralelni 132
-, dijagonali na 152
-, normalni 138
-, sprotivni yidovi na 152
Re{avawe na sistem linearni ravenki
Piramida
-, metod na zamena 104
-, pravilna 170
-, metod na sprotivni koeficienti 106
-, osnovni rabovi na 169
-, metod na grafi~ko re{avawe 109
-, vrv na 168
Pitagorova teorema 30 Sistem linearni ravenki
Plo{tina na - so dve nepoznati 100
- konus 194 -, homogen 101
- piramida 174 Sistem linearni neravenki 77
- prizma 155 Sli~nost
- sfera 201 -, koeficient na 17
- cilindar 185 -, priznaci za 19
Povr{ina Statisti~ka masa 119
-, bo~na 168, 148, 191 Sfera
-, rotaciona 181 -, mala kru`nica na 194
-, kru`na cilindri~na 181 -, golema kru`nica na 194
-, kru`na konusna 190 -, plo{tina na 195
Poliedar Talesova teorema 11
-, yidovi na 146 Tetraedar 163
-, rabovi na 146 -, pravilen 194
-, temiwa na 146 Tangentna ramnina
Populacija 124 - na topka 194
-, obem na 125 Topka
Pravi 133 -, radius na 193
-, paralelni 137 -, dijametar na 194
-, razminuva~ki 137 -, golem krug na 194
Primerok 125 -, mal krug na 194
-, obem na 125 -, centar na 193
Presek -, volumen na 197
-, paralelen 150, 183 Triagolnik
-, dijagonalen 150 -, Egipetski 31
-, oskin 183, 191 -, Indiski 31
Prizma Figuri 127
-, prava 149 -, prostorni 127
-, kosa 149 -, ramninski 127
-, pravilna 150 -, sli~ni 17
-, paralelen presek na 150 Funkcija
Proektirawe - na pravata proporcionalnost 81
- na to~ka 135 -, linearna 81
-, ortogonalno 135 -, nula na 88
-, paralelno 134 Cilindar
-, koso 135 -, prav kru`en 176
Proekcija 135 -, radius na 176
Proektira~ki pravec 135 -, oska na 176
Proporcionalni otse~ki 7, 28 -, ramnostran 174
Proporcija 6 -, oskin presek na 177
Razmer
-, obraten 5
-, vrednost na 5
-, prodol`en 6
Ravenka
- so edna nepoznata 47
- so dve nepoznati 47
-, nepoznati na 47
-, re{enie na 48
-, definiciona oblast na 46

218
SODR@INA

A PROPORCIONALNI OTSE^KI

I.1. Razmer ...................................................................................................................... .... 5


I.2. Proporcionalni otse~ki ........................................................................................ 7
I.3. Delewe na otse~ka na ednakvi delovi ............................................................... 9
I.4. Talesova teorema ................................................................................................... 11
I.5. Primena na Talesovata teorema ....................................................................... 13

B SLI^NI TRIAGOLNICI

I.6. Sli~ni figuri. Sli~ni triagolnici ................................................................ 16


I.7. Priznaci za sli~nost na triagolnicite ......................................................... 19
I.8. Odnos na perimetrite na dva sli~ni triagolnika ....................................... 21
I.9. Odnos na plo{tinite na dva sli~ni triagolnika ........................................ 24
I.10. Primena na sli~nosta na triagolnici ............................................................ 25

V PITAGOROVA TEOREMA

I.11. Sli~nost vo pravoagolen triagolnik (Evklidovi teoremi) .....................28


I.12. Pitagorova teorema ..............................................................................................30
I.13. Primena na Pitagorovata teorema vo konstruktivnite zada~i ............ 33
I.14. Primena na Pitagorovata teorema na ramninski geometriski figuri ...... 36
Zada~i za povtoruvawe i utvrduvawe - I .................................................. 42
Zada~i za samokontrola - I ................................................................................... 43
LINEARNI RAVENKI

219
A LINEARNI RAVENKI

II.1. Ravenstvo. Identitet. Ravenka .........................................................................45


II.2. Vidovi ravenki .......................................................................................................47
II.3. Re{enie na ravenka ...............................................................................................48
II.4. Ekvivalentni ravenki ..........................................................................................50
II.5. Osnovni svojstva na ravenstvata i ravenkite ..............................................51
II.6. Op{t vid na linearna ravenka so edna nepoznata .......................................54
II.7. Re{avawe na linearni ravenki so edna nepoznata .....................................55
II.8. Primena na linearna ravenka so edna nepoznata ........................................58

B LINEARNI NERAVENKI SO EDNA NEPOZNATA

II.9. Neravenstvo i neravenka .....................................................................................62


II.10. Svojstva na brojnite neravenstva .....................................................................64
II.11. Brojna oska. Intervali ........................................................................................67
II.12. Re{enie na neravenka. Ekvivalentni neravenki ........................................69
II.13. Teoremi za ekvivalentni neravenki ................................................................70
II.14. Re{avawe na linearna neravenka so edna nepoznata .................................73
II.15. Primena na linearni neravenki so edna nepoznata ....................................75

V SISTEM LINEARNI NERAVENKI SO EDNA NEPOZNATA

II.16. Re{enie na sistem linearni neravenki so edna nepoznata ......................77


II.17. Re{avawe na sistem linearni neravenki so edna nepoznata ...................78

G LINEARNA FUNKCIJA

II.18 Linearna funkcija .. ....................................................................................80


II.19. Grafik na funkcijata u = ah ........................................................................... 82
II.20. Grafik na funkcijata u = ah + b .................................................................... 84
II.21. Zaemna polo`ba na graficite na nekoi linearni funkcii .................86
II.22. Rastewe, opa|awe i nula na linearna funkcija ...........................................88

220
II.23. Grafi~ko re{avawe na linearna ravenka so edna nepoznata .................. 89
Zada~i za povtoruvawe i utvrduvawe - II .................................................. 91
Zada~i za samokontrola - II ......................................................................... 93

A LINEARNI RAVENKI SO DVE NEPOZNATI

III.1. Ekvivalentni linearni ravenki so dve nepoznati ...................................... 97


III.2. Linearna ravenka so dve nepoznati .................................................................. 95

B SISTEM OD DVE LINEARNI RAVENKI SO DVE NEPOZNATI

III.3. Sistem od dve linearni ravenki so dve nepoznati ................................... 100


III.4. Grafi~ko re{avawe na sistem linearni ravenki so dve nepoznati ... 102
III.5. Ekvivalentni sistemi ....................................................................................... 105
III.6. Re{avawe na sistem linearni ravenki so dve nepoznati
so metod na zamena .............................................................................................. 107
III.7. Re{avawe na sistem linearni ravenki so dve nepoznati
so metod na sprotivni koeficienti .............................................................. 109
III.8. Primena na sistemite linearni ravenki so dve nepoznati ................... 111

V RABOTA SO PODATOCI

III.9. Princip na Dirihle ........................................................................................... 114


III.10. Populacija. Primerok ....................................................................................... 116
III.11. Nastan. Siguren i slu~aen nastan ................................................................. 117
III.12. Sistemski i slu~aen izbor na podatoci ....................................................... 119
III.13. Verojatnost na slu~aen nastan ........................................................................ 120
III.14. Eksperimentalna proverka na rezultatite ................................................ 122
Zada~i za povtoruvawe i utvrduvawe - III ............................................... 124
Zada~i za samokontrola - III ....................................................................... 126

221
A TO^KA, PRAVA I RAMNINA VO PROSTOROT

IV.1. Zaemni polo`bi na to~ki, pravi i ramnini vo prostorot ...................... 127


IV.1.1. Zaemna polo`ba na to~ka i prava .................................................................. 128
IV.1.2. Zaemna polo`ba na to~ka i ramnina ............................................................. 129
IV.1.3. Zaemna polo`ba na prava i ramnina ............................................................. 129
IV.1.4. Zaemna polo`ba na dve pravi vo prostorot ................................................ 130
IV.1.5. Zaemna polo`ba na dve ramnini ..................................................................... 132
IV.2. Paralelno proektirawe i ortogonalno proektirawe ............................. 133
IV.2.1. Normala na ramnina ........................................................................................... 133
IV.2.2. Paralelno proektirawe .................................................................................... 134
IV.2.3. Ortogonalno proektirawe ............................................................................... 135
IV.3. Agli me|u pravi i ramnini ............................................................................... 137
IV.4. Geometriski tela ................................................................................................ 139

B PRIZMA

IV.5. Poim i elementi na prizmite ......................................................................... 141


IV.6. Vidovi i preseci na prizmite ......................................................................... 145
IV.7. Paralelopiped ..................................................................................................... 145
IV.8. Mre`a na prizma ................................................................................................. 147
IV.9. Plo{tina na prizma ........................................................................................... 149
IV.9.1. Plo{tina na kvadar i kocka ............................................................................ 150
IV.9.2. Plo{tina na nekoi pravilni i nekoi pravi prizmi ................................. 151
IV.10. Volumen na prizma .............................................................................................. 154
IV.10.1. Op{to za volumen na telata ........................................................................... 154
IV.10.2. Volumen na kvadar i kocka ............................................................................... 155
IV.10.3. Volumen na prava prizma .................................................................................. 158

222
V PIRAMIDA

IV.11. Poim i elemeneti na piramidata ................................................................... 162


IV.12. Vidovi i svojstva na piramidite. Dijagonalen presek ............................ 163
IV.13. Mre`a na piramida ............................................................................................. 166
IV.14. Plo{tina na piramida ...................................................................................... 168
IV.15. Volumen na piramida .......................................................................................... 172

G CILINDAR

IV.16. Poim i elementi na cilindarot. Preseci na cilindar ........................... 175


IV.17. Mre`a na cilindar ............................................................................................. 178
IV.18. Plo{tina na cilindar ....................................................................................... 179
IV.19. Volumen na cilindar .......................................................................................... 181

D KONUS

IV.20. Poim i elementi na konusot. Oskin presek ................................................ 184


IV.21. Mre`a na konus .................................................................................................... 186
IV.22. Plo{tina na konus ............................................................................................. 188
IV.23. Volumen na konus ................................................................................................. 190

\ TOPKA

IV.24. Poim za sfera i topka ....................................................................................... 193


IV.25. Plo{tina na sfera ............................................................................................ 195
IV.26. Volumen na topka ................................................................................................. 197
Zada~i za povtoruvawe i utvrduvawe - IV .................................................. 199
Zada~i za samokontrola - IV ............................................................................ 201

Odgovori i upatstva ............................................................................................................... 203

Azbu~nik na poimite .............................................................................................................. 217

223

You might also like